Está en la página 1de 334

Universidad La Salle.

Facultad Mexicana de Medicina.


Curso de Extensión Universitaria para la Preparación del Examen Nacional para
Aspirantes a Residencias Médicas.
Examen Final 1º Septiembre 2010.
Modalidad a distancia.

1.- La profilaxis de una neumonía por Pneumocystis en un paciente de cinco años de edad, se
encuentra indicada cuando los linfocitos ajustados para la edad son de:

a) < 1500 Células/µl


b) < 750 Células/µl
c) < 500 Células/µl
d) < 200 Células/µl

La NPC en niños y en el embarazo.

La NPC es infección diagnóstica de SIDA en un alto porcentaje de niños, sobre todo en el


primer año de vida. Los niños menores de 1 año con CD4+ por debajo de 1500// l tienen un
90% de riesgo de padecerla.

Las manifestaciones clínicas, el diagnóstico y el tratamiento no difieren de los del adulto.


Para la prevención deben seguirse las siguientes recomendaciones:

• Se aconseja la profilaxis primaria en todos los niños menores de 1 año hijos de


madre VIH+
• La profilaxis debería comenzar a las 4-6 semanas de vida y suspenderse en caso de
que posteriormente no resulten estar infectados.
• Los niños infectados y aquellos cuyo estado de infección por VIH se desconozca
deberán seguir recibiendo tratamiento profiláctico durante el primer año de vida.
• En los mayores de esta edad se considerará continuar con la profilaxis atendiendo
al recuento de linfocitos CD4+ según la edad: de 1 a 5 años, cuando el recuento sea
< 500 células/ l o el porcentaje < 15%; en niños de 6 a 12 años, cuando el recuento
sea inferior a 200 células/ l o el porcentaje < 15%.

• No se ha estudiado la seguridad de interrumpir la profilaxis en niños infectados con


el VIH y sometidos a tratamiento antirretroviral.
• Los niños con un historial que incluya episodios de NPC deberán ser tratados de por
vida con quimioprofilaxis para evitar casos de recurrencia.
NCP en el embarazo

Como ya se ha dicho, la quimioprofilaxis anti-NPC debe administrarse a las mujeres


embarazadas al igual que al resto de adultos y adolescentes. En este caso, el agente
profiláctico recomendado sigue siendo el cotrimoxazol, con dapsona como alternativa.
Debido a la posibilidad teórica de una posible teratogenicidad asociada a la exposición a los
fármacos durante el primer trimestre de gestación, durante dicho período puede
considerarse la alternativa de pentamidina en aerosol debido a que dicho agente no se
absorbe sistémicamente por lo que el feto en desarrollo no sufre exposición al fármaco.

Lectura recomendada:
Neumonía por Pneumocystis carinii en niños infectados por el virus de
inmunodeficiencia humana (VIH). Gac Med Mex 2004; 140 (1): 59-70.

2.- El Tratamiento de elección para la Isosporosise es:

a) Trimetoprim Sulfametoxazol.
b) Albendazol.
c) Metronidazol.
d) Pentamidina.

El tratamiento más efectivo contra la isosporiasis consiste en una combinación de dos


medicamentos: trimethoprim y sulfamethoxazole (TMP-SMX, Bactrim, Septra). Para tratar
la isosporiasis, se toman dos pastillas de TMP-SMX de doble potencia dos veces al día.
Otra posibilidad es tomar una pastilla de doble potencia tres veces al día. El tratamiento
con TMP-SMX suele continuarse durante dos a cuatro semanas.

Referencia Bibliográfica:

Título Del Libro: Guía De Antimicrobianos, Antivirales, Antiparasitarios,


Antimicóticos E Inmunomod.
Autor Principal: Dr. Napoleón González Saldaña
Editorial: Mc Graw Hill Interamericana. Edición: 6ª. Año: 2004 Páginas 257-278.

3.- Un estudiante es tacleado mientras jugaba football y desarrollo dolor en rodilla severo.
A la EF la rodilla se encuentra edematizada y el paciente tiene dolor a la palpación directa
del aspecto lateral de la rodilla. Cuando se flexiona la rodilla 30°, la aducción pasiva
ocasiona dolor en la misma área, y la pierna puede ser aducida más que la pierna
contralateral. Cajon anterior, cajón posterior y Lachman negativos. ¿Cuál es el sitio más
probable de lesión?

a) Ligamento cruzado anterior.


b) Ligamento colateral lateral.
c) Menisco lateral.
d) Ligamento cruzado posterior.

Se produce por un trauma en varo al golpearle en la parte externa de la pierna por debajo
de la rodilla o al caerle otro jugador o contusión en la cara interna de la rodilla. Presenta
dolor y generalmente sensación de parestesia en el territorio del nervio ciático poplíteo
externo por distensión, que puede ser una parestesia transitoria en traumas de menor
energía o una lesión definitiva con pie caído en traumas de alta energía. El paciente
presenta dolor, derrame leve a moderado y equimosis en cara lateral de la rodilla.

El diagnostico se hace al encontrar:

1. Bostezo en varo en 30 grados de flexión: para evaluar el ligamento colateral lateral,


el dolor es en trayecto del ligamento. Puede ser de diferente magnitud:

• Grado I: muy discreta apertura de la interlinea comparado con lado contra lateral.

• Grado II: apertura de 5 mm de la interlinea articular.

• Grado III: apertura de 10 mm de la interlinea articular.

2. Bostezo en varo en extensión completa: para evaluar esquina postero lateral, capsula
postero lateral y ligamento fibulo poplíteo.

Las estructuras laterales tienen un potencial de cicatrización menor que las estructuras
mediales.

En las lesiones grado I y II se debe inmovilizar y proteger el apoyo durante seis semanas.

Se debe tener un alto índice de sospecha y en los casos grado III y donde se presente
compromiso de la capsula postero lateral, se debe realizar cirugía para reparo primario
en las primeras dos semanas, pues el reparo primario funciona mejor que cualquier
reconstrucción anatómica.
4.- ¿Cuál es la tríada clínica clásica de la rotura espontánea del esófago o síndrome de
Boerhaave?

a) Náuseas, vómitos y fiebre.


b) Vómitos, dolor torácico y enfisema subcutáneo.
c) Fiebre, dolor abdominal y hematemesis.
d) Dolor torácico, dolor abdominal y vómito.

Definición: ruptura espontánea del esófago que ocurre sobre todo como consecuencia de un
violento vómito.

Este síndrome fue descrito por primera vez en 1724 por Herman Boerhaave. Se
diferencia del síndrome de Mallory-Weiss en que en este caso se tratada de una
perforación transmural mientas que el síndrome de Mallory-Weiss es tan solo una
laceración. Dado que frecuentemente está asociado al vómito, no puede decirse que en
realidad se trate de una ruptura espontánea. Hay que distinguirlo de la perforación
yatrogénica que supone el 85-90% de los casos de rupturas esofágicas. Casi siempre en el
lado izquierdo del tercio inferior esofágico a 2 o 3 cm de la unión gastroesofágica. A igual
que el síndrome de Mallory-Weiss, se ha relacionado con los aumentos bruscos de la presión
intrabdominal y con la hernia de hiato. Es más frecuentemente observado en pacientes
alcohólicos o que abusan de la comida.

Se trata de un síndrome relativamente raro pero con un alto índice de mortalidad (35%).
De hecho, es considerada como la más letal de todas las perforaciones del tracto digestivo.

Los síntomas son vómitos, dolor torácico y enfisema subcutáneo cervical.

REFERENCIAS

C. Yeo. Shackelford' surgery of the Alimentary Tract, 2007. Saunders, 2700 páginas
Khan, Aamir Z.; Forshaw, Mathew J.; Davies, Andrew R.; Youngstein, Taryn; Mason,
Robert C.; Botha, Abraham J. Transabdominal Approach for Management of Boerhaave's
Syndrome . American Surgeon , May2007, Vol. 73 Issue 5, p511-513,
Hill, Andrew G.; Tiu, Albert T.; Martin, Iain G. Boerhaave's syndrome : 10 years
experience and review of the literature: H. ANZ Journal of Surgery , Dec2003, Vol. 73
Issue 12, p1008-1010,
Ochiai, T.; Hiranuma, S.; Takiguchi, N.; Ito, K.; Maruyama, M.; Nagahama, T.; Kawano, T.;
Nagai, K.; Nishikage, T.; Noguchi, N.; Takamatsu, S.; Kawamura, T.; Teramoto, K.; Iwai, T.;
Arii, S. Treatment strategy for Boerhaave's syndrome.. Diseases of the Esophagus,
May2004, Vol. 17 Issue 1, p98-103.
5.- Ante un paciente portador de úlcera gástrica ¿Cuál de las siguientes es la indicación
quirúrgica?

a) Sangrado de tubo digestivo alto con inestabilidad hemodinámica.


b) Menor de 60 años.
c) Biopsia negativa a tumor maligno.
d) La úlcera no cicatriza después de 12 semanas de tratamiento bien llevado por el
paciente.

Úlcera Péptica:

50-75 % de casos.

La hemorragia por úlcera duodenal es 4 veces más frecuente que la úlcera gástrica, pero
ambas tienen la misma tendencia a la hemorragia.

La Hemorragia Masiva tiene una frecuencia del 10-15% y generalmente es producida por
horadación por el proceso inflamatorio de la arteria regional.

* Localización:

a) Bulbo duodenal.
b) Curvatura menor.
c) Zona prepilórica.

De ellas, la curvatura menor hace las hemorragias masivas más frecuentes, pero la
duodenal, en general, es la más frecuente.

La relación entre la localización de la úlcera y la gravedad de la hemorragia se debe a la


lesión de las arterias principales, de la curvatura menor, la coronaria estomáquica, la
pilórica y la gastroduodenal.

TRATAMIENTO DE LA HEMORRAGIA DIGESTIVA ALTA.

Es una emergencia Médico-Quirúrgica, de gran importancia y frecuencia, cuya morbilidad y


mortalidad están influenciadas por un manejo oportuno, coherente, en el que
necesariamente concurren varios especialistas, en lo posible organizados en equipo.
Es evidente que el manejo en equipo y la identificación de los pacientes de alto riesgo se
convierten en las armas más importantes en el manejo actual de esta patología.
TRATAMIENTO QUIRURGICO:

El momento de la indicación y la técnica a realizar son los pilares principales de una desición
correcta. Las técnicas a realizar serán de acuerdo a la experiencia del cirujano y el riesgo
quirúrgico del enfermo. Tales como: Vagotomía troncular piloroplastía, es más rápida y
menos traumática, elimina factor vagal, baja mortalidad postoperatoria, pero alto índice de
recidiva del sangrado.

Vagotomía más Gastrectomía 4/5 elimina todo el factor sangrante, usada en fracaso de
otras técnicas.

Gastrectomía total: en casos especiales. Estado general del paciente, edad, obesidad,
repetición de sangrado indican la técnica a usar.

En Úlcera Duodenal la Vagotomía más piloroplastía y sutura de la úlcera es lo más adecuado.

En Úlcera Gástrica, Gastrectomía y Sutura de la Úlcera: Gastrectomía subtotal Bilroht I,


resección en cuña de úlcera.

Várices esofágicas: sonda de Sengstaken o de Linton.


Técnica de desconección portoacigos o Tanner modificada.

Lectura recomendada:
Yamada Manual de gastroenterologia pagina 337.

6.- El agente causal más frecuentemente encontrado como causa de Nocardiosis es:

a) Nocardia Brasiliensis.
b) Nocardia Asteroides.
c) Nocardia Farcinica.
d) Nocardia Otitidis Cavarum.

La nocardiosis es una enfermedad infecciosa relativamente poco común, con una frecuencia
anuale stimada entre 500 y 1000 casos por año en los Estados Unidos de América. Las dos
especies más prevalentes son Nocardia asteroides y N. brasiliensis (10) y pueden causar un
amplio espectro de manifestaciones clínicas localizadas y diseminadas. El complejo N.
asteroides ha sido considerado el responsable de la mayoría de las infecciones invasivas
severas tales como neumonía y abscesos cerebrales Post-traumatismo, diseminándose
frecuentemente a otros sitios en pacientes inmunocomprometidos (3, 8). En Contraste, N.
brasiliensis está generalmente asociada a infecciones de piel y partes blandas,
comunicándose pocos casos de infecciones disemina- das (6, 8).
Aquí se describe el caso de una niña de 4 años de edad previamente sana, que presentó
linfadenitis necrotizante postraumática causada por N. asteroides y que evolucionó
satisfactoriamente con cefalotina.

Referencia Bibliográfica:

Título Del Libro: Micología Médica Básica


Autor Principal: Dr. Alejandro Bonifaz
Editorial: Méndez Cervantes.Edición: 1ª. Año: 1991, páginas 389-395.

7.- Masculino portador de una Micosis producida por una levadura capsulada que afecta
pulmón y sistema nervioso, ¿La patología de este paciente es?

a) Micetoma.
b) Histoplasmosis.
c) Criptococosis.
d) Criptosporidiosis.

La criptococosis es una enfermedad sistémica causada por una levadura encapsulada de


distribución cosmopolita, Cryptococcus neoformans, cuyo estado perfecto es el
Basidiomycete, Filobasidiella neoformans que tiene dos variedades: neoformans y
bacillispora. Se han reportado cinco serotipos y dos variedades biológicamente distintas: C.

neoformans variedades neoformans (A, B y AD) y gattii (B y C)1,3. Crece a 25º y a 37º en
agar-Saboraud y agar-sangre, y se encuentra en el suelo, vegetales descompuestos,
excremento de aves y murciélagos. El agente es capaz de sobrevivir varios años
En ambiente oscuro y húmedo, y puede transportarse por el polvo. La enfermedad se
adquiere por inhalación de esporas, afecta en primer lugar al pulmón y se disemina luego por
vía hematógena afectando otros órganos y sistemas1.
Arenas R. Micología médica ilustrada 2º edición. Mc Graw Hill 2003;205-211.

8.- Masculino de 40 años recién llegado del estado de Chiapas, Méx. Acude a consulta por
referir presentar fiebre elevada de tres días de tres días de evolución refiere artralgias
francas y cefalea intensa. El mismo día de su visita a urgencias comenzó a presentar un
exantema maculo-papuloso pruriginoso. El examen de la sangre mostró los siguientes datos:
Valor hematocrito 38%, Leucocitos 3600 p.mm3 con 82% neutrófilos y 12% linfocitos.
Plaquetas 115.000 p.mm3. Placa de tórax normal. El paciente fue dado de alta con el
diagnóstico de cuadro viral, en tratamiento con paracetamol y antihistamínicos para el picor.
A las 48 horas volvió a urgencias con muy mal aspecto: estaba afebril, tenía confusión
mental, se observaban petequias en antebrazos y piernas, edema en pies, TA 85/70. Pulso
110 l.p.min. de amplitud pequeño. En los nuevos exámenes de la sangre destacaban: Valor
hematocrito 46%, leucocitos 3600 p.mm3 sin cambios en la fórmula y plaquetas 65.000
p.mm3. Glucosa 106 mg.p. dl. Creatinina 1,8 mg.p.dl. Sodio 126 mEq/l.Potasio 4,2 mEq/l. La
placa de tórax mostraba un pequeño derrame pleural bilateral. ¿Cuál es, entre los
siguientes, el diagnóstico más probable?:

a) Dengue.
b) Meningoencefalitis bacteriana.
c) Fiebre tifoidea.
d) Neumonía por Legionella Neumophila.

El dengue es actualmente la más importante arbovirosis que afecta al hombre. Su agente


etiológico son los 4 serotipos del virus del dengue (D1-4). Se transmite entre humanos
mediante la picadura del mosquito Aedes aegypti. Se estima que el 40% de la población
mundial vive en áreas de riesgo de esta entidad (1,2).

La forma clínica más grave, la fiebre hemorrágica de dengue/síndrome de choque del


dengue (FHD/SCD), fue casi exclusiva del Sudeste Asiático y el Pacífico Occidental hasta
1981, cuando una gran epidemia de dengue, la mayor de las Américas, que incluyó más de
10.000 casos de FDH/SCD, ocurrió en Cuba. Esta constituyó la primera epidemia de DH en
la región (3). Posterior a este brote y hasta la actualidad la FHD continúa presentándose de
forma endémica en diferentes países, ocurriendo epidemias de forma frecuente (4).

Cuadro clínico

El curso de la enfermedad, desde el punto de vista clínico, se puede dividir en cuatro fases:
Inicial, crítica, de recuperación y de convalecencia.

Fase inicial: En ella el enfermo tiene un síndrome febril sin localización. Es muy
sintomática y además de la fiebre predominan las manifestaciones generales como cefalea,
artromialgias, dolor retroocular y malestar, puede aparecer rash.

Suele durar alrededor de 3 días, tras lo cual la fiebre cede y algunos casos de dengue
clásico comienzan a presentar manifestaciones hemorrágicas leves, pero la mayoría tienden
a mejorar. Otro grupo menor de pacientes desarrollará la FHD/SCD. Los llamados signos de
alarma que preceden al choque pueden comenzar en esta fase.

Fase crítica: Transcurre entre el 4to y 7mo día de la enfermedad, en ella se presentan los
síntomas que definen al DH, como son la extravasación de plasma, la trombocitopenia, y las
manifestaciones hemorrágicas. Algunos pacientes desarrollan el síndrome de choque por
dengue que es la forma más severa de la enfermedad. Las manifestaciones hemorrágicas
pueden ser tan leves que solo se hacen evidentes a través de la prueba del torniquete, o tan
severas como sangramientos digestivos graves con compromiso hemodinámico.
Los signos de extravasación de líquidos (derrame en serosas, hemoconcentración) son
indispensables para hablar de FHD/SCD, ya que es la fuga de líquido la que casi siempre
lleva el paciente al choque y no las hemorragias. Este hecho es de gran importancia pues
influenciado por el nombre de fiebre hemorrágica de dengue, en no pocas ocasiones los
médicos asistentes esperan las grandes hemorragias que nunca llegan y el paciente cae en
choque por la extravasación de plasma.

Fase de recuperación: Se inicia cuando cesa el escape de líquido y las manifestaciones de


sangrado comienzan a disminuir. Pueden aparecer edemas o agravarse los derrames serosos
producto de la sobre hidratación. También se puede apreciar un rash tardío asociado a
prurito intenso. Se recuperan el apetito y el número de plaquetas.

Fase de convalecencia: Puede prolongarse hasta más de 6 meses, se caracteriza por


cefalea discreta, cansancio fácil y artromialgias. No se presenta en la totalidad de los
casos.

Resumimos los signos de alarma de la siguiente manera:

1. Signos clínicos tempranos del choque.


· Caída brusca de la fiebre.
· Dolor abdominal intenso.
· Irritabilidad, somnolencia u otras alteraciones mentales.
· Fatiga extrema.
· Lipotimias.
· Dolor torácico.

2. Signos clínicos que agravan el choque.


· Vómitos frecuentes.
· Diarreas frecuentes.

3. Signos de laboratorio e imagenológicos.


· Aumento progresivo del hematócrito y
disminución progresiva del conteo de plaquetas.
· Engrosamiento de la pared de la vesícula biliar.

Referencias

1. Guzman MG, Kouri G. Dengue-an update. The Lancet Inf Dis 2002;2:33-42.
2. Gubler DJ, Clark CG. Dengue/dengue hemorrhagic fever: the emergence of a global
health problem. Emerging Infectious Diseases. Atlanta USA. 1995;1:55-57.
3. Kourí GP, Guzmán MG, Bravo JR, Triana C. Dengue haemorrhagic fever/ dengue shock
syndrome: lessons from the Cuban epidemic. 1981. Bull World Health Organ 1989;87:375-
80.
4. Organización Panamericana de la Salud. 2003: Number of reported cases of dengue and
dengue hemorrhagic fever (DHF), region of the Americas (by country and subregion).
[citado del 25 de agosto de 2003].
5. Valdes L, Guzman MG, Kouri G, Delgado J, Carbonell I, Cabrera MV, Rosario D, Vazquez
S. La Epidemiología del Dengue en Cuba en 1997. Rev Panam Salud Publica/Pan American
Journal of Public Health 1999;6:16-25.
6. Pelaez O et al. Havana dengue 3 epidemic, 2001. Enviada a Emerging Infection Diseases.
7. George R, Lum LCS. Clinical spectrum of dengue infection. In: Gubler DJ, Kuno G, eds.
Dengue and dengue hemorrhagic fever. London: CAB International, UK, 1997; 89-113.
8. Kourí G, Guzmán MG, Bravo J. Why dengue haemorrhagic fever in Cuba? 2: An integral
analysis. Trans R Soc Trop Med Hyg 1987;81:821-23.

9.- Femenino de 35 años con antecedentes de trasplante renal hace mes y medio, acude al
servicio por referir fiebre desde hace más de tres días, dolor en epigastrio así como
cansancio leve.. En la analítica practicada destaca una moderada leucopenia (2400/mm3)
con una leve elevación en la cifra de transaminasas (ALT 75 UI/l; AST 89 Ul/l) ¿Cuál sería
el primer diagnóstico de sospecha?

a) Tuberculosis pulmonar.
b) Infección por Helicobacter pilorii.
c) Infección por Citomegalovirus.
d) Hepatitis por VHC.

El Citomegalovirus (CMV) es una infección oportunista. Aproximadamente el 50 por


ciento de la población general y el 90 por ciento de las personas VIH positivas son
portadoras del CMV. En general, un sistema inmune sano puede mantener la infección bajo
control. Pero cuando el VIH o cualquier otra enfermedad debilitan al sistema inmune, el
CMV puede atacar varias partes del cuerpo.

Causas y Factores de Riesgo:

La transmisión del CMV puede ser: Fetal, perinatal a través de secreciones cervicales y por
la leche materna. Posteriormente por vía respiratoria, sexual (semen y secreciones
cervicales), transfusiones de sangre, trasplante de órganos. Los individuos que
particularmente tienen mayor riesgo de adquirir una infección seria por CMV son:

1. Pacientes infectados con el VIH, especialmente aquellos que desarrollan SIDA.

2. Pacientes que han recibido un trasplante de médula ósea ó de un órgano sólido (Hígado,
Riñón, Corazón).

3. Pacientes con Cáncer, en especial aquellos que involucran las células sanguíneas.

4. Por transmisión Intrauterina de CMV.


Las infecciones adquiridas después del nacimiento o en la fase adelantada de la vida,
frecuentemente son asintomáticas. Una enfermedad febril aguda, denominada
mononucleosis por citomegalovirus o bien hepatitis por citomegalovirus, puede depender
del contacto iatrógeno o espontáneo con el CMV. Por su parte, el síndrome posperfusión se
presenta de 2 a 4 semanas después de una transfusión con sangre fresca que contenga
CMV, y se caracteriza por fiebre que dura de 2 a 3 semanas, hepatitis de intensidad
variable, con ictericia o sin ella, linfocitosis atípica con característica similar a la de la
mononucleosis infecciosa y, en ocasiones, una erupción.12,24 Entre los episodios febriles, el
paciente, aunque cansado, no se siente muy enfermo.23,25

Las infecciones por CMV son frecuentes en todas las formas de aloinjertos, y puede causar
neumonitis intersticial, hepatitis, encefalitis, retinitis y enfermedad difusa de inclusión
citomegálica. Se ha detectado en homosexuales y en pacientes con el síndrome de
inmunodeficiencia adquirida (SIDA).

De acuerdo con esto, gran parte de la morbilidad y cierta mortalidad relacionadas con el
SIDA se atribuyen a infecciones por CMV de hígado, cerebro (con nódulos gliales), aparato
digestivo (lesiones ulcerosas) pulmones, pneumonitis intersticial difusa que coexiste con
frecuencia con la infección por Pneumocystis carinii, y en los ojos (retinitis). Se considera
que desempeña alguna función en la aterogenesis.10

Crumpacker CS, Wadhwa S. Cytomegalovirus. In: Mandell GL, Bennett JE, Dolin R, eds.
Principles and Practice of Infectious Diseases. 6th ed. Philadelphia, Pa: Elsevier Churchill
Livingstone; 2005: chap 134.

10.- Masculino de 77 años portador de virus C y diabético desde hace 10 años, que
presenta lesiones ampollosas de contenido serohemorrágico en extremidades superiores e
inferiores, y erosiones a nivel de la mucosa bucal. La biopsia cutánea muestra una ampolla
subepidérmica, se realiza inmunofluorescencia directa que muestra la presencia de
depósitos de IgG a nivel de la membrana basal. El diagnóstico más probable es:

a) Dermatitis herpetiforme.
b) Penfigoide ampolloso.
c) Toxicodermia ampollosa.
d) Porfiria cutánea tarda.

Penfigoide ampolloso

Casi siempre afecta a mayores de 60 años. Se trata de una enfermedad ampollosa crónica
autoinmune causada por la unión de autoanticuerpos tipo IgG y complemento C3 al antígeno
del penfigoide, una glucoproteina transmembrana de 180 (antígeno menor) o 230 kD
(antígeno mayor, desmoplaquina I) situada en los hemidesmosomas del queratinocito basal,
a nivel de la lámina lúcida. Aunque se había dicho que el penfigoide podía ser paraneoplásico,
existen estudios epidemiológicos que demuestran que el incremento de carcinomas
asociados se debe unicamente a que ambas patologías, el penfigoide y el cáncer, se dan con
mayor frecuencia al mismo grupo de edad.

Patogénesis

Se cree que la unión de los auto anticuerpos IgG al antígeno del penfigoide produce
activación del complemento, lo que da lugar a la síntesis de C3a y C5a que causan
desgranulación de los mastocitos. Éstos a su vez liberan mediadores entre los que destaca
el factor quimiotáctico de los eosinófilos. Los eosinófilos reclutados liberan enzimas
lisosomales que destruyen los hemidesmosomas y los filamentos de anclaje con lo que se
separa la unión dermoepidérmica en la lámina lúcida.

Clínica:

Se caracteriza por lesiones ampollosas grandes y tensas de contenido seroso o


serohemático, localizadas o generalizadas, con distribución preferente en flexuras y que
curan sin dejar cicatriz residual. La aparición de ampollas puede ir precedida de un cuadro
urticarial o eczematoso. Pueden ser o no pruriginosas. En cerca del 30% de los casos existe
afectación mucosa (generalmente mucosa oral).

Penfigoide ampolloso

Diagnóstico

El diagnóstico se establece por criterios clínicos, histológicos e inmunopatológicos.

En el estudio histológico de la biopsia cutánea de una ampolla se observa una hendidura


subepidérmica en el interior de la cual se aprecia un rico infiltrado inflamatorio mixto con
predominio de eosinófilos que también abundan en la dermis.

Mediante inmunofluorescencia directa se comprueba la existencia de depósitos lineales de


IgG y/o C3 en la región de la membrana basal.

La inmunofluorescencia indirecta, utilizando fluido de la ampolla u orina, pone de relieve en


cerca de la mitad de los pacientes la existencia de IgG y/o C3 circulantes.
Bibliografía

1. Worjnarowska F, Kirtschig G, Highet AS, Venning VA, Khumalo NP. Guidelines for the
management of bullous pemphigoid. Br J Dermatol 2002; 147:214-221.

2. Downham TF, Chapel TA. Bullous pemphigoid. Arch Dermatol 1978; 114:1639-1642.

3. Chosidow O, Saas V, Diquet B. Correlation between the pretreatment number of blisters


and the time to control bullous pemphigoid with Prednisone 1mg/Kg/day. Br J Dermatol
1992; 127:185-195.

4. Fiveson D, Breneman D, Rosen G et al. Nicotinamide and tetracycline therapy of bullous


pemphigoid. Arch Dermatol 1994; 130:753-758.

5. Fleming TE, Korman NJ. Cicatricial pemphigoid. J Am acad Dermatol 2000; 43:571-591.

6. Eisen D, Ellis CN, Voorhees JJ. Topical Cyclosporine for oral bullous disorders. J Am
Acad Dermatol 1990; 23:936-937.

11. - Is the most common genetic feature in ankylosing spoondylitis

a) BRCA-1.
b) HLA B27.
c) HNF 4alfa.
d) K-ras de novo mutation.

• La espondilitis anquilosante es un padecimiento reumático inflamatorio,


generalizado y crónico, que afecta primordialmente el esqueleto axial, con la
presencia de daño de las articulaciones S-I (sacroiliitis) como su hallazgo
fundamental

• Raíces griegas: ankilos (inclinado-fusión), espondilos (vértebra)


• PREVALENCIA: Africanos y esquimales: 0.1 %. Blancos: 0.5 – 1 %. Indios Haida
(Norte de Canadá): 6 %
• PREVALENCIA DEL HLA-B27: 6 – 8 %
• PREVALENCIA EN FAMILIARES DE ENFERMOS, HLA-B27 +: 10 – 20 %
• INCIDENCIA: 6.3 – 6.9/100 habitantes por año.
• CONCORDANCIA en gemelos: Monocigotos, 63 %; dicigotos, 12.5 %
• EDAD: Se inicia en la adolescencia o la edad adulta temprana. Rara después de los
40 años.
• GÉNERO: 3 – 5:1 a favor del hombre.

• FACTORES GENÉTICOS:

™ Antígenos del CPH:

- HLA-B27.
- HLA-B60.
- HLA-DRB1.
- TNF.
- Otros de clase III: MICA, TAP, LMP2, LMP7, HSP70, complotipos.

™ Antígenos fuera del CPH:

- Regiones en los cromosomas 1, 2, 6, 9, 10, 16 y 19; 16q.

• FACTORES AMBIENTALES.
• ALTERACIONES INMUNOLÓGICAS.

Brown MA, Crane A, Wordswoth BP.


Curr Opin Rheumatol 2003;14:354-60.

12.- Un hombre de 50 años acude al servicio de urgencias por presentar fiebre de 39.0c,
exantema máculo-papuloso generalizado, incluyendo palmas y plantas. El paciente labora en
el campo ordeñando vacas frecuentemente parasitadas por garrapatas. Señale la
enfermedad a la que se refiere, el germen causante y el tratamiento adecuado:

a) Kala-azar, Leishmaniae Donovani: Antimoniales.


b) Fiebre Q, Coxiella Burnetti, Doxiciclina
c) Fiebre botonosa, Ricckettsia Conori: Doxiciclina.
d) Fiebre de Malta, Brucella Mellitensis: Cotrimoxazol.

Las rickettsias son organismos coco-bacilares, Gram negativos, de 2-3 micras de diámetro,
intracelulares, incapaces de crecer en ausencia de células vivas del huésped.
El género Rickettsia es ubicado taxonómicamente en la familia Rickettsiaceae, junto a
otros dos géneros:
Coxiella, con la especie C.burnetii, responsable de la fiebre
Q, y Ehrlichia con las especies E.chaffeensis, agente de la ehrlichiasis monocítica, y
E.phagocytophila, productora de la ehrlichiasis granulocítica humana.
Junto a la familia Rickettsiaceae está la familia Bartonellaceae con tres especies
principales: Bartonella henselae, agente de la enfermedad por arañazo de gato (“cat
scratch

disease”); B.quintana, responsable de la angiomatosis bacilar(1), y B.bacilliformis,


productora de la bartonelosis o enfermedad de Carrión (verruga peruana).
Las rickettsiosis son zoonosis transmitidas desde los huéspedes o reservorios animales al
hombre a través de picaduras de artrópodos diversos, que varían con cada enfermedad
(piojos, pulgas, garrapatas, otros ácaros, esencialmente).
La rickettsiosis por R.conorii es conocida con el nombre de fiebre botonosa o manchada del
Mediterráneo o fiebre de Marsella, siendo transmitida al hombre desde el perro que
constituye su reservorio por garrapatas de los géneros Amblyomma y Riphicephalus
principalmente. En ellas el germen cumple un ciclo que incluye el pasaje transovárico a los
descendientes por lo cual representan también otro verdadero reservorio del parásito.
Clínicamente la enfermedad se caracteriza por la aparición en el sitio de la picadura de la
garrapata de una lesión inicial indurada con centro necrótico muchas veces (“mancha negra”
o “tache noir”) rodeada de aureola inflamatoria, seguida de adenopatías regionales de
carácter inflamatorio en los días subsiguientes. Concomitantemente, fiebre
frecuentemente alta de 39º-40ºC, malestar general, cefaleas a veces intensas, dolores
musculares y articulares.
Es relativamente frecuente la observación de un exantema máculo-papuloso que explica el
nombre de fiebre botonosa y que puede afectar varios territorios. Es una afección
endémica en Sudáfrica, Europa del Sur y Medio Este (2). El diagnóstico se confirma
esencialmente por la técnica de inmunofluorescencia indirecta (IFI) empleando láminas que
contienen antígenos de R.conorii y utilizando, siempre que sea posible, sueros pareados
obtenidos al inicio del cuadro y 20-30 días después para investigar la seroconversión. La
histopatología de las lesiones iniciales o “taches noires” fue estudiada en detalle por
Montenegro y colaboradores en 1983(3). En otro trabajo posterior, Montenegro y
colaboradores (4) demuestran en ratones inoculados con R.conorii la importancia crucial de
la inmunidad celular con respecto a la humoral en el control de la infección experimental y
reducción del índice de mortalidad. El ratamiento de elección es dicloxacilina, alternativas
macrólidos y quinolonas (ciprofloxacino).

Bibliografia:

1 . Sampaio SAP, Rivitti EA. Dermatologia. São Paulo: Artes Médicas, 1998: 1155.
2 . Harris RL, Kaplan SL, Bradshaw MW, Williams Jr, Temple W. Boutonneuse fever in
american travelers. J Infect Dis 1986; 153:126-8.
3 . Montenegro MR, Mansueto S, Hegarty BC, Walker DH. The histology of “taches
noires” of boutonneuse fever and demonstration of Rickettsia conorii in them by
immunofluorescence. Virchows Arch (Pathol Anat) 1983; 400:309-17.
4 . Montenegro MR, Walker DH, Hegarty BC. Infection of genetically immunodeficient
mice with Rickettsia conorii . Acta Virol 1984; 28:508-14.
5. Conti Díaz IA, Rubio I, Somma Moreira RE, Pérez Bormida G. Rickettsiiosis
cutáneo-ganglionar por Rickettsiaconorii en el Uruguay. Rev Inst Med Trop (São Paulo),
1990.
13.- En una consulta prenatal de rutina, una mujer de 28 años de edad, G5 P4, con 28 SDG,
refiere que no ha sentido movimiento fetal durante los últimos 2 días. Su embarazo ha sido
complicado debido a que padece hipertensión crónica, para lo cual se le recetaron tabletas
de alfa-metildopa 2 veces al día. Al examen, su FU es de 30cm, y las maniobras de Leopold
demuestran que el feto se encuentra en situación transversa. Su TA es 145/85mmHg. No
se encuentra latido cardiaco con el Doppler. ¿Cuál de los siguientes es el paso más
apropiado a seguir en el manejo?

a) Realizar un test sin estrés.


b) Amniocentesis.
c) USG.
d) Beta-HCG (cuantitativa)

Probable óbito:

Sintomatología y diagnóstico Signos funcionales: No se perciben movimientos fetales por


12-24 horas. Disminución o ausencia de síntomas y/o signos como nauseas vómito,
hipertensión, albuminuria) Paraclínicos: *ecografía: diagnóstico precoz y exacto: Doppler.
*Radiología: hay 3 signos: +deformación del cráneo +curvatura y torsión de la columna
+presencia de gas en el feto *líquido amniótico: puede estar meconiado, o sanguinolento
Signos locales: en los senos hay secreción calostral, sangrado leve y oscuro por vagina, el
feto se vuelve blando a la palpación, fetocardia (-), puede haber detención y/o disminución
de la altura uterina, bajo peso corporal, entre otros... Reducción de la perfusión útero
placentaria Luisa Fernanda Anaya Admadé Dr. Orlando Peinado, ginecólogo Peso bajo: 15.7%,
RCIU: 11.4%, Anencefalia: 3.9%, Genopatías: 2.7%, circular de cordón: 2%, Fetopatía
diabética: 1.6%, Hipoplasia pulmonar: 1.2%, hemólisis: 1.2%, mielomeningocele:
2%Enfermedades maternas: Enf. Hipertensiva del embarazo: 20.7%, cardiopatías: 16.7%,
RPM 13%, Infección urinaria: 12%, diabetes: 11.3%, desprendimiento de placenta NI: 6%,
LES:

Bibliografía: 1. OBSTETRICIA, Schwarcz R, editorial El ateneo, 2003. 2. Sociedad


española de ginecología y obstetricia, junio 2002. 3. OBSTETRICIA CLÍNICA, Llaca V,
edición 2000, capítulo 24; Pág, 315-316.
14.- Una mujer de 21 años, nuligrávida, acude a consulta para hablar sobre anticoncepción.
Es sexualmente activa desde hace 2 semanas y actualmente utiliza condón. Tiene
antecedente de asma, la cual se ha mantenido inactiva por 2 años. No toma medicamentos y
niega alergias. No hay AHF de cáncer. Su EF es normal. Después de una plática con su
médico, escoge tomar anticonceptivos orales combinados, y continúa tomándola por 6 años.
Ahora ha disminuido su riesgo de desarrollar:

a) Cáncer de mama.
b) Cáncer cervical.
c) Cáncer hepático.
d) Cáncer ovárico.

PROTECCION CONTRA EL CANCER EPITELIAL OVARICO.

Debido a la falta de estrategias eficaces para el diagnostico y tratamiento temprano del


cáncer de ovario, es de capital importancia la prevención (76). Esta patología es una
importante causa de morbilidad y mortalidad. Se estimó que para 1980 ocurrieron 137.600
casos nuevos en el mundo (86). Estudios a gran escala realizados por el Centro para el
Control de las Enfermedades de EE.UU. y el Royal Collage of General Practitioners del
Reino Unido (RCGP) indican que la supresión de la ovulación causada por los anticonceptivos
orales protege contra el desarrollo del cáncer epitelial ovárico (20,76). Este efecto
benéfico es directamente proporcional al tiempo de uso y persiste muchos años después de
suspendida la planificación con este método (87). La evaluación norteamericana denominada:
Estudio sobre cáncer y hormonas (CASH) demostró que el uso de uno a cinco años de
anovulatorios orales disminuye en un 50 a 70% el riesgo de cáncer ovárico (88). Este efecto
protector aumenta entre más sea el tiempo de uso y se extiende por lo menos hasta diez
años después de interrumpido (70,89). La Organización Mundial de la Salud también realizó
un estudio multicéntrico confirmando el efecto protector de los anticonceptivos orales
contra el cáncer epitelial ovárico (90). Dos estudios de Cohortes realizados en Gran
Bretaña, confirmaron el efecto protector de la píldora al encontrar riesgos relativos de 0.3
y o.6 en mujeres que habían usado el método en algún momento (83,91). El efecto protector
es tanto para tumores malignos como para Bordenline (92) y cada uno de los principales
subtipos histológicos de cáncer epitelial (70,93).

TABLA Nº 3

BENEFICIOS NO CONTRACEPTIVOS DE LOS ANTICONCEPTIVOS ORALES


COMBINADOS.
MEJORIA DE LA DISMENORREA.
CORRECCION DE LOS CICLOS MENSTRUALES IRREGULARES.
PREVENCION DE QUISTES OVARICOS FUNCIONALES.
PROTECCION CONTRA EL CANCER EPITELIAL OVARICO.
MEJORIA DEL MITTELSCHMERZ.
PROTECCION CONTRA EL CANCER ENDOMETRIAL.
PROTECCION CONTRA TUMORES BENIGNOS MAMARIOS.
DISMINUCION DE LA ENFERMEDAD PELVICA INFLAMATORIA.
DISMINUCION EN LA INCIDENCIA DE EMBARAZO ECTOPICO.
PREVENCION DE ANEMIA FERROPENICA.
MENOR INCIDENCIA DE ARTRITIS REUMATOIDEA.
MENOR INCIDENCIA DE OSTEOPOROSIS POST-MENOPAUSICA.
MEJORIA DEL SINDROME PREMENSTRUAL.
PREVENCION DE LA MIOMATOSIS UTERINA.
MEJORIA DEL ACNE.

1. Bagshaw S. the combined oral contraceptives. Risk and adverse effects in


perspective. Drug-Saf 1995; 12 (2): 91 - 96.
2. American Collage of Obstetricians and Gynecologist. Hormonal contraception.
ACOG technical bulletin Nº. 198 - October de 1994 Int J Gynaecol Obstet 1995; 48 (1):
115 - 126.
3. Mishell Jr DR. Oral contraception: past, present and future perspectives. Int J
Fertil 1992; 37 (1) Suppl: 7 - 18.
4. Melo NR, Pinotti J.Advances in hormonal contraception. Adv. in contraception 1994;
10 (suppl 1): 33 - 39.
5. Winkler UH, Schindler AE, Endrikat J, et al. A comparative study of the effects
of the hemostatic system of two monophasic Gestodene oral contraceptive containing 20
ug and 30 ug Etinil-Estradiol. Contraception 1996; 53: 75 - 84.
6. Coenen CMH, Thomas CMG, Borm GF, et al. Changes in androgens during treatment
with four low-dose contraceptives. Contraception 1996; 53: 171 - 176
7. Wilde MI, Balfour JA. Gestodeno. A review of its pharmacology, efficacy and
tolerability in combined contraceptive preparation. Drug 1995; 50 (2): 364 - 395.
8. Monterrosa A. Anticoncepción hormonal. EN : Caraballo J, Parra E, Taylor H.
Memorias del 1º Curso de actualización en Ginecología y Pediatría. Imprenta U. de
Cartagena. Cartagena.1994; 241 - 250.
9. Hannaford PC, Combined oral contraceptives : do we know all of their effects.
Contraception 1995; 51: 325-327.

Rosenberg MJ, Waugh MS, Meehan T. Use and misuse of oral contraceptives : risk
indicators for poor pill taking and discontinuation. Contraception 1995; 51: 283- 288.
15.- Una mujer de 32 años de edad es llevada a quirófano por una laparoscopía diagnóstica
debido a dolor pélvico y en cuadrante superior izquierdo crónicos (durante los últimos 2
años). No tiene alteraciones funcionales vesicales o intestinales. Tiene antecedente de 2
episodios de gonorrea previos. Bebe una cerveza al día. Labs: HCG urinaria negativa; Hto
39%; Leuc. T 8 000; Plt 200 000; AST 12; ALT 14. Intraoperatoriamente se observan
adhesiones densas que involucran los oviductos, ovarios y útero. También se observan
adhesiones perihepáticas que se extienden desde la superficie hepática hacia el diafragma.
¿Cual de los siguientes es el diagnóstico más probable?

a) Sx de Fitz-Hugh-Curtis.
b) Hepatitis.
c) Carcinoma hepatocelular.
d) Sx Wolff-Parkinson-White.

El síndrome de Fitz-Hugh-Curtis se define como la presencia de una perihepatitis asociada


a salpingitis. Los agentes etiológicos reconocidos hasta la fecha son Chlamydia trachomatis
y Neisseria gonorrhoeae. El cuadro clínico de este síndrome es inespecífico y puede ser
confundido con procesos inflamatorios o infecciosos del tubo digestivo, aparato urinario y
respiratorio, en los cuales la manifestación sintomática fundamental es el dolor en
hipocondrio derecho. El diagnóstico debe de sospecharse en aquella mujer joven con vida
sexual activa que tenga antecedentes de promiscuidad en ella o en su pareja, que se queje
de dolor subcostal derecho. Es más probable el diagnóstico si se cuenta con el antecedente
de enfermedad pélvica inflamatoria y más aún, si se tiene evidencia de que ésta sea
causada por Neisseria gonorrehoeae y/o Chlamydia trachomatis. El diagnóstico definitivo
se realiza con la visualización directa de la adherencia perihepática por laparoscopía o
laparotomía. Se recomienda la primera. El tratamiento médico es a base de cefalosporinas y
dicloxacilina y en algunos casos se requiere de la extirpación quirúrgica del proceso
adherencial para mitigar el dolor.

El Síndrome de Fitz-Hugh-Curtis. Causa frecuente de error de diagnóstico en hepatología y


gastroenterología / The Fitz-Hugh-Curtis Syndrome. a frequent misdiagnosis in hepatology
and gastroenterology
Rev. gastroenterol. Méx;60(4):223-8, oct.-dic. 1995.

16.- Una mujer de 67 años refiere intenso prurito vulvar y sensación quemante, al examen
el introito vaginal se encuentra estenótico. ¿Cuál de los siguientes es el tratamiento
apropiado?

a) 5-fluoracilo.
b) Testosterona tópica.
c) Corticoesteroides fluorados.
d) Estrógeno tópico.
Vulvovaginitis atrófica

El hipoestrogenismo conduce a atrofia de la vagina y el vestíbulo vulvar, que los hace


fácilmente irritables y susceptibles a infecciones secundarias. Las pacientes refieren
sensación de quemadura, prurito, disuria, hipersensibilidad y dispareunia. Puede
encontrarse al examen físico atrofia, fisuras superficiales, y un flujo vaginal acuoso1. Hay
disminución del tamaño del introito2, pérdida de la rugosidad y la vagina toma una apariencia
lisa y brillante.

Los hallazgos histológicos revelan un epitelio vaginal delgado, disminución de los lechos
capilares, y la citología muestra, a medida que la atrofia progresa, aumento de las células
basales y disminución o ausencia de las células superficiales2.

Se aconseja evitar el uso de jabones y demás irritantes de la piel. Se pueden utilizar


lubricantes simultáneamente con los estrógenos o como terapia única, si hay alguna
contraindicación a las hormonas.

El tratamiento con estrógenos por vía sistémica o transvaginal mejora y restaura los signos
y síntomas, y una a dos semanas después de iniciar el tratamiento los cambios de atrofia
empiezan a mejorar rápidamente, se reduce el pH y se induce maduración vaginal y de la
mucosa uretral, reduciendo la frecuencia de las infecciones urinarias3. La dosis y vía de
administración debe ser debidamente individualizada4. Contraindicaciones al tratamiento
con estrógenos, incluyen: la presencia de tumores estrógenosensibles, falla hepática
terminal y antecedentes de tromboembolización relacionada con ellos.

Menopausia y Piel.
ParteII:Manifestaciones- clínicas
dermatológicas durante la menopausia.

MARÍA ISABEL BARONA C. Docente adjunto. Dermatóloga Universidad del Valle-Cali.

17.- .- El dolor localizado, las parestesias, la sensación de cuerpo extraño faríngeo, el


prurito nasal, la sialorrea, el nistagmus, las fasciculaciones linguales, se presentan en una
intoxicación por:

a) Cocaína.
b) Víbora de cascabel.
c) Araña Loxosceles reclusa.
d) Alacrán.

El veneno del alacrán actúa sobre los canales de sodio, potasio, calcio.
Existe liberación de acetilcolina y catecolaminas, con efectos neuro y cardiotóxicos. Es
producido en el telson e inyectado a través del aguijón del alacrán.
Grado I O Leve
¾ Dolor local intenso
¾ Parestesias locales y a distancia
¾ Inquietud
¾ Prurito nasal y faríngeo
¾ Sialorrea

Grado II o Moderado
¾ Diaforesis
¾ Sensación de cuerpo extraño en faringe
¾ Nistagmus
¾ Dislalia
¾ Distensión abdominal
¾ Fasciculaciones linguales

Grado III o Grave


¾ Taquicardia
¾ Hipertensión arterial
¾ Visión de halos rojos
¾ Ceguera transitoria
¾ Disnea
¾ Dolor retroesternal
¾ Espasmos musculares
¾ Edema agudo pulmonar
¾ Priapismo
¾ Malestar vaginal

1.- Montoya-Cabrera MA. Intoxicaciones y envenenamientos en niños. México,


Intersistemas , 2000.
2.-Montoya CMA. Toxicología clínica. 2ª. Ed, México, Méndez Editores, 1997.

18.- Paciente de 4 años de edad tiene lesiones eczematosas crónicas en flexuras de brazos
y piernas que producen intenso picor, asociadas a una queilitis descamativa de labios. ¿Cuál,
entre los siguientes, es el diagnóstico más probable?

a) Una dermatitis atópica.


b) Un eczema seborréico.
c) Un prúrigo nodular.
d) Un eczema microbiano
La dermatitis atópica llamada comúnmente eccema (atópico), es una enfermedad que
consiste en un estado reaccional de la piel caracterizado por erupciones pruriginosas y con
aspecto de escamas, más frecuente en niños, multifactorial, en la cual intervienen factores
tanto ambientales como constitucionales. Las personas con eccema a menudo tienen
antecedentes de condiciones alérgicas como asma, fiebre del heno o eccema. La dermatitis
atópica fue originalmente conocida como prúrigo de Besnier y eccema constitucional,
actualmente también es llamada neurodermatitis diseminada, por las escuelas europeas.

Clínica
Las manifestaciones clínicas típicas de la dermatitis atópica se dividen en tres etapas, que
suelen denominarse del lactante, infantil y del adulto. Junto a ellas se encuentran otras,
con frecuencia llamadas atípicas, a pesar de que muchas, como la xerosis, son muy
constantes.

Dermatitis atópica del lactante:


Suele empezar hacia los cinco meses de vida, pero puede hacerlo antes. Algunos niños
desarrollan lesiones de eccema seborreico, que de forma gradual va adquiriendo el aspecto
de la dermatitis o eccema atópico.

La localización más habitual es en la cara, respetando las zonas alrededor de los ojos, la
nariz y la boca (. Son también frecuentes en el cuero cabelludo, las orejas, el dorso de las
manos y las zonas de extensión de las extremidades.
Las lesiones suelen ser pápulas o placas eritematosas y edematosas, muchas veces con
erosiones, exudación y costras.
Es muy raro que se aprecien las vesículas características del eccema. El prurito es un
síntoma constante.

Dermatitis atópica infantil:


Este periodo suele considerarse con un inicio hacia los dos años y un final entre los siete
años y la pubertad. Las lesiones características se observan sobre todo en las flexuras, en
especial en los codos y las rodillas (Fig. ), pero pueden aparecer en otras zonas.
En esta fase es más fácil ver lesiones eccematosas con vesículas, pero el intenso prurito
hace que enseguida se transformen en erosiones, con exudación y formación de costras.

Bibliografía
1. Bielsa Marsol I. Eccemas (II). En: Ferrándiz C, ed. Dermatología Clínica. Madrid, Mosby/
Doyma Libros 1996, 113-124.
2. Fernández Vozmediano JM y cols. Dermatitis atópica. Madrid, Jarpyo 1994.
3. Fernández Vozmediano JM, Armario Hita JC. Tacrolimus. Piel 2001;16:48-54.
4. Fonseca Capdevila E. Dermatitis atópica. Protocolo terapéutico. (En línea) (14.02.2001).
Disponible en www.especialistasdermatologia.com.
5. Fonseca E. Dermatitis atópica en la infancia. Salud Rural 1997;14:92-105.
6. Guerra Tapia A. Dermatitis atópica. En: Fonseca Capdevila E, ed. Dermatología
Pediátrica.
Madrid, Aula Médica 1999, 83-180.
19.- Femenino de 62 años de edad el cual, experimenta episodios frecuentes de ptosis,
diplopia y fatiga generalizada. A la EF se encuentra timo palpable y parálisis del nervio
oculomotor que se corrige transitoriamente al administrar edrofonio.
¿Cuál es el diagnóstico más probable en su caso?

a) Miastenia gravis.
b) Parálisis de Bell.
c) Síndrome de Horner.
d) Lupus.

Aunque la miastenia gravis puede afectar cualquier músculo voluntario, los músculos que
controlan el movimiento de los ojos y los párpados, la expresión facial y el deglutir se ven
afectados con mayor frecuencia. El inicio del trastorno puede ser repentino. A menudo, los
síntomas no se reconocen inmediatamente como miastenia gravis.

En la mayoría de los casos, el primer síntoma perceptible es la debilidad en los músculos


oculares (de los ojos). En otros, la dificultad para tragar e impedimentos en el habla pueden
ser los primeros síntomas. El grado de la debilidad muscular de la miastenia gravis varía
sustancialmente entre los pacientes, pudiendo manifestarse desde una forma localizada,
limitada a los músculos oculares (miastenia ocular), hasta una forma grave o generalizada en
la cual se afectan muchos músculos-incluyendo a veces los músculos que controlan la
respiración.

Los síntomas, que varían en tipo y gravedad, pueden incluir la caída de uno o ambos
párpados (ptosis), visión nublada o doble (diplopia) a consecuencia de la debilidad de los
músculos que controlan los movimientos oculares, marcha inestable o irregular, debilidad en
los brazos, las manos, los dedos, las piernas y el cuello y un cambio en la expresión facial,
dificultad para deglutir y respirar y trastornos en el habla (disartria).

Una de las pruebas paa realizar el diagnóstico llamada la prueba del edrofonio. Este
esquema requiere la administración intravenosa de cloruro de edrofonio o Tensilon(r), un
medicamento que bloquea la degradación (interrupción) de la acetilcolina y aumenta
temporalmente los niveles de acetilcolina en las uniones neuromusculares. En individuos que
padecen de miastenia gravis en los músculos oculares, el cloruro de edrofonio tiende a
aliviar transitoriamente la debilidad. Otros métodos para confirmar el diagnóstico incluyen
una versión del estudio de la conducción nerviosa que examina el nivel de fatiga de un
músculo en específico mediante una estimulación repetida de los nervios. Esta prueba
registra respuestas de debilitamiento muscular cuando los nervios se estimulan en forma
repetida y ayuda a distinguir entre trastornos nerviosos y los trastornos musculares. La
estimulación repetida de los nervios durante un estudio de conducción nerviosa puede
demostrar disminuciones en el potencial de acción muscular debidas a un deterioro en la
transmisión del nervio al músculo.
GUIA PRÁCTICA PARA LA MIASTENIA GRAVE.

Por: John E. Keesey, M.D. y Rena Sonshine.

Traducción al Español: Oné R. Pagán-Ojeda.

20. Un hombre de 55 años acude a consulta por una historia de 2 meses de dificultad para
tragar. En un principio, la dificultad era únicamente con bocados grandes de alimentos
sólidos, pero ahora tiene problema hasta con los líquidos. Tiene una historia de uso de
alcohol y tabaco. La esofagoscopía demuestra una masa polipoide grande e irregular que
ocluye casi completamente el tercio superior del esófago. ¿Cual de los siguientes es el tipo
histológico de este tumor?

a) Linfoma de células gigantes.


b) Carcinoma de células pequeñas.
c) Linfoma de células pequeñas.
d) Carcinoma de células escamosas.

Hay varios subtipos, principalmente adenocarcinoma (aproximadamente 50-80% de todos


los cánceres del esófago) y el cáncer de células escamosas. El cáncer de células escamosas
surge de las células que recubren la parte superior del esófago. El adenocarcinoma se
deriva de las células glandulares que están presentes en la unión del esófago y el estómago.
Los tumores de esófago por lo general llevan a la disfagia (dificultad para tragar), dolor.

Clasificación:
Los cánceres de esófago son generalmente carcinomas que surgen a partir del epitelio o
revestimiento de superficie, del esófago. La mayoría de los cánceres del esófago caen en
una de dos clases: Los carcinomas de células escamosas, que son similares a cáncer de
cabeza y cuello en su apariencia y su asociación con el tabaco y el consumo de alcohol, y los
adenocarcinomas, que se asocian a menudo con antecedentes de enfermedad por reflujo
gastroesofágico y esófago de Barrett. Una regla del pulgar general es que un cáncer en la
parte superior de dos tercios es un carcinoma de células escamosas y uno en el tercio
inferior es un adenocarcinoma.
REVISTA ESPAÑOLA DE PATOLOGÍA
Vol. 37, n. º 4, 2004
Protocolo e información sistematizada para los estudios histopatológicos relacionados con
el carcinoma esofágico
Francisco Colina, Guadalupe López Alonso, Carolina Ibarrola.

21.- En un paciente masculino de 35 años que se presenta con meningitis bacteriana, el


tratamiento antibiótico empírico de elección que deberá ser indicado es:

a) Imipenem.
b) Penicilina.
c) Ciprofloxacino.
d) Cefotaxima.

La meningitis puede definirse como una inflamación de las leptomeninges (piamadre y


aracnoides) con afectación del LCR que ocupa el espacio subaracnoideo; puesto que tanto
las leptomeninges como el LCR se extienden por el cerebro y canal medular, el término
implica siempre una afectación cerebroespinal. La meningitis bacteriana es quizás una de
las más claras emergencias de todas las enfermedades infecciosas puesto que el
tratamiento tardío o inadecuado incrementa el riesgo de muerte o de morbilidad
neurológica en aquellos que sobreviven. La meningitis bacteriana es más frecuente en las
edades extremas y entre los inmunodeprimidos, pero puede ocurrir en cualquier grupo de
edad.

ETIOLOGÍA

Tres organismos,Haemophilus influenzae, Neisseria meningitidis (meningococo) y


Streptococcus pneumoniae (neumococo), son los responsables del 70-85% de los casos de
meningitis bacterianas . Cada organismo predomina en una población específica, que puede
ser establecida en función de la edad y las condiciones de base del huesped.

Examen físico

Fiebre, cefalea, signos de irritación menígea, y alteración del nivel de conciencia se


presentan en más del 85% de los adultos con meningitis. En una amplia serie, el rango del
estado de conciencia observado a la admisión fué la siguiente: alerta (17.8%), irritable o
letárgico (52.1%), estuporoso/obnubilado (20.3%), o comatoso (9.7%).
Pueden presentarse signos de incremento agudo de la presión intracraneal (PIC) que incluye
pérdida de conciencia, pupilas dilatadas o con reactividad perezosa, oftalmopléjia,
afectación de la función respiratoria, inestabilidad cardiovascular, posturas motoras
anómalas, hiperreflexia y espasticidad.

Aproximadamente el 50% de los adultos con meningitis desarrollan complicaciones


neurológicas. A la infección del SNC se asocia con frecuencia neumonia (25-50%) y otitis
media (33%). Las complicaciones sistémicas incluyen shock septico (11.6%), SDRA (3.5%) y
CID (8.15%)

Pruebas de laboratorio

La valoración del LCR es esencial para el diagnóstico de meningitis, permitiendo establecer


el diagnóstico de infección del SNC y diferenciar las infecciones bacterianas de las no
bacterianas. El riesgo de herniación cerebral debe ser siempre considerado antes de la
realización de la punción lumbar (PL), mediante el exámen neurológico e incluso estudios de
neuroimagen. Al realizar la punción, si la presión de apertura estuviese muy elevada (>40 cm
de H2O), el LCR se extraerá lentamente y se reducirá al mínimo el volumen obtenido. Deben
obtenerse al menos dos muestras de 2-4 ml en tubos estériles: uno para recuento-fórmula
celular y bioquímica, y otro para pruebas microbiológicas.

Tratamiento antimicrobiano

El conocimiento de la escasa actividad opsonica del liquido cerebroespinal, y considerando


que el LCR es un área con resistencia disminuida a la infección, los antibióticos utilizados
deben tener actividad bactericida en el LCR. En la tabla 5, se muestran las
recomendaciones para el tratamiento antibiótico en pacientes con meningitis bacteriana
que tienen tinción de Gram o cultivo de LCR positivo, y en las tablas 6 y 7, las
recomendaciones para el tratamiento empírico en pacientes con sospecha de meningitis
bacteriana con tinción de Gram o cultivo de LCR no diagnóstico (o la realización de la
punción lumbar ha sido postpuesta), pudiendo ser dirigida la antibioterapia contra el
patógeno probable en función de la edad y las condiciones de base del paciente.
Tabla 5
Antibioterapia en meningitis bacterianas
Antibioterapia basada en el organismo sospechado o aislado
Organismo Antibiótico
Cefotaxima, ampicilina, penicilina G,
S. pneumoniae
vancomicina
N. meningitidis
Penicilina G, ampicilina, cefotaxima,
H. influenzae
cloranfenicol
S. aureus (MS)
Cefotaxima, ceftriaxona, ampicilina
S. aureus (MR)
Oxacilina
L.
Vancomicina + Rifampicina
monocytogenes
Ampicilina
Streptococos
Penicilina G, ampicilina
Bacilos Gram-neg
Ceftriaxona, cefotaxima, TMP-SMZ
Enterobacterias
Ceftriaxona, cefotaxima
P. aeruginosa
Ceftazidima
S. epidermidis
Vancomicina + Rifampicina
Anaerobios
Cefotaxima + metronidazol + rifampicina

Tabla 6
Antibioterapia en meningitis bacterianas
Tratamiento antibiótico empírico*
Niños

Cefotaxima (75-100 mg/kg/6 h) o


Cefotaxima (50 mg/kg/6 h) + vancomicina (15/mg/kg/6 h) o
Ceftriaxona (100 mg/kg/24 h) + vancomicina (15 mg/kg/6 h)
Niños alérgicos a la penicilina

Vancomicina (15 mg/kg/6 h) + aztreonam (120 mg/kg/6 h) o


Vancomicina (15 mg/kg/6 h) + cloranfenicol (100 mg/kg/6 h) en los
niños vacunados contra H. influenzae
Adultos

Cefotaxima** (75 mg(kg/6 h; máximo, 24g/día) + ampicilina (60


mg/kg/6 h), si se desea cubrir L. monocytogenes
Adultos alérgicos a la penicilina

Vancomicina(15 mg/kg/12 h) + aztreonam(120 mg/kg/6 h; máximo,8


g/dia)
*Tinción de Gram y pruebas de detección de antígeno en LCR
negativas.
**Si la etiología neumocócica se considera muy improbable puede
administrarse ceftriaxona (100 mg/kg/6 h; máximo, 8 g/día)

Tabla 7
Antibioterapia en meningitis bacterianas
Antibioterapia empírica según edad y factores predisponentes:

Edad y factor
Tratamiento Gérmenes a cubrir
predisponente
Enterobacterias,
Ampicilina +
Neonatos (<1 mes) Estreptococo B
Cefotaximaa
Listeria
Ampicilina +
1-3 meses Meningococo, H. influenzae
Cefotaximaa
Meningococo, H. influenzae,
3 meses-5 años Cefotaximaa
Neumococo
Meningococo, Neumococo,
> 5 años y adultos Cefotaximaa
Estreptococo A
Neumococo, Meningococo,
Ancianos Cefotaximaa,b,c Bacilos gram-negativos,
Listeria
Fractura de cráneo
Neumococo
cerrada o fístula Cefotaximaa,d
Estreptococo A
de LCR
Fractura de cráneo Vancomicina + Enterobacterias
abierta o craneotomía. Ceftazidima + Pseudomonas S.aureus y
Neutropenia Amikacina epidermidis
Inmunodepresión sin Igual que en adultos +
Cefotaxima +
neutropenia. enterobacterias, H.
Ampicilina
Alcoholismo. influenzae y Listeria
a
Puede usarse tambien ceftriaxona
b
Muchas autoridades recomiendan antibioterapia de amplio espectro
c
Si se quiere cubrir Listeria, debe añadirse amplicilina
d
En pacientes graves, cubrir también estafilococos
22.-Is the easiest method to dermine if a patient is in prerenal azotemia:

a) Plasma creatinine 24 hours cuantification.


b) GFR cuantification.
c) Granular casts in urinary sediment.
d) BUN: Cr ratio.

La IRA prerrenal, también llamada azoemia prerrenal, es la causa más frecuente de IRA,
representando en realidad una respuesta fisiológica a la hipoperfusión renal. Por definición
el tejido renal se mantiene íntegro, como lo prueba el que estos riñones trasplantados a
otras personas funcionan adecuadamente en el receptor y el que la función renal se
normaliza rápidamente si se corrigen las causas que originaron la hipoperfusión renal. De
mantenerse estas, la isquemia continuada puede terminar lesionando el parénquima renal,
conduciendo a la situación de NTA isquémica. Por lo tanto la IRA prerrenal y la NTA
isquémica son parte de un mismo espectro de hipoperfusión renal, que en casos extremos
puede llegar a la necrosis cortical.

La relación BUN: Crea en la azotemia prerenal es superior a 20. La razón para esto yace en
los mecanismos de filtración de ambos compuestos. Los niveles de filtración glomerular
disminuyen por la hipoperfusión, conduciendo a un mayor incremento en el BUN que en la
creatinina. Como el riñón está funcionando correctamente, la respuesta al disminuir la
filtración glomerular es el incremento en la reabsorción. El aumento en la reabsorción de
sodio produce también un aumento en la reabsorción de agua y urea a nivel del túbulo
proximal. En contraste, la creatinina es secretada en el mismo nivel de los túbulos, teniendo
una fracción excretada mayor. Este mecanismo de autoregulación renal, lleva a que BUN: Cr
sea mayor a 20 además de una fracción de sodio excretado menor a 1%, junto a una elevada
osmolaridad urinaria (debido principalmente a la reabsorción de agua).

Kumar, Vinay; Fausto, Nelson; Fausto, Nelso; Robbins, Stanley L.; Abbas, Abul K.; Cotran,
Ramzi S. (2005). Robbins and Cotran Pathologic Basis of Disease, 7th edición, Philadelphia,
Pa.: Elsevier Saunders, pp. 960,1012.

23.- Masculino de 42 años como antecedentes refiere malos hábitos alimenticios comenta
que con mucha frecuencia consume alimentos en la vía pública, actualmente se presenta con
fiebre, edema facial, fotofobia y mioartralgias estos datos son sugestivos de:

a) Larva migrans visceral.


b) Fasciolosis.
c) Criptosporidiosis.
d) Trichinellosis.
DEFINICIÓN

Se denomina triquinosis a la Infección parasitaria producida por nemátodos del género


Trichinella, transmitida por carnivorismo, y caracterizada por un síndrome febril, signos
oculopalpebrales, mialgias y eosinofilia elevada. Sinonimia: Trichinellosis.

AGENTE Y CLASIFICACIÓN

La Trichinella es un pequeño nemátodo blanquecino y filiforme, con su extremidad anterior


más adelgazado que la posterior. La hembra mide entre tres y cuatro milímetros, en tanto
que el macho es de menor tamaño.

Originalmente, se reconocía como única especie a la Trichinella spiralis (Owens, 1835),


pero en diversas áreas geográficas se han descrito recientemente triquinas que, aunque
morfológicamente similares, presentan sutiles diferencias en sus características biológicas.
Así, en la actualidad se distinguen:

1. Trichinella spiralis, propia de las zonas geográficas templadas.

2. Trichinella pseudospiralis, la cual, aunque no es frecuentemente observada afecta


más a las aves que a los mamíferos y se caracteriza por ser de menor tamaño y por no
formar quistes en la musculatura del hospedador.

3. Trichinella nelsoni del Africa tropical, la cual se encuentra en los grandes carnívoros
de la región, presenta un bajo grado de infectividad para los cerdos domésticos y ratas
de laboratorio, y en el hombre provoca intensas infecciones con un gran número de
larvas por gramo de músculo; aunque ha sido fatal en ocasiones, es muy bien tolerada a
pesar de la masividad de la infección.

4. Trichinella nativa de las zonas árticas que, distintivamente, presenta una considerable
resistencia a la congelación, tiene bajo grado de infectividad para el cerdo doméstico, y
en el hombre provoca importantes síntomas digestivos, principalmente diarreas
prolongadas.

Si bien T. spiralis, T. nelsoni y T. nativa son morfológicamente similares, se ha


establecido la diferenciación entre ellas por sus características isoenzimáticas y
mediante el uso de anticuerpos monoclonales. En cambio, existen diferencias
estructurales entre esas tres "especies" y la T. pseudospiralis.

Ciclo doméstico: El cerdo adquiere la infección, principalmente, por la ingestión de ratas


infectadas, lo que es posible cuando es criado en malas condiciones higiénicas o,
simplemente, cuando debe buscar su propia fuente de alimentación en sitios eriazos o
basurales; además, el cerdo se infecta con carnes de otros animales que encuentra en los
criaderos o en los basurales. Las ratas, debido principalmente a sus hábitos de canibalismo,
mantienen y propagan la infección en la naturaleza.
Los principales huéspedes domésticos de la T. spiralis son la rata, el cerdo y el hombre. El
hombre adquiere la infección a través de la ingestión de carne de cerdo cruda o
insuficientemente cocida, con larvas de triquina. Los jugos digestivos digieren la carne y
las larvas quedan en libertad en el intestino, donde rápidamente, ya a las cuarenta y ocho
horas, se diferencian en hembras y en machos adultos. Copulan en el lumen intestinal y,
mientras los machos son eliminados con las deposiciones del huésped luego de cumplida su
función genésica, las hembras grávidas - que son vivíparas- se localizan en el interior de la
mucosa del duodeno y del yeyuno. Entre el tercero y el quinto día, comienza la postura de
larvas. Cada hembra coloca alrededor de 1.500. Estas larvas miden entre 80 y 120
micrones, se profundizan en la mucosa intestinal, penetran a través de los capilares
linfáticos y venosos y llegan a la circulación general, diseminándose por todo el organismo,
pero enquistándose sólo en la musculatura, esquelética. Las larvas se localizan en el
interior de las fibras musculares, destruyéndolas parcialmente; al cabo de unos quince días,
quedan rodeadas por una envoltura constituida por el sarcolema. Así se origina el quiste
larval, que mide entre 250 a 400 micrones y que, en consecuencia, no es visible a simple
vista, aunque puede observarse con una lupa o con un microscopio de poco aumento. Tiene
un aspecto fusiforme o alargado, que recuerda la forma de un limón, y contiene, enrollado
en su interior, una o varias larvas de triquina.

La invasión de la musculatura esquelética por las larvas, comienza alrededor del séptimo día
de ocurrida la infección y continúa mientras existan hembras grávidas en el intestino. Al
cabo de un mes, las larvas completan su encapsulamiento y a los seis meses, se inicia el
depósito de calcio en las paredes del quiste. La calcificación total se alcanza en un plazo
aproximado de un año.

En consecuencia, un mismo individuo es, sucesivamente, huésped definitivo e intermediario


del parásito. Es hospedero definitivo cuando alberga en su intestino las formas adultas, y
es intermediario cuando las larvas se localizan en su musculatura. Sin embargo, para
completar todo su desarrollo, la Trichinella requiere siempre de dos huéspedes

1. Leo X. Liu Peter F. Weller. Helminthic Infections: Trichinella and other tissue
Nematodes. In Baunwald E, Faucy AS, Kasper DL, Hauser SL, Longo DL, Jameson JL,
editors. Harrison´s Principles of Internal Medicine. 15 Th ed. New York . McGraww-Hill;
2001; 1231-3.

2. Pozio E. New patterns of Trichinella infections. Vet Parasitol 2001; 98: 133-48.

3. Murrel KD. Trichinellosis: now and forevermore?. Parasite 2001; 8 (2 supl.): 11-3.

4. Sánchez Rodríguez A, Martínez López de Letona J, Arias paciencia M, Sánchez García


AM, Paz Bouza J, Jarrin J, et al. Triquinosis. Estudio de 21 casos en un mismo brote. Rev
Clin Esp 1982; 165: 79-84.
5. Perucha González M, Lezaun Larrumbe ME, Torres Baile JL, Campo Hernández JM,
Bernal
Martínez A. Brote de Triquinosis en varias localidades de la Rioja Baja. Rev Sanid Hig
Pública (Madrid) 1987; 61: 1035-47.

6. De la Torre Cecilia C, Espino Aguilar R, Cárdenas Talaverón C, Canuelo Ruiz O, Garrido


Palomo R, Baena Sáez J, et al. Triquinosis: presentación de 2 casos. An Esp Pediatr 1989;
30: 227-8.

24.-Una mujer de 27 años llega al servicio de urgencias con disnea y dolor torácico
pleurítico. También refiere que en los cuatro días previos tuvo tumefacción e
hipersensibilidad en la pantorrilla y el muslo derechos. Con base en la presentación clínica
se sospecha una trombosis venosa profunda que podría haber causado embolia pulmonar.
¿Cuál de los siguientes fragmentos de información de los antecedentes de la enferma
apoyan mejor este diagnóstico?

a) Antecedente de tabaquismo.
b) Antecedente de diabetes mellitus en la familia de la paciente.
c) Antecedente de lesión en la extremidad inferior.
d) Antecedente de hipertensión.

Allen R. M. MMS Medicina Interna. 5ª. Edición. National Medical Series. Mc. Graw Hill.
2006. (capítulo 1 VIIIA 2 d, 3 a) La lesión de las extremidades inferiores puede producir
formación de coágulos sanguíneos y desarrollo de tromboflebitis. Otros factores que
contribuyen al desarrollo de trombosis venosa profunda incluyen uso de compuestos con
estrógenos (p. ej., anticonceptivos orales) o inmovilización de la extremidad inferior (p. ej.,
durante cirugía o reposo en cama prolongado) que da lugar a estasis venosa. La
hipertensión, la diabetes mellitus y el abuso de drogas intravenosas no tienen relación con
la trombosis venosa profunda. La disnea y el dolor pleurítico sugieren que la trombosis
venosa profunda ha causado embolia pulmonar, que es provocada por desplazamiento de un
trombo de las venas de las extremidades inferiores o la pelvis a la arteria pulmonar.
25.- Una mujer de 24 años refiere que sus manos se tornan blancas y luego azules en el frío.
¿Cuál de los siguientes datos sugiere más fuertemente esclerodermia como causa de
síndrome de Raynaud en esta paciente?

a) Engrosamiento cutáneo distal que se extiende en dirección proximal hasta las


articulaciones metacarpofalángicas.
b) Anticuerpos anticentrómero en suero.
c) Anticuerpos antinucleares en suero.
d) Cambios capilares distales en la valoración del lecho ungueal.

Allen R. M. MMS Medicina Interna. 5ª. Edición. National Medical Series. Mc. Graw
Hill. 2006. (capítulo 10 VIII F 1). El engrosamiento de la piel es una característica
definitoria de esclerodermia; si se limitan las áreas distales puede ser manifestación de la
variante CREST de esclerodermia (esclerodermia que coexiste con calcinosis subcutánea,
fenómeno de Raynaud, disfunción de la motilidad esofágica, esclerodactilia y telangiectasia).
Los pacientes con fenómeno de Raynaud seropositivos para anticuerpos antinucleares (ANA)
o anticuerpos anticentrómero o que muestran cambios capilares digitales, están en más alto
riesgo de presentar esclerodermia que un paciente con fenómeno de Raynaud sin esas
características. La dismotilidad, el espasmo o la estenosis esofágicos distales son posibles
datos de esclerodermia, pero no son específicos.

26.- La rigidez cadavérica alcanza su mayor extensión e intensidad.

a) 4 hrs.
b) 6 hrs.
c) 12 hrs.
d) 24 hrs.

La rigidez cadavérica suele ser completa en un periodo de 8 a 12 horas, alcanzando su


máxima intensidad a las 24 horas y casi siempre inicia su desaparición de 36 a 48 horas,
siguiendo el mismo orden en que se instauró.
La LEY de la RIGIDEZ CADAVËRICA indica que cuando esta comienza precozmente, es de
intensidad escasa y de duración limitada. Existen excepciones claras a esta ley, como
ocurre con las muertes por frío, calor o determinadas intoxicaciones. (Ley de NYSTEN)
Martínez Murillo - Saldivar S. Medicina Legal. 16ª. Edición, Francisco Méndez
Oteo, 2003. (Capítulo V, p. 43).
27.- El cáncer de tiroides que puede producir un síndrome paraneoplásico y que se asocia a
elevaciones de calcitonina es:

a) Cáncer anaplásico.
b) Cáncer de células de Hürttle.
c) Cáncer medular.
d) Cáncer papilar.

El cáncer medular de tiroides surge de las células parafoliculares de la tiroides, que


normalmente producen calcitonina. La medición de calcitonina es importante sobre todo en
el seguimiento de los pacientes para detectar enfermedad residual o recidivante.

Jiménez RSA, Gómez VE, Bolaños GF. Tiroides. En Flores JF, Cabeza A, Calarco Z
(ed): Endocrinología. 5ª ed. México. Méndez Oteo México, 2005: 584-92.

28.- Paciente que acude por presentar herida por mordedura de perro de 8 cms de longitud
que afecta piel y tejido celular subcutáneo, localizada en pierna derecha con 48 hrs de
evolución. , la herida se encuentra eritematosa, dolorosa y con secreción verde amarillenta
la primera medida terapéutica es:

a) Lavado quirúrgico y cierre primario con drenaje con aplicación de toxoide tetánico y
antibióticos.
b) Lavado quirúrgico y cierre primario sin drenaje con aplicación de toxoide tetánico y
antibióticos.
c) Lavado quirúrgico y debridación con aplicación de toxoide tetánico y antibióticos.
d) Lavado quirúrgico y afrontamiento de bordes con drenajes.

Current Pediatric Diagnosis and Treatment 17 Ed Mc Graw Hill. Pag 336 . 2005.

29.- Un niño de 12 años en Yautepec Morelos, se encuentra dormido sobre una toalla a la
orilla de una alberca, súbitamente presenta dolor intenso en muslo derecho y se observa
una pequeña pápula eritematosa El niño refiere sensación de ardor intenso en el muslo, su
madre aplica una pomada con antihistamínico y le administra un antihistamínico oral.
Dos horas después el paciente inicia parestesias, nausea y vómito, comienza a presentar
dolor abdominal intenso, en episodios, síntomas que en una hora se hacen más intensos, la
causa más probable de este cuadro es:

a) Picadura de viuda negra.


b) Picadura de alacrán.
c) Picadura de cara de niño.
d) Picadura de nauyaca.
El veneno que inocula es 15 veces más potente que el veneno de una serpiente de cascabel y
puede condicionar efectos sistémicos graves e incluso la muerte.

El primer síntoma generalmente es un dolor similar a una punción con un alfiler y la


sensación se experimenta cuando realmente se ha efectuado la picadura de la araña. Sin
embargo, es posible que algunas personas no lo sientan. Puede haber hinchazón y
enrojecimiento leve y una lesión en forma de diana.
De 15 minutos a una hora más tarde, un dolor muscular sordo se irradia desde el área de la
picadura a todo el cuerpo.

Síntomas
Náuseas
Ataque al estado general
Diaforesis
Contracturas musculares
Dolor muscular
Retención urinaria
Estreñimiento
Taquicardia
Insuficiencia cardiaca
Hipertensión arterial
Inquietud
Ansiedad
Sensación de muerte
Inminente

Current Pediatric Diagnosis and Treatment 17 Ed Mc Graw Hill. Pag 346 . 2005

30.- La indicación quirúrgica es obligada en un paciente afectado de enterocolitis


necrotizante cuando presente uno de los siguientes signos clínicos:

a) Heces mucosanguinolentas.
b) Vómitos biliosos.
c) Distensión abdominal.
d) Neumoperitoneo.
Enfermedad adquirida que afecta principalmente a RN prematuros o patológicos y que se
caracteriza por necrosis de la mucosa o de capas incluso más profundas del intestino, sobre
todo en el íleon terminal y con menos frecuencia del colon y del intestino delgado proximal.

Síntomas, signos y diagnóstico

La enfermedad puede iniciarse con un íleo que se manifiesta con distensión abdominal,
residuos gástricos biliosos (tras las tomas) que pueden progresar a vómitos de bilis o
presencia de sangre macroscópica o microscópica en las heces. La sepsis puede ponerse de
manifiesto con letargia, inestabilidad térmica, aumento de las crisis de apnea y acidosis
metabólica.

La detección sistemática de sangre oculta o de sustancias reductoras en las heces de los


prematuros (que han recibido alimentación oral o enteral) puede ayudar a diagnosticar la
ECN. Las radiografías iniciales pueden ser inespecíficas o mostrar sólo el íleo. Sin embargo,
un asa intestinal fija y dilatada que no cambia en las radiografías posteriores indica una
ECN. Las radiografías diagnósticas son las que muestran neumatosis intestinal y gas en el
territorio de la vena porta. El neumoperitoneo es un signo de perforación intestinal e indica
la necesidad urgente de una intervención quirúrgica.

BOL PEDIATR 2006; 46(SUPL. 1): 172-178.


Protocolos de Neonatología.
Enterocolitis necrotizante neonatal.

I. FERNÁNDEZ JIMÉNEZ1, I. DE LAS CUEVAS TERÁN2.

31.- El principal estímulo para la síntesis y secreción de prolactina es:

a) La menstruación.
b) El embarazo.
c) La succión.
d) El trabajo de parto- el dolor.

La función principal de la prolactina en la mujer es estimular y mantener la lactancia


puerperal, acción directa sobre las células acidofílicas conocidas como lactotrofas de la
glándula mamaria. Para que aumente el sistema ductal se requiere de estrógenos, hormona
del crecimiento, corticoides, lactógeno placentario y prolactina. Para el desarrollodel
sistema lóbulo alveolar se requiere de estrógenos, progesterona y prolactina, por lo que se
deben considerar los niveles de estas hormonas en estados patológicos como mastopatía
fibroquística, mastodinia (dolor mamario), carcinoma mamario, etc.
NIVELES DE PROLACTINA DURANTE EL EMBARAZO

Durante la gestación las concentraciones de prolactina aumentan gradualmente a partir de


la implantación y siguen en ascenso casi lineal hasta el parto. Preparando a la glándula
mamaria para la lactancia. Las células lactotrofas se ven aumentadas en número y tamaño
considerablemente, tal vez inducidas por el aumento de estrógenos hasta 10 veces más que
en la etapa pregestacional. El mayor estímulo para su producción refleja es la succión y
estimulación del pezón, efectuada por las vías neurales desde la mama hasta los centros
cerebrales.

Durante la gestación normal en el primer, segundo y tercer trimestre los niveles están
alrededor de 75, 116 y 216 ng/mL. Respectivamente según reportan Tyson y col. El nivel
máximo lo alcanza en la semana 25 y empieza a disminuir hacia la 38ª semana. El ritmo
circadiano y su secreción por pulsos se mantiene durante todo el embarazo.

BIBLIOGRAFIA

Azíz D.C. Use and Interpretation of Tests in Endocrinology. 1997. Specialty Laboratories.
Capítulo 10 Desordenes pituitarios Páginas 129-130.

Benson R.C. Diagnóstico y Tratamiento Ginecoobstétricos. 1990. Editorial el Manual


Moderno,S.A. Capítulo 3 Fisiología del sistema reproductor de la mujer. Pagina 64 y
Capítulo 37 El puerperio Paginas 818-820.

Comparato M.R. Terapéutica Hormonal en Ginecología. 1988. Editorial "El Ateneo". Capítulo
2 Hormonas sexuales. Páginas 36-37 Capítulo 13 Terapéutica hormonal en endocrinología
ginecológica Páginas 207-216.

Farreras P.Valenti y Rozman C. Medicina Interna.1982. Décima Edición. Ediciones Doyma.


Capítulo 15 Endocrinología. Páginas 1805, 1806, 1811, 1812, 1814.

32.- Le envían a su consultorio paciente femenino de 62 años con diagnóstico de miastenia


gravis. ¿Cuál de los siguientes medicamentos está indicado?

a) Curare.
b) Neostigmina.
c) Quinidina.
d) Sumatriptán.
e) Succinilcolina.
Manejo farmacológico: Existen diferentes pautas terapéuticas dirigidas a contrarrestar
los síntomas de la enfermedad o el mecanismo inmunológico que la gatilla. Los fármacos
utilizados son:

- Inhibidores de la Acetilcolinesterasa (Neostigmina, Piridostigmina). Dirigidos al manejo


sintomático de la MG, mejorando la fuerza motora pero no la progresión de la enfermedad.
Su mecanismo de acción es la inhibición reversible de la acetilcolinesterasa, lo cual genera
un aumento de ACh en la placa motora. La dosis a utilizar es variable y debe modificarse en
distintas etapas de la enfermedad, incluso siendo frecuente no lograr un efecto uniforme
en los diferentes grupos musculares en un mismo paciente. El objetivo por lo tanto será
utilizar la dosis mínima con la que se genere la mejor respuesta clínica. El efecto se obtiene
de 30 minutos a 2 horas de la administración y tiene una duración de hasta 6 horas. Las
dosis recomendadas de Piridostigmina son de 15-60 mg cada 4-6 horas vía oral y de
Neostigmina 0,5-2 mg/kg cada 4-6 horas intramuscular. Las reacciones adversas asociadas
son: dolor abdominal, hipersalivación, aumento de las secreciones respiratorias y
bradicardia y se relacionan con el efecto colinérgico generado, por lo que es necesario
administrar concomitantemente atropina2,3.

1.-Ponsetia JM: Miastenia Gravis. Manual Terapéutico. Barcelona; Springer Verlag Ibérica,
1995.

2.- Ponsetia JM, Espina E, Armengola M: Diagnóstico y Tratamiento de la Miastenia grave.


Med Clin (Barc) 2000; 115: 264-70.

3.- Drachman DB: Myasthenia gravis. N Engl J Med 1994; 330: 1797-810.

4.- Andrews PI: Autoimmune myasthenia gravis in childhood. Semin Neurol 2004; 24: 101-
10.

5.- Anlar B: Juvenile myasthenia: diagnosis and treatment. Paediatr Drugs 2000; 2: 161-9.

6. - Gajdos P: Myasthenic syndrome. Diagnosis trends. Rev Prat 2000; 50: 419-23 7. -
Arroyo H: Myasthenia gravis in childhood and adolescence. Rev Neurol 1996; 24: 1385-9.

33.- El examen microscópico de una biopsia pulmonar de un paciente con mucormicosis


pulmonar mostrará:

a) Micelio Dicotomizado Septado Con Conidias.


b) Micelio Dicotomizado Hialino Cenocitico.
c) Micelio Con Clamidoconidios Y Blastoporas.
d) Micelio Septado Y Esclerotes De Mediar.
Examen directo. Se realiza a partir de exudados o secreciones nasales, expectoración,
lavados bronquiales y heces, inclusive se puede hacer a partir de biopsias. La muestra se
debe aclarar con KOH al 10 %. Al microscopio se observan numerosas hifas no tabicadas,
hialinas, dicotómicas, de aproximadamente 5F de ancho por 20-25 de largo, esta imagen es
patognomónica.

Harada M, Manabe T, Yamashita K, Okamoto N. Pulmonary mucormycosis with fatal massive


hemoptysis. Acta Pathol 1992;42(1):49-54.

34.- En la infección por virus de la inmunodeficiencia humana por lo común la linfadenopatía


difusa en una persona clínicamente sana suele ser un signo de:

a) Linfoma.
b) Sarcoma de Kaposi.
c) Tuberculosis.
d) No indica infección o tumor específicos.

Allen R. M. MMS Medicina Interna. 5ª. Edición. National Medical Series. Mc. Graw Hill.
2006. (capítulo 8 VIII G 1 b). La linfadenopatía difusa en una persona infectada por virus
de la inmunodeficiencia humana (VIH) que se encuentra clínicamente bien suele ser signo de
que no hay infección específica o tumor. Aunque todas esas respuestas pueden ser ciertas,
las personas con múltiples ganglios aumentados de tamaño, tuberculosis o trastornos
malignos tienden a encontrarse enfermas. Con mayor frecuencia también experimentan
pérdida de peso y fiebre. Es más probable que el linfoma se presente con afección orgánica
en pacientes infectados con VIH que en otros. El sarcoma de Kaposi puede producir
afección linfática, pero en general sólo se encuentra en etapa tardía de la enfermedad con
lesiones cutáneas y mucosas extensas. La linfadenopatía moderada es un dato común en
infección por VIH en etapa media. Se desconoce su causa exacta, pero la desaparición de la
linfadenopatía prolongada puede preceder al deterioro clínico. La sífilis produce adenopatía
local o difusa en pacientes con infección por VIH o sin ella. Sin embargo, esta adenopatía
siempre se acompaña de algún otro dato de sífilis.

35.- Esteroide de origen principalmente placentario:

a) Progesterona.
b) Estrona.
c) Estradiol.
d) ACTH.

DeCherney A. (1999) Diagnóstico y tratamiento ginecoobstétricos.México. Ed. Manual


Moderno. Pag 187. La progesterona es el esteroide de origen principalmente placentario, la
estrona y el estradiol son hormonas de origen maternoplacentofetal, la lipotropina y la
ACTH son de origen fetales.
36.- Acude a su consulta paciente femenino de 26 años que cursa el tercer trimestre del
embarazo tiene edema en miembros inferiores sin sintomatología agregada, usted indica a
la paciente:

a) Elevación de los miembros inferiores en decúbito lateral.


b) Restricción Hídrica.
c) Tiacidas.
d) Diurético de asa.

DeCherney A. (1999) Diagnóstico y tratamiento ginecoobstétricos. México. Ed. Manual


Moderno. Pag 237. El edema de las partes bajas producido por la impedancia del retorno
venoso es común en la parte final del embarazo, La paciente debe tratarse solo si está
molesta. La elevación de los miembros inferiores (especialmente en decúbito lateral)
mejora la circulación. Están contraindicados los diuréticos.

37.- En un paciente preescolar se debe sospechar el diagnóstico de adenoiditis si éste


presenta:

a) Roncus,fiebre y dificultad respiratoria.


b) Tos nocturna, respiración oral y voz nasal.
c) Rinorrea, conjuntivitis y disfonía.
d) Halitosis,sibilancias y tos húmeda.

Signos y síntomas de adenoiditis


Respiración bucal y dificultad para la respiración por la nariz.
Voz nasal.
Respiración ruidosa.
Ronquidos durante el sueño.
Apnea (períodos en que deja de respirar durante unos segundos mientras duerme).
Resfriados frecuentes.
Tos nocturna.
Mordida Abierta.
Otitis Media Aguda y Sinusitis a repetición.
Hiperactividad.
Dificultad para la concentración.
Somnolencia diurna.

Nelson. Tratado de Pediatría. Tomo 1. 529-534. Ed. Interamericana


38.- La manifestación clínica más frecuente del hiperparatiroidismo es:
a) Crisis convulsiva.
b) Osteoporosis y fracturas.
c) Arritmias y movimientos anormales.
d) Litiasis renoureteral.

La hipersecreción de hormona paratiroidea, por aumento del umbral en que la calcemia


suprime a la PTH o por aumento de la masa de tejido paratiroideo, provoca hipercalcemia, la
que es la responsable principal de la sintomatología del HPT1°. Que puede presentarse en
forma: a) sintomática o b) asintomática (mínimamente sintomática); esta última es la más
frecuente desde que se dosa en forma sistemática la calcemia. No obstante, la expresión
clínica no esta relacionada con el grado de elevación de la calcemia (68).
Forma sintomática
Las manifestaciones óseas son más frecuentes en mujeres, y la litiasis renal y úlcera
gástrica en el hombre. Las manifestaciones óseas, condrocalcinosis, falla renal y episodios
agudos son más frecuentes en viejos, y la litiasis renal en jóvenes (42).
a) Manifestaciones renales: la hipersecreción de PTH, especialmente por aumento de
absorción intestinal de calcio, provoca hipercalcemia, si esta supera el umbral de filtración
glomerular y resorción tubular se produce hipercalciuria, la que se acompaña de orina
alcalina debido a la acidosis renal tubular proximal, causada por el exceso de HPT, llevando
a nefrocalcinosis y/o nefrolitiasis. Estas se presentan en el 30 a 70% de los HPT1°, la
función renal se afecta hasta llegar a la insuficiencia. Era la causa más frecuente de
diagnóstico de HPT en la década del 60 a 70 y lo sigue siendo, actualmente, en nuestro
medio.
b) Manifestaciones óseas: el exceso de PTH moviliza el calcio óseo por aumento de la
osteólisis osteocítica y por estimulación de la proliferación de osteoclastos, lo que lleva
inicialmente a osteopenia difusa y finalmente a osteítis fibrosaquítica. Antes de 1960 esta
era la forma más frecuente de presentación o sea quistes, tumores óseos, fracturas
múltiples y deformación esquelética con disminución de estatura. En la mayoría de los
pacientes aún sin enfermedad ósea clínica, se comprueba pérdida progresiva de masa
mineral ósea.
Revista Médica Universitaria, Volumen 1, Número 1, Diciembre 2005, ISSN 1669-8991
Hiperparatiroidismo primario, secundario y terciario: actualización
Perinetti H.A.
Instituto de Patología de la Tiroides. Departamento de Medicina Quirúrgica.
F.C.M. UNCuyo.

39.- Le reportan larvas de Ancylostoma braziliensisi y Ancylostoma caninum son los


agentes causales en el hombre de:

a) Dermatitis de los nadadores.


b) Oncodermatitis.
c) Dermatitis verminosa reptante.
d) Larva currens cutánea.
Helmintos

Dermatitis verminosa reptante

Agnte causal:

Ancylostoma caninum, A. braziliense, Uncinaria stenocephala, Gnatostoma spinigerum, A.


tubaeforme, Bunostomum phlebotomum.

Introducción
La geohelmintiasis denominada larva migrans cutánea o dermatitis verminosa reptante (DVR)
es una entidad clínica que forma parte del complejo síndrome de las larvas migratorias
cutáneas y viscerales.

Se define como una erupción autolimitada de la piel producida por larvas de nemátodos
parásitos de diversos animales, cuyo huésped natural no es el hombre, por lo que no pueden
completar en él su ciclo evolutivo.

Los agentes etiológicos predominantes son A. caninum y A. braziliense. Se ha comprobado


que el primero puede alcanzar el tubo digestivo y producir el cuadro clínico conocido como
enteritis eosinofílica.

40.- Son indicaciones para oxigenoterapia suplementaria a largo plazo:

a) PaO2 menor a 55 y SaO2 menor a 88.


b) Pa02 mayor a 55 y SaO2 menor a 88.
c) PaO2 mayor a 55 y SaO2 mayor a 88.
d) PaO2 menor 20 y SaO2 menor 80.

Las indicaciones para oxigenoterapia suplementaria a largo plazo incluyen una Pao2 de 55 o
Menor y Sao2 (SATURACION DE OXIGENO) de 88 o menor.

Fishman, A; Manual de Enfermedades pulmonares, tercera edición, Mc Graw Hill, pags


142-143, 2004.

41.- El cáncer de tiroides más frecuente es:

a) Medular.
b) Papilar.
c) Medular.
d) De células de Hürtle.
El cáncer papilar de tiroides es el tipo de cáncer de tiroides más frecuente y constituye 70
a 90% de los tumores malignos bien diferenciados de tiroides. El cáncer papilar suele ser
multifocal e invadir localmente la glándula tiroides y extenderse a través de la cápsula
tiroides invadiendo estructuras adyacentes. Sin embargo la mayor parte de los casos este
cáncer no es agresivo cuando se detecta en etapas tempranas y alcanza una sobrevida de
más del 90% a 20 años.

Jameson JL, Weetman AP. Transtornos de la glándula tiroides. En Jameson JL (ed):


Harrison. Endocrinología. 1a ed. Madrid. MacGraw-Hill España, 2006: 106.

42.- La imagen en cabeza de medusa puede observarse en un ultrasonido en casos de:

a) Síndrome de hiperinfección por Strongyloides.


b) Oclusión por Ascaris lumbricoides.
c) Fasciolosis múltiple biliar.
d) Uncinariasis masiva.

Ascaris lumbricoides es el agente etiológico de la ascariasis; es un nematodo intestinal


cosmopolita y es uno de los más comunes helmintos que parasitan al hombre. Esta
helmintiasis se adquiere por la ingestión de huevos larvados; las larvas durante su migración
pasan por el pulmón para completar su maduración, ascienden por el árbol respiratorio para
posteriormente ser deglutidas y llegar al intestino delgado en donde se transforman en
adultos.

Las radiografías abdominales muestran niveles hidroaéreos y múltiples imágenes lineales de


A. lumbricoides en las asas intestinales dilatadas (Khuroo, 1996).En el ultrasonido
abdominal se puede demostrar un asa dilatada, con pared engrosada y una masa de gusanos
que causa la obstrucción. Las imágenes de helmintomas se describen como una masa
ecogénica compleja de aire intestinal, helmintos y materia fecal, con morfología de cabeza
de medusa en el eje longitudinal y de roseta en el corte transaxial (Malde y Chadha, 1993).
Los helmintos se identifican como estructuras ecogénicas que revelan un canal anecoico, el
cual representa el canal digestivo del parásito, y presentan movilidad (Khuroo 1996). El
tratamiento primeramente es conservador con manejo hidroelectrolítico adecuado, succión
nasogástrica, antibióticos y terapia antihelmíntica. Se ha llegado a utilizar el Gastrografín,
que es una sustancia hiperosmolar que produce un exceso de líquido en la vecindad y
alrededor de la masa de gusanos condicionando su separación, a razón de 15 a 30 ml
introducidos en el estómago, a través de una sonda nasogástrica para tratar la obstrucción
intestinal parcial (Maor y col., 1984).
REFERENCIAS

American Academy of Pediatrics. Ascaris lumbricoides infections. In: Peter G.


Ed. 1997. Red Book: Report of the Committe on Infectious Diseases 24th
edition. American Academy of Pediatrics. Khuroo, S. 1996. Ascariasis.
Gastr. Clin. North Am. 25:553-577.

Malde, H. and Chadha, D. 1993. Roundworm obstruction: sonographic diagnosis.


Abdom. Imagin. 18:274-276.

Maor , B., de Carvalho, F. and Chapell, J. 1984. Gastrofín treatment of intestinal


obstructions due to Ascaris lumbricoides. J. Ped. Surg. 19:174-176.

Marinho, HA., Shrimpton, R., Giuliano, R. and Burini, RC. 1991. Influence of
enteral parasites on the blood vitamin A levels in preschool orally supplemented
with retinol and/or zinc. Eur. J. Clin. Nutr. 45:539-544.

Rao, P.L., Satyanarayana, G. and Venkatesh, A. 1988. Intraperitoneal Ascariasis.


J. Ped. Surg. 23:936-38.

Rosner, B. 1995. Hypothesis testing: Categorical data. In: Rosner Bernard (ed.)
Fundamental of Biostatistic. Duxbury de. Fourth ed. Harrisonburg, VA.: 345-
440.

Salman, B. 1997. Management of intestinal obstruction caused by ascariasis. J.


Ped. Surg. 32:585-587.

43.- Masculino de 44 años originario del Edo de México, ganadero, se le diagnostica


Fasciola Hepática, su forma infectante es:

a) Quiste.
b) Ooquiste.
c) Metacercaria.
d) Esporoblasto.

Fasciola hepatica o duela del hígado es una especie de platelminto trematodo (duela) de la
subclase Digenea, caracterizado por su forma lanceolada, con dos ventosas, una bucal y
otra ventral, y un ciclo biológico con dos generaciones (digeneo) en dos hospedadores, un
molusco gasterópodo anfibio y un mamífero. Es parásito de los canales biliares y la vesícula
biliar de herbívoros y omnívoros, incluido el hombre; es el agente causal de una de las
parasitosis más difundidas del ganado, la fascioliasis (o fasciolosis), que es considerada
como una de las enfermedades parasitarias más importantes del mundo de los rumiantes
domésticos.
Metacercaria

La metacercaria es la forma infectante para el hombre y para los demás animales que
sirven de hospedador definitivo. Generalmente se encuentran enquistadas en la vegetación
acuática semisumergida que normalmente comen los animales, pero el hombre también
acostumbra a ingerirlas. También se adquiere la infección tomando aguas contaminadas. Al
llegar al duodeno se desenquistan liberando un parásito juvenil que perfora la pared
intestinal y en unas 3 horas, se aloja en la cavidad peritoneal en donde pasa de 3 a 16 días;
posteriormente avanza por el peritoneo, llega a la cápsula de Glisson, la perfora, penetra al
parénquima hepático del cual se alimentan los parásitos juveniles durante su migración
hacia los conductos biliares en donde se desarrolla hasta el estado adulto, lo que sucede en
unos 2 meses; después empezará a reproducir huevos que salen al exterior con la bilis y
materias fecales, complementando así el ciclo biológico.

Bibliografía

• Abrous, M.; Rondelaud, D. and Dreyfuss, G. 1999. Paramphistomum daubneyi and


Fasciola hepatica: influence of temperature changes on the shedding of cercariae
from dually infected Lymnaea truncatula. Parasitol. Res., 85(8-9). p765-9.
• Acuna, R. 1998. Human fascioliasis: seasonal variations and female preponderance
of complicated forms. J. Infect.; 37(1). p88-9.
• Agatsuma, T. 2000. Molecular evidence of natural hybridization between Fasciola
hepatica and F. gigantica. Parasitol.Int., p231-38.
• Anderson, H.R.; Fairweather, I.; Bamford, D.R. and Montgomery, W.J. 1998. The
effect of diamphenethide on protein synthesis by the liver fluke, Fasciola hepatica.
International Journal for Parasitology. 23. p1053-1062.

Basso, N.; Calceta Resio, E.; Dughetti, R.P.; Giménez, R.A.; Peres Tort, G.B.; Rosa, A.B. y
Welch, E.L. 1992. Fundamentos de Parasitología Veterinaria. Ed. Hemisferio Sur. Argentina

44. - A 6-year-old child presents with flesh-colored papules on the hand that are not
pruritic. Examination reveals lesions that are approximately 4 mm in diameter with central
umbilication. A halo is seen around those lesions undergoing regression. Which of the
following is the most likely diagnosis?

a) Verruca vulgaris.
b) Molluscum contagiosum.
c) Keratoacanthoma.
d) Herpetic whitlow.

El Molusco Contagioso es el nombre de una infección viral del grupo de los Poxvirus. Es
frecuente, transmisible, autoinoculable (uno mismo la puede trasmitir a diferentes partes
del cuerpo), se puede curar sola y es benigna.
Afecta principalmente a los niños menores de 10 años, adultos sexualmente activos y a
pacientes inmunosuprimidos (con defensas bajas) como los pacientes con SIDA, ocurriendo
en un 5 ó 18 por ciento. Afecta más a hombres que a mujeres, y su frecuencia aumenta
también en climas tropicales y durante el verano.

CLINICA

El periodo de incubación de la infección es de 14 a 50 días, aunque hay datos de recién


nacidos con lesiones al cabo de 7 días postparto.

Las lesiones se inician como neoformaciones que miden generalmente de 2 a 6 mm, aunque
pueden llegar a medir 3 cm (13,14), son hemisféricas, cupuliformes, lisas, del color de la piel
o perladas, algunas (20%) tienen una umbilicación central(15); la base es levemente
eritematosa y son de consistencia firme(13,14). Se localizan en cualquier parte del cuerpo e
incluso pueden afectar mucosas, generalmente se agrupan en un área específica, pero
pueden estar diseminadas en personas infectadas con el virus de la inmunodeficiencia
humana adquirida, siendo un marcador de enfermedad avanzada (16). En el caso de los niños
las lesiones se localizan normalmente en cara, tronco, brazos y piernas a diferencia de los
adultos jóvenes en quienes el molusco contagioso se adquiere por transmisión sexual, las
lesiones tienen predilección por genitales, abdomen y cara interna de los muslos; en otras
series no hay diferencia (17).

Las lesiones del molusco suelen aparecer entre los 14 días a 6 meses después de la
exposición, hay datos de recién nacidos con lesiones al cabo de 7 días postparto. Se
pueden propagar por auto inoculación, pero es típico que se resuelvan espontáneamente en
pocos meses.

Las lesiones son asintomáticas en la mayoría de los pacientes (13,18), aunque en el 10% de
los casos puede haber prurito y desarrollarse una reacción eccematosa (14).

CRITERIO DIAGNÓSTICO

El diagnóstico se hace clínicamente y en algunos casos dudosos puede efectuarse biopsia


con tinción H-E donde se encuentran los cuerpos de molusco ( inclusiones
intracitoplasmáticas grandes) o de Hendersen-Paterson; el 90% de los pacientes posee Ac
tipo Ig G. Puede realizarse microscopía electrónica, PCR, Elisa, e inmunohistoquímica.

REFERENCIAS

1. Schotz J, Rosen-Wolft A, Bugert J et al. Molecular epidemiology of molluscum


contagiosum. J Infect Dis 1988; 158: 898-900.

2. Porter CD, Nlake NW, Archard LC et al. Molluscum contagiosum virus type in genital and
non genital lesions. Br J Dermatol 1989; 120: 37-41.

3. Overfield TM, Briody JA. An epidemiologic study of molluscum contagiosum in Achorage,


Alaska. J Pediatr 1966; 69: 640-42.

4. Gottlieb SL, Myskowwski PL. Moluscum contagiosum. Intern J Dermatol 1994; 33: 453-
61.
5. Telner P, Solomon LM. Eruptive molluscum contagiosum in atopic dermatitis. Can Med
Assoc J 1966; 95: 978-79.

6. Pauly CR, Artis WM, Jones HE. Atopic dermatitis, impaired cellular immunity and
molluscum contagiosum. Arch Dermatol 1978; 114: 391-93.

7. Dohil, P. Lin, J. Lee, A. Lucky, A. Paller, L The epidemiology of molluscum contagiosum in


children. J Am Acad Dermatol. 2006;54:47-54.

8 Rosenberg EW, Yusk JW. Molluscum contagiosum. Eruption following treatment with
prednisone and methotrexate.Arch Dermatol 1970; 101: 439-41.

9. Koopman JJ, Van Merrienboer FCJ, Vreden SGS, Dolmans WMV. Molluscum contagiosum:
a marker for advanced HIV infection. Br J Dermatol 1992; 126: 528-29.

10. Schwartz JJ, Myskowski PL. HIV-related molluscum contagiosum presenting as a


cutaneous hom. Int J Dermatol1992; 31: 142-44.

45.- Femenino de 43 años portadora de DM tipo II, e HTAS, es ingresada al servicio de


Medicina Interna por cetoacidosis. Posterior a su recuperación metabólica inicia con
fiebre, cefalea, dolor facial, disminución del nivel de conciencia y enrojecimiento nasal con
lesión negruzca en fosa nasal derecha. El diagnóstico más probable en ésta paciente es:

a) Endocarditis por S. aureus.


b) Infección por Mucor.
c) Carcinoma epidermoide.
d) Infección por M. tuberculosis.

Mucormicosis es el nombre común dado a varias diferentes enfermedades causadas por


hongos de la orden de los Mucorales. Muchas diferentes especies han sido implicadas como
agentes de síndromes clínicos similares.

Los Mucoraceos son hongos que se encuentran en todas partes y son comunes habitantes de
materia en descomposición. Por ejemplo, Rhizopus sp. Frecuentemente puede ser
recuperado de pan mohoso. Por su rápido crecimiento y prolífica capacidad de formar
esporas, inhalación de conidias debe ser una experiencia cotidiana. La presencia de esporas
Mucorales en cinta adhesiva no estéril se demostró fue la fuente de mucormicosis cutánea
primaria. Aun cuando estos hongos crecen en muchos nichos ecológicos, la infrecuencia de
enfermedad debida a estos organismos da fe de su baja potencial virulencia en el huésped
humano. En contraste a la amplia distribución de este hongo, la enfermedad en humanos
esta limitada, en muchos casos a población con severo inmunocompromiso, diabetes mellitus
o trauma.
Manifestaciones clínicas

En 1973 Meyer y Armstrong12 categorizaron las diferentes presentaciones clínicas de


mucormicosis, considerando el órgano involucrado y haciendo una división en seis entidades:
rinocerebral, cutánea, gastrointestinal, pulmonar, diseminada y formas misceláneas.

Existe una fuerte asociación entre la entidad subyacente y la forma de presentación. La


mucormicosis rinocerebral se presenta más frecuentemente en pacientes diabéticos con
acidosis, y debido al advenimiento de quimioterapias cada vez más potentes, se aprecia con
más frecuencia en pacientes leucémicos con neutropenia prolongada, en aquellos que
reciben múltiples esquemas de antibióticos y corticoesteroides, así como en pacientes con
trasplantes de órganos. En pocas ocasiones se ha documentado esta forma invasora en
personas sin enfermedades subyacentes.

La forma pulmonar puede presentarse en pacientes con leucemia y neutropenia; la


mucormicosis gastrointestinal se observa más comúnmente en pacientes con desnutrición
calórico-proteica y en prematuros, y la diseminada en pacientes con déficit inmunológico
grave como trasplantados, leucémicos y nefrópatas tratados con deferoxamina.1,2

La infección rinocerebral es la presentación más frecuente y característica de


mucormicosis, siendo la rinoorbitaria y la mucormicosis paranasal estadios tempranos de
esta.

La infección generalmente inicia en senos paranasales o paladar duro, y se extende a senos


adyacentes con diseminación a través de senos etmoidales y zona retroorbitaria; puede
tener acceso al cerebro a través del ápex orbitario, lámina cribosa y por vía vascular. Una
costra necrótica sangrante en paladar o en mucosa nasal y un drenaje ocular de pus
negruzco orientan al diagnóstico. Puede haber una progresión rápida y presentarse el
deceso en pocos días o ser indolente si la enfermedad subyacente se logra controlar.

Inicialmente puede haber dolor facial, cefalea, fiebre y algún grado de celulitis orbitaria,
conforme progresa la invasión de la órbita la pérdida de la función del II, III, IV y VI
nervios craneales puede ocurrir, así como también puede haber pérdida de la función de
músculos extraoculares, proptosis, quemosis progresiva, congestión nasal, epistaxis y
letargia. La disfunción de nervios craneales, especialmente el V y VII, ocurre de manera
tardía manifestando ptosis y midriasis, lo cual es un factor pronóstico grave, en caso de
trombosis retiniana, hay pérdida de la visión y puede haber afección intraocular.

El compromiso cerebral es la complicación más seria y puede manifestarse como: infarto,


absceso, trombosis del seno cavernoso, hematoma subdural y necrosis del lóbulo frontal.2
Otras complicaciones son trombosis de arteria carótida interna y vena yugular, e incluso se
ha reportado infarto de miocardio por oclusión de coronaria y aborto séptico.

Puede haber presentaciones crónicas y secuelas tardías a pesar del tratamiento


aparentemente satisfactorio por lo cual siempre debe realizarse un seguimiento del
paciente a largo plazo.
BIBLIOGRAFIA:

• Mayo Clinic. Pulmonary Diseases.


• Mandele Douglas. Infections Diseases.
• Pennington. Respiratory infections, Diagnosis and Management.
• Ronald B. george., Richard W. Light, Michael A. Matthay. Chest Medicine 3era. edición.
• Scott E Davis. Neumonía Micótica. Clínicas médicas de Norteamérica 1997; 5: 1092?1094.

46.- Los agentes causales más frecuentes de meningitis viral son:

a) Virus de la parotiditis epidémica.


b) Enterovirus.
c) Arbovirus.
d) Herpesvirus.

Introducción

Con el término meningitis aséptica (MA) se denomina al síndrome clínico caracterizado por:
cefalea, fiebre, signos meningeos y líquido cefalorraquídeo (LCR) de aspecto claro pero con
pleocitosis linfocitaria y que se presenta como consecuencia de la inflamación de las
leptomeninges (aracnoides y piamadre).

Implícita a la definición es la ausencia de signos de compromiso parenquimatoso (encefalitis)


o inflamación de la médula espinal (mielitis) (1).

La principal causa de la MA es la viral. Pero se debe tener en cuenta que ante un paciente
con MA se debe plantear un diagnóstico diferencial.

Epidemiología y etiología

Respecto a la etiología viral, las series iniciales publicadas hasta fines de los años cuarenta
identificaban sólo un 25% de los agentes etiológicos, entre los cuales se contaban el virus
de la parotiditis, de la coriomeningitis linfocitaria y de la polio; causas infrecuentes hoy en
día (2).

En las décadas siguientes, en la medida que se ha logrado identificar a los enterovirus,


estos han emergido como la principal etiología reconocible, más aún, la aplicación de la
reacción de polimerasa en cadena (PCR) ha permitido identificar al enterovirus no polio
como el agente causante en un 85%-95% de los casos (2,3).

Otros agentes virales son: Arbovirus, Herpes Simple, Varicela Zoster, Herpesvirus humano
tipo 6, virus de la inmuno deficiencia humana (VIH), y virus Influenza, ver tabla 2 (1,2,3). A
continuación revisaremos los virus que con mayor frecuencia producen una MA.
Enterovirus

Los enterovirus (EV) son los virus que predominan como agente etiológico en la MA,
causando entre un 85-95% de los casos. Pertenecen a la familia de los picornavirus, se
clasifican en 2 clases: el grupo poliovirus (tipo 1, 2 y 3) y el grupo no poliovirus (coxsackie,
echovirus y enterovirus no clasificados). Se han identificado 71 serotipos de EV. El ser
humano es el único reservorio (4,5).

Los individuos suelen infectarse por vía de contaminación fecal-oral, y menos


frecuentemente por vía respiratoria. La superficie celular del tracto gastrointestinal sirve
como receptor viral, observándose que la replicación viral inicial se desarrolla en el tejido
linfático local. Aproximadamente al tercer día de la infección, el virus penetra al sistema
circulatorio causando la primera viremia e invadiendo diferentes órganos, viremias
sucesivas durante la primera semana, se asocian a una mayor replicación y se correlacionan
con los síntomas y signos de la infección viral, en esta etapa inicial ocurre la invasión del
SNC (1,2,3). No existe variación por raza ni sexo, la población de lactantes y niños es la
más susceptible a la infección (4).

Referencias:

1. Harley A, Rotbart MD. Viral Meningitis. Sem Neurol. 2000; 20: 277-92.
2. Rotbart HA. Viral meningitis and aseptic meningitis syndrome. In: Scheld WM,
Whitley RJ, Durack DT. (Eds). Infections of the central nervous system.
Philadelphia, Lippincott —Raven. 1997: 23-46.
3. Tunkel AR, Scheld WM. Acute meningitis. En: Mandell G, Bennett J, Dolin R.
Principles and Practice of Infectious Diseases. New York. Churchill Livingstone.
2000: 959-89: 1016-28.
4. Mowad M. Enteroviral Infections. December 20, 2002. Emedicine.com.
5. Nigrovic L. What`s new with enteroviral infections?. Curr Op Pediatr. 2001; 13: 89-
94.
6. Dolin R. Enterovirus-71. Emerging infections and emerging questions. N Engl J
Med.1999; 341: 984-5.
7.

Sawyer MH, Saez-Llorenz X, Aviles CL. Oral pleconaril reduces the duration and severity
of enteroviral meningitis in children. Proceedings of 1999 APS/SPR meetings, San
Francisco; 1999.
47.- Se trata de paciente femenino de la tercera edad que acude por presentar durante
las últimas 3 semanas astenia, febrícula vespertina, cefalea global y, durante los últimos
días, le han notado confusión intermitente y somnolencia progresiva. A la exploración se
observa una temperatura de 38ºC, somnolencia, dudosa rigidez de nuca y paresia de VI par
derecho. La placa de tórax muestra un discreto engrosamiento pleural apical derecho. La
biometría hemática y química sanguinea se encuentra normal. Se realiza una punción lumbar
con los siguiente resultados en LCR: presión de apertura 170mmHg, proteínas 140 mg/dL,
glucosa 42 mg/dL, 270 células (70% mononucleares), tinción de gram, de Zielhl-Nielsen, así
como investigación de antígenos bacterianos son negativos. ¿Que diagnóstico es el más
probable?:

a) Meningitis tuberculosa.
b) Encefalitis herpética.
c) Meningitis bacteriana.
d) Meningitis vírica.

Meningitis TB

Causas, incidencia y factores de riesgo

La meningitis tuberculosa es un trastorno muy poco común, causado por la diseminación de


la Mycobacterium tuberculosis al cerebro, desde otro sitio en el cuerpo. Generalmente, sus
síntomas se inician gradualmente. Entre los factores de riesgo se pueden mencionar
antecedentes de tuberculosis pulmonar, consumo de alcohol en forma excesiva, SIDA u
otros trastornos que comprometen el sistema inmunitario.

MANIFESTACIONES CLINICAS.

Los fenómenos clínicos evolucionan en varios periodos o etapas cuya duración y contenido
semiológico son muy variables, pero que tienen unas características generales que permiten
definirlos.En 1948 el British Medical Council definió los tres estadios evolutivos de la
enfermedad.(7)

Estadio I: consciente, síntomas inespecíficos, ausencia de afectación neurológica.

Estadio II: cierto grado de afectación de la conciencia, aparición de signos de afectación


neurológica.

Estadio III: afectación profunda del estado de conciencia, convulsiones, signos de


focalidad neurológica.
La recuperación total es la regla en los casos en estadio I mientras que las secuelas y
muerte son muy elevadas en los estadios II y III. En esta última etapa el porcentaje de
fallecimientos puede ser superior al 25%.(8,9).

Estadio I

En esta fase, los síntomas son muy inespecíficos, sin que puedan atribuirse a alteración del
SNC, salvo por la aparición de alteraciones del carácter, que si bien no son específicas si
son bastante constantes.

Fiebre, generalmente de escasa entidad. Otros síntomas de este periodo son cefalea
moderada, apatía, pérdida de la alegría, anorexia, y con mucha frecuencia en el niño dolores
abdominales, acompañados o no de vómitos, estos últimos sin características que sugieran
un origen central. Los síntomas de esta etapa en el lactante pueden pasar desapercibidos.

Estadio II

Junto a la persistencia de otros signos, entre ellos la fiebre, se ponen de manifiesto los
signos de afectación neurológica, coincidiendo con los signos meníngeos. Al principio
predominan la cefalea y los vómitos, estos últimos sobre todo en menores de tres años.
Aparece rigidez de nuca, que puede ser muy intensa, y signos de Kernig y Brudzinski.

En más de 1/3 de pacientes con meningitis tuberculosa hay afectación de pares craneales.
El sexto par, es el que se afecta con más frecuencia, seguido por los pares III y IV, de
forma uni o bilateral. El facial se afecta más raramente, así como otros pares craneales.
Puede afectarse el nervio óptico, conduciendo a la atrofia óptica y ceguera.

Las convulsiones son en el niño una manifestación habitual y precoz de la


enfermedad.(10)Los signos de hipertensión endocraneal se van acentuando, apreciándose
una fontanela tensa en el lactante, aumento del perímetro craneal, edema de papila en el
niño mayor, diplopia y visión borrosa.

Loa déficits motores que pueden presentarse a lo largo de la evolución son secundarios a
los fenómenos vasculares descritos, que conducen a isquemia e infarto. Puede verse
hemiplejía por afectación del territorio de la arteria cerebral media o carótida interna.

Síntomas de afectación medular pueden existir en casos muy evolucionados, con


presentación progresiva de paraplejía.

Durante este estadio las alteraciones del sensorio aun son discretas, y su empeoramiento
es el que define el paso al siguiente periodo.
Estadio III.

Presidido por la alteración profunda del estado de conciencia, pasando de la apatía e


irritabilidad, a la confusión, estupor y coma profundo en los casos avanzados. Los casos
terminales se caracterizan por coma profundo, rigidez de descerebración, siendo la muerte
inminente.

Encefalopatía tuberculosa.

En los niños hay un cuadro, denominado encefalopatía tuberculosa por Udani en 1958 (11),
que se caracteriza por la presencia de signos de afectación difusa con convulsiones estupor
o coma, y signos moderados de afectación meníngea. El coma sobreviene precozmente, y la
muerte sobreviene entre 1 y 2 meses después del comienzo, aun con terapéutica específica.
Dominan el cuadro los fenómenos de hipertensión intracraneal.

Anatomopatológicamente el cuadro está constituido por severo edema de la sustancia


blanca, sin presencia de hemorragias. Microscópicamente hay una pérdida difusa de mielina
en la sustancia blanca, atribuyéndose la causa a fenómenos de hipersensibilidad frente a la
tuberculoproteinas

Es una infección de las meninges, las membranas que recubren el cerebro y la médula
espinal, causada por la bacteria Mycobacterium tuberculosis que produce la tuberculosis.

Signos y exámenes

Para cualquier paciente con meningitis, es importante llevar a cabo una punción lumbar:

• Tinción del líquido cefalorraquídeo (LCR) positiva para Mycobacterium .


• LCR con alto nivel de proteínas, bajo nivel de glucosa y aumento en el número de
linfocitos.
• Cultivo de LCR que muestra la proliferación de Mycobacterium tuberculosis.
• Reacción en cadena de la polimerasa (PCR) que muestra LCR positivo para M.
tuberculosis .
• Prueba cutánea positiva para tuberculosis.
• Biopsia cerebral o meníngea que muestra la presencia de M. tuberculosis.
BIBLIOGRAFIA

1. Sudre P,Dam G,Kochi A. La tuberculose aujourd'hui dans le monde.WHO Bull


1992;70:297-308.

2. Dolin PJ,Raviglione MC,Kochi A. Global tuberculosis incidence and mortality during 1990-
2000.WHO Bulletin 1994;72:213-220.

3. March Ayuela P. Trend in tuberculous meningitis in Barcelona in children aged 0-4


years:correlation with the annual risk of tuberculous infection. Tubercle and Lung Disease
1994;75:423-428.

4. Barry R Bloom.Tuberculosis: Pathogenesis, Protection, and Control. Tuberculosis.


American Society for Microbiology. Washington DC 2005.

5. Anggard E. Nitric oxide:mediator,murderer,and medicine. Lancet. 1994;343: 1199-206.

6. Jinkins JR,Gupta R,Hyun Chang K,Rodriguez-Carvajal J.MR imaging of cental nervous


system tuberculosis. Radiol Clin North Am 1995;33:771-786.

7. Medical Research Council.Streptomycin in Tuberculosis Trials Committee:Streptomycin


treatment of tuberculous meningitis.Lancet. 1948;1:582-596.

48.- Es la patología que asocia distensión de los espacios aéreos respiratorios distales a los
bronquíolos terminales acompañada de destrucción de los tabiques alveolares:

a) Bronquitis crónica.
b) Enfisema.
c) Asma.
d) Asbestosis.

Un enfisema se define en términos anatomopatológicos por el agrandamiento permanente


de los espacios aéreos distales a los bronquiolos respiratorios, con destrucción de la pared
alveolar, con o sin fibrosis manifiesta.1 Es una enfermedad crónica comprendida junto con
la bronquitis crónica en la Enfermedad Pulmonar Obstructiva Crónica (EPOC).2 El nombre
viene del griego emphysema que significa "soplar el aire" o "insuflar'.3

Fisiopatología del enfisema


Le enfermedad por sus efectos incapacita y debilita enormemente la calidad de vida del
paciente, sometiéndolo a una vida restringida y sedentaria. El deterioro progresivo de su
función pulmonar, desencadena no sólo cambios físicos en el paciente, sino también
alteraciones a nivel psicológico.
El enfisematoso, se ve ceñido a realizar mínimos esfuerzos. Cualquier tipo de actividad
física, se convierte en verdadero sufrimiento y evita realizarlas. Las más elementales
necesidades fisiológicas, requieren de enormes esfuerzos y gastos extras de energía.
El enfisema es una enfermedad crónica, progresiva, caracterizada por un agrandamiento
anormal y permanente de los espacios aéreos distales al bronquiolo terminal, acompañado
de destrucción de sus paredes sin fibrosis (8)
La característica más relevante de la enfermedad, es su limitación al flujo aéreo durante
los movimientos respiratorios. Esta limitación, medido en valores del volumen espiratorio
forzado al primer segundo (VEF 1) por debajo de los valores predictivos del paciente (< 1,0
l), ocasiona una gran morbilidad, con deterioro severo de su capacidad vital (CV).
El 25% de la resistencia total pulmonar al flujo aéreo, ocurre normalmente en las vías
aéreas < 3mm, pero estos valores se aproximan al 80% en los pacientes enfisematosos
(9,10). Este flujo, esta determinado por el juego entre presión – retroceso pulmonar
elástico. Como el volumen pulmonar durante la expiración disminuye, ocurre cierre
prematuro de las vías aéreas periféricas, debido a la pérdida de elasticidad pulmonar y a la
destrucción de los alvéolos que se encuentran fijados a las delgadas paredes de los
bronquiolos. Al final, estos pulmones enfisematosos son de mayores dimensiones a lo normal.
Ello se traduce en disminución del VEF 1, y otros flujos (FEF 25-75, FEF 50), aumento del
volumen residual (VR) de la capacidad residual funcional (CRF) y de la capacidad pulmonar
total (CPT), así como, una disminución a la prueba de difusión al monóxido de carbono
(DLCO).
El atrapamiento de aire en las zonas enfisematosas, se traduce en hallazgos radiológicos
típicos; un pulmón hiperaireado, con escasa vasculatura pulmonar, aplanamiento de los
diafragmas y un tórax más alargado.
Al examen físico, vemos un tórax con dimensiones mayores en el diámetro antero posterior
y escaso desarrollo de la masa muscular. Hay hipersonoridad a la percusión y los sonidos
pulmonares se encuentran muy disminuidos.
Todos estos cambios en la función pulmonar, ocasionan desigualdad en la relación
ventilación / perfusión (V/Q), resultado de un incremento de espacio muerto, hipoxemia
sola en fases tempranas, acompañadas de hipercapnia en las fases terminales.
Basándose en estos datos, la indicación quirúrgica no sólo se limita al pulmón enfisematoso
puro, sino también, al pulmón con enfermedad bulosa (11-16).

Gordon LS. Emphysema: The first two centuries and beyond. A historial overview,
with suggestions for future research: Part I. Am Rev Respir Dis 1992; 146:1334-
1344.
Gaast A, Molard-Dietmenn A, Pelletier A, Pauli G, Bieth JG. The antielastase screen of the
lower respiratory tract of alpha I-proteinase inhibitor suficiente patients with emphysema
or pneumotorax. Am Rev Respir Dis 1990;141:880-883.
Janoff A. Elastases and emphysema, current assesment of the Protease-Antiprotease
hypothesis. Am Rev Respir Dis 1985;417-433.
Silverman EK, Speizer FE. Risk factors for the development of chronic pulmonary disease.
Med Clin North Amer 1996;80:501-522.
Deslaries J. A perspective on the role of surgery in chronic obstructive lung disease.
Chest Surg Clin North Amer 1995;5:575-602.
Brantigan OC, Müller E. Surgical treatment of pulmonary emphysema. Am Surg
1957;23:789-804.
Cooper JD, Trulock EP, Triantafillou AN, Patterson GA, Pohl MS, Doloney PA, et al.
Bilateral pneumonectomy (volume reduction) for chronic pulmonary disease. J Thorac
Cardiovasc Surg 1995,109:106-119.

49.- Masculino de 64 años, jubilado es ingresado al servicio de urgencias. Los familiares


refieren, que el día de ayer de manera brusca el paciente inició con confusión, comenzó a
decir cosas raras, a no responder a lo que le preguntaban, incluso presentó confusión de su
propio nombre. Al interrogatorio se muestra distraído y parece no entender lo que se le
pregunta, no recuerda nada de lo que le ha pasado y no sabe ni el día, ni el lugar en el que
está. El diagnóstico más probable de éste paciente es:

a) Delirium.
b). Psicosis breve.
c) Alzheimer.
d) Esquizofrenia.

MANIFESTACIONES CLÍNICAS
Los hallazgos cardinales del delirium incluyen su aparición aguda y la inatención. Para
definir el momento exacto del inicio es necesario recurrir a la información dada por el
cuidador del paciente. Otra de las características es la fluctuación que presenta el cuadro
a lo largo del día. El individuo fácilmente se distrae ante los diferentes estímulos externos,
y es muy difícil que sostenga una conversación y obedezca órdenes. Aunado a lo anterior
puede presentarse desorganización del pensamiento, alteraciones de la sensopercepción y
malteración del estado de conciencia (generalmente letárgico). Aunque no son síntomas
principales, también presentan desorientación, déficit cognoscitivo, agitación o retardo
psicomotriz, delirios, labilidad emocional e inversión del ciclo sueño-vigilia.

Criterios para el diagnóstico de


F05.0 Delirium debido a... (indicar enfermedad médica) (293.0)

A. Alteración de la conciencia (p. ej., disminución de la capacidad de atención al entorno)


con disminución de la capacidad para centrar, mantener o dirigir la atención.

B. Cambio en las funciones cognoscitivas (como déficit de memoria, desorientación,


alteración del lenguaje) o presencia de una alteración perceptiva que no se explica por la
existencia de una demencia previa o en desarrollo.
C. La alteración se presenta en un corto período de tiempo (habitualmente en horas o días)
y tiende a fluctuar a lo largo del día.

D. Demostración a través de la historia, de la exploración física y de las pruebas de


laboratorio de que la alteración es un efecto fisiológico directo de una enfermedad médica.

Nota de codificación: Si el delirium está superpuesto a una demencia vascular


preexistente, indicarlo codificando F01.8 Demencia vascular, con delirium [290.41].

Nota de codificación: Incluir el nombre de la enfermedad médica en el Eje I, por ejemplo,


F05.0 Delirium debido a encefalopatía hepática [293.0]; codificar también la enfermedad
médica en el Eje III.

50.- El mecanismo de acción de la glibenclamida es:

a) Estimula la liberación de insulina de las células beta del páncreas.


b) Bloque la acción del péptido C con lo que disminuye la acción periférica de la insulina.
c) Bloquea a la leptina.
d) Estimula el glucagón del hepatocito.

Goodman & Gilman’s. The Pharmacology Diseases and Therapeutics. USA: Medical
Publishing Division. Mc. Graw-Hill.2005: 1700-1707.2.

La Glibenclamida es un agente antidiabético debido a su acción hipoglicemiante. Estimula la


liberación de insulina de las células β del páncreas; aumenta los niveles de insulina mediante
la reducción de la liberación hepática de la hormona. Incrementa la sensibilidad de los
tejidos periféricos a la acción de la insulina y disminuye la glucogenólisis hepática y la
gluconegénesis. Su efecto global es una reducción de la concentración sanguínea de glucosa
en pacientes diabéticos cuyo páncreas es capaz de sintetizar insulina.

51.- Pseudomonas aeuroginosa es una bacteria frecuentemente responsable de:

a) Otitis media aguda.


b) Otitis serosa.
c) Otitis media crònica.
d) Otitis externa necrotizante.
La infección crónica del oído medio y la mastoides por lo general se desarrolla como
consecuencia de otitis media aguda recurrente, aunque puede presentarse después de
cualquier enfermedad y trauma. La perforación de la membrana timpánica es común, suele
acompañarse de cambios en la mucosa como degeneración polipoidea y tejido de granulación,
y cambios óseos como osteítis y esclerosis. La bacteriología de la otitis media crónica
difiere de la otitis media aguda. Los microorganismos más frecuentes incluyen P. aeruginosa,
especies de Proteus, Staphylococcus aureus e infecciones mixtas por anaerobios. El
hallazgo clínico de la otitis media crónica es una otorrea purulenta. El drenaje puede ser
continuo o intermitente, con aumento en la intensidad durante una infección de vías
respiratorias superiores o después de una exposición al agua. El dolor es poco frecuente
excepto durante las exacerbaciones agudas. La pérdida conductiva de la audición es
resultado de la destrucción de la membrana timpánica y de la cadena de los huesecillos. El
tratamiento médico de la otitis media crónica incluye remoción continua de los restos
infectados, uso de tapones para protección contra exposición al agua y gotas de antibiótico
tópico para las exacerbaciones. La actividad de la ciprofloxacina contra Pseudomonas puede
auxiliar a secar un oído crónicamente purulento, cuando se administra en dosis de 500 mg,
2 veces al día, por 1 a 6 semanas.

Bibliografía
Acuin J. Chronic suppurative otitis media. Clin Evid. 2006 Jun;(15):772-87. [PubMed]
Agrawal S, Husein M, MacRae D. Complications of otitis media: an evolving state. J.
Otolaryngol. 2005 ;34 Suppl 1:S33-9. [PubMed]
Dynamed. Otitis media with efusión . Ebsco; Updated 2008 Feb [acceso 26/2/2008].
Disponible en: http://www.ebscohost.com/dynamed/
Hannley MT, Denneny JC 3rd, Holzer SS. Use of ototopical antibiotics in treating 3
common ear diseases. Otolaryngol Head Neck Surg. 2000; 122(6):934-40
Jahn AF, Abramson M. Medical managment of chronic otitis media. Otolaryngol Clin N Am
1984; 17: 673-678.

52.- Usted tiene la sospecha de diagnóstico de criptosporidiosis, el examen de laboratorio


de elección para confirmar éste es:

a) Coproparasitoscópico De Faust.
b) Coproparasitoscópico De Stoll.
c) Coproparasitoscópico Directo.
d) Frotis De Heces Teñido Con Kinyoun.

Las aves infestadas con Cryptosporidium sp. no tienen historia, signos clínicos o lesiones
macroscópicas específicas Para el diagnóstico de criptosporidiosis existen
aproximadamente 15 técnicas para la observación de oocistos de Cryptosporidium, pero las
más usadas son las técnicas de Kinyoun y Ziehl-Neelsen.
Jorge Tay Zavala, Parasitología Médica. Editorial: Méndez Editores, Edición: 5ª Año: 1996.
Páginas: 81-96

53.- Se observa un incremento en las infecciones de vías urinarias posterior al uso de:

a) Preservativos.
b) Espermicidas.
c) Esponjas vaginales.
d) Diafragmas.

Beck W. (1997) NMS. Obstetrics and Gynecology. USA. Williams & Wilkins. Pag 243.
Existen pocos efectos secundarios al uso del diafragma. Se reporta un incremento en la
frecuencia de infecciones de vías urinarias probablemente resultado de compresión uretral.

54.- Masculino de 66 años que acude al servicio de urología, al realizar un PSA en sangre
demuestra 20 ng/L. Se realiza ecografía la cual muestra un nódulo de 2 cms, localizado en
el lóbulo izquierdo, en el que la biopsia demuestra carcinoma.Se realizan estudios de
extensión manifiesto de metástasis a distancia, adenopatías sospechosas ni invasión de
órganos vecinos. Se le realiza prostatectomía radical. ¿Cuál de los siguientes parámetros
tiene importancia pronóstica?:

a) La estadificación anatómica en la pieza de resección y el volumen total de la glándula.


b) La estadificación anatómica en la pieza de resección y el score de Gleason.
c) La estadificación anatómica en la pieza de resección y el grado tumoral según Bloom-
Richardson.
d) El volumen tumoral estimado en comparación con el volumen glandular total.

Grado Gleason

Los análisis univariantes y multivariantes de factores pronósticos en el cáncer de próstata


identifican el índice de Gleason como uno de los marcadores pronósticos más significativos,
con peores resultados de supervivencia, extensión tumoral y periodo libre de enfermedad
cuanto más indiferenciado esté el tumor4. La utilización de índices Gleason combinados (que
indican la proporción relativa de muestras con cáncer de alto grado) nos proporciona una
información pronóstica más precisa.
Si valoramos el índice Gleason junto con el estadio clínico haremos estimaciones
pronósticas aún más acertadas35. Sin embargo, se ha encontrado que cuando el tumor es de
alto grado, el pronóstico será desfavorable incluso cuando exista órgano-confinación.

El índice Gleason más preciso se obtiene con la pieza de prostatectomía radical. Cuando se
intenta estimarlo a partir de la muestra obtenida con biopsia por punción se comete un alto
porcentaje de errores, superior incluso al 50%. Algunos estudios sugieren que el error más
frecuente ocurre cuando la biopsia por punción sugiere un Gleason <7, en los que en muchos
casos, tras analizar la pieza quirúrgica se clasificará como Gleason ≥7.

Foco de origen

La próstata está dividida en tres partes: zona periférica, zona transicional y zona central
(ver figura 1). Los tumores de la zona transicional tienen datos de mejor pronóstico
(malignidad, extensión del tumor, supervivencia libre de recidiva bioquímica) que los de la
zona periférica.

3.1.4. Multifocalidad

Un alto porcentaje (67%) de cánceres prostáticos tienen varios focos de origen, que
pueden tener distintos grados histológicos (heterogeneidad).

La existencia de multifocalidad se asocia con mayores tasas de recurrencia, y con un grado


y un estadio más avanzados.

3.1.5. Extensión extracapsular

Es un indicador de mal pronóstico, con mayores tasas de fallo bioquímico y progresión de la


enfermedad. Esta relación desfavorable aumenta cuando existe un mayor nivel de invasión y
penetración de la cápsula por parte del tumor.

Algunos autores creen que la importancia pronóstica de la extensión extracapsular se debe


a su asociación con otras variables, como el volumen tumoral o la infiltración de vesículas
seminales, pero otros encuentran peores resultados en los pacientes con penetración
capsular, con independencia de las posibles variables loco-regionales asociadas.
3.1.6. Invasión de vesículas seminales

Es un factor de mal pronóstico, asociado a mayores tasas de progresión de la enfermedad y


de fallo bioquímico.

Este mayor riesgo de resultados desfavorables se debe a su asociación con otros


marcadores de mal pronóstico, como el índice Gleason, extensión extracapsular, el volumen
tumoral, márgenes quirúrgicos positivos o PSA preoperatorio.

Puede que el significado pronóstico de la invasión de vesículas seminales no sea constante, y


que dependa de la zona de las vesículas que se ve afectada: si la invasión es en la porción
distal el pronóstico es peor que cuando ocurre en la zona proximal.

3.1.7. Márgenes quirúrgicos positivos

Algunos estudios han encontrado que son un factor predictor de mayor riesgo de
progresión de la enfermedad o fallo bioquímico. Aunque para algunos autores este efecto
de los márgenes quirúrgicos positivos se debe a su asociación con otras variables que
empeoran el pronóstico, como invasión de vesículas seminales, extensión extracapsular, PSA
preoperatorio, grado Gleason o volumen tumoral, otros han encontrado significación
pronóstica de forma independiente.

3.1.8. Volumen tumoral

Un mayor volumen tumoral en la pieza de prostatectomía se asocia a mayor riesgo de


progresión de la enfermedad y de fallo bioquímico. Sin embargo, diversos estudios han
encontrado que este efecto desfavorable se debe a su asociación con varios factores
pronósticos como la existencia de penetración capsular, márgenes quirúrgicos positivos,
invasión de vesículas seminales o un grado Gleason avanzado.
Bibliografía

1. 22. Wallen MJ, Linja M, Kaartinen K, Schleutker J, Visakorpi T. Androgen receptor


gene mutations in hormone-refractory prostate cancer. J Pathol. 1999;189(4):559-
63.
2. 23. Segawa N, Mori I, Utsunomiya H, Nakamura M, Nakamura Y, Shan L, et al.
Prognostic signifi cance of neuroendocrine differentiation, proliferation activity and
androgen receptor expression in prostate cancer. Pathol Int. 2001;51(6):452-9.
3. 27. Miyoshi Y, Ishiguro H, Uemura H, Fujinami K, Miyamoto H, Miyoshi Y, et al.
Expression of AR associated protein 55 (ARA55) and androgen receptor in prostate
cancer. Prostate. 2003;56(4):280-6.
4. 25. Culig Z, Hobisch A, Cronauer MV, Radmayr C, Trapman J, Hittmair A, et al.
Androgen receptor activation in prostatic tumor cell lines by insulin-like growth
factor-I, keratinocyte growth factor, and epidermal growth factor. Cancer Res.
1994;54(20):5474-8.
5. 26. Sadi MV, Barrack ER. Image analysis of androgen receptor immunostaining in
metastatic prostate cancer. Heterogeneity as a predictor of response to hormonal
therapy. Cancer. 1993;71(8):2574-80.

55.- Una mujer de 27 años G4 P0 con 6 SDG acude a consulta para su primera visita
prenatal. Su historia obstétrica pasada es importante porque tiene tres pérdidas en
segundo trimestres. Refiere que en las tres ocasiones al presentarse al hospital presentaba
dilación cervical completa. No recuerda haber tenido contracciones dolorosas. Niega
antecedentes médicos y quirúrgicos. El examen físico es normal incluyendo un examen
pélvico que muestra un cervix largo y cerrado. Después de una larga discusión con la
paciente ella pide que se le practique un cerclaje durante este embarazo. Cual de los
siguientes es el momento más apropiado para realizarlo.

a) Inmediatamente.
b) 10 a 14 semana.
c) 24 a 28 semanas.
c) 32 a 36 semanas.
La incompetencia cervical se define como la incapacidad de llevar un embarazo a término
debido a anormalidades funcionales o estructurales del cuello uterino. Esta patología fue
descrita por primera vez en 1950 por Lash y Lash (1), fecha en que comenzaron las
primeras correcciones quirúrgicas para resolverla. Tiene una incidencia entre 0,1 a 2% de
todos los embarazos, manifestándose clínicamente por abortos repetidos del segundo
trimestre o por dilatación cervical indolora con parto prematuro. En la mayoría de los
casos, la etiología es desconocida.

Cerclaje profiláctico:

El cerclaje profiláctico generalmente se realiza basado en los antecedentes obstétricos


positivos alrededor de la semana 10 a la 14 de gestación, antes de que la dilatación,
reblandecimiento o acortamiento del cervix se lleven al cabo y antes de cualquier sangrado
o contracciones.

Manual de embarazo y parto de alto riesgo.


Escrito por Elizabeth S. Gilbert,Judith S. Harmon.

56.- Una paciente de 22 años G5P2A3 tiene una historia de abuso de sustancias prenatal. A
las 37 sdg tiene una parto vaginal obteniéndose una neonato pequeño para la edad
gestacional, de género masculino con péqueñas aperturas palpebrales, pliegues epicánticos,
con cara aplanada, filtrum hipoplásico y borde del vermillon delgado. Estos hallazgos son
característicos en neonatos cuyas madres tuvieron abuso prenatal de cual de las siguientes
sustancias.

a) Tabaquismo.
b) Alcohol.
c) Marihuana.
d) Narcóticos.

Criterios diagnósticos de los FASD:

Desde Jacqueline Rouquette1 en adelante, el diagnóstico clínico del FAS reposa en una
tríada de elementos semiológicos que constituyen el conjunto nuclear de las
manifestaciones del síndrome.
Estas manifestaciones son: a) el retraso del crecimiento intrauterino y extrauterino; b) un
dismorfismo facial característico; y c) anomalías morfofuncionales del neurodesarrollo en
el SNC.
La cara de los niños afectados tienen características particulares durante la primera
infancia: frente pequeña asociada a (y consecuencia de) la microcefalia; la raíz nasal es
aplanada o incluso presenta un aspecto en silla de montar similar al de la sífilis congénita;
las alas de la nariz son pequeñas; los ojos son diminutos (microftalmia) y están muy
próximos entre sí, las hendiduras palpebrales son más cortas que lo normal y el párpado
superior frecuentemente está en ptosis; a veces, puede haber un pliegue epicanto o
epicanto invertido o blefarofimosis; la región media de la cara es pequeña; el labio superior
es característicamente fino, delgado, recto y está como “arremangado” hacia adentro,
mostrando muy poco el bermellón (a veces puede haber labio leporino con paladar hendido o
sin él); el surco nasolabial está notablemente aplanado o, incluso, es inexistente, por lo que
la zona facial superior a la boca adquiere un aspecto como de “planchado”; los dientes
pueden ser pequeños, hipoplásicos y con esmalte defectuoso; las orejas pueden tener
implantación baja y los pabellones estar malformados; al nacer suele existir un notable
hirsutismo, más marcado en la piel perifacial. Sin embargo, todas estas características
faciales, que expresan defectos en el desarrollo del mesodermo facial, se suavizan con el
crecimiento y pueden desaparecer, de modo que en la adultez ya no es posible distinguir a
los nacidos con FAS sólo por el aspecto facial. La maduración del macizo facial se completa
lentamente.

Arch Argent Pediatr 2010; 108(1):61-67 / 61


Criterios diagnósticos del síndrome alcohólico fetal y los trastornos del espectro
del alcoholismo fetal Diagnostic criteria for fetal alcohol syndrome and fetal alcohol
spectrum disorders.

57.- ¿Cuál, entre las siguientes, es la prueba más específica para el diagnóstico de
miastenia grave?

a) Electromiograma de fibra muscular aislada.


b) Determinación de anticuerpos anti-receptor de acetilcolina.
c) TAC torácico.
d) Electromiograma con estimulación repetitiva.

La miastenia gravis es una enfermedad autoinmune que se caracteriza por presentar


debilidad muscular fluctuante y fatiga de distintos grupos musculares. La miastenia gravis
afecta a individuos de todas las edades, con una predilección por mujeres entre los 20 y 40
años. Los músculos oculares, faciales y bulbares son los más frecuentemente afectados por
la enfermedad. Los pacientes con miastenia gravis manifiestan empeoramiento de la
debilidad muscular, con infecciones intercurrentes, fiebre y agotamiento físico o
emocional. La infección respiratoria (bacteriana o vírica) es la causa más frecuente de
provocación. La presencia de anticuerpos contra receptores de acetilcolina en un paciente
con manifestaciones clínicas compatibles con la miastenia gravis confirma el diagnóstico.
El tratamiento de esta entidad es controvertido y debe ser individualizado, ya que no
existe un régimen terapéutico uniforme para todos los pacientes. Entre los tratamientos
disponibles testacan los fármacos anticolinesterásicos, los corticosteroides, la
plasmaféresis, la inmunoglobulina, los inmunosupresores y la timectomía.

REFERENCIAS

1. Drachman DB. Myasthenia gravis. N Engl J Med 1994;330:1797-8102.-

2. Engel AG. Myasthenic syndromes. In Engel AG, Franzini-Armstrong C, eds. Myology. 2


ed. New York: McGraw-Hill;1994. P. 1798-835.

3. Engel AG. Ohno K. Milone M, Sine SM. Congenital myasthenic syndromes caused by
mutations in acetylcholine receptor genes. Neurology 1997;48 (Suppl 5):S28-35.

4. Steinman L, Mantegazza R. Prospects for specific inmunotherapy in myasthenia gravis.


FASEB J 1990;4:2726-31.

5. Massey JM. Treatment of acquired myasthenia gravis. Neurology 1997;48 (suppl 5):S46-
51.

58.- Ocasiona rechazo al alimento, irritabilidad, llanto constante, inquietud, delirio,


convulsiones y estado epiléptico.

a) Capulín tullidor.
b) Té de anís estrella.
c) Papaver somniferum.
d) Picadura de alacrán.

La intoxicación se presenta en forma crónica. Más frecuente en recién nacidos y lactantes.


La sintomatología por intoxicación de té de anís de estrella se caracteriza por el siguiente
cuadro clínico:
¾ Gran irritabilidad
¾ Llanto constante
¾ Inquietud
¾ Delirio
¾ Convulsiones
¾ Rechazo al alimento
¾ Estado epiléptico
El tratamiento consiste en:

¾ Estabilizar signos vitales


¾ Ayuno
¾ Soluciones parenterales
¾ Anticonvulsivos:
Benzodiacepinas ó barbitúricos
Estabilizar signos vitales
¾ Ayuno
¾ Soluciones parenterales
¾ Anticonvulsivos:
Benzodiacepinas ó barbitúricos
• 1.- Montoya-Cabrera MA. Intoxicaciones y envenenamientos en niños. México,
Intersistemas , 2000.
• 2.-Montoya CMA. Toxicología clínica. 2ª. Ed, México, Méndez Editores, 1997.

59.- Una mujer de 40 años con DM acude a consulta quejándose de descarga vaginal
prurítica y blanquecina, ¿cual de los siguientes exámenes de diagnósticos es mas útil para
identificar el patógeno?

a) Wet prep.
b) Tincion de Gram.
c) PH.
d) KOH.

Prueba del KOH es un procedimiento en el cual hidróxido del potasio (KOH) se utiliza
detectar hongos disolviendo las células humanas en una cultura. La diferencia en la
composición de la pared de célula de células humanas y de células fungicidas permite que
este procedimiento ayude a distinguir las dos células. El KOH desnaturaliza las proteínas en
la célula humana; solamente sigue habiendo las células fungicidas ser considerado debajo
del microscopio.

1. Bernal B. Fisiología y ecología de la vagina. Rev Chil Obstet Ginecol 1986; 51:56-60.

2. Ibárcena E. Vaginosis bacteriana; diagnóstico y prevalencia. XII Congreso Peruano de


Obstetricia y Ginecología 1996; 204-6.

3. Scapini JC, Guzmán CA. Detección de bacilos Gram negativos curvos anaerobios en
pacientes con vaginosis. Obstet Ginecol Latinoam 1986; 44: 320-5.

4. Soihet S. El flujo vaginal en la consulta ginecológica. Ginecol Obstet (Perú) 1986; 30: 50-
60.
5. Linaldi CAS, Urbina JR, Castañeda JL. Vaginitis por Gardnerella vaginalis en niñas y
adolescentes. Bol Med Infant Mex 1988; 45: 101-3.

6. Sobel J. Vaginitis and vaginal flora. Controversies abound. Current Opin Infect Dis 1996;
9: 42-7.

7. Faro S. Leucorrea. Causas infecciosas e imbalances en el ecosistema vaginal. Tribuna


Médica. Cahners Healthcare ed 1996; 1 – 10.

60.- Son manifestaciones de púrpura vascular aguda ó anafilactoide:

a) Hematomas en tronco.
b) Petequias en cara.
c) Pápulas hemorrágicas en miembros pélvicos.
d) Hemortrosis en miembros torácicos.

La Púrpura de Schönlein-Henoch tiene comienzo agudo o insidioso, los síntomas pueden


seguir una secuencia de semanas o meses, más de la mitad presenta febrícula y astenia. (1)

Las lesiones por vasculitis provocan angioedema local, que puede preceder a la púrpura,
especialmente en los miembros inferiores en niños y adultos (zonas declives), esto se
atribuye a la mayor presión hidrostática que conduciría al depósito de los inmunocomplejos.

El edema agudo hemorrágico del lactante caracterizado por fiebre, lesiones cutáneas
purpúricas grandes y edema, en su forma típica afecta a lactantes con historia reciente de
infección de vías respiratorias altas. La erupción cutánea se inicia con la aparición súbita de
pápulas y placas edematosas y purpúricas que van adquiriendo una morfología anular en
medallón, escarapela o iris, de distribución simétrica, respetando el tronco y las mucosas.
No hay participación visceral.

BIBLIOGRAFIA

1.-Miller ML, Pachman LM. Vasculitis. Púrpura de Schönlein-Henoch. En: Nelson. Tratado de
Pediatría.17ª ed. Madrid: Elsevier; 2005: p.826-830.
2.-Enciclopedia Médica: Púrpura de Henoch-Schönlein
http://www.nlm.nih.gov/medlineplus/spanish/ency/article/000425.htm.

3.- J. Vila Cots, A. Giménez Llort, J.A. Camacho Díaz y A. Vila Santandreu. Sección de
Nefrología. Unidad Integrada de Pediatría. Hospital Sant Joan de Déu-Hospital Clínic.
Universidad de Barcelona. España. Nefropatía en la púrpura de Schönlein-Henoch: estudio
retrospectivo de los últimos 25 años An Pediatr (Barc). 2007; 66(3):290-3.

4.-Gussinye P, Catalá M, Codina X, Corral S, Domínguez O, Puig A. Púrpura de Schönlein-


Henoch: estudi retrospectiu dels darrers 20 anys. Pediatria Catalana. 2002; 62:109-14.

61.- En el diagnóstico clínico de feocromocitoma ¿Cual de las siguientes síntomas clásicos


se presentan?

a) Palpitaciones, diaforesis, hipertensión y cefalea.


b) Palpitaciones, hipotensión postural y diarrea.
c) Hipotensión postural, bradicardia y palpitaciones.
d) Constipación, bradicardia y diaforesis.

Manifestaciones clínicas:

La hipertensión arterial es la más frecuente. Un 60 % de estos tumores presenta


hipertensión mantenida, y la mitad de estos pacientes presenta crisis o paroxismos de
hipertension. El 30% restante presenta una elevación de la tensión arterialunicamente
durante el ataque. Un 10% son silentes (incidentaloma). La HTA suele ser grave, a veces
maligna, y casi siempre resistente al tratamiento convencional.

Los paroxismos o ataques hipertensivos pueden ser frecuentes o esporádicos, los síntomas
son similares en cada crisis y con el tiempo aumentan de intensidad, suelen ser súbitos y de
duración variable.

Aparece cefalea, sudoración profusa, palpitaciones, angustia, sensación inminante de


muerte, palidez, dolor torácico y abdominal acompañado de náuseas y vómitos. La tensión
arterial se eleva hacia cifras alarmantes. Las crisis suelen estar desencadenadas por
actividades que desplazan el contenido abdominal, el estrés psicológico, el ejercicio físico,
los cambios posturales, los estornudos, las maniobras de Valsalva, algunos alimentos (queso),
bebidas alcohólicas, la anestesia, las angiografías, la cirugía, fármacos, en otros casos no se
puede determinar el factor desencadenante. Puede existir en algunos casos hipotensión
ortostática.

Más del 50% de los pacientes presenta intolerancia glucídica secundaria al efecto
contrainsular de las catecolaminas. Suele desaparecer tras el tratamiento quirúrgico y no
suele precisar tratamiento con insulina.
Puede existir taquicardia sinusal, bradicardia sinusal, arritmiais supraventriculares y
extrasístoles ventriculares. Como consecuencia del aumento del consumo de oxígeno por el
miocardio inducido por las catecolaminas, puede aparecer angina e infarto. En el ECG
aparecen alteraciones inespecíficas del ST y de la onda T, ondas U prominentes,
crecimiento ventricular izquierdo y bloqueos de rama.

62.- Al referirnos al concepto de estereotipia como síntoma psiquiátrico nos referimos a la


psicopatología de:

a) La orientación.
b) La memoria.
c) La psicomotricidad.
d) El pensamiento.

La estereotipia motora consiste en una o varias secuencias de movimiento de carácter


reiterativo, que no configura un programa de acción específico.

Está presente en trastornos psiquiátricos y neurológicos. Puede ser secundaria a fármacos


antipsicóticos (diskinesia tardía o al uso abusivo de sustancias psicoestimulantes como la
cocaína o anfetamina (punding). Cuando se trata de la reiteración insistente de
pensamientos, acompañada de una viscosidad en el desplazamiento atencional y rigidez
ideativa, se constituye la forma clínica cognitiva, que puede o no acompañar a la motora.
Esta perseveración cognitiva es un signo deficitario o negativo del propio cuadro clínico de
la esquizofrenia. Sin embargo el uso al largo plazo de agentes antipsicóticos típicos puede
generar el síndrome deficitario por neurolépticos que incrementa el fenómeno de la
perseverancia cognitiva y causa un déficit motor simultáneo.

En el trastorno obsesivo compulsivola perseveración cognitiva se presenta bajo la forma


compulsiva de reiteración de pensamientos, que puede generar rituales motores. El
trastorno de Tourette también cursa con estereotipias, ya sean los tics motores o vocales.
La enfermedad de Parkinson o la corea de Huntington también pueden presentar diskinesia
como expresión de eventos de neuroplasticidad. En el Parkinson pueden presentarse en el
curso del tratamiento con L-Dopa, y en el Huntington forman parte del cuadro clínico
coreo-atetósico.

Las estereotipias motoras y cognitivas pueden ser desencadenadas por aumento del tono
dopaminérgico en el estriado, y pueden ser neutralizadas por agentes bloqueantes
dopaminérgicos que actúan en el estriado. Asimismo pueden ser generadas por la
disminución del tono dopaminérgico en el estriado.
Ejemplos de la estimulación dopaminérgica en el estriado lo constituyen sustancias de
abuso como la anfetamina o cocaína, fármacos como L-dopa, apomorfina, enfermedades
como la corea de Huntington y el T. de Tourette.

Ejemplos de bloqueo dopaminérgico en el estriado lo constituyen los neurolépticos típicos


usados al largo plazo que son capaces de generar diskinesia tardía.

-American Psychiatric Association. Diagnostic and Statistical Manual of Mental Disorders.


4th ed. Washington, DC: American Psychiatric Association; 1994.

-Albert, M.S., Levkoff, S.E. & Reilly, C.H. (1992): The delirium symptom interview. An
interview for the detection of delirium in hospitalized patients. Journal of Geriatric
Psychiatry and Neurology, 5, 14-21.

- American Psychiatric Association (2002). Diagnostic and statistical manual of mental


disorders (DSM-IV-TR). Washington, D.C.: American Psychiatric Association.

- Belsky J:K. (1996): Psicología del envejecimiento: Teoría, investigaciones e intervenciones.


MASSON, S.A.

- Berg R, Franzen M., Wedding D. (1990): Exploración del deterioro orgánico cerebral.
Biblioteca de Psicología. Editorial Desclee de Brouwer, S.A., Bilbao.

- Berggren D, Gustafson Y, Eriksson B et al. (1987): Postoperative confusion after


anesthesia in the elderly patients with femoral neck fractures. Anesth Analg; 66: 497-504

- Breitbat W, Rosenfeld B, Roth F et al. (1997): The memorial delirium assessment scale. J
Pain Symptom Manage; 13: 128-137.

63.- Motivo por el cual se administran progestágenos en los últimos 10 a 13 días de un


régimen con estrógenos:

a) Disminuir el riesgo de hiperplasia y neoplasia del endometrio.


b) Disminuir el incremento de peso.
c) Disminuir la mastalgia.
d) Disminuir el riesgo de neoplasia mamaria maligna.

Berek J. (2002) Ginecología de NOVAK. México. Ed. Mc Graw Hill Interamericana. Pag 303
Las mujeres que están recibiendo tratamiento de restitución hormonal durante la
menopausia pueden recibir estrógenos los primeros 25 días de cada mes. Se añade un
progestágeno a menudo acetato de medroxiprogesterona, durante los 10 a 13 últimos días
de este régimen en un esfuerzo por reducir el riesgo de hiperplasia y neoplasia del
endometrio.

64.- El factor predisponente más importante en la mucormicosis (zigomicosis) es:

a) Cetoacidosis diabética.
b) Acidosis metabólica.
c) Desnutrición.
d) Uso de esteroides.

La mucormicosis es una infección infrecuente causada por hongos oportunistas que


pertenecen al orden Mucorales, familia Mucoraceae, cuyas características son invasión
vascular por hifas, lo que determina trombosis e infartos. En general afecta pacientes con
enfermedades subyacentes y produce infecciones severas invasoras y a menudo
fulminantes. Algunas condiciones de riesgo para contraer la mucormicosis son: cetoacidosis
diabética, terapia inmunosupresora, leucemia y linfomas con neutropenia prolongada. Se
describen varias presentaciones clínicas, según su localización anatómica, siendo la forma
rinocerebral la más frecuente, especialmente en pacientes diabéticos descompensados.
Aunque esta infección tiene una alta morbimortalidad, su pronóstico ha mejorado los
últimos años, en base a medidas terapeuticas como la oportuna corrección de los factores
predisponentes, uso de anfotericina B y una cirugía precoz y agresiva. Es muy importante
sospechar esta infección en pacientes con factores predisponentes de manera de hacer un
diagnóstico precoz. (AU).

1. Béjar V, Campomanes L. Manual de práctica de micología. Lima, Universidad Nacional


Mayor de San Marcos 1994.

2. Fernández Guerrero. Casos clínicos: Enfermedades infecciosas. Barcelona, España,


Salvat 1991.

3. Sastre J, Pérez 0 et al. Mucormicosis pulmonar en pacientes leucémicos. Sangre 1994;


39 (1). Barcelona.

4. Levy E, Bia MJ, Isolated renal mucormycosis: case report and review. J Am Soc Nephrol
1995; 5 (12): 2014.

5. Baraia J, Muñoz P. Cutaneous mucormycosis in a heart transplant patient associated with


a peripheral catheter, Eur J Clin Microbiol Infect Dis, sep. 1995; 14 (9); 813-815.

6. Williams JC, Schned AR. Fatal genitourinary mucormycosis in a patient with undiagnosed
diabetes. Clin Infect Dis, sep. 1995; 21 (3): 682-684.

7. Sugar A. Mucormycosis. Clin Infect Dis 1992; 220 (14): 5126.


8. Mandell D, Bennett's. Principles and practice of infectious diseases, cuarta edici6n. New
York 1995.

9. Singh N, G. Invasive gastrointestinal zygomycosis in a liver recipient: case report and


review of zygomycosis in solid organ transplant recipients. Clin Infect Disl mar. 1995: 617-
620

65.- Doce días después de una infección de vías aéreas altas un niño de 12 años de edad
desarrolla debilidad de miembros inferiores , la cual es progresiva en días hasta afectar el
tronco, a la exploración física se encuentra arreflexia atrofia muscular y dolor en
miembros inferiores. El LCR solo muestra proteinuria, el diagnostico más probable es:

a) Parálisis de Bell.
b) Sx. de Guillian Barre.
c) Enfermedad de Charcot Marie Tooth.
d) Enfermedad de Werdning Hoffman.

Resumen:
El síndrome de Guillain-Barré es una serie heterogénea de neuropatías periféricas
mediadas inmunológicamente.

El hallazgo común es la polirradiculopatía de evolución rápida, que se manifiesta


posteriormente a un suceso disparador, que con frecuencia es un proceso infeccioso. Se
manifiesta generalmente con una parálisis motora simétrica con o sin afectación sensorial y
autonómica. Su diagnóstico se basa en el examen electrodiagnóstico. El tratamiento consta
de un tratamiento se soporte y uno específico. Si los pacientes superan la fase aguda, la
mayoría recuperará la función. Sin embargo, la neuropatía puede avanzar tan rápido que
será necesaria la intubación endotraqueal y la ventilación mecánica en las 24 horas
posteriores al inicio del proceso El tratamiento específico se basa en la inmunoglobulina
intravenosa y la plasmaféresis.

Estudios diagnósticos
Las pruebas de laboratorio son de poca ayuda al inicio del proceso. Después de la primera
semana de aparición de los síntomas, el estudio del líquido cefalorraquídeo (LCR) detecta
típicamente elevación de las proteína (mayor de 50 mL/dL), máxima entre la segund y la
cuarta semanas, con escasas células (menor de 10 células mononucleares/dL.. Algunos
pacientes presentan anticuerpos antigangliósidos especialmente GM1 y GM1b.
Los estudios electrofisiológicos son los exámenes más sensibles y específicos para el
diagnóstico con este estudio se demostró una gran variedad de anomalías que indican
desmielinización multifocal, la cual incluye:
1. Velocidad de conducción nerviosa enlentecida.
2. Bloqueo parcial de la conducción motora.
3. Dispersión temporal anormal.
4. Latencias distales prolongadas.

Revista de Posgrado de la VIa Cátedra de Medicina. N° 168 – Abril 2007.

BIBLIOGRAFIA

1. Valls JS, Casademont JP, Berciano BD. Enfermedades de los nervios periféricos. En:
Farreras VP, Rozman.
CR y col. Medicina Interna. 14 ed. Madrid: Ed. Harcourt SA; 2000: Vol 2: 1753-70.
2. Newswanger DL. Guillain-Barré Syndrome. Am Fam Physician [online] mayo 2004 [fecha
de acceso 15 de febrero 2007]; 69(10) URL.
3. Melano Carranza E, Carrillo Maravilla E, Gulias Herrero A y col. Síndrome de Guillain-
Barré en el anciano: un estudio retrospectivo. Arch. Neurocien. (Mex DF)] junio 2004.

66.- Las espondilitis anquilosante se puede acompañar de las siguientes manifestaciones:

a) Uveitis e insuficiencia pulmonar.


b) Uveitis e insuficiencia aórtica.
c) Escleromalacia e insuficiencia mitral.
d) Epiescleritis y estenosis aórtica.

ESPONDILITIS ANQUILOSANTE

La característica principal de la espondilitis anquilosante (EA) es el desarrollo de artritis y


entesitis en la columna vertebral, articulaciones sacroliacas y de las periféricas, las
extremidades inferiores. Las manifestaciones articulares usuales son la uveítis, la
enfermedad inflamatoria intestinal, la aortitis e insuficiencia aórtica, miocarditis, defectos
de conducción y la fibrosis pulmonar. La EA se asocia fuertemente con el HLA-B27.
EspondiloartropatíasPacMG-1.
Parte D Libro 5 Pags. 20 a 23.

67.- Una primigrávida de 31 años de edad. Con 35SDG, acude a consulta quejándose de
dolor y de en los 3 primeros dedos de su mano derecha. Ha tenido estos síntomas
intermitentemente las últimas 2 semanas. Por lo demás, el curso prenatal no ha presentado
complicaciones. El examen físico y neurológico son normales. Cual de los siguientes es el
diagnóstico más probable.

a) Síndrome del tunel del carpo.


b) EVC.
c) Convulsiones.
d) Fractura de muñeca.

Síndrome del túnel carpiano:

El túnel carpiano es un conducto estrecho entre los huesos y los ligamentos de la muñeca. El
nervio que controla la sensación en los dedos y en algunos músculos de la mano pasa a través
de este túnel, junto con algunos tendones de los dedos. Cuando una persona mueve o usa la
mano o la muñeca una y otra vez, los tendones se pueden inflamar y comprimir el nervio
contra el hueso. La presión contra el nervio causa dolor y entumecimiento en la mano y los
dedos. A esto se le llama síndrome del túnel carpiano o CTS por sus siglas en inglés.

Los síntomas de este síndrome incluyen:


• Entumecimiento u hormigueo en los dedos de una o ambas manos, excepto el dedo meñique.
• Dolor en la muñeca que también puede afectarle los dedos y extenderse hasta el brazo.
• Dolor en la mano o en la muñeca que con frecuencia es peor en la noche y temprano en la
mañana.
El síndrome del túnel carpiano puede ser causado por cualquier cosa que produzca una
hinchazón que afecte el nervio; por ejemplo, un quiste en el tendón, o la artritis reumatoide.
Pero en la mayoría de los casos, este síndrome se debe a la inflamación producida por el uso
excesivo de los tendones debido a los movimientos repetitivos de las manos y de los dedos
en una posición con la muñeca doblada. El embarazo, la diabetes, las enfermedades de la
tiroides y el uso de píldoras anticonceptivas aumentan el riesgo de padecer de este
síndrome.

Síndrome del túnel del carpo


Carpal tunnel syndrome
Fisioterapia 2004;26(3):170-85.
68.- El factor predisponente más frecuentemente asociado a casos de aspergilosis
pulmonar invasiva es:

a) Neutropenia severa.
b) Sarcoma de Kaposi.
c) Retinoblastoma.
d) Uso de esteroides.

La entrada de Aspergillus spp. en el organismo se produce por inhalación de las


esporas que se encuentran en el aire, suelo o plantas, por lo que la afectación
pulmonar es la principal presentación de la infección por este hongo, que puede
adoptar 5 formas clínicas diferentes: aspergilosis broncopulmonar alérgica,
aspergiloma, aspergilosis crónica necrotizante, traqueobronquitis invasiva (o
aspergilosis bronquial invasiva) y la aspergilosis pulmonar invasiva (API)1,2. Otras
presentaciones clínicas como la sinusitis, la afectación cutánea y las formas
diseminadas a partir de pulmón que pueden afectar al cerebro, hígado, bazo y
tracto gastrointestinal son mucho menos frecuentes.

Entre las distintas formas de presentación de la infección por Aspergillus spp.,


que afectan a pacientes críticos, la API constituye un reto para los médicos
intensivistas, tanto por las dificultades en su diagnóstico como por la escasa
efectividad de los tratamientos propuestos hasta la actualidad.

Los factores de riesgo de adquisición de una infección por Aspergillus spp. se


reflejan en la tabla 1 tomada del trabajo de Perfect el al3. De esos factores, el
más importante es la profundidad y duración de la neutropenia. En enfermos
inmunocompetentes ingresados en las Unidades de Cuidados Intensivos (UCI),
los pacientes con mayor riesgo de infectarse por Aspergillus spp. son aquellos
que han estado en tratamiento con glucocorticoides.
Bibliografía

1. Soubani AO, Chandrasekar PH. The clinical spectrum of pulmonary


aspergillosis. Chest. 2002;121:1988-99.

2. Lumbreras C, Gavalda J. Aspergilosis invasora, formas clínicas y tratamiento.


Rev Iberoam Micol. 2003; 20:79-89

3. Perfect JR, Cox GM, Lee JY, Kauffman CA, de Repentigny L, Chapman SW, et
al. The impact of culture isolation of Aspergillus species: a hospital-based
survey of aspergillosis. Clin Infect Dis. 2001;33:1824-33

4. Vandewoude KH, Blot SI, Benoit D, Colardyn F, Vogelaers D. Invasive


aspergillosis in critically ill patients: attributable mortality and excesses in
length of ICU stay and ventilator dependence. J Hosp Infect. 2004; 56:269-76

5. Azoulay E, Thiery G, Chevret S, Moreau D, Darmon M, Bergeron A, et al. The


prognosis of acute respiratory failure in critically ill cancer patients. Medicine
(Baltimore). 2004; 83:360-70.

6. Janssen JJ, Strack van Schijndel RJ, van der Poest Clement EH, Ossenkoppele GJ,
Thijs LG, Huijgens PC. Outcome of ICU treatment in invasive aspergillosis. Intensive Care
Med. 1996;22: 1315-22.

69. - Thrombocytopenia that is caused by increased platelet destruction is most closely


associated with which of the following conditions?

a) Combination chemotherapy.
b) Acute leukemia.
c) Systemic lupus erythematosus.
d) Excessive ethanol intake.

El diagnóstico de LEG se hace mediante la identificación de manifestaciones clínicas de


la enfermedad acompañadas de uno o más autoanticuerpos típicos (Tablas 2 y 3).
Tabla 3
Criterios para la Clasificación del Lupus Eritematoso Generalizado
Eritema malar
Eritema discoide
Fotosensibilidad
Ulceras orales
Artritis
Serositis
a. pleuritis
b. pericarditis

Renal:
c. proteinuria persistente > 0,5 g/24 h o +++
d. cilin

Neurológicos:
e. convulsiones
f. psicosis

Hematológicos:
g. Anemia hemolítica con reticulocitosis
h. Leucocitopenia < 4.000 por dos veces
i. Linfocitopenia < 1.500 por dos veces
j. Trombocitopenia < 100.000

10. Inmunológicos:
a) Anti-DNA elevado
b) Anti-Sm
c) Antifosfolípidos: Anticardiolipinas (IgG o IgM) o Anticoagulante
lúpico o VDRL falso positivo por 6 meses.
11. Anticuerpos antinucleares (en ausencia de lupus por drogas)
70.- Se evalúa a un paciente que padece esquizofrenia resistente al tratamiento, la primera
medida a considerar será:

a) Verificar que el paciente estuviera cumpliendo el tratamiento prescrito.


b) Pasar al paciente a un antipsicótico atípico.
c) Uso de la Terapia Electro convulsiva.
d) Ingresar al paciente en un hospital.

Esquizofrenia resistente al tratamiento farmacológico.

(Pharmacological treatment resistant schizophrenia.)

En la práctica clínica son tres los términos para caracterizar a los pacientes
esquizofrénicos que no mejoran con medicación antipsicótica: resistencia al tratamiento,
tratamiento refractario y no respondedor. La resistencia al tratamiento no es sinónima de
cronicidad ni de severidad o gravedad, de tal manera que para que un paciente sea
considerado resistente deben tenerse en cuenta varios puntos: a) si la esquizofrenia es
primaria o secundaria; b) la naturaleza de la misma; c) si hubo o no historia de abuso de
sustancias; d) cumplimiento y tolerancia de los tratamientos, y e) presencia de signos
neurológicos menores.

Los criterios mayoritariamente aceptados para definir la resistencia al tratamiento en la


esquizofrenia fueron desarrollados inicialmente por Kane en 1988. Actualmente para la
evaluación de los niveles de falta de respuesta o resistencia al tratamiento se utiliza la
Brief Psychiatric Rating Scale (BPRS) y la Independent Living Skills Survey (ILSS).

Para alcanzar una evolución terapéutica favorable en ensayos con fármacos antipsicóticos
en pacientes esquizofrénicos resistentes al tratamiento se deben seguir las siguientes
directrices:

Identificar claramente los síntomas y utilizar fármacos en dosificación adecuada y tiempo


suficiente.

Tener en cuenta que la resistencia al tratamiento puede confundirse bien con intolerancia a
la medicación, no adherencia al tratamiento, un inadecuado apoyo social o bien un
tratamiento psicosocial inapropiado.
Agotar la utilización de fármacos solos antes que tratamientos farmacológicos combinados.

Prevenir los efectos extrapiramidales mediante una apropiada elección del tratamiento
primario.

FUENTE: ACTAS ESPAÑOLAS DE PSIQUIATRÍA. 2006 ENE;34(1):48-54.

71.- .- Se trata de paciente femenino de 6 años de edad la cual presenta ceguera nocturna
(hemeralopia) en la exploración oftalmológica se observa constricción del campo visual con
escotoma anular, pérdida de la agudeza y electrorretinograma anómalo asi mismo arterias
estrechadas ¿Cuál de las siguientes patologías es la más probable?

a) Retinoblastoma.
b) Catarata congénita o infantil.
c) Retinosis pigmentaria (retinitis pigmentaria).
d) Persistencia de vítreo primario.

Retinosis pigmentaria. Bilateral, de curso lento y progresivo, comienza en la edad escolar,


pudiendo causar ceguera hacia los 40 años. Es una alteración de los bastones. Puede
presentarse aislada o asociada a otras malformaciones, como la polidactilia, el síndrome de
Laurence-Moon-Bield.
Se hereda bajo diferentes patrones. Clínicamente presentan mala visión nocturna
(hemeralopía) y escotoma anular en el campo visual .Oftalmoscópicamente se aprecian
arterias estrechadas, atrofia de papila y acumulaciones de pigmento que asemejan
osteocitos en la retina periférica. Evolucionan hacia una reducción del campo visual y
finalmente ceguera. Además presentan complicaciones adicionales como aparición temprana
de cataratas y glaucoma.

Retinosis pigmentaria
Retinosis pigmentaria, preguntas y respuestas, Universidad Miguel Hernández de Elche,
2007.
Science Daily (ed.): «Retina Transplants Show Promise In Patients With Retinal
Degeneration» (11-7-2008).

72.- Ante un paciente con diagnóstico de glaucoma el siguiente grupo de fármacos tópicos,
se encuentran contraindicados:

a) Betabloqueantes no selectivos.
b) Betabloqueantes selectivos beta-1.
c) Adrenérgicos alfa-2-agonistas.
d) Parasimpaticolíticos.

http://www.bvs.sld.cu/revistas/mgi/vol16_3_00/mgi14300.htm
Se anexa dirección para para revisión de artículo.

DEFINICIÓN:
Fármacos que inhiben las acciones de la acetilcolina debidas a la activación de receptores
muscarínicos. Sólo los compuestos derivados de amonio cuaternario interfieren con las
acciones de acetilcolina en los receptores nicotínicos.
2. CLASIFICACIÓN:
2.1. Alcaloides naturales:
* Atropina y Escopolamina
2.2. Derivados sintéticos y semisintéticos
* Aminas terciarias: Ciclopentolato, Tropicamida, Pirenzepina, Homatropina
* Aminas cuaternarias: bromuro de ipratropio, metilbromuro de escopolamina,
metilbromuro de homatropina, bromuro de propantelina, bromuro de metantelina.
3. ALCALOIDES NATURALES:
3.1. RELACIÓN ESTRUCTURA-ACTIVIDAD: Son ésteres orgánicos formados por la
combinación de un ácido aromático (ác. trópico) y una base orgánica (tropina o escopina).
3.2. MECANISMO DE ACCIÓN: Son antagonistas competitivos de los receptores
muscarínicos.
3.3. PROPIEDADES FARMACOLÓGICAS:
3.3.1. SNC:
Atropina. A dosis terapéuticas produce excitación vagal leve. Si se aumenta mucho la dosis,
la excitación central aumenta (inquietud, irritabilidad, desorientación, alucinaciones) y
puede ir seguida de depresión, colapso circulatorio, fallo respiratorio, parálisis y coma.
Escopolamina. A dosis terapéuticas produce depresión del SNC (somnolencia, amnesia,
fatiga y sueño). También produce euforia, lo que puede originar abuso. Con altas dosis
aparecen efectos excitatorios.

3.3.2. OJO:
Bloquean los efectos de acetilcolina sobre el músculo esfínter del iris y el músculo ciliar del
cristalino, originando midriasis (dilatación pupilar) y cicloplejia (parálisis de la acomodación)
de larga duración. Los efectos de escopolamina (a igualdad de dosis) son mayores que para
atropina. El incremento de la presión intraocular sólo es importante en pacientes con
glaucoma de ángulo agudo.

APLICACIONES TERAPÉUTICAS

5.2. OFTALMOLOGÍA:
- exploración de cristalino y fondo de ojo
- tratamiento de la queratitis e iridociclitis
- algunos tipos de estrabismo
¡Contraindicados en ancianos y pacientes con glaucoma de ángulo estrecho!

REFERENCIAS BIBLIOGRÁFICAS

1. Vaughan D, Taylor A, Khalid FT. Glaucoma. En: Oftalmología general. México, DF:
Editorial El Manual Moderno, 1991:193-203.
2. Cuba. Ministerio de Salud Pública. Guía terapéutica. La Habana: Editorial Orbe,
1981.
3. . Guía terapéutica para la atención primaria de salud en Cuba. La Habana: Editorial
José Martí, 1994.
4. Rosenstein SE. Diccionario de especialidades farmacéutica. 41 ed. México, DF: DEF,
1995.
5. Vademecum Internacional: Medicom. 34 ed. Madrid: 1993.

73.- En relación con el cáncer de endometrio ¿cuál de las siguientes respuestas es la


verdadera?

a) La contracepción oral durante más de tres años es un factor de riesgo.


b) El tratamiento con tamoxifeno es un factor de riesgo.
c) La multiparidad es un factor de riesgo.
d) Es menos frecuente que el cáncer de ovario.
El tamoxifeno es un tratamiento hormonal muy utilizado en el cáncer de mama.Cuanto más
se prolonga su uso en el tiempo, mayor riesgo existe de padecer cáncer de endometrio y de
grados más avanzados y peor pronóstico.

El tamoxifeno viene siendo en los últimos veinte años el tratamiento hormonal de cáncer de
mama precoz y del cáncer de mama metastásico. A pesar de sus buenos resultados tiene
como efectos secundarios principales un mayor riesgo de tromboembolismo pulmonar,
trombosis venosa profunda y 2 a 7 veces mayor riesgo de cáncer de endometrio. En este
último caso el riesgo aumenta cuanto mayor es el tiempo de uso del tamoxifeno.

Aunque se sabe que el tamoxifeno aumenta el riesgo de cáncer endometrial, pocos estudios
han determinado la relación entre el riesgo y pronóstico del cáncer de endometrio con la
dosis, duración y uso reciente del tamoxifeno.

El departamento de Epidemiología del Instituto del Cáncer holandés ha realizado un


estudio de casos y controles nacionales que incluía a 309 mujeres con cáncer de endometrio
tras haber padecido un cáncer de mama y 860 con cáncer de mama pero sin cáncer
endometrial, todas tratadas con tamoxifeno.

Como conclusión de este estudio se confirma que cuanto más prolongado es el tratamiento
con tamoxifeno mayor es el riesgo de cáncer de endometrio y peor pronóstico tiene ya que
se dan estadíos más avanzados del tumor. En el caso de mujeres con cáncer de mama
compensa el uso del tamoxifeno , que mejora de forma importante la supervivencia y reduce
la incidencia del cáncer en la mama contralateral, frente al aumento del riesgo del cáncer
endometrial. No ocurre lo mismo con las mujeres sanas ,en ellas no está muy claro que sea
beneficioso el uso del tamoxifeno como prevención del cáncer de mama por los riesgos que
conlleva el tratamiento.

FUENTE:

" Risk and prognosis of endometrial cancer after tamoxifen for breast cancer"

Liesbeth Bergman, Maureen L R Beelen, Maarten PW Gallee, Harry Hollema, Jantien


Benraadt, et al.

THE LANCET. Vol 356. September 9, 2000.


74.- En relación con la gonadotropina coriónica humana durante el embarazo ¿cual de las
siguientes considera correcta?
a) Tiene un máximo alrededor de la 10ª semana.
b) Tiene su valor mínimo al final del embarazo.
c) Comienza a aumentar de forma evidente a partir de la 30ª semana.
d) Aumenta hasta la 10ª semana y luego se mantiene constante hasta el parto.

La gonadotropina coriónica humana es una glucoproteína sintetizada en las células del


sincitiotrofoblasto de la placenta, formada por dos cadenas una & y otra ß. Aumenta en
sangre y orina poco tiempo después de la implantación y sirve para pruebas de embarazo.
Actualmente se la denomina hCG por las siglas de su nombre en inglés: Human Chorionic
Gonadotropin.

La hormona tiene como objetivo mantener la funcionalidad del cuerpo lúteo como ente
endócrino en la secreción de progesterona durante el primer trimestre de embarazo.

La hCG es la base histórica y actual del diagnóstico de embarazos, su diferenciación de los


falsos embarazos que pueden constituirse en tumores así como de los cánceres de próstata,
molahidatiforme y cariocarcinoma.

Los valores esperados para este procedimiento en el periodo de gestación son los
siguientes:

SEMANAS DE EMBARAZO RANGO mIU/ml.

3 –4 9 -130

4 – 5 75 – 2,600

5 -6 850 –20,800

6 – 7 4,000 –100,200

7 – 12 11,500 –289,000

12 –16 18,300 – 137,000

16 –29 1,400 – 53,000

29 –41 940 – 60,000


Reference values in adults.

MeSH: HEMATOLOGIC TESTS, REFERENCE VALUES, LABORATORY TEST, CLINICAL


CHEMISTRY TESTS.

75.- Un estudiante universitario de 20 años de edad acude a consulta debido a tos seca,
fiebre, cefalea y dolor muscular durante las últimas 2 semanas. Refiere que sus
compañeros de casa han desarrollado síntomas similares. Niega el uso de drogas ilícitas y
no es homosexual. Su temperatura es de 38.2°C, FC 90lpm, FR 18x’. Se auscultan murmullo
vesicular bilateral. Una RX de tórax muestra opacidades intersticiales multifocales.
Presenta leucocitosis y un test de aglutinina frío +. ¿Cual de los siguientes es el patógeno
más probable?

a) Influenza virus.
b) Bacterias anaeróbicas.
c) Mycoplasma pneumoniae.
d) Pneumocystis carinii.

La neumonía es la principal manifestación clínica de la infección por este microorganismo y


también es el síndrome mejor descrito en la infección por Mycoplasma. La neumonía es de
inicio incidioso, con síntomas constitucionales que aumentan hasta la aparición de la tos (2 ó
4 días después del inicio de la enfermedad) que se convierte en el síntoma cardinal y se
caracteriza por ser seca, aunque en ocasiones también productiva (hemoptoica,
mucopurulenta o mucoide), hay dolor torácico retroesternal, sin embargo el dolor pleurítico
franco es raro, la fiebre varía de 38 a 40 grados. El cuadro clínico se puede acompañar de
coriza, odinofagia y otalgia (por miringitis bullosa en la semiología torácica por lo general se
limitan a la ausculturación de estertores, roncus y sibilancias, aunque en muchas ocasiones
el examen es normal, en algunos casos la consolidación lobar caracterizada por el aumento
de la matidez en la percusión, disminución del murmullo vesicular, estertores, soplo
tubárico y broncofonía de la zona afectada1,2,4. Asociado a este cuadro clínico podemos
encontrar mialgias, artralgias, vómito. En el 15% de los casos, erupciones cutáneas
(masculares, eritematosas, maculopapulares, vesiculares), y síndrome nodoso, urticaria,
pitiriasis rosada, y síndrome de Steven-Johnson2. Desde el punto de vista extrapulmonar
también puede existir compromiso del SNC (meningoencefalitis, encefalitis, Síndrome de
Guillan-Barré, psicosis, ataxia, radiculopatías)1,2,4.
A nivel articular poliartritis en rodillas, codos, tobillos y hombra1,2,4, también se ha
reportado pericarditis y miocarditis hasta en un 4,5% de los infectados1,4. La hemólisis
intravascular ocurre generalmente en pacientes con títulos altos de crioaglutininas1,4, otras
complicaciones vasculares incluyen (coagulación intravascular diseminada) CID y fenómeno
de Raynaud. También s e han documentado, casos de hepatitis, pancreatitis, nefritis,
polimiositis, púrpura trombocitopénica1,4; algunas en un marco séptico producto de la
infección por este microorganismo.
Rayos X de tórax: Presenta hallazgos variables desde exámenes normales hasta
infiltrados intersticiales, paracardíacos, retículonodulares, mixtos lobares o multilobares o
francamente condensaciones lobares basales y efuciones pleurales en un 20-25% de los
casos, sin embargo es importante recordar la pobre correlación existente entre los
síntomas referidos, la clínica encontrada, los hallazgos pulmonares y al radiografía de
tórax.

Exámenes paraclínicos de "rutina": El cuadro hemático generalmente presenta un


recuente leucocitario normal, ocasionalmente leucocitos o leucopenia leve, el diferencial es
normal o con predominio de linfocitos, es frecuente la trombocitosis y aumento de la VSG
(velocidad de eritrosedimentación), anemia hemolítica en ocasiones correlacionada con
niveles altos decrioagutininas2. El Coombs directo puede ser positivo, según el nivel de
crioaglutininas circulantes. El parcial de orina revela en ocasiones el albuminuria
(relacionada con la fiebre), el estudio del líquido cefalorraquídeo (L.C.R.) puede revelar
pleocitosis de meningoencefalitis1,4.

76.- Paciente masculino de 39 años de edad, alcohólico conocido, se presenta al


servicio de urgencias por dolor en epigastrio de 12 horas de duración. El
episodio surgió posterior a la ingesta de alcohol, seguido de náuseas, vómito y
fiebre. Los hallazgos abdominales incluyen signos de irritación peritoneal,
rigidez muscular, disminución de los ruidos peristálticos, distensión abdominal,
y hallazgos rectales normales. Los hallazgos de laboratorio incluyen leucocitosis
y aumento de la amilasa y lipasa séricas. Sus radiografías muestran pocas asas
dilatadas.¿Cuál puede ser el diagnóstico?

a) Gastroenteritis.
b) Pancreatitis aguda.
c) Perforación de úlcera péptica.
d) Ruptura de una aneurisma aórtica.
CLINICA
Dolor abdominal (95-100%) de instauración súbita, que se intensifica rápidamente hasta
alcanzar su pico de unos minutos a una hora, continuo, y se localiza en epigastrio, donde
queda fijo o se irradia a uno o ambos hipocondrios, espalda, zona periumbilical y en
cinturón.. A la exploración abdominal, el dolor a la palpación en epigastrio siempre aparece y
con frecuencia se percibe sensación de ocupación o empastamiento a ese nivel o incluso una
masa epigástrica en algunos casos, a veces con defensa pero sin claros signos de reacción
peritoneal. Los ruidos intestinales son escasos o están ausentes.
Ictericia (25-30%) ó rubicundez facial, relacionada esta última con la liberación de
histamina y otras sustancias vasoactivas. En los casos graves, el compromiso hemodinámico
es evidente, con hipotensión, taquicardia o incluso shock establecido
Vómitos y náuseas están presentes 70-90% de los casos y suele haber detención del
tránsito intestinal desde el comienzo del cuadro
Distensión abdominal (60-70%)
Febricula (70-85%)
Hematemesis (5%)
Hipotensión (20-40%)
Cuadros confusionales (20-35%). El dolor suele no aparecer generalmente en los más
graves, y el cuadro clínico se ve dominado por los vómitos, síntomas de manifestaciones
sistémicas, shock o por la descompensación cetoacidótica, sin causa aparente, en un
diabético.

. En los casos graves también están presentes la Disnea (<10%) taquipnea, los signos de
insuficiencia cardiaca, la confusión mental o la tetania, todo lo cual expresa el compromiso
multisistémico. Las equimosis abdominales de localización periumbilical (signo de Cullen)
(<5%) o en los flancos (signo de Gray-Turner) (<5%) son muy características, de la
pancreatitis aguda así como manifestaciones tardías de la enfermedad, generalmente en
extremidades inferiores, con aspecto similar al eritema nudoso o a la paniculitis supurada.
(Necrosis grasa subcutánea (<1%)
IV PRUEBAS COMPLEMENTARIAS ( En Urgencias)
1-Hemograma Leucocitosis de moderada intensidad hasta encontrase reacciones
leucemoides
Hemoglobina y Hematocrito, encontrándose al inicio elevado y posteriormente en relación a
reposición de líquidos, podemos encontrar niveles inferiores.
2-Bioquimica -Hiperglucemia
-Hipocalcemia: Un descenso marcado es indicador de mal Pronóstico
-Hiperamilasemia
-Hiperbilirrubinemia (en los casos graves y en obstrucción vía biliar)
Rx simple Abdomen - Datos para Diagnostico Pancreatitis y or otro lado nos ayuda en el
diagnóstico diferencial con otros procesos (perforación víscera hueca)
Rx de Tórax - Datos de Derrame pleural, Atelectasia, etc...
Ecografía abdominal
- Es técnica de elección para diagnóstico de litiasis biliar (causante de
PA) y para detección y seguimiento de Pseudoquistes pancreáticos
- No existe relación entre los hallazgos ecográficos y gravedad clínica
- Visualiza el Páncreas en un 60 -70 % correctamente. (Interposición de gas)
TAC - Más sensible y especifica que la ecografía.
- Técnica de elección para diagnóstico de PA y Necrosis Pancreática.
- No todos los pacientes con pancreatitis aguda, precisan TC, ya que la indicación se hace
en función de la gravedad

77.- Femenino de 38 años con diagnóstico de endometriosis, en ésta patología podemos


encontrar elevación sérica de:

a) CA-125.
b) Tc 99.
c) AFP.
d) TGP.

Beck W. (1997) NMS. Obstetrics and Gynecology. USA. Williams & Wilkins. Pag 278. Se
encuentra elevación sérica de CA-125 en pacientes con endometriosis, se trata de un
antígeno de superficie encontrado en derivados del epitelio celómico, incluyendo el
endometrio, se correlacionan sus niveles con el grado de enfermedad y la respuesta a
tratamiento, pudiendo ser un marcador de recurrencia de la enfermedad.

78.- Masculino de 43 años con diagnóstico de isosporosis el tratamiento de elección es:

a) Trimetoprim sulfametoxazol.
b) Albendazol.
c) Metronidazol.
d) Pentamidina.

ISOSPOROSIS o ISOSPORIASIS

Isospora belli representa un problema de salud en personas inmunocomprometidas. Su


distribución es cosmopolita, aunque es más común en zonas tropicales y subtropicales,
entre ellas Haití, El Salvador, Brasil, México, Africa tropical, Este Medio y el sudeste de
Asia.
Ciclo biológico.

Al igual que los otros coccidios, tiene un un ciclo de vida complejo. El ooquiste inmaduro
(forma diagnóstica), de forma oval y translúcido, mide 22 - 33 µm y contiene un cuerpo
esférico, el esporoblasto, el cual se divide posteriormente en dos.

Una vez eliminado con las heces fecales, en ambientes adecuados se desarrollan 2
esporoquistes, cada uno con 4 esporozoítos móviles (quiste maduro, forma infectante).

El protozoo habita en intestino delgado.

Mecanismo de transmisión: ingesta de agua y alimentos contaminados con ooquistes


esporulados.

Cuadro clínico.

En pacientes inmunocompetentes, generalmente se presentan cuadros diarreicos limitados;


en infantes y preescolares, puede presentarse una enfermedad severa, con diarrea acuosa,
sin sangre, esteatorrea, cefalea, fiebre, dolor abdominal, vómito y pérdida de peso.

Algunos sujetos presentan eosinofilia. Puede haber recaídas, relacionadas probablemente


con ooquistes extraintestinales en reposo, no susceptibles al tratamiento.
Es una enfermedad oportunista asociada al inmunocompromiso (SIDA, por ejemplo), con
frecuencia con caracter crónico; la diarrea, más abundante y líquida puede conducir a
deshidratación y desequilibrio hidroelectrolítico importantes, requiriendo de
hospitalización. Los pacientes con antecedente de infección por Pneumocystis jiroveci y
sujetos a tratamiento preventivo con trimetoprim-sulfametoxazol estarán recibiendo a un
tiempo tratamiento contra la isosporosis y las recaídas.

Existen reportes de la enfermedad extraintestinal en pacientes con SIDA, involucrando


pulmones, hígado,vías biliares, bazo, ganglios linfáticos, colon.
Histopatológicamente se observan atrofia de microvellosidades, infiltrados de eosinófilos y
desorganización del epitelio en las criptas y en la superficie.
Diagnóstico de laboratorio.

I. belli. I. belli en heces


I. belli. Examen
Esporoblasto. fecales.
en fresco
Tinción ácida. Esporoblasto. Tinción
Chiang Mai
Chiang Mai ácida
University,
University, S.J. Upton, Kansas
Thailand
Thailand University

Se fundamenta en la observación microscópica de los ooquistes no esporulados; su


eliminación con la materia fecal es irregular y en poca cantidad, por lo que son necesarios
varios exámenes.

Las preparaciones en fresco son útiles en la visualización de los ooquistes cuando se


emplean contraste diferencial, epifluorescencia (detección de autofluorescencia).

Son de mayor utilidad los métodos de concentración y posterior tinción de los extendidos.
Las técnicas empleadas son Ziehl-Neelsen modificado, safranina-azul de metileno,
auramina-rodamina (costoso).

Métodos tales como el aspirado duodenal, Enterotest y la biopsia intestinal son útiles
también, cuando los exámenes coproparasitoscópicos son negativos y existe evidencia
clínica suficiente para ameritar su empleo.

Tratamiento.

Trimetoprim-Sulfametoxazol y tratamiento de sostén.


En patientes con SIDA y otras patologías que producen inmunosupresión, el tratamiento se
continúa en las recaídas con Trimetoprim-Sulfametoxazol, 2-3d/ semana/3 semanas. Es
muy importante el tratamiento antirretroviral.
Isosporiasis. eMedicine. Feb 6, 2008.

Oliveira-Silva MB, et al. Seasonal profile and level of CD4+ lymphocytes in the occurrence
of cryptosporidiosis and cystoisosporidiosis in HIV/AIDS patients in the Triângulo Mineiro
region, Brazil. Rev. Soc. Bras. Med. Trop. [serial on the Internet]. 2007 Oct [cited
2008 Feb 10]; 40(5):512-515.

Isosporiasis. DPDx.
Bennett JA, Krafczyk T, Thomas JG. Canfield PM. Case study. A 36-Pound Weight Loss
and Debilitating Painful Rash in a 42-Year-Old Male. Labmedicine, Aug 2004;35(8):484 -
487.

79.- El agente causal más frecuentemente encontrado como causa de Nocardiosis es:

a) Nocardia Brasiliensis.
b) Nocardia Asteroides.
c) Nocardia Farcinica.
d) Nocardia Otitidis Cavarum.

La nocardiosis es una enfermedad infecciosa relativamente poco común, con una frecuencia
anuale stimada entre 500 y 1000 casos por año en los Estados Unidos de América. Las dos
especies más prevalentes son Nocardia asteroides y N. brasiliensis (10) y pueden causar un
amplio espectro de manifestaciones clínicas localizadas y diseminadas. El complejo N.
asteroides ha sido considerado el responsable de la mayoría de las infecciones invasivas
severas tales como neumonía y abscesos cerebrales Post-traumatismo, diseminándose
frecuentemente a otros sitios en pacientes inmunocomprometidos (3, 8). En Contraste, N.
brasiliensis está generalmente asociada a infecciones de piel y partes blandas,
comunicándose pocos casos de infecciones disemina- das (6, 8).
Aquí se describe el caso de una niña de 4 años de edad previamente sana, que presentó
linfadenitis necrotizante postraumática causada por N. asteroides y que evolucionó
satisfactoriamente con cefalotina.
Dr. Alejandro Bonifaz, Micología Médica Básica, Editorial: Méndez Cervantes, Edición: 1ª-
1991. Páginas 389-395

80.-El dolor abdominal intermitente, la anemia, la proteinuria, la necrosis tubular, la


esterilidad, los trastornos de la conducta, la polineuropatía periférica se encuentran en la
intoxicación por:

a) Organoclorados.
b) Sulfuro de hidrogeno.
c) Plomo.
d) Etilenglicol.

La intoxicación por plomo es la más común de las exposiciones a metales, el cual tiene
muchos usos, las fuentes más frecuentes vienen de las minas y del reciclado de materiales
conteniendo plomo. Este metal es absorbido por pulmones y del tracto gastrointestinal. El
mecanismo de acción es por unión a los grupos sulfhidrilo y tóxico para las enzimas
dependientes de zinc. Diagnóstico: La toxicidad aguda se presenta luego de una exposición
respiratoria a altas concentraciones, con encefalopatía, insuficiencia renal y síntomas
gastrointestinales. La toxicidad crónica es la más frecuente y se manifiesta con
compromiso multisistémico: hematopoyético, del sistema nervioso, gastrointestinal, riñón y
sistema reproductor. (Astenia, dolor abdominal, irritabilidad, náusea, vómitos, pérdida de
peso, cefalea, anemia, neuropatía periférica, ribete de Burton, IRC, proteinuria, Nefritis
intersticial,etc). En los exámenes auxiliares podemos encontrar anemia, punteado basófilo,
aumento del ácido úrico, etc.

REFERENCIA BIBLIOGRÁFICAS

1. Keogh JP y Boyer LV. «Lead» en Sullivan y Krieger editores: Clinical Environmental


Health and Toxic Exposures. Lippincott Williams & Wilkins, 2da edición, 2001
2. Decreto Supremo N° 019-98-MTC. Dispone eliminar del mercado la oferta de gasolina 95
RON con plomo y reducir el límite máximo de contenido de plomo en la gasolina 84 RON.
(14/07/98)
3. Shannon Michael. «Lead» en Haddad, Shanon y Winchester editores: Clinical
Management of Poisoning and Drug Overdose. WB Saunders, 3ra edición, 1998.
4. Krantz A, Dorevitch S. Metal exposure and common chronic diseases: A guide for the
clinician. Dis Mon 2004; 50:215- 262.
5. Nogué S. Burton’ s Line. N Engl J Med 2006; 354:e21.
6. Rempel D. The lead-exposed worker. JAMA 1989; 262:532-4.

81.- El tratamiento de elección del cáncer papilar de tiroides es:

a) Tiroidectomía total y yodo radioactivo.


b) Hemitiroidectomía y radioterapia externa.
c) Quimioterapia y radioterapia.
d) Yodo radioactivo.

El tratamiento para el cáncer papilar usualmente incluye:

Cirugía - para extirpar la glándula total o parcialmente (tiroidectomía).


Terapia de hormona de la tiroides - para suprimir la secreción por la glándula pituitaria de
más hormona estimulante de la tiroides, la cual puede estimular la recurrencia del cáncer
papilar.
Administración de yodo radioactivo - para destruir cualquier remanente de tejido tiroideo.

Bibliografía
1.Sherman, S.I., Thyroid carcinoma. Lancet, 2003. 361(9356): p. 501-11.
2.Jemal, A., et al., Cancer statistics, 2004. CA Cancer .J Clin, 2004. 54(1): p.8-29.
3.Fardella, C., Jimenez M, González H, León A, Goñi I, et al. Características de
presentación del microcarcinoma papilar del tiroides: Experiencia retrospectiva de los
últimos 12 años.
Rev. Méd. Chile, 2005. 133: p. 1305- 1310.
4.Mosso L, Jimenez M, González H, Solar A, Torres J, Fardella C. Microcarcinoma tiroideo
de evolución agresiva. Rev. Méd. Chile. 133: 232- 326, 2005.

82.- Cual de las siguientes es una aseveración cierta en relación al osteosarcoma:

a) Es muy raro que produzca metastasis.


b) Lo más común es que aparezca en la metafisis distal del fémur y proximal de la
tibia.
c) Es mas frecuente en mujeres en la segunda y tercera decada de la vida.
d) Se ha asociado a traumatismos.
El osteosarcoma se define como un tumor mesenquimal maligno en el que las células
cancerosas producen matriz ósea. Es el tumor óseo maligno primario más común, a
excepción del mieloma y el linfoma, y representa el 20% de los tumores óseos primarios. El
osteosarcoma se produce en todos los grupos de edad, pero su distribución es bimodal; el
75% de los casos se produce en pa-cientes menores de 20 años. El segundo grupo más
pequeño se produce en ancianos, que frecuentemente tienen patologías que se saben están
asociadas al desarrollo de osteosarcoma. Su localización más frecuente es la región
metafisiaria de los huesos largos, preferentemente en miembros inferiores (metafisis
distal del fémur y proximal de la tibia). Se caracteriza por un crecimiento rapido del
tumor con extensión locore-gional y difusión metastásica a pulmón; generalmente, 20% de
los pacientes presentan metástasis en el momento del diagnóstico

BIBLIOGRAFIA

• KUMAR Vinay, Abbas Abul, Fausto Nelson, “Patología funcional y estructural”, Madrid-
España, Elsevier, 7ma Edición, Pag.1297-1298. • Documento “Osteosarcoma”. • GOODMAN.
Snyder. “Patología médica para fisioterapeutas”. España. McGraw-Hill Interamericana. 3ra
Edición. 2002. Pag. 368. • PICCI P. Sangiorgi, P. Calroda, L.. Benassi M. S y Campanacci M.-
“Histo-patologia del osterosarcoma”- Centro de Tumores Óseos. Instituto Ortopé-dico
Rizzoli, Bolonia, Italia.

83.- ¿Cuál de las siguientes aseveraciones respecto a la hipertensión renovascular es


correcta?

a) Se relaciona con mayor liberación de renina.


b) Se ve a menudo en jóvenes como complicación de enfermedad fibromuscular.
c) Es fácil de controlar.
d) Rara vez produce hipertensión grave.

Allen R. M. MMS Medicina Interna. 5ª. Edición. National Medical Series. Mc. Graw Hill.
2006. (capítulo 6, parte I: XIV C, parte III: II B 1). La hipertensión renovascular se
relaciona con un incremento en la liberación de renina. La disminución del flujo sanguíneo
renal estimula la liberación renal de cantidades en aumento de renina, que activan el
sistema renina-angiotensina-aldosterona y producen hipertensión. El captopril (IECAS)
impide la conversión de angiotensina I a angiotensina II (un vasoconstrictor que se
esperaría fuera de especial eficacia en esta enfermedad).
Las mujeres jóvenes, mucho más a menudo que los varones jóvenes, presentan obstrucción
fibromuscular de las arterias renales, lo que provoca hipertensión reno-vascular.
Típicamente ésta causa hipertensión muy grave y es notablemente difícil de controlar; este
hecho hace sospechar que hay hipertensión renovascular.

84.- ¿Cuál de las siguientes aseveraciones describe mejor a los pacientes con apnea
obstructiva durante el sueño?

a) Tienen diferentes presentaciones clínicas que los sujetos con otras formas de apnea
durante el sueño.
b) Responden bien al tratamiento con estimulantes respiratorios.
c) Responden bien a la presión positiva continua en las vías nasales.
d) No tienden a quedarse dormidos durante el día.

Allen R. M. MMS Medicina Interna. 5ª. Edición. National Medical Series. Mc. Graw Hill.
2006. (capítulo 2 XV B-E). Los pacientes con apnea obstructiva evolucionan bien con pre
positiva continua por vía nasal (CPAP), que actúa como férula de la parte posterior
obstruida de la faringe personas con apnea durante el sueño, ya sea central, obstructiva, o
mixta, son indistinguibles desde el pi de vista clínico; se requieren estudios del sueño para
averiguar qué tipo de apnea está presente. Aun cuado algunos pacientes con apnea
obstructiva pueden mejorar con estimulantes respiratorios, otros requieren m das más
drásticas. Los enfermos con todas las formas de apnea tienen somnolencia durante el día y,
por razones desconocidas, están en riesgo de sufrir cor pulmonale.

85.- Contraindicación absoluta para el remplazo hormonal en la postmenopausia:

a) Hipertrigliceridemia > 400mg/dl.


b) Colecistitis aguda.
c) Cáncer de mama y enfermedad hepática activa.
d) HTAS.

Braunwald, Eugene et al (2006). Harrison’s principles of internal medicine. New York:


McGraw-Hill. Pag 2212. Son contraindicaciones absolutas para el reemplazo hormonal,
sangrado vaginal de causa desconocida, enfermedad hepática activa, tromboembolismo
venoso o historia de cáncer endometrial. La hipertrigliceridemia y la enfermedad vesicular
activa son contraindicaciones relativas.
86.- Factores de riesgo en el recién nacido para hemorragia suprarrenal:

a) Macrosómicos y nacidos por parto de nalgas.


b) Macrosómicos y nacidos por cesárea.
c) Prematuro y nacidos por parto de nalgas.
d) Prematuro y nacidos por cesárea.

Manual CTO Pediatría, p.1295.


La hemorragia suprarrenal afecta a productos macrosómicos nacidos por parto de nalgas y
se manifiesta con anemia, hipoglucemia, trastornos electrolíticos y anemia.

87.- Recién nacido de 32 semanas de gestación, que presenta de manera súbita hipotensión,
anemia y abombamiento fontanelar. A al EF presenta una FC de 100 x’, una FR de 20 x’ y
comienza a convulsionar ¿Cuál es el diagnóstico más probable?

a) Hemorragia de la matriz germinal.


b) Leucomalacia periventricular.
c) Hidrocefalia.
d) Hipotiroidismo congénito.

Manual CTO pediatría, 7° edición p.1295


La matriz germinal es una estructura del cerebro inmaduro hasta la semana 34 de
gestación. Su lesión produce hemorragia. Se manifiesta con deterioro del estado general,
aparición de pausas de apnea, bradicardia, hipoventilación, convulsiones, disminución del
reflejo del Moro y fontanelas a tensión. Debe sospecharse ante un paciente pretérmino que
presenta de forma súbita anemia, hipotensión y abombamiento fontanelar. Puede derivar en
la aparición de leucomalacia periventricular.

88.- ¿Cuál de los siguientes es un factor de riesgo que requiere la realización del test de
O‘Sullivan al comienzo del embarazo?

a) IMC<19.

b) Gestante menor de 25 años.

c) Glucemia en ayunas >126 mg/dl en dos ocasiones.

d) Infección de vías urinarias de repetición.


McPhee S, Papadakis M, Tierney L. Currrent Medical diagnosis and treatment. McGraw
Hill 2007 p. 803.

Los factores de riesgo que requieren la realización del test de O’Sullivan son: obesidad
(IMC>30), padres o hermanos con DM, hipertensión crónica, preeclampsia, candiadiasis,
IVU’s de repetición, antecedentes de diabetes gestacional o intolerancia a la glucosa, y
antecedentes obstétricos: abortos de repetición, fetos muertos, muerte neonatal no
explicada, prematuridad, malformaciones congénitas, hidramnios, macrosomía fetal. La
detección de una glucemia basal > 126 mg/dl en 2 ocasiones hace el diagnóstico de diabetes
gestacional directamente, sin necesidad de realizar el test, al igual que dos valores al azar
de 200 mg/dl. (CTO 665, 666).

89.- Paciente masculino de 52 años con diagnóstico de esquizofrenia desde hace 25 años,
tratado con diversos neurolépticos en dosis elevadas. Hace dos años inició con movimientos
repetitivos de labios y lengua, y movimientos coreoatetósicos de piernas y tronco que no
han mejorado a pesar de múltiples fármacos. ¿Cuál de estos factores es predisponente de
discinesia tardía?

a) Administración prolongada de neurolépticos y anticolinérgicos.


b) El diagnóstico de Esquizofrenia.
c) Una distimia agregada.
d) Una probable neoplasia cerebral.

Kaplan H I, Sadock B J. Sinopsis de Psiquiatría. Panamericana. 8° edición. Madrid 2000. p.


711. Los factores predisponentes son la edad avanzada, existe mayor probabilidad en
pacientes mayores de 50 años, y la administración prolongada de neurolépticos y
anticolinérgicos, que ocurre varios meses o años después de iniciado el tratamiento con
agentes del bloqueo de la dopamina. Se cree que es por aumento de las concentraciones
sanguíneas circulantes de dopamina, aumento de liberación de dopamina desde las neuronas
dopaminérgicas, aumento de la capacidad de reacción a la dopamina a nivel de sitios de
receptores postsinápticos y síntesis de nuevos receptores de dopamina de bajo umbral.

90.- El tratamiento de elección en el síndrome de hiperinfección por Strogyloides es:

a) Triclabendazol.
b) Prazicuantel.
c) Ivermectina.
d) Metronidazol
El tratamiento de strongyloides se dirige a la erradicación de la infección.Estudios
recientes han demostrado un tratamiento efectivo con el uso de Ivermectina100 a 200
mcg/kg/día, 2 días. V.O. por lo que es el fármaco de elección en la estrongiloidosis no
complicada, presentando menos efectos secundarios queel tiabendazol 50mg xkg divididos
en 2 tomas v.o. x 2 días. El tratamiento en hiperinfección debe prolongarse de 7 a 10 días.

Nelson Tratado de Pediatria Escrito por Richard E. Behrman, Robert M. Kliegman,Hal B.


Jenson pag. 1160 -2004.

91.-Masculino de 39 años diagnosticado de esclerodermia La siguiente es una manifestación


patognomónica de esta enfermedad:

a) Síndrome de “asa ciega”


b) Pseudoobstrucción intestinal.
c) Divérticulos de “boca ancha” en el colon.
d) Disfunción del esfínter esofágico inferior.

Definición:
Enfermedad adquirida, rara, crónica multisistémica caracterizada por engrosamiento y
fibrosis de la piel, compromiso digestivo, pulmonar y vascular ( F. De Raynaud)

Epidemiología:
Baja Incidencia: Chile 4-12 pac/millón hab. En USA 19 casos/millón

Relación-mujer___hombre=__15-20:1
Mayor incidencia entre 2° y 5° decada.

CLASIFICACIÓN
Esclerodermia sistémica difusa.

Esclerodermia sistémica limitada (CREST)

Eslerodermia Localizada (Morfea y Esclerodermia Lineal)


Sd. De Sobreposición (EMTC)

Mesenquimopatía Indiferenciada.

Sd. Esclerodermia like.

Desordenes asociados al ambiente (quarzo,vinil clorido,mercurio,solventes)


Fascitis difusa con Eosinofilia.
Formas epidémicas (Sd. Mialgia eosinofilia y sd. Aceite toxico)
Escleredema (DMID)
Alteraciones gastrointestinales en esclerodermia.
El compromiso visceral más conspicuo le corresponde al aparato digestivo. El esófago
perderá su natural elasticidad y aparecerán signos de disfunción: pirosis, regurgitación,
dolor retroesternal o bien una sensación de disfagia. Estos síntomas no son universales y
solamente la mitad de los pacientes se percata del problema aunque la alteración
peristáltica del tercio inferior del esófago se puede constatar radiológicamente. El resto
del aparato digestivo también se afecta: el estómago y el intestino delgado con atonía, el
grueso con divertículos de “boca ancha”.

Lecturas:

Esclerosis sistémica y tracto gastrointestinal. Abordaje diagnóstico y terapéutico

Olga Lidia Vera-Lastraa Departamento de Medicina Interna. Hospital de Especialidades


Antonio Fraga Mouret. Centro Médico Nacional La Raza. IMSS. México DF. México.

92.- Los antimaláricos se utilizan en el lupus eritematoso para tratar las manifestaciones:

a) Neurológicas.
b) Cutáneas y articulares.
c) Cardíacas.
d) Digestivas.

Son un grupo de medicamentos (también llamados antipalúdicos) que se han usado


clásicamente para tratar el paludismo o malaria, enfermedad parasitaria que afecta a más
de 500 millones de personas en el tercer mundo. Algunos antimaláricos (cloroquina e
hidroxicloroquina) se han mostrado útiles en el tratamiento de algunas enfermedades
reumáticas (artritis reumatoide, lupus eritematoso sistémico, reumatismo palindrómico,
Sindrome de Sjögren, artritis psoriásica, sarcoidosis, osteoartrosis erosiva,...).
Estos medicamentos tienen múltiples mecanismos de acción, algunos no bien entendidos.
Tienen efecto antiinfeccioso (base del tratamiento del paludismo), antiiinflamatorio y
sobre la función inmune inhibiendo la activación de linfocitos, monocitos y la producción de
factores mediadores de la inflamación. Otros efectos conocidos son su capacidad de
disminuir los niveles de lípidos y de inhibir la agregación de plaquetas, ayudando a prevenir
fenómenos trombóticos.
Lecturas:

ANTIMALÁRICOS COMO DROGAS INDUCTORAS DE REMISIÓN EN LA ARTRITIS


REUMATOIDEA.

José Félix Restrepo Suárez1, Renato Guzmán Moreno2, Antonio Iglesias Gamarra3
1. Médico Internista_Reumatólogo. Profesor Asociado de Medicina Interna y
Reumatología. Coordinador Unidad de Reumatología. Universidad Nacional de Colombia.
2. Internista _ Reumatológo. Clínica Salud-Coop-Bogotá.

93. - A 60-year-old woman presents to a physician complaining of a swelling in her neck.


Her past medical history is significant for rheumatoid arthritis and Sjögren syndrome.
Physical examination reveals a mildly nodular, firm, rubbery goiter. Total serum thyroxine
(T4) is 10 mg/dL, and third-generation thyroid-stimulating hormone (TSH) testing shows a
level of 1.2 mIU/mL. Antithyroid peroxidase antibody titers are high. Which of the
following is the most likely diagnosis?

a) Euthyroid sick syndrome.


b) Graves disease.
c) Hashimoto thyroiditis.
d) Silent lymphocytic thyroiditis.

Tiroiditis de Hashimoto
Es la causa más frecuente de hipotiroidismo en las zonas donde no existe déficit dietario
de yodo y los síntomas de hipotiroidismo suelen ser el motivo de su diagnóstico. Es
frecuente su asociación con otras enfermedades autoinmunes.
Es característica la presencia de un pequeño bocio, firme, irregular y no doloroso al tacto
con sensación de plenitud en la garganta. Los Ac anti-TPO (anticuerpos antiperoxidasa) son
positivos en el 90% de pacientes y los antitiroglobulina en un 20 a 50%. La VSG es normal o
casi normal. La ecografía muestra una glándula hipoecogénica sin nódulos.
Una vez el hipotiroidismo está establecido debe tratarse con hormona tiroidea a dosis
sustitutiva: 50 microgramos diarios inicialmente (25 si edad avanzada, enfermedades
cardiovasculares o consuntivas o niveles previos muy bajos de T4 libre) aumentando 25
mcrgr/ día cada 15 a 30 días según los mismos criterios, hasta 75-100 micrgr/día,
ajustándose posteriormente según controles que deben dilatarse al menos 2-3 meses
desde el inicio del tratamiento ya que las respuestas son lentas. Los pacientes con
hipotiroidismo subclínico con niveles altos de Ac anti-TPO debe ser tratados también por
su frecuente evolución a hipotiroidismo clínico y mayor riesgo arteriosclerótico por
elevación lipídica. Si el bocio es grande debe administrase hormona tiroidea en dosis
suficiente para frenar la TSH, lo que puede hacer disminuir en 6 meses hasta un 30% el
tamaño de la glándula.

Si se observa un nódulo tiroideo debe ser estudiado mediante PAAF a fin de descartar
carcinoma tiroideo o linfoma (raro, pero con un riesgo relativo de 67 en este tipo de
tiroiditis)

Lectura recomendada.
Tiroiditis de Hashimoto y síndrome de Sjögren.
¿Asociación o síndrome múltiple autoinmune?
L. SIERRA SANTOS, A. SENDINO REVUELTA, R. PACHECO CUADROS,
G. APARICIO JABALQUINTO, F. J. BARBADO HERNÁNDE
Servicio de Medicina Interna. Hospital la Paz. Universidad Autónoma. Madrid
Sierra Santos L, Sendino Revuelta A, Pacheco Cuadros R, Aparicio Jabalquinto G, Barbado
Hernández FJ. Tiroiditis de Hashimoto y síndrome de Sjögren. ¿Asociación o síndrome
múltiple autoinmune? An Med Interna (Madrid) 2001; 18: 86-87
HASHIMOTO’S THYROIDITIS AND SJÖGREN´S SYNDROME.
ASSOCIATION OR MULTIPLE AUTOIMMUNE SYNDROME?

Bibliografía

Beers MH, Berkow R. Editores. El manual Merck. 10ª ed. Madrid: Harcourt; 1999. p. 96-7.
Fauci AS, Braunwald E, Isselbacher KJ, Wilson JD, Martín JB, Kasper DL et al. Editores.
Harrison Principios de Medicina Interna. 14ª ed. Madrid: McGraw Hill; 1998. p. 2310-2.
Larsen PR, Davies TF. Hipotiroidismo y tiroiditis. En: Larsen PR, Kronemberg HM, Melmed
S, Polonsky KS. Williams Tratado de Endocrinología. 10ª ed. Madrid: Elsevier; 2004. p. 465-
500.
Pearce EN, Farwell AP, Braverman LE. Thyroiditis. N Engl J Med 2003; 348:2646-55

94. - An 18-year-old woman complains of myalgias, a sore throat, and painful mouth sores
for 3 days´duration. Her temperature is 38.2 C (100.8 F), blood pressure is 110/80 mm Hg,
pulse is 84/min, respirations are 15/min. Her gingival are edematous and erythematous,
and there are vesicles on her right upper and lower lips. Her pharynx is mildly
erythematous but without exudates, and there is tender mobile cervical lymphadenopathy.
Her breath is not fetid, and the dentition is normal. Which of the following is the most
likely causal agent?

a) Actinomyces israelii.
b) Herpes simplex virus 1.
c) Nocardia asteroids.
d) Streptococcus pyogenes.
Las infecciones por herpes simplex son comunes en la práctica diaria, y con frecuencia el
paciente acude a los servicios de urgencia. Estas infecciones son ocasionalmente
recurrentes, generalmente dolorosas y asociadas con síntomas sistémicos, por lo cual el
médico de urgencias debe estar familiarizado con el cuadro clínico y su manejo.

Existen dos variedades de virus del Herpes simplex (VHS) capaces de causar infección en
el hombre: el tipo 1 (VHS-1) y el tipo 2 (VHS-2) que se distinguen entre sí por varias
características, incluyendo sus comportamientos clínico y epidemiológico, antigenicidad,
composición del ADN y la sensibilidad a diferentes agentes físicos y químicos (Cuadro No.1)

Cuadro No. 1

DIFERENCIAS ENTRE LOS VIRUS HERPES SIMPLEX TIPOS 1 Y 2

Características clínicas VHS-1 VHS-2

Vía de transmisión Oral Genital


Síndromes Oral-facial Genita
característicos Ocular PerianaL
Encefalitis Neonata
Paroniquia Paroniquia
Reactividad a antígenos VHS-1 específico l VHS-2 específico
monoclonales específicos

El VHS-1 es de localización primordialmente extragenital, con predilección por los tejidos


de origen ectodérmico, mientras que el VHS-2 corresponde al "Herpes progenitalis"
descrito por separado, dentro de las infecciones de transmisión sexual.

La región oral es la localización habitual del Herpes simplex 1, el cual es causa frecuente de
lesiones orofaciales recurrentes y de otro tipo de enfermedades (encefalitis).

EPIDEMIOLOGIA

El ser humano es el único reservorio natural conocido del virus herpes simplex, aunque
algunos animales de experimentación pueden infectarse con facilidad.

La infección primaria del VHS-1 ocurre sobre todo durante la infancia, mientras que el tipo
2 se presenta en la adolescencia y adultos jóvenes activos sexualmente. Las tasas de
infección son inversamente proporcionales al estrato socioeconómico.
El principal mecanismo de transmisión es el contacto directo con las secreciones infectadas.
El VHS-1 se trasmite por saliva y el VHS-2 por vía genital. Aunque los títulos virales son
más altos cuando existen lesiones activas, también es frecuente la liberación viral en
infectados asintomáticos. Por lo tanto, la transmisión viral puede efectuarse aún en
ausencia de lesiones activas.

La persistencia de la infección y la recurrencia de las lesiones son un fenómeno frecuente


tanto para el VHS-1 como para el VHS-2 y por lo común se producen por reactivación
endógena. Los factores que la precipitan van desde la luz solar, el viento, traumatismos
locales, fiebre, menstruaciones y hasta estrés emocional.

DIAGNOSTICO

Cuadro Clínico. Los cuadros clínicos causados por este virus se suelen dividir en dos grupos:
el debido a la infección primaria y el correspondiente a la infección recurrente. En el
primer grupo se incluyen la gingivoestomatitis aguda, la vulvovaginitis aguda y la infección
herpética del ojo, que puede llegar a queratitis. Las recurrentes se circunscriben al
"Herpes labialis", queratitis, blefaritis y queratoconjuntivitis. Todos los cuadros son
autolimitados, pero tanto las formas primarias como las recurrentes, se pueden complicar.
Una de estas complicaciones es la Encefalitis herpética y el Eczema herpeticum.

Infección primaria. El primer contacto clínico de infección por virus del herpes simple
suele ser el más grave. Los enfermos aquejan fiebre, malestar general, artralgias y por
último la presencia de un grupo de vesículas sobre una base eritematosa, dolorosa,
inflamada y sensible. La gingivoestomatitis es la manifestación más común, cuya gravedad
varía desde la erosión de pequeñas áreas a la ulceración extensa de la boca, lengua y encías.
La infección puede ser bastante grave como para dificultar la ingesta de alimentos y
líquidos (odinofagia). La curación tiene lugar en 7 a 14 días, a menos que las lesiones se
sobreinfecten con estafilocos o estreptococos.

Infección recurrente. Generalmente existe prurito, dolor o molestias focales que


preceden la aparición de las vesículas. Las vesículas se rompen espontáneamente después
de unos cuantos días y sanan en una semana sin dejar secuelas.

LECTURAS RECOMENDADAS

1. Callen JP, Cooper Ma. Dermatologic emergences. Emerg Med. Clin North Am
3:641, 1985
2. Guzmán M. Herpes simple, varicela zoster. En: Medicina Interna. Segunda
Edición. Editado por F Chalem, JE Escandón, J Campos, R Esguerra.
Fundación Instituto de Reumatología e Inmunología. Editorial Presencia
Ltda. Santafé de Bogotá, 1992
3. Guerra Flecha J, Lizarraga Bonelli S. Enfermedades de transmisión sexual:
herpes genital Trib Med 79:29, 1989
4. Jaramillo AC. Infecciones virales de la piel y sus anexos. En: Fundamentos
de Medicina. Enfermedades Infecciosas. Cuarta edición. Corporación para
Investigaciones Biológicas. CIB. Medellín, 1989

95.- Masculino de 48 años de edad, presenta erupción cutánea descamativa asintomática,


con afectación del cuero cabelludo, cejas, pestañas, eminencias malares y pliegues
nasolabiales. En la exploración se pone de manifiesto una descamación seborreica sobre una
piel moderadamente eritematosa en las áreas afectadas. En la región anterior del tórax se
observa un moderado enrojecimiento con descamación. El diagnóstico más probable es:

a) Dermatitis seborreica.
b) Psoriasis.
c) Lupus eritematoso.
d) Tiña corporis.

CLÍNICA
La dermatitis seborreica se puede clasificar en: 1) Dermatitis Seborreica en la infancia: a)
Costra láctea. B) Dermatitis seborreica infantil. C) Falsa tiña amiantácea. D) Eritrodermia
descamativa de
Leiner-Mousses. 2. Dermatitis Seborreica en el adulto: Pitiriasis cápitis. B) Pitiriasis
esteatoide.
C) Dermatitis seborreica facial, mediotóracica y medioescapular y púbica. 3. Dermatitis
Seborreica en enfermos inmunocomprometidos.
La costra láctea se manifiesta en los primeros meses de vida y se manifiesta mediante
placas escamosas, untuosas al tacto, de color grisáceo que localizan en cuero cabelludo (Fig.
1). También pueden existir lesiones eritemato-descamativas centrofaciales y
mediotorácicas. La falsa tiña amiantácea se manifiesta como gruesas escamocostras en
cuero cabelludo, de color blanco-grisáceo que al despegarse dejan zonas húmedas. La
eritrodermia descamativa de Leiner Mousses comienza de forma repentina, a los pocos
meses de vida, con lesiones en cuero cabelludo, evolucionando de forma rápida a lesiones
eritematosas y descamativas que ocupan casi todo el tegumento.

Suele acompañarse de candidiasis de pliegues, infecciones locales y sistémicas, diarreas. A


veces el pronóstico es malo. Las manifestaciones clínicas de la dermatitis seborreica
infantil o clásica del adulto sólo se manifiestan en la edad de presentación.
Conforman lesiones eritemato-descamativas que localizan en la zona de inserción de cuero
cabelludos, pliegues retroauriculares, conducto auditivo externo, surcos nasolabiales y
nasogeniamos, cejas, glabela, zona mediotorácia e interescapular, axilas, ingles, ombligo,
pubis . Suele ser pruriginosa. En pacientes HIV (+) las manifestacionesclínicas de la
dermatitis seborreica son mucho más intensas.
DERMATITIS SEBORREICA
Dres. E. Herrera y G. Ruíz del Portal.

96.- Mecanismo de transmisión de la escabiasis:

a) Persona a persona, fomites.


b) Agua contaminada, piojos.
c) Carne mal cocida de animal contaminado.
d) Ganado vacuno al hombre.

Definición: Sarna o escabiosis es la enfermedad producida por la presencia en la piel del


hombre del ectoparásito Sarcoptes scabiei. Aún cuando existen variedades de esta misma
especie que atacan a los animales domésticos (var.cati, canis, suis, equis y otros) la
variedad hominis es la única que afecta al hombre en forma permanente.

Mecanismos de transmisión

El hombre es el principal reservorio del Sarcoptes scabiei variedad hominis. La propagación


es de persona a persona a través del contacto directo de la piel, incluido el contacto sexual,
de por lo menos 10 minutos. La transmisión mediante prendas íntimas y ropa de cama es
rara y se produce si están contaminadas con ácaros juveniles o adultos de personas
infestadas, que han usado la ropa hasta 48 horas antes. La transmisión sexual es un
mecanismo posible, pero poco frecuente 16. Las personas pueden infestarse por los ácaros
de los animales de compañía, principalmente los perros 17.

Referencias

1. Heukelbach J, Feldmeier H. Scabies. Lancet 2006; 367: 1767-74


2. Van Neste DJ. Inmunology of Scabies. Parasitology Today 1986; 2: 194-6.
3. Walton S, Currie BJ. Problems in diagnosing scabies, a global disease in human and
animal populations. Clin Microbiol Rev 2007; 20: 268-79.
4. Saavedra T. Sarna y otras acarosis. Atías A. Parasitología Médica. 1ª Edición,
Santiago de Chile. Edit Méditerráneo, 1999. Capítulo 54, págs 484-9.
5. Meinking TL, Taplin D. Advance in pediculosis, scabies and other mite infestations.
Adv Dermatol 1990; 5: 131-52.
6. Chosidow O. Scabies and pediculosis. Lancet 2000; 355: 819-26.
7. Commens CA. We can get rid of scabies: new treatment available soon. Med J Aust
1994;160: 317-8.
8. Chosidow O. Scabies. N Engl J Med 2006; 354: 1718-27.
9. Huynh TH, Norman RA. Scabies and pediculosis. Dermatol Clin 2004; 22: 7-11.
10. Neira P, Arenas C, Neira G, Valenzuela N, Subercaseaux B. Infecciones parasitarias
en niños de una escuela de Reñaca Alto. Rev Med Valparaíso 1981, 34: 56-60.
97.- Femenino de año y medio de edad refiere la madre que inició con tos seca, rinorrea
hialina, temperatura axilar de 37.8°C. Por la tarde muestra tos intensa y en accesos, dolor
subesternal al toser, estridor inspiratorio; al explorarle se le observa pálido, con aleteo
nasal, hundimiento intercostal bilateral a la inspiración y taquicardia. Faringe hiperémica
roja; en tórax se escuchan algunos estertores gruesos diseminados en ambos hemitorax y
disminución del murmullo vesicular. Usted sospecha de:

a) Bronquiolitis.
b) Asma bronquial.
c) Neumonía bilateral.
d) Laringotraqueobronquitis.

La laringotraqueobronquitis también conocida como laringitis subglótica o crup vírico, es


una enfermedad infecciosa aguda de etiología viral que afecta fundamentalmente a la
laringe y a las estructuras que se localizan por debajo de las cuerdas vocales, en donde los
tejidos blandos subglóticos inflamados producen obstrucción de la vía aérea en grado
variable. Este padecimiento afecta con mayor frecuencia al grupo de edad comprendido
entre seis meses y tres años, con un pico de incidencia a los dos años de edad; no hay
predominio de sexo, y se presenta más frecuentemente en otoño e invierno.

CUADRO CLÍNICO

El cuadro inicial es el de una rinofaringitis e incluye irritación nasal, coriza, fiebre


generalmente menor de 39 °C, tos seca y odinofagia, con poca o nula afección del estado
general; 24 a 48 horas después aparece disfonía, la tos se hace "crupal" (traqueal, perruna
o en ladrido) y se presentan en forma gradual estridor inspiratorio creciente (estridor
laríngeo) y signos de dificultad respiratoria de intensidad variable. En la exploración física
son evidentes la disfonía, el estridor laríngeo que en ocasiones es audible a distancia, la
disminución del murmullo vesicular y datos de dificultad respiratoria.1-6.

Forbes describe la progresión del cuadro clínico de la siguiente manera:

Etapa 1: Fiebre, ronquera, tos crupal y estridor inspiratorio al molestar al paciente.

Etapa 2: Estridor respiratorio continuo, participación de músculos accesorios de la


respiración con retracción de costillas inferiores y de tejidos blandos del cuello.

Etapa 3: Signos de hipoxia e hipercapnia, inquietud, ansiedad, palidez, diaforesis y


taquipnea.

Etapa 4: Cianosis intermitente, cianosis permanente, paro respiratorio.

Por lo general la enfermedad es leve, en pocas ocasiones progresa más allá de la etapa 1,
más del 95% reciben tratamiento ambulatorio; del 5% que requiere hospitalización sólo del
1 al 1.5% requieren medidas de apoyo ventilatorio (intubación endotraqueal o
traqueostomía).
La duración del padecimiento es muy variable, desde tres a siete días en casos leves, hasta
siete a catorce días en casos graves

1. PAC Infecciones de Vías Respiratorias en Niños Parte B Libro 2

98.- Una mujer de 55 años, se queja de presión pélvica y una masa en la entrada vaginal,
hace 3 años tuvo su último periodo menstrual. No tomo terapia de reemplazo, tiene
dificultad para evacuar. Tiene una tos crónica y una historia de tabaquismo positivo a razón
de 30 cajetillas año. Ha tenido 3 partos y el ultimo bebe pesó 4,500 gramos. Tiene una
orina de 60 cc. ¿Cuál de los siguientes hallazgos es más probable que encuentres en la
exploración pelvica?

a) Rectocele.
b) Cistocele.
c) Enterocele.
d) Uretrocele.

El Rectocele es una hernia de la pared anterior del Recto hacia la porción posterior de la
vagina. La incidencia real de esta “deficiencia anatómica” es desconocida y en muchas
ocasiones es un resultado del paso del tiempo. Es un hallazgo muy frecuente del examen
perineal, siendo en múltiples ocasiones asintomático.
El rectocele puede ser un hallazgo importante del síndrome de Obstrucción Defecatoria
(SOD). No debe tomarse como una deficiencia anatómica única, sino como parte importante
de un problema anatomo-fisiológico complejo.

Un principio importante es la etiología y anatomo-patología del rectocele. Existen varias


teorías sin consenso principal. La existencia, deficiencia o alteraciones del septo
rectovaginal son controversias importantes. No existe una fascia visceral que separe el
recto de la vagina o que forme un septo específico. Existe frontera entre donde termina la
pared anterior del recto y donde comienza la pared de la vagina, pero múltiples estudios no
han encontrado un septo rectovaginal específico. El septo puede estar formado de una
película casi transparente hasta una pared de consistencia fibromuscular fuerte.
El rectocele es un hallazgo común. Se presenta en el 80% de las pacientes femeninas y
13% de los masculinos en una defeco grafía (> a 1cm.) Entre mas grande es el rectocele,
mayor son los síntomas asociados, Dificultad en la evacuación, constipación crónica, dolor
rectal y perineal, sensación de masa y en ocasiones sangrado. La necesidad de presión
manual para ayudar a la evacuación o para vaciar el rectocele es común en más del 50% de
los pacientes.
El diagnostico se hace con un simple tacto rectal e inspección vaginal, pero debe recordarse
que rara vez es el rectocele un hallazgo aislado. La presencia cistocele, peritoneocele,
enterocele u otros prolapsos perineales y problemas funcionales deben ser descartados
antes de proponer la reparación quirúrgica del rectocele. Defeco grafía,
Pruebas de función fisiológica del piso pélvico, pruebas de función urinaria, evaluación del
esfínter anorectal por ultrasonido y hasta la resonancia magnética han sido propuestos
antes de la cirugía.
Bibliografía:
Rectocele: Pathogenesis and surgical managment. Zbar AP, Linemann A, Fritsch H,
Beer-Gabel M, Pescatori M. Int J Colorectal Dis. (2003) 18:369-384.
Evaluation and Treatment of Women with rectocele. Cundiff GW, Fenner D, Obstetrics
and Ginecology 104(6): 1403-1416
Stapled transanal rectal resection to treat obstructed defecation caused by rectal
intussusseption and rectocele. Renzi A, Izzo D, Di Sanrno (2—6) 21:661-667
Rectocele repair using biomaterial augmentation. Altman D, Melgren A, Zetterstrom J.
Obstet Gynecol (2005) 60(11)753-760.

99.- La causa mas frecuente de dolor de retropié (talón) en el paciente pediátrico es

a) Osgood Schlater.
b) Enfermedad de Köhler.
c) Enfermedad de Sever.
d) Enfermedad de Snding Larsen.

Esta enfermedad se llama de Sever en honor al médico que la describió por primera vez en
1912. Se denomina enfermedad de Sever, apofisitis posterior del calcáneo u osteocondritis
calcánea a la inflamación e irritación del cartílago de crecimiento del hueso calcáneo que
cursa con dolor localizado en el talón. Existen diversos factores que condicionan su
desarrollo, tales como la edad, el exceso deportivo, la retracción de la musculatura
posterior de la pierna o un calzado inadecuado. No obstante, con la colocación de una pieza
elevadora de talón así como con reposo físico durante 2-3 semanas y terapia anti-
inflamatoria, se resuelven la mayoría de los casos consultados en la bibliografía. Así mismo,
se hace necesaria la planificación deportiva anual de los niños para evitar la aparición de
esta patología

BIBLIOGRAFIA
Lowell and Winters's Pediatric Orthopaedics
Chapter 29. Vincent S. Mosca: The Foot
Raymond T. Morrissy and Stuart L. Weinstein editors
Fifht edition, Volume 2, pag 1206
Lippincott Williams and Wilkins, 2001.
100. - A 20-year-old man comes to the physician he has noticed blood in his urine on
several occasions in the past year. Each episode of hematuria occurred in association with
an upper respiratory tract infection or a flulike illness. Physical examination is
unremarkable. A urine dipstick test shows mild proteinuria and microhematuria. Serum
levels of electrolytes, creatinine, and blood urea nitrogen are within normal limits. Serum
levels of IgA are elevated. Which of the following is the most likely diagnosis?

a) Berger disease.
b) Goodpasture syndrome.
c) Postinfectious glomerulonephritis.
d) Wegener granulomatosis.

HEMATURIA GLOMERULAR AISLADA PERSISTENTE:

Definición

Presencia de hematuria de origen glomerular (con acantocitos, y ocasionalmente cilindros


hemáticos), sin otro elemento de inflamación glomerular, es decir, sin HTA, ni edema, con
proteinuria menor a 1 gm /24 hs, y función renal normal, estable.

En caso de hematuria persistente, aislada, las posibilidades principales es que se trate de


Glomerulonefritis por IgA (lo más frecuente), enfermedad de membrana basal fina, Sd.
de Alport, (raro) o el inicio de una glomerulonefritis crónica (< probable) (15) .También
puede tratarse de una GN Post-Streptocócica, con persistencia de hematuria.

A continucación se describirán brevemente:

Nefropatía por IgA: Enfermedad de Berger y síndrome de Shönlein-Henoch, se considerán


por algunos como espectro de una misma enfermedad, siendo Berger la forma limitada al
riñón.
Es la Glomerulonefritis aguda más frecuente. Se da entre los 15 y 35 años, y es 3 veces
más frecuente en hombres, tiene tres patrones clínicos de presentación:

1. Como hematuria macroscópica, 24 – 48 hs. posterior a cuadro


respiratorio alto, asociado a dolor lumbar. Hematuria dura 2 – 6
días, rara vez presenta HTA o deterioro de función renal, y tiene
proteinuria mínima. Recurre hasta en un 50%. De esta forma se
presenta un 40-50%.

2. Como hematuria microscópica, con proteinuria leve, detectada solo


en Sed. Orina, por rutina. 30 – 40 % se presenta así.
3. Como Sd. Nefrítico (HTA, Edema, Hematuria), pudiendo
evolucionar algunos de estos casos como GNRP. Se presentan así un
10 %.

Generalmente cursan con proteinuria menor a 1-2 gm/24 hs,. Pocos desarrollan Sd.
nefrótico (10 %)

Existe un grupo de pacientes (20 – 40 %), que evoluciona con falla renal progresiva,
llegando a IRT en 5 – 25 años. Se ha detectado como factor de riesgo de esta evolución la
presencia de Sd. nefrótico o proteinuria > 1gm/24h, , edad, HTA, deterioro de la función
renal, ausencia de hematuria macroscópica. Si no tiene ninguno de estos factores, tiene
poca probabilidad de desarrollar falla renal, y no se ha demostrado que el tratamiento, en
este grupo de bajo riesgo, altere la evolución de la enfermedad.

Al laboratorio, el complemento es generalmente normal, rara vez elevado. Existe aumento


de IgA circulante entre un 30 – 50 %, sin ser específico de esta enfermedad.
Para el diagnóstico, se requiere de biopsia renal, donde se identifican por
inmunohistoquímica, la presencia de depósitos de IgA. Biopsia de piel carece de
especificidad y de sensibilidad para diagnóstico de enfermedad de Berger.

BIBLIOGRAFÍA

1.Jennette C, Falk R: Small Vesel Vasculitides. N Engl J Med 337:1512, 1997.


2.Kashtan, CE Alport Síndrome and thin glomerular basement Membrane disease. JAM Soc
Nephrol 1998;9:1736.
3.Falk R et al: Primary glomerular disease, en “Breneer & Rector´s, The Kidney, 6a ed, BM
Brenner (editor). Phyladelfia, Saunders, 2000. pp 1263-1349.
4.Hricik, DE et al: Glomerulonephritis. N Eng J Med 339:889,1998.
5.Antony BF. Attack rates of acute nephritis after type 49 streptoccocal infection of the
skin and of the respiratory tract. J Clin Inv, 1969;48:1697.
6.Oliviera DBG. Poststreptococcal glomerulonephritis:getting to know an old enemy. Clin
Rxp Immunol 1997;107:8-10.
7.LangeK, et all. Evidence for the in situ origin of poststreptococcal glomerulonephritis:
glomerular localization of endpstreptosin and the clinical significance of the subsequent
antibody response. Clin Nephrol 1983;19:3-10.
8.Rodriguez-Iturbe, B. Epidemic poststreptococcal glomerulonephritis, Kidney int 1984,
25:129.
9.Tejani A. Poststreptococcal glomerulonephritis: current clinical and pathologic concepts.
Nephron 1990;55:15.
10. Potter EV. Twelve to seventeen year follow up of patients with poststreptococcal
acute glomerulonephritis in Trinidad. N Engl J Med 1982; 307:725-9.
Universidad La Salle.
Facultad Mexicana de Medicina.
Curso de Extensión Universitaria para la Preparación del Examen Nacional para
Aspirantes a Residencias Médicas.
Examen de Gineco.obstetricia.

Nombre: Examen del Módulo I.


Número de intentos: 3.
Vigencia: 27 de Febrero del 2013.
Horario: 7:00 p.m. a 9:00 p.m.
Programar aleatorio. (4 bloques con 20 preguntas cada uno).

1.- Femenino de 51 años, con mioma uterino de tamaño equivalente a una gestación de 12
semanas, que presenta hipermenorreas y hemoglobinemia de 9 gr%. No se demuestra
patología asociada. Se encuentra en espera para la práctica de una histerectomía
programada a realizar en 4 meses. En esta paciente está indicado el tratamiento
preoperatorio con:

a) Estrógenos.
b) Inhibidores de la fibrinólisis.
c) Derivados del cornezuelo del centeno.
d) Análogos de la GnRH.

Análogos de la GnRH: Son derivados de la hormona GnRH en donde se ha realizado una


sustitución peptídica en posición 6 y en algunos casos en la 10, obteniendo compuestos
hasta unas 200 veces más potentes debido a mayor afinidad por los receptores y a su
resistencia a la degradación por peptidasas. Aunque su acción inicial produce un incremento
en la producción de FSH-LH (efecto flure-up o llamarada) tras 5-6 días de exposición
contínua, los receptores son internalizados produciéndose un estado de hipogonadismo
hipogonadotropo y niveles de estradiol similares a los de la postmenopausia. Los análogos de
GnRH están disponibles en distintas fórmulas: administración nasal (varias aplicaciones al
día), subcutánea (aplicación diaria) o intramuscular (preparados depot mensuales o
trimestrales) (Shaw RW 1999).

Marco Filicori y sus colaboradores de la Universidad de Bolonia fueron los primeros en


utilizar en 1983 los aGnRH en un estudio que confirmó su eficacia para reducir el tamaño
de los miomas uterinos y secundariamente síntomas como alteraciones menstruales, dolor
pélvico y síntomas de presión local. Otros autores como Minaguchi H y colaboradores
continúan comprobando la efectividad de los análogos de la GnRH en el tratamiento del
mioma uterino tras evaluar en el año 2000 seis estudios con un total de 602 pacientes
tratadas con nafarelina. La disminución del tamaño se calcula entre un 30-70%, y se ha
observado como el mayor porcentaje de reducción ocurre tras el primer mes de
tratamiento, no existiendo reducciones o siendo éstas mínimas después del tercer mes
(Healy et al 1986; Friedman et al 1989; Matta et al 1989; Williams y Shaw 1990). En
miomas pediculados o con gran proporción de calcio o colágeno (hialinización) la repuesta es
también menor. Debe tenerse en cuenta que si después de dos meses de tratamiento no se
ha producido un significativo descenso del tamaño del mioma, éste ya no debe ser esperado
y debe pensarse en la posibilidad de la existencia de un tumor muscular maligno no
diagnosticado (Messia AF et al 1998). Tras finalizar el tratamiento y recuperarse el estado
de hipogonadismo, el mioma retorna rápidamente a su tamaño inicial (Friedman AJ et al
1987; Matta WH et al 1989).

En casos próximos a la menopausia, la reducción del tamaño del mioma y su sintomatología,


permitiría hablar de una solución médica del problema, pero en todo caso la utilización de
aGnRH facilitaría la intervención quirúrgica al acortar el tiempo de intervención, la
hemorragia y el acceso a localizaciones complicadas como el caso de miomas
interligamentarios o situados en istmo o cérvix. En el caso de la cirugía histeroscópica la
reducción del tiempo de cirugía permitiría reducir el volumen de fluidos aportado a cavidad
uterina y los riegos de absorción e hiponatremia.
Los mecanismos de acción por los que los aGnRH actúan son: o Hipoestrogenemia: es
necesario mantener la hipoestrogenemia, pues la elevación de sus niveles lleva a un rápido
incremento del tamaño del mioma. El crecimiento del mioma es dependiente de los niveles
de estrógenos (aumentan de tamaño con el embarazo y se reducen durante la menopausia o
el tratamiento con aGnRH, pudiendo volver a crecer durante la THS), pero aunque los
estrógenos parecen ser importantes en el crecimiento del mioma, su relación debe ser algo
más compleja pues no se han descrito incrementos significativos del tamaño de miomas
durante el tratamiento con gonadotrofinas en RA (situaciones con elevados niveles de E2),
algunos de ellos no se modifican durante el embarazo o incluso decrecen y se han
encontrado crecimientos después del tratamiento con citrato de clomifeno (antiestrógeno).
En relación con la hipoestrogenemia podrían estar los cambios inducidos en el flujo vascular
uterino (incrementos en el índice de resistencia de las arterias uterinas) que suponen una
reducción de la vascularización o las modificaciones de distintos factores de crecimiento. o
Cambios histológicos: el tratamiento con aGnRH puede producir degeneración roja,
infiltración linfocitaria, y necrosis, así como reducción de la proliferación celular e
incremento de la apoptosis. Pero en otras circunstancias no es posible encontrar 7
diferencias. No se ha encontrado una relación entre los cambios histológicos y el
porcentaje de reducción del tamaño del útero, y existe una gran variabilidad entre
distintas pacientes o entre distintos miomas de una misma paciente, no existiendo pues un
patrón histológico característico de respuesta ante el tratamiento con aGnRH.
BIBLIOGRAFÍA:

1. Abad L, Abad de Velasco L, Parilla JJ. Etiopatogenia. Papel de las hormonas


esteroideas, factores de crecimiento y otras sustancias. Cuad Med Reprod
1999;5(1):15-29.
2. Albano C, Platteau P, Devroey P. Gonadotropin-releasing hormone antagonist: how good is
the new hope? Curr Opin Obstet Gynecol 2001;13(3):257-62.
3. Coutinho EM.Treatment of large fibroids with high doses of gestrinone.
Gynecol Obstet Invest 1990;30(1):44-47.
4. Chavez NF, Stewart EA. Medical treatment of uterine fibroids. Clin Obstet
Gynecol 2001;44(2):327-84.
5. De Leo V, la Marca A, Morgante G. Shortterm treatment of uterine
fibromyomas with danazol. Gynecol Obstet Invest 1999;47(4):258-262.
6. Eldar-Geva T, Healy DL. Other medical management of uterine fibroids.
Baillieres Clin Obstet Gynaecol 1998;12(2):269-88.
7. Felberbaum RE, Germer U, Ludwig M, Riethmuller-Winzen H, Heise S,
Buttge I, Bauer O, Reissmann T, Engel J, Diedrich K. Treatment of uterine fibroids with a
slow-release formulation of the gonadotrophin releasing hormone
antagonist Cetrorelix.HumReprod 1998;13(6):1660-8.

2.- Femenino de 25 años G-3, P-1, A-1 con 39 SDG por FUR. Reporta contracciones
uterinas que han sido regulares las últimas tres horas. Al examen encuentras que las
contracciones son cada tres minutos y duran 50 segundos y son firmes a la palpación. Tuvo
ruptura de membranas hace una hora y lo demuestras con papel de nitrazina. El examen
digital cervical demuestra una dilatación de 5 cm, con borramiento del 100% y presentación
en vértex en estación 0. ¿Cual de los siguientes criterios es el más preciso para decir que
se encuentra en la fase activa del trabajo de parto?

a) Dilatación cervical mayor de tres centímetros


b) Borramiento cervical más de 90%
c) Duración de las contracciones de más de 30 seg
d) Ruptura de membranas
FASES DEL TRABAJO DE PARTO
El trabajo de parto se divide en tres fases:

Fase 1 ó latente
Es llamado así al periodo que sirve para la preparación uterina del parto, ocurre al final del
embarazo y va hasta el inicio de las contracciones del trabajo de parto. Los aspectos a
destacar en este lapso es el reblandecimiento cervical, el aumento importante en el número
de receptores para oxitocina a nivel de las células endometriales, un aumento sustancial en
los puentes de unión y el número de conexinas a nivel miometrial y por consiguiente una
mayor sensibilidad a los agentes uterotónicos.

Fase 2 ó activa
Es el lapso que representa el trabajo de parto activo, y se acepta que se inicie cuando
existen 3 cm de dilatación y las contracciones uterinas son aptas para producir un avance
en el trabajo de parto; se divide en tres periodos:
Primer periodo. Se inicia cuando las contracciones uterinas alcanzan la frecuencia,
intensidad y duración suficientes para causar borramiento y dilatación del cuello uterino, y
finaliza cuando éste se encuentra en completa dilatación.
El lapso de tiempo que dura es variable, pero se acepta como normal hasta diez horas en
primigrávidas y ocho horas en multigrávidas; pero independientemente de esto, se debe
considerar como adecuado si el borramiento y la dilatación cervical son progresivos e
ininterrumpidos.
Segundo periodo. Se inicia con una dilatación cervical completa y termina con la expulsión
del feto; tiene una duración variable, pero se acepta como normal una hora en pacientes
primíparas y 30 minutos en multíparas; y tiene como característica que debe de ser
progresivo e ininterrumpido.

Tercer periodo. Este comienza inmediatamente finalizada la expulsión fetal y termina con
la expulsión total de la placenta y las membranas corioamnióticas; a este periodo se le
conoce también como de “alumbramiento” y es el más corto de los periodos del parto; como
norma general se acepta que no debe de extenderse más allá de 10 minutos.
Existen algunos autores que incluyen un “cuarto periodo” dentro del trabajo de parto, el
cual abarca aproximadamente la hora posterior al alumbramiento, y comprende el lapso de
tiempo cuando ocurre la contracción y retracción de las fibras miometriales, así como la
trombosis de los vasos adyacentes, lo cual es un efectivo control de la hemorragia del sitio
de implantación de la placenta.
Fase 3

Este periodo es el que representa el regreso de la mujer a su estado previo al embarazo, y


se caracteriza por la involución uterina, la eyección láctea y por último la restauración de la
fertilidad; existen estudios que involucran en esta fase a la endotelina-1 y a la oxitocina
como substancias responsables de estos cambios postparto.
PROGRAMA DE ACTUALIZACION CONTINUA PARA GINECOLOGÍA Y OBSTETRICIA
PAC GO-1 Libro 3 Obstetricia 2005

3.- Femenino de 64 años de edad acude a consulta externa con la siguiente


sintomatología: plenitud, estreñimiento, distensión abdominal se acompaña de USG. Con
resultado de imagen quística en ovario derecho de 15 por 15 cms. El diagnóstico más
probable es:

a) Teratoma quístico.
b) Disgerminoma.
c) Endometrioma.
d) Cistadenoma seroso.

Los Tumores de Ovario son una patología frecuente dentro del contexto de la patología
femenina. Por esta causa consultan un grupo elevado de mujeres, tanto las consultas de
ginecología como las de Cirugía propiamente dicha. Las edades oscilan desde las tempranas
hasta las ya avanzadas, siendo el riesgo de degeneración maligna muy variable y relacionado
con le edad. La experiencia de la clínica revela la alta incidencia de tumores de ovario en la
etapa del climaterio, comprendida entre los 35 y 65 años de edad 1.
El cistoadenoma seroso de ovario (CSO) es un tipo de tumor derivado del epitelio
superficial (celómico), formado por áreas quísticas. El cistoadenoma seroso de ovario es el
tumor más frecuente de aquellos que provienen del epitelio celómico superficial. Hay
tumores pequeños macroscópicamente y tumores masivos que ocupan toda la pelvis e incluso
la cavidad abdominal. Estas frecuentes neoplasias quísticas uniloculares están tapizadas
por células epiteliales altas, cilíndricas y ciliadas, llenas de un líquido seroso claro y de
superficie lisa con abundantes vasos. Las variedades benigna, limítrofe y maligna
representan, en conjunto, 30% aproximadamente de todos los tumores del ovario. El riesgo
de presentar tumores epiteliales se incrementa con el paso de la edad, ya que pese a que la
declinación de la función ovárica marca el envejecimiento gonadal progresivo, el ovario
humano nunca pierde su capacidad para generar tumores. Por lo general, cuando es
detectado, su tamaño es grande, en donde la imagenología puede ayudarnos a considerar su
diagnóstico.

1. Capítulo 22 Tumores Benignos de Ovario. En: Novak ER, Jones G., Jokes HW.
Tratado de Ginecología. 9 ed. Ciudad de la Habana. Editorial Científico Técnica;
1977.p.432 – 66.
2. MedlinePlus Enciclopedia Médica en Español: Quistes Ováricos. Disponible en:
http://vsearch.nlm.nih.gov/vivisimo/cgibin/querymeta?v%3Aproject=medlineplusspanish&s
pell=spell&query=Quistes+Ov%C3%A1ricos Acceso: Actualizado 20/6/06.

3. Capítulo XL Tumores Ováricos En: Llusiá Botella J, Núñez Clavero JA. Tratado de
Ginecología. Ciudad de la Habana. Editorial Científico Técnica. 1983; T 3.1; p. 751 – 803.

4.- Femenino de 23 años acude al servicio de ginecología, por referir ciclos opso-
menorreicos, desde el inicio de su menarquia, en los últimos 7 días ha incrementado 15 Kg.
de lo que pesaba habitualmente. Exploración Física: acné facial importante, así como
bigote.

El diagnostico más probable en esta paciente es:

a) Sx. De Asherman
b) Sx. Stein Leventhall
c) Sx. Amenorrea Galactorrea
d) Sx. Karman

Síndrome de Ovario Poliquístico (SOP) es uno de los más comunes trastornos endocrinos
que afectan a las mujeres alrededor del 5% al 10% de las mujeres en edad reproductiva
(12-45 años) y se piensa que es una de las principales causas de la infertilidad femenina.
Las características principales son la obesidad, anovulación (dando lugar a la menstruación
irregular) o amenorrea, acné, y las cantidades excesivas o los efectos de androgénicos
(masculinizantes) hormonas. Los síntomas y la severidad del síndrome varían mucho entre
las mujeres. Si bien las causas son desconocidas, resistencia a la insulina, la diabetes y la
obesidad están fuertemente correlacionadas con el SOP.

Bulun SE, Adashi EY. The physiology and pathology of the female reporductive axis.
In: Kronenberg HM, Melmed S, Polonsky KS, Larsen PR, eds. Williams Textbook of
Endocrinology. 11th ed. Philadelphia, Pa: Saunders Elsevier; 2008:chap 16.

5. - Femenino de 19 años, atendida en sala de urgencias ginecoobstétricas, Antecedente:


cursa embarazo de 38 SDG. Exploración Física: en trabajo de parto. Repentinamente
presenta sangrado profuso transvaginal y dolor abdominal. Si la paciente presenta abruptio
placentae ¿EL factor de riesgo más frecuente es?

a) Edad materna
b) Elevada paridad
c) Trauma abdominal
d) Hipertensión materna
Se han sugerido numerosos factores que desempeñan un papel causal en el abruptio
placentae, pero no existe una explicación etiológica satisfactoria para cada uno de ellos:
• Traumatismo.
• Malformación o tumoración uterina.
• Brevedad de cordón umbilical (menor de 20 cm)
• Descompresión brusca del útero (RPM, expulsión de un 1º gemelo)
• Compresión de la vena cava inferior (sumamente raro y no demostrado)

• Hipertensión materna: más del 50% de los casos de desprendimientos están asociados a
HTA.
• Deficiencia de ácido fólico. No demostrado.
• Tabaquismo. Asociado por la necrosis de vasos deciduales (anomalías deciduales).
• Paridad y edad materna. En general la asociación de mayor edad y paridad, es no
demostrable,
No se puede descartar, que la paciente, por lógica, que sufre un desprendimiento,
generalmente
Es mayor de 20 años (no excluyente) y multípara. Sin embargo, si hay relación en que la
paciente que sufrió un DPPNI, tiene 5 veces más probabilidades de sufrir otro DPPNI,
independientemente de la edad y paridad.

• Iatrogenia. Versión externa y PTC inducción.

Obstetricia. Scwarcz, Sala, Duverges. 7ª edic. Edit. El Ateneo. (Biblioteca Fac.


Med. UNNE).

6.- Mujer de 21 años, que presenta una tumoración de 2 cm de diámetro en el cuadrante


ínfero-externo de la mama izquierda, indolora, de consistencia firme, superficie lisa,
forma ovoidea, móvil y bien delimitada del parénquima vecino, sin antecedentes de
derrame por el pezón, sin “piel de naranja” ni retracción del pezón, El diagnóstico más
probable es:

a) Fibroadenoma.
b) Carcinoma.
c) Ectasia de los conductos mamarios.
d) Quiste solitario.

FIBROADENOMA MAMARIO
Tumor benigno más frecuente en las mujeres entre los 20 y 35 años.
ETIOLOGIA
Existen múltiples teorías siendo la más aceptada la hormonal, generalmente son únicos, solo
el 20% son múltiples o bilaterales. De tamaño variable hasta de 10 cm. Ocupa el 13.6% de la
patología mamaria benigna.
CUADRO CLÍNICO
Lesión nodular de consistencia dura, de larga evolución y no dolorosa. Normalmente llegan a
los 3 cm. De diámetro. Durante la fase tardía del ciclo menstrual el tumor suele presentar
un leve aumento de tamaño. Durante la menopausia presentan regresión hasta la
calcificación (signo de palomitas de maíz).

DIAGNOSTICO

Es clínico, se presenta como un tumor bien delimitado, desplazable, no adherido a piel ni a


planos profundos, liso o multilobulado en ocasiones. Se localiza frecuentemente en
cuadrantes externos.
EXÁMENES DIAGNÓSTICOS
ULTRASONIDO MAMARIO.- Identifica un nódulo sólido, bien delimitado de bordes
regulares.
TRATAMIENTO.-
Conservador con vigilancia estrecha dependiendo del tamaño y en caso de ser necesario
exéresis del nódulo para estudio histopatológico.

hospitalgeneral.salud.gob.mx/

BIBLIOGRAFIA:

1. Sánchez BC. Tratado de Enfermedades de la glándula mamaria. Ed. Manual Moderno.


Cap. 13- 15.
2.- De Vita V. Cancer of the Breast. In Cancer: Principles and Practice of Oncology: Fifth
Ed. Philadelphia: Lippincott-Raven, Chapter 36; pp: 1521-1616.
3.-Consenso Nacional Acerca del Tratamiento de Cáncer de Mama. En Tumores de mama:
Diagnóstico y Tratamiento. 2ª Ed. McGraw-Hill Interamericana; pp: 119-126.
4.-Eberlein T. Current management of carcinoma of the breast. Ann Surgery 1994; 220:
121-136.
5. Encyclopedie Medico. Chirurgicale Praxis Medica, Editions Techiques de Mexico, tomo 5,
año 2005.

7.- Femenino de 42 años, gesta- 5, partos-3, abortos-1, con diagnóstico de anemia


ferropénica, de 9.5 g/dl, refiere ciclos menstruales de 31,32 x 8,9 días de duración,
acompañados de coágulos, los cuales aparecieron después del nacimiento de su segundo hijo
hace 13 años. E.F.: Buen estado general, TA 130/80, genitales con evidencia de sangrado
activo, al tacto vaginal se detecta útero de consistencia firme voluminoso, irregular,
aproximadamente de 12 cm. anexos libres. En esta paciente el diagnóstico más probable es:

a) Adenomiosis uterina.
b) Cáncer cervicouterino.
c) Miomatosis uterina.
d) Hiperpalsia adenomatosa de endometrio.
MIOMATOSIS UTERINA

Definición:

Tumor benigno que se origina en el miometrio, por lo que su componente histológico


predominante es el tejido muscular y, en menor medida, el conectivo y fibroso. El único
tratamiento efectivo es el quirúrgico; sin embargo, sólo requieren ser tratados aquellos que
producen síntomas.

Evaluación y Diagnóstico:

Historia:

1. El síntoma más frecuente suele ser la hemorragia uterina.


2. Los síntomas principales están relacionados con el crecimiento del tumor.
3. La paciente puede notar una masa en hipogastrio o abdomen inferior.
4. La masa se puede asociar a dolor pélvico, o manifestaciones por compresión de
órganos o estructuras vecinas.
5. Puede haber alteración de la fertilidad.

Examen Físico:

1. Se debe realizar con la vejiga y el recto vacuo.


2. El hallazgo primordial es el aumento de volumen y consistencia del útero, el cual
puede ser simétrico (nódulos submucosos) o irregular (nódulos intramurales o
subserosos).

Exámenes Auxiliares:

1. Papanicolau cérvico–vaginal: Indicado siempre; permite descartar neoplasia epitelial


cervical o cáncer infiltrante de cérvix.
2. Ultrasonido transabdominal y transvaginal: Indicado siempre; permite evaluar la
localización, tamaño y número aproximado de miomas.
3. Hemoglobina, hematocrito: Indicado cuando hay historia de sangrado; orienta en la
severidad del sangrado y anemia.
4. Hemograma y VSG: Indicado cuando hay historia de fiebre; si es anormal sugiere
infección o necrosis del mioma (puede ser apropiado descartar infección de otro
órgano o sistema).
5. Grupo sanguíneo y factor Rh: Si hay anemia severa o en el preoperatorio.
6. Perfil de coagulación (tiempo de protrombina y de tromboplastina parcial, recuento
de plaquetas): Si hay historia de sangrado exagerado.
7. Gonadotrofina coriónica (sub–unidad ] sérica): Permite descartar posible embarazo
en casos de ciclos irregulares, retraso menstrual o útero de consistencia blanda.
Diagnóstico Diferencial:

1. Embarazo.
2. Tumor de ovario.
3. Enfermedad inflamatoria pélvica, complejo inflamatorio anexial pélvico.
4. Endometriosis.
5. Adenomiosis.
6. Tumor extragenital: colon, retroperitoneo.

Referencias Bibliográficas:

1. Hillard PA. Benign Diseases of the Female Reproductive Tract: Symptoms and
Signs. En: Berek JS, Adashi EY, Hillard PA, eds. Novak's Gynecology. Baltimore:
Williams and Wilkins, 1996:331-97.
2. Hutchins FL, Greenber MD. Miomas Uterinos: Diagnóstico e Indicaciones de
Tratamiento. Clinicas de Ginecología y Obstetricia. Temas Actuales. 1995;5:609-14.
3. Davis KM, Sclass WD. Tratamiento Médico para Miomatosis Uterina. Clinicas de
Ginecología y Obstetricia. Temas Actuales. 1995;5:671-81.
4. Selwyn P, Oskowitz MB. Leiomyomata Uteri. En: Friedman EA, ed. Gynecological
Decision Making. St. Louis: Mosby, 1983:148-9.
5. Diaz Huamán V. Tumores Benignos del Aparato Reproductor Femenino. En: Ludmir
A, Cervantes R, Castellano C, eds. Ginecología y Obstetricia, Prevención -
Diagnóstico - Tratamiento. Lima: Concytec, 1996:907-25.

8.- Femenino de 34 años con antecedentes patológicos de hipertensión arterial sistémica


de 4 años de evolución, bien controlada tratada con IECAS , actualmente cursa con 7
semanas de gestación , signos vitales dentro del parámetro normal y exámenes de
laboratorio sin alteraciones, se refiere asintomática , la conducta más adecueda a seguir
es:

a) Mantener el tratamiento y asociar alfametildopa para disminuir los riesgos fetales


de los IECAs.
b) Mantener el tratamiento y asociar hidralacina para disminuir los riesgos maternos
de los IECAs.
c) Mantener el tratamiento dado el buen control tensional.
d) Suspender los IECAs dado el riesgo que presentan para el feto.
El uso de IECA y ARAII durante el segundo y tercer trimestre de embarazo está
contraindicado, debido a que estos medicamentos inducen toxicidad fetal (descenso de la
función renal, oligohidramnios, retraso en la osificación del cráneo) y toxicidad neonatal
(insufi ciencia renal, hipotensión, hiperpotasemia).
En cuanto a su uso durante el primer trimestre de embarazo, un estudio publicado en el
año 2006, 1 mostraba un incremento de la incidencia de malformaciones congénitas, en
particular malformaciones cardiacas, en niños nacidos de madres expuestas a IECA
durante el primer trimestre de embarazo en comparación con las mujeres que no
recibieron tratamiento antihipertensivo o que recibieron tratamiento con otros
medicamentos antihipertensivos. Estudios posteriores realizados no han confirmado a día
de hoy los resultados de este estudio. En lo referente a los ARAII, no se dispone de
estudios epidemiológicos analíticos apropiados, por lo que no se puede descartar que exista
el mismo riesgo que para los IECA.
A pesar de estas incertidumbres, el Comité de Medicamentos de Uso Humano (CHMP) de la
Agencia Europea de Medicamentos (EMEA) ha recomendado prudencialmente evitar el uso
de IECA y ARAII durante el primer trimestre del embarazo.

Cooper WO et al. Major congenital malformations after fi rst-trimester exposure to ACE


inhibitors. N Engl J Med 2006; 354 (23): 243- 51. ref.: 2008/10, junio.

9.- Femenino de 23 años, G1 en trabajo de parto prematuro con embarazo de 30 semanas


de gestación. A pesar del uso de agentes tocolíticos, estos no han dado resultado. La
inducción para la maduración pulmonar del producto se debe realizar por medio de:

a) Betametasona
b) Sulfato de magnesio
c) Hidroxiprogesterona
d) Clorprocaína

La utilización de betametasona como inductor de madurez pulmonar fetal (IMPF) disminuye


la morbilidad neonatal relacionada con prematurez pero su efecto diabetógeno materno ha
sido poco estudiado.

La revisión Cochrane de un ciclo único de corticosteroides se actualizó en 2006. En esta


actualización se incluyeron 21 estudios con un total de 3885 mujeres y 4269 lactantes.
En la revisión se descubrió que la administración de determinados corticosteroides a
mujeres con riesgo de tener un parto prematuro reduce considerable los riesgos de
complicaciones relacionadas con la prematurez como muerte fetal y neonatal combinada,
síndrome de dificultad respiratoria, hemorragia cerebroventricular, enterocolitis
necrotizante, infecciones sistémicas y retraso en el desarrollo durante la niñez. Los
beneficios estaban presentes cuando el tratamiento se iniciaba entre las 26 y las 35
semanas de gestación y en los niños que nacían entre 1 y 7 días después de haber
comenzado el tratamiento; también se observaron beneficios en los subgrupos de mujeres
con rotura prematura de membranas y trastornos hipertensivos. La muerte fetal y neonatal
combinada se redujo incluso en neonatos que nacieron a menos de las 24 horas de haber
administrado la primera dosis.

No se demostraron beneficios cuando el tratamiento comenzó antes de las 26 semanas de


gestación, tampoco se observaron beneficios en los recién nacidos antes de las 26 semanas
de gestación ni en los que nacieron después de 7 días o más de la administración del
tratamiento.

En el caso de los neonatos que nacieron después de las 36 semanas hubo una tendencia a
aumentar la muerte fetal y neonatal combinada.

Se observó una reducción en el peso al nacer en los neonatos que nacieron entre los días 1 y
7, al igual que en los que nacieron más de 7 días después del primer tratamiento.

Un estudio que reclutó mujeres con preeclampsia severa sugirió que las mujeres tratadas
tenían un mayor riesgo de sufrir diabetes gestacional.

La evidencia epidemiológica y en animales sugiere que pueden haber efectos adversos a


largo plazo por la exposición prenatal a los corticosteroides, entre ellos la alteración de la
tolerancia a la glucosa y la hipertensión. Los estudios en animales también han sugerido que
afecta el crecimiento del cerebro.

1. National Institute of Health (NIH). Consensus Conference; Effect of corticos-


teroide for fetal maturation on perinatal outcomes. JAMA 1994;(12):1-19.
2. White A, Marcucci G, Andrews E, Edwards K. Antenatal steroids and neonatal
outcomes in controlled clinical trials of surfactant replacement. Am J Obstet
Gynecol 1995; (173):286-90.
3. Klauss MH, Fanaroff AA, Martin RJ. Problemas respiratorios. En: Asistencia del
recién nacido de alto riesgo. 2 ed. La Habana: Editorial Científico-Técnica,
1981:194.
4. Avery M, Frank N, Gribetz I. The inflationary force produced by pulmonary
vascular distention in excised lungs. The possible relation of this force to that
needed to inflatc the lungs at birth. J Clin Invest 1959;38:456.
5. Chu J, Clements J, Cotton E. Neonatal pulmonary ischemia. Pediatrics 1965;40:733.
6. Liggins GC, Howle RN. A controlled trial of antepartum glucocorticoid treatment
for prevention of respiratory distress syndrome in premature infants. Pediatrics
1972;50: 515-25.
7. Wright LL, Verter J, Younes N. Antenatal corticosteroids administration and
neonatal outcome in infants 501 to 1500 g. Am J Obstet Gynecol 1995; (173):263.

10.- Femenino de 31 años se envía de alta con diagnóstico de enfermedad inflamatoria


pélvica, regresa a los 15 días con temperatura de 38.5 °c, mal estado general y datos de
irritación peritoneal, ¿El diagnóstico más probable es?

a) Endometritis
b) Absceso tubo-ovárico
c) Hidrosalpinx
d) Ooforitis aguda

La enfermedad inflamatoria pélvica (EIPA) es un síndrome clínico caracterizado por la


infección del tracto genital superior que se produce casi siempre por vía ascendente desde
el cuello uterino. El impacto que la infección pélvica ejerce sobre la condición física de la
mujer va desde la infección asintomática o silente a una mayor morbilidad que en algunos
casos puede llegar hasta la muerte. Incluye una variedad de condiciones inflamatorias que
afectan el tracto genital superior. Los Centros de Control de Enfermedades (C .D. E.) la
definen como un síndrome agudo debido al ascenso de microorganismos de la vagina o el
cuello uterino al endometrio, trompas uterinas y en ocasiones a las estructuras vecinas
(ovarios, peritoneo y cavidad pelvianas).

En el momento actual se incluyen como principales agentes etiológicos de la E.I.P.A la


Neisseria gonorrhedae, las clamydias y los anaerobios. Otros microorganismos como los
microplasmas y los actinomices se están observando con frecuencia.

La presencia de anaerobios así como de bacterias aerobias puede deberse a un fenómeno


de sobre infección secundaria. Hay autores que señalan que excepto para el gonococo y la
Clamydia trachormatis, no existen datos suficientes que permitan afirmar que otras
bacterias tengan un papel primario en la infección de unas trompas sanas. Una vez alterada
la integridad anatómica de la trompa, se produciría la infección mixta o poli microbiana.
El absceso tubo ovárico es una formación inflamatoria que compromete el ovario y la
trompa y puede ser uni o bilateral. En este absceso las estructuras comprometidas
están infectadas y contienen pus. Este proceso inflamatorio es secundario a un proceso
infeccioso de la pelvis, habitualmente producido por gérmenes muy patógenos, que llegan
al tracto genital a través de una relación sexual, es decir corresponde a una complicación
severa de una enfermedad de transmisión sexual.

Se caracteriza por aumento de volumen del ovario y trompa, los que se encuentran
adheridos entre sí producto de esta infección, además el proceso infeccioso se extiende
habitualmente a otras estructuras y órganos pelvianos, los que están muy inflamados y
adheridos í formando lo que se denomina plastrón.

El tratamiento se inicia médicamente con antibióticos de amplio espectro para cubrir


tanto gérmenes aeróbicos como anaeróbicos, generalmente requiere de hospitalización
para iniciar una terapia agresiva endovenosa con los antibióticos y para monitorizar
adecuadamente a la paciente, pues la infección produce compromiso del estado general
pudiendo llegar hasta la sepsis generalizada.

Diagnóstico

El cuadro clínico se sospecha cunado una paciente consulta por dolor abdominal intenso,
progresivo, fiebre y compromiso de su estado general, habitualmente en el examen se
encuentra un distensión abdominal y a la palpación del abdomen hay dolor, y signo de
blumberg positivo o irritación peritoneal. El Tacto vaginal demuestra fondos de saco
vaginales abombados y dolorosos y habitualmente el cuello del útero lateralizado y
doloroso a la movilización si el compromiso es unilateral, además de palpar una masa para
uterina irregular y sensible.
BIBLIOGRAFÍA:

1. Botella Llusiá,J.Clavero Núñez,J.A:Tratado de Ginecología.14ª edición. Ed.Diaz de


Santos.pág 833-844.Madrid,1993.
2. Brunham,R,C:Infectionin woman and ectopic pregnancy. Am J Obstet
Gynecol.67:722,1999.
3. Cates,W,Wasserheit,J,N:Genital Infection Epidemiology and sequeale.Am J Obstet
Gynecol 164-1771,1998.
4. Keit,L,G; Berger,G,S:On the causation of pelvic inflammatory disease. Am J Obstet
Gynecol 149-215,2002.
5. Muller,B,R;Allen,J,et al.Pelvic Inflamatory disease after histerosalpingography.Brit
J Obstet Gynecol,91-1181,1999.
6. Toth,A,O Leary,W,M: Evidence of microbial transfer by espermatozoo.Am J O
bstet Gynecol 59-556,2003.
7. Varela,R,et col:Abceso Tuboovárico,Acta médica Portuguesa ,p:537-542,Vol.
8,2001.
8. Sopper,D,E:Pelvic Inflamatory disease.Infections disease.Clin of North America
.831-840,vol 8;n 4.Dec 2003.

11.- Paciente femenino de 28 años de edad con deseo de un embarazo, antecedentes de


G3 A2 - P1 se le realiza una histerosalpingografía, se constata que existe un síndrome de
Asherman. Ello significa que se trata de:

a) Útero bicorne
b) Endometriosis en la trompa
c) Sinequias uterinas
d) Insuficiencia istmico cervical

El síndrome de Asherman es una enfermedad ginecológica rara que se caracteriza por la


presencia de sinequias (adherencias) intrauterinas que pueden ocasionar amenorrea
(ausencia de períodos menstruales regulares) e infertilidad.

En 1894 Heinrich Fritsch describe por primera vez la presencia de sinequias intrauterinas
de tipo postraumático, en una paciente que desarrolló una amenorrea secundaria a un
curetaje. Posteriormente en 1927 Bass informó de veinte casos de atresia (oclusión de una
abertura natural) cervical tras abortos inducidos, pero no fue hasta 1948, cuando Joseph
G. Asherman recopiló la información hasta entonces existente y acuñó el nombre con el que
se conoce actualmente a la enfermedad.

Asherman describió originalmente dos tipos diferentes de amenorrea secundaria, en


función de su etiología (estudio de las causas de las enfermedades): la amenorrea
traumática atrética, debida a estenosis del orificio cervical interno y la amenorrea debida
a adherencias intrauterinas. Posteriormente ambas entidades se agruparon en una única
entidad bajo el nombre de síndrome de Asherman.

Suele presentarse en mayor proporción tras dilataciones y curetajes uterinos de repetición


y sobre todo si se realizan durante el embarazo o si existe infección uterina en el momento
en el que se realizan estas intervenciones.

Las adherencias intrauterinas pueden producirse debido a cualquier factor que lleve a una
destrucción de las paredes del miometrio (capa muscular de la pared del útero). Sin
embargo, hay que distinguir entre factores predisponentes, siendo el principal de ellos el
embarazo y factores causales, entre los que se encuentran: traumatismos uterinos,
intervenciones quirúrgicas que afecten al útero, agentes físicos o químicos e infecciones
uterinas por tuberculosis o esquistosomiasis. En cualquier caso, el factor más importante es
el trauma uterino en el momento del parto o el puerperio.

El cuadro clínico es muy variable y las manifestaciones clínicas varían con el grado de
oclusión de la cavidad uterina y la severidad de las adherencias, pudiendo presentarse:
esterilidad cuando la oclusión de la cavidad uterina incluye porciones proximales (más cerca
de un centro, tronco o línea media) de las trompas de Falopio o cuando las adherencias
impiden la nidación del huevo; las pacientes presentan con frecuencia amenorrea,
oligomenorrea (disminución de la frecuencia de las menstruaciones), dismenorrea
(menstruación dolorosa) y abortos repetidos.

Hysteroscopic treatment of severe Asherman's syndrome and subsequent fertility.


Capella-Allouc S; Hum Reprod, 1999 May.
12.- Femenino de 37 años, es atendida en consulta externa con reporte de papanicolaou que
reporta un NIC I, la especuloscopía se observa cérvix con ectropión periorificiario.
El método más sensible para corroborar el diagnóstico en esta paciente es:

a) Papanicolaou.
b) Exudado vaginal.
c) Prueba de koh.
d) Colposcopía.
9.5.2 Las pacientes a quienes se les realizó citología cervical, cuyo resultado es LEIBG
(infección por VPH, displasia leve o NIC 1); LEIAG (displasia moderada y grave o NIC 2 y 3)
o cáncer deben enviarse a una clínica de colposcopía, para realizar estudio colposcópico.
9.5.3 Si el resultado de la citología es LEIBG, la colposcopía es satisfactoria y sin
evidencia de LEIBG, se realizará control citológico en un año (Apéndice Normativo A)
9.5.4 Si la citología es de LEIBG, la colposcopía es satisfactoria y existe evidencia de
lesión, se debe tomar una biopsia dirigida.
9.5.4.1 Si la biopsia dirigida es negativa, se realizará nueva colposcopía para verificar el
diagnóstico y en caso necesario, tomar nueva biopsia dirigida y revalorar.
9.5.4.2 Si la biopsia dirigida es reportada como LEIBG se podrá dar tratamiento
conservador: criocirugía, electrocirugía o laserterapia (sólo si cumple con las condiciones
referidas en el Apéndice 1) o se podrá mantener a la paciente en vigilancia en la clínica de
colposcopía, con colposcopía y estudio citológico cada seis meses, durante 24 meses.
Jueves 31 de mayo de 2007 DIARIO OFICIAL (Primera Sección)
9.5.4.3 Si la biopsia dirigida es reportada como LEIAG (Lesión Intraepitelial Escamosa de
Alto Grado) se realizará tratamiento conservador (electrocirugía o laserterapia). En las
mujeres posmenopáusicas, dependiendo de las condiciones anatómicas del cérvix, se
realizará tratamiento conservador en la clínica de colposcopía o tratamiento quirúrgico
(histerectomía extrafascial) en el servicio que corresponda.
9.5.4.4 Si la biopsia dirigida reporta cáncer microinvasor o invasor, la paciente se
transferirá a un Servicio o Centro Oncológico para su tratamiento correspondiente.
9.5.4.5 Si la citología reporta LEIBG y la colposcopía es no satisfactoria, se tomará
cepillado endocervical (Apéndice Normativo A)
9.6 En caso de colposcopía no satisfactoria, negativa a LEIBG y con cepillado endocervical
negativo, se continuará su control en la clínica de colposcopía en seis meses, con colposcopía
y citología.
9.6.1.1 Si el cepillado endocervical reporta LEIBG se tratará a la paciente como LEIAG,
con métodos conservadores escisionales.
Jueves 31 de mayo de 2007 DIARIO OFICIAL (Primera Sección)
Modificación a la Norma Oficial Mexicana NOM-014-SSA2-1994, Para la prevención,
detección, diagnóstico, tratamiento, control y vigilancia epidemiológica del cáncer
cérvico uterino.
Al margen un sello con el Escudo Nacional, que dice: Estados Unidos Mexicanos.- Secretaría
de Salud.
MODIFICACION A LA NORMA OFICIAL MEXICANA NOM-014-SSA2-1994, PARA LA
PREVENCION,
DETECCION, DIAGNOSTICO, TRATAMIENTO, CONTROL Y VIGILANCIA
EPIDEMIOLOGICA DEL CANCER CERVICO UTERINO.

13.- Es el caso de paciente femenino de 17 años la cual presenta amenorrea acompañada


de profundas alteraciones del olfato. ¿Cuál de los siguientes diagnósticos es el más
probable?

a) Síndrome de amenorrea-galactorrea.
b) Amenorrea de causa uterina.
c) Síndrome de ovario poliquístico.
d) Amenorrea por alteración hipotalámica.

• AMENORREA HIPOTALAMICA. (Hipogonadotrópica)


– Psicógena. (Stress emocional)
– Anorexia nerviosa. (Deficiencia nutricional)
– Ejercicio excesivo. (Carrera de fondo, natación, gimnasia, ballet)
– Fármacos. (Fenotiazina, reserpina, bloqueadores ganglionares,
anticonceptivos)
– Pseudociésis.
– Síndrome de Kallman (Deficiencia selectiva de gonadotropinas y anosmia).
El ejemplo clásico de la alteración hipotalámica que lleva a desórdenes del ciclo menstrual
es el Síndrome de Kallman.

Lectura Recomendada:
Etiopatogenia de la amenorrea hipotalámica funcional Interacción de las respuestas
hormonales del Sistema Nervioso Central y Neuropéptidos Periféricos.

Revista Argentina de Endocrinología y Metabolismo


Copyright © 2008 por la Sociedad Argentina de Endocrinología y Metabolismo
Vol 45 • No. 2.
14.- Femenino de 40 años de edad G.3 P.1 C 2, la cual es diagnosticada por miomatosis
uterina de pequeños y medianos elementos sintomáticos, sus antecedentes refieren
cirugías pélvicas previas, el tratamiento de elección es:

a) Progesterona.
b) Histerectomía total abdominal.
c) Observación
d) Análogos de GnRH.

CUADRO CLINICO

La miomatosis uterina muestra manifestaciones clínicas en menos del 50%, de estas las más
frecuentes son:

1. Hemorragia uterina anormal.


2. Dolor.
3. Distensión abdominal.
4. Compresión genitouterina.
5. Compresión gastrointestinal.
6. Compresión pélvica.

10. DIAGNOSTICO

El diagnóstico se realiza a través de imagenología:

1-Ecografía.
2-TAC
3-Rayos X
4-Histeroscopia.

11. TRATAMIENTO

La miomatosis uterina debe ser tratada cuando produzca cualquiera de las manifestaciones
clínicas anotadas, toda paciente que se programe para histerectomía debe tener
previamente legrado biopsia

Lo podemos dividir en Conservador o Radical.


1. CONSERVADOR:

Este tratamiento se puede instaurar en pacientes con deseo de preservar el útero.


Igualmente se puede subdividir:
-Expectante
-Quirúrgico: miomectomía
-Medico

Tratamiento expectante: Esta indicado en pacientes, cuyos síntomas son leves y no deseen
o tengan alguna contraindicación medica para tratamiento quirúrgico. En ellas se
recomienda controles clínicos y ecográficos cada 6 meses a 1 año.

Tratamiento médico.

AINES

Análogos GnRH:

Progestágenos:
Andrógenos.

Antiandrógenos

Tratamiento quirúrgico:

MIOMECTOMIA:

1. Criterios del ACOG en pacientes infecundas.

Procedimientos:

• Vía endoscópica:
Laparoscopia: Miomas subserosos sesiles o pediculados < 5cm.
Histeroscopia: miomas submucosos

• Vía laparotomía
Aquellos miomas que se salgan de las características anteriores.

Indicaciones:

 Hemorragia anormal.
 Perdida reproductiva.
 Infecundidad.
 Dolor.
El tratamiento previo con análogos esta indicado cuando se desee disminuir el tamaño del
mioma para prevenir sangrado quirúrgico.

Contraindicaciones:

 Embarazo.
 Cáncer endometrial.
 Infecciones.
 Dificultad técnica.

2. Criterios del ACOG para Miomectomía en pacientes que desean conservar el útero.

Procedimiento:

• Vía endoscópica.
• Vía abdominal.
• Vía vaginal.

Indicación:

a. Presencia de uno o dos Leiomiomas asintomáticos de tamaño tal que se pueden palpar
por vía abdominal y constituyen una preocupación para la paciente.

b. Pacientes ovulatorias con miomas como posible causa de hemorragia uterina excesiva,
demostrada por cualquiera de las siguientes circunstancias:
• Hemorragia profusa: de duración mayor de 8 días.
• Anemia por pérdida sanguínea aguda o crónica.

2. RADICAL:

HISTERECTOMIA

Para pacientes post menopáusicas, con paridad satisfecha o sin deseo de preservar el
útero.

Criterios del ACOG para Histerectomía por miomas.

1. Presencia de 1, 2, o 3 Miomas asintomáticos de tamaño tal que son palpables por vía
abdominal y preocupan a la paciente.

2. Hemorragia uterina excesiva.

• Duración mayor de 8 días.


• Anemia por pérdida sanguínea aguda o crónica.
1. Molestias pélvicas producidas por los miomas: signos compresivos.

Contraindicaciones:

1. Deseo de conservar la fecundidad.


2. Miomas asintomáticos.
3. Contraindicación médica o dificultades técnicas para la cirugía.

Guarnaccia M. and Rein M. Traditional Surgical Approaches to Uterine Fibroids


Abdominal. Myomectomy and Hysterectomy. Clinical Obstetrics and Gynecology
2001. 44.2. 385-400.
- Milad. M and Sankpal R. Laparoscopic Approaches to Uterine Leiomyomas.
Clinical Obstetrics and Gynecology. 2001. 44-2. 401-411.
- Carlson K. et al. Indications for Hysterectomy. N. Engl. J. Med. 1993. 328(12) 56-

15.- En la sala de urgencias recibe a una paciente que inicia con convulsiones por
preclampsia usted decide administrar el siguiente fármaco ya que es el de elección en
ésta patología:

a) Sulfato de magnesio.
b) Diacepam.
c) Fenitoína.
d) Donadores de óxido nítrico.

Manejo de la Preeclampsia

1. Manejo ambulatorio: HTA sin proteinuria significativa, se recomienda el reposo en cama.


Monitoreo de TA, peso, presencia de proteínas en orina. Ecografías periódicas para ver el
feto y evaluar posibles retardo de crecimiento.
2. Manejo hospitalario: para mujeres con HTA inducida por el embarazo y 2+ o más o
proteinuria significativa y en quienes falló el manejo ambulatorio.
3. Laboratorio y evaluación del peso: debe realizarse diariamente. Evaluación de la
dinámica fetal. Monitoreo de síntomas como cefalea, alteraciones visuales y dolor
epigástrico.
4. El parto es el tratamiento de elección: el cual debe realizarse cuando el feto está
maduro pero puede realizarse en forma temprana si la salud de la madre está en peligro o si
hay evidencia de distress fetal. El parto está indicado cuando la paciente cumple con los
criterios de preeclampsia severa. Betametasona 12.5 mg IM dos veces por día puede
estimular la maduración de los pulmones fetales.
5. Terapia antihipertensiva: está indicada sólo si la TA es persistentemente > 160/110 , es
importante disminuir la TA hasta una diastólica de 90 a 100 mmHg porque la presión normal
podría resultar en hipoperfusión de la placenta. Los diuréticos nunca están indicados, estas
pacientes ya son hipovolémicas. Los IECA no deben ser usados durante el embarazo. Las
medicaciones de largo plazo, incluyen alfa metildopa, atenolol y labetalol.
6. Terapia anticonvulsivante:
A- Profilaxis de las convulsiones: está indicada en todas las pacientes pre-eclámpticas
durante el trabajo de parto y el parto y por un mínimo de 24 hs luego del mismo. Algunos
mantienen la terapia con magnesio hasta que comienza la diuresis. El Sulfato de Magnesio
es la droga de elección. La dosis profiláctica es de 4 a 6 g de sulfato de magnesio IV y
continúa con 2 g c/ hora.
B- Tratamiento de las convulsiones: Sulfato de Magnesio 1 g/min IV hasta controlar
las convulsiones hasta un máximo de 4 a 6 g. El nivel terapéutico es de 4 meq/l. Toxicidad
del magnesio: ausencia de reflejo patelar, debilidad muscular, parálisis respiratoria y
depresión cardíaca, 10 ml al 10 % de gluconato de calcio puede ser administrada IV. La
terapia con sulfato de magnesio continúa por lo menos 24 horas en el post parto, la terapia
puede detenerse si la excreción urinaria es > 200 ml/h por cuatro horas consecutivas.
C- Prevención: 81 mg de aspirina diarios pueden ser administrados luego del primer
trimestre en mujeres con hipertensión crónica o historia previa de preeclampsia, sin
embargo la eficacia de esta indicación ha sido cuestionada.

Bibliografía:

Myers JE, Baker PN. Hupertensive diseases and eclampsia. Curr Opin Obstet Gynecol
2002; 14: 119-125.

2. Tierney, McPhee, Papadakis. Diagnóstico clínico y tratamiento 2003. 38ª ed, México,
Manual Moderno, 2003: 770-773.

3. Wilson MI, Goodwin TM, Pan VI, Ingles SA. Molecular epidemiology of preeclampsia.
Obstet and Gynecol Survey 2003; 58(1):39-66.

4. Burrow GM. Complicaciones médicas durante el embarazo. 4ª ed, México, McGraw-Hill


panamericana: 1996: 1-25.

5. Guyton AC, Hall JE. Embarazo y lactancia en: Tratado de fisiología médica, 10ª ed,
México, McGraw-Hill Interamericana 2001: 1135-45.

6. Vaticon D. Fisiología de la fecundación, embarazo, parto y lactancia, en: Tresguerres


JAF. Fisiología Humana. México, Interamericana McGraw-Hill, 1992: 1086-1109.

7. Pridjian G, Puschett JB. Preeclampisa. Part 1: Clinical and Pathophysiologic


Considerations. Obstet and Gynecol Survey 2002; 57 (9): 598-618.
8. Pridjian G, Puschett JB. Preeclampisa. Part I1: Experimental and Genetic Considerations.
Obstet and Gynecol Survey 2002; 57 (9): 619-40.

9. IMSS. Embarazo de alto riesgo. Guía diagnóstica terapéutica. Rev Med IMSS 1998;
36(1):45-60.

16.- An 18-year-old woman complains of myalgias, a sore throat, and painful mouth sores
for 3 days´duration. Her temperature is 38.2 C (100.8 F), blood pressure is 110/80 mm Hg,
pulse is 84/min, respirations are 15/min. Her gingival are edematous and erythematous,
and there are vesicles on her right upper and lower lips. Her pharynx is mildly
erythematous but without exudates, and there is tender mobile cervical lymphadenopathy.
Her breath is not fetid, and the dentition is normal. Which of the following is the most
likely causal agent?

a) Actinomyces israelii
b) Herpes simplex virus 1
c) Nocardia asteroids
d) Streptococcus pyogenes

Las infecciones por herpes simplex son comunes en la práctica diaria, y con frecuencia el
paciente acude a los servicios de urgencia. Estas infecciones son ocasionalmente
recurrentes, generalmente dolorosas y asociadas con síntomas sistémicos, por lo cual el
médico de urgencias debe estar familiarizado con el cuadro clínico y su manejo.

Existen dos variedades de virus del Herpes simplex (VHS) capaces de causar infección en
el hombre: el tipo 1 (VHS-1) y el tipo 2 (VHS-2) que se distinguen entre sí por varias
características, incluyendo sus comportamientos clínico y epidemiológico, antigenicidad,
composición del ADN y la sensibilidad a diferentes agentes físicos y químicos (Cuadro No.1)

Cuadro No. 1

DIFERENCIAS ENTRE LOS VIRUS HERPES SIMPLEX TIPOS 1 Y 2

Características clínicas VHS-1 VHS-2

Vía de transmisión Oral Genital


Síndromes Oral-facial Genita
característicos Ocular PerianaL
Encefalitis Neonata
Paroniquia Paroniquia
Reactividad a antígenos VHS-1 específico l VHS-2 específico
monoclonales específicos
El VHS-1 es de localización primordialmente extragenital, con predilección por los tejidos
de origen ectodérmico, mientras que el VHS-2 corresponde al "Herpes progenitalis"
descrito por separado, dentro de las infecciones de transmisión sexual.

La región oral es la localización habitual del Herpes simplex 1, el cual es causa frecuente de
lesiones orofaciales recurrentes y de otro tipo de enfermedades (encefalitis).

EPIDEMIOLOGIA

El ser humano es el único reservorio natural conocido del virus herpes simplex, aunque
algunos animales de experimentación pueden infectarse con facilidad.

La infección primaria del VHS-1 ocurre sobre todo durante la infancia, mientras que el tipo
2 se presenta en la adolescencia y adultos jóvenes activos sexualmente. Las tasas de
infección son inversamente proporcionales al estrato socioeconómico.

El principal mecanismo de transmisión es el contacto directo con las secreciones


infectadas. El VHS-1 se trasmite por saliva y el VHS-2 por vía genital. Aunque los títulos
virales son más altos cuando existen lesiones activas, también es frecuente la liberación
viral en infectados asintomáticos. Por lo tanto, la transmisión viral puede efectuarse aún en
ausencia de lesiones activas.

La persistencia de la infección y la recurrencia de las lesiones son un fenómeno frecuente


tanto para el VHS-1 como para el VHS-2 y por lo común se producen por reactivación
endógena. Los factores que la precipitan van desde la luz solar, el viento, traumatismos
locales, fiebre, menstruaciones y hasta estrés emocional.

DIAGNOSTICO

Cuadro Clínico. Los cuadros clínicos causados por este virus se suelen dividir en dos
grupos: el debido a la infección primaria y el correspondiente a la infección recurrente. En
el primer grupo se incluyen la gingivoestomatitis aguda, la vulvovaginitis aguda y la infección
herpética del ojo, que puede llegar a queratitis. Las recurrentes se circunscriben al
"Herpes labialis", queratitis, blefaritis y queratoconjuntivitis. Todos los cuadros son
autolimitados, pero tanto las formas primarias como las recurrentes, se pueden complicar.
Una de estas complicaciones es la Encefalitis herpética y el Eczema herpeticum.

Infección primaria. El primer contacto clínico de infección por virus del herpes simple
suele ser el más grave. Los enfermos aquejan fiebre, malestar general, artralgias y por
último la presencia de un grupo de vesículas sobre una base eritematosa, dolorosa,
inflamada y sensible. La gingivoestomatitis es la manifestación más común, cuya gravedad
varía desde la erosión de pequeñas áreas a la ulceración extensa de la boca, lengua y encías.
La infección puede ser bastante grave como para dificultar la ingesta de alimentos y
líquidos (odinofagia). La curación tiene lugar en 7 a 14 días, a menos que las lesiones se
sobreinfecten con estafilocos o estreptococos.

Infección recurrente. Generalmente existe prurito, dolor o molestias focales que


preceden la aparición de las vesículas. Las vesículas se rompen espontáneamente después
de unos cuantos días y sanan en una semana sin dejar secuelas.

LECTURAS RECOMENDADAS

1. Callen JP, Cooper Ma. Dermatologic emergences. Emerg Med. Clin North Am
3:641, 1985
2. Guzmán M. Herpes simple, varicela zoster. En: Medicina Interna. Segunda
Edición. Editado por F Chalem, JE Escandón, J Campos, R Esguerra.
Fundación Instituto de Reumatología e Inmunología. Editorial Presencia
Ltda. Santafé de Bogotá, 1992
3. Guerra Flecha J, Lizarraga Bonelli S. Enfermedades de transmisión sexual:
herpes genital Trib Med 79:29, 1989

Jaramillo AC. Infecciones virales de la piel y sus anexos. En: Fundamentos de Medicina.
Enfermedades Infecciosas. Cuarta edición. Corporación para Investigaciones Biológicas.
CIB. Medellín, 1989.

17.- Masculino de 33 años, acude al servicio de urgencias por referir dolor en pabellón
auricular izquierdo, dificultad para mover la hemicara del mismo lado. Antecedentes: sin
importancia. Exploración física: TA 120/80mmHg, FC 72 x´, FR 16 x´, Temp 36 °, se
observan vesículas en concha auricular y paresia de la hemicara izquierda. El agente causal
más probable en este caso es:

a) Haemophylus influenzae
b) Herpes virus
c) Virus del Papiloma Humano
d) Epstein Barr

Síndrome de Ramsay-Hunt: Representa 6.8% de las causas de parálisis faciales


intratemporales. La enfermedad se presenta en adultos entre 20 y 30 años y 50 a 70 años
de edad. Se caracteriza por la presencia de vesículas herpéticas en pabellón auricular,
conducto auditivo externo y, con menor frecuencia paladar blando y cara. Por lo general se
inicia con otalgia intensa seguida en pocos días de la erupción vesicular y parálisis facial
periférica; además puede haber hipoacusia, acúfenos y vértigo.
Escajadillo J, Oídos, naríz, garganta y cirugía de cabeza y cuello, Ed. Manual Moderno, 2ª
Edición, Pág. 152
18.- Masculino de 38 años diagnosticado por trastorno obsesivo-compulsivo. Los actos
obsesivos más frecuentes en este trastorno son:

a) Recuentos mentales.
b) Evitar pisar las cruces de las baldosas.
c) Acumulación y colección de objetos.
d) Comprobaciones y rituales de limpieza.

Síntomas del TOC

Obsesiones

Las obsesiones son ideas o impulsos no deseados que aparecen repetidamente en la mente
de la persona que padece TOC. Los pacientes suelen tener miedo a sufrir daño ellos mismos,
o alguien al que quieren, se preocupan irracionalmente por no contaminarse, o tienen una
necesidad excesiva de hacer las cosas correctamente o con perfección. Una y otra vez, la
persona piensa algo que le inquieta, como por ejemplo, "Mis manos pueden estar
contaminadas--debo lavarlas" o "Puedo haber dejado el gas abierto" o "Estoy haciéndole
daño a mi hijo." Estos pensamientos angustiosos se inmiscuyen con los otros pensamientos
del paciente, y causan ansiedad. A veces, las obsesiones son de carácter violento o sexual, o
tienen que ver con enfermedades.

Compulsiones

En respuesta a sus obsesiones, la mayoría de las personas con TOC recurren a


comportamientos repetitivos llamados compulsiones. Los más frecuentes son los rituales de
limpieza y las comprobaciones. Otros comportamientos compulsivos incluyen recuentos (a
menudo al mismo tiempo que se realizan otras acciones compulsivas, tales como lavarse las
manos), hacer repeticiones, acaparamiento, y recolocaciones sin fin de objetos en un
esfuerzo para mantenerlos perfectamente alineados. También son corrientes los problemas
mentales, tales como repetir frases mentalmente y hacer listas. Estos comportamientos, en
general, tienen por objeto proteger de peligros a la persona que padece TOC, o a los otros.

Algunas personas con TOC tienen rituales establecidos; otros tienen rituales que son
complejos y cambiantes.

El ejecutar estos rituales sólo proporciona alivio temporal de la ansiedad, pero no hacerlos
incrementa la ansiedad de la persona.
19.- El estudio donde se toman un grupo de personas sanas que se clasifican en subgrupos
según su exposición a una causa o enfermedad se conoce como:

a) Cohorte
b) Prospectivo
c) Observacional
d) Doble ciego

En los estudios de cohorte se eligen dos grupos uno de expuesto y otro de no expuestos los
cuales son seguidos a través del tiempo para detectar las posibles consecuencias.
Posteriormente se analiza la información calculado la incidencia en el grupo de expuestos y
en el grupo de no expuestos; y una vez obtenidos se calcula el Riesgo Relativo.

Ruiz M. A. Epidemiología Clínica, Panamericana, 1ª. Ed. 2004; págs: 287-289

20.- Debemos sospechar un retinoblastoma en un niño que presenta los siguientes síntomas:

a) Dolor, fotofobia y lagrimeo.


b) Lagrimeo, fotofobia y aumento del diámetro corneal.
c) Fotofobia y quemosis conjuntival.
d) Estrabismo y leucoria.

RETINOBLASTOMA

TUMOR OCULAR MÁS FRECUENTE EN INFANCIA.


1/20.000 RN
80% en < 3 años
Uni o bilaterales.
Herencia
1. AD alta penetrancia (90-95%)
2. Esporádicos
Diagnóstico:
•leucocoria
•estrabismo
•mala AV
•ojo rojo y doloroso
•celulitis orbitaria
•examen de rutina

Annals d.Oftalmologia 2001;9(2):74-92


N. Martín, MD. Coll, J. García, J. Sánchez de Toledo, E. Triviño, M. Guitart, JJ. Gil.

1Unidad Oftalmología Pediátrica. Hospital


Maternoinfantil Vall d.Hebron
2Departamento de biología-celular, fisiología e inmunología de la Universidad
Autónoma de Barcelona
3Servicio oftalmología Hospital General Vall d.Hebron
21.- Se trata de paciente de 34 años que cursa con 39 SDG; a la exploración física
reflejos patelares hiperactivos, inquieta, se reportan cifras de TA 145/95, se realiza
laboratorio que reporta proteinuria 2+,. El tratamiento mas adecuado para esta paciente
es:

a) Reposo en cama
b) Sulfato de magnesio oral
d) Propranolol
e) Inducir el trabajo de parto

Interrupción del embarazo


La mayor parte de las guías de práctica clínica que contestan esta pregunta se basan en
estudios retrospectivos o recomendaciones de los comités de expertos (Nivel III/IV). La
interrupción del embarazo se considera la mejor opción de tratamiento para la
preeclampsia y, sin duda, lo es para la embarazada al prevenir la preeclampsia severa y la
eclampsia. Sin embargo, para el feto no siempre es la mejor opción. Dos estudios
observacionales retrospectivos mostraron que a las 37 semanas la terminación del
embarazo es la mejor opción, tomando en cuenta que si existen condiciones cervicales
favorables, la inducción del parto es una via segura y de bajo riesgo para el feto (Nivel
III). En embarazos pretérmino debe considerarse la prolongación del embarazo hasta
alcanzar el mayor peso y madurez fetal.
Atención conservadora de pacientes con preeclampsia leve
Cuando el embarazo cursa entre las 28 y 34 semanas, el manejo conservador puede reducir
las complicaciones propias de la prematuridad. La decisión debe basarse en la estabilidad
del estado materno y fetal, así como en la capacidad del centro hospitalario para brindar
atención optima a un recién nacido prematuro (Nivel III). Durante el periodo del monitoreo
ambulatorio se indica a la paciente que se realice en forma externa una medición diaria de
la presión arterial, y que acuda semanalmente a la consulta para descartar evolución o
agravamiento de la enfermedad (Nivel IV). No existen estudios controlados que hayan
determinado la magnitudde los riesgos maternos y fetales durante el tratamiento
conservador de la paciente con preeclampsia leve.
Recomendaciones
La paciente con preeclampsia leve, seleccionada y clasificada después de su estancia
hospitalaria, puede atenderse en forma ambulatoria, con vigilancia semanal en la consulta,
como una medida efectiva y segura.
(Grado de recomendación B)
En las pacientes con embarazo ≥ 37 semanas y preeclampsia leve, la terminación del
embarazo es la mejor opción, valorando la inducción del parto como una medida de bajo
riesgo para el feto.
(Grado de recomendación C)
La atención conservadora del embarazo menor de 34 semanas, complicado por preclampsia
leve, mejora el pronóstico del feto.
(Grado de recomendación C)
Ginecol Obstet Mex 2010;78(6):S461-S525
Guías de práctica clínica.

Diagnóstico y tratamiento de la
preeclampsia-eclampsia
Fecha de busqueda de información: marzo 2009
Fecha de elaboracion: Septiembre 2009
Fecha de actualizacion: Junio 2012
Institución responsable: Colegio Mexicano de Especialistas en Ginecología y Obstetricia.

Coordinador del grupo


Dra. Maria Teresa Leis Marquez
Especialista en Ginecología y Obstetricia y Medicina Materno Fetal. Certificada por el
Consejo Mexicano de Ginecología y Obstetricia. Jefa de la Clínica de Medicina Materno
Fetal del Hospital Ángeles Lomas.
Coordinadora del Comité para la elaboración de las Guías de Práctica Clínica del Colegio
Mexicano de Especialistas en Ginecología y Obstetricia, periodo 2008-2010. Miembro de la
Junta de Gobierno y del Comité de Exámenes del Consejo Mexicano de Ginecología y
Obstetricia. Vicepresidenta para América del Norte de la Sociedad Iberoamericana de
Diagnóstico y Tratamiento Prenatal.

22.- Femenino de 32 años segunda gesta a término sin anormalidades en el transcurso de


éste. Inicia trabajo de parto de forma espontánea, con evolución normal hasta que se
rompe la bolsa, con una dilatación de 4 cm. A partir de entonces, comienza con hemorragia
de sangre roja, en moderada cantidad y aparecen signos de sufrimiento fetal agudo. El
estado general de la mujer es bueno y la dinámica uterina es normal. Este cuadro
corresponde a:

a) Abruptio placentae.
b) Rotura de vasa previa.
c) Rotura uterina.
d) Placenta previa central
La vasa previa es una condición de alto riesgo obstétrico en la cual vasos fetales o
placentarios cruzan el segmento uterino por debajo de presentación.
Estos vasos están desprotegidos de gelatina de Wharton o tejido placentario, lo que los
hace altamente vulnerables y susceptibles de ruptura o laceraciones en cualquier período
del embarazo, principalmente en el momento del parto. También es frecuente la compresión
de estos vasos, especialmente durante el tercer trimestre de la gestación, lo que puede
condicionar asfixia y muerte fetal.

Esta condición ocurre como resultado de que vasos velamentosos cruzan por el segmento
uterino debido a una inserción velamentosa del cordón, situación en la cual el cordón
umbilical se inserta en las membranas ovulares en vez del tejido placentario (vasa previa
tipo I), o por el cruce de vasos fetalesentre uno o más lóbulos accesorios de la placenta
(vasa previa tipo II) (Figura 1).

Figura 1. A, Vasa previa tipo I, debida a inserción velamentosa.


B, Vasa previa tipo II, debida a cotiledones aberrrantes. Reproducido de Daly-Jones y cols.
Ultrasound 2008.

Vasa previa se puede presentar si existe alguna (o ninguna) de las siguientes condiciones:
placenta baja (que puede ser causa de abortos previos seguidos por legrado o por
operaciones uterinas, que provocan cicatrices en el útero), placenta bilobada o de lóbulo
succensuriado, embarazos resultado de fertilización in vitro, o embarazos múltiples (5-6).
El sangrado por vasa previa no es doloroso. Otros sangrados por complicaciones o por
nacimiento no necesariamente son sin dolor.
Referencias:

1. Oyalese Y, Smulian JC. Placenta previa, placenta acreta, and vasa previa. Obstet Gynecol
2006; 107:
927-941.
2. Oyalesse KO, Turner M, Less C, Campbell S. Vasa previa: an avoidable obstetric tragedy.
Obstet Gynecol Surv 1999; 54: 138-145.
3. Sepúlveda W, Sebire NJ, Harris R, Nyberg DA. The placenta, umbilical cord, and
membranas. In Diagnostic Imaging of Fetal Anomalies, Nyberg DA, MaGahan JP, Pretorius
DH, Pilu G (eds). Philadelphia, PA: Lippicont Williams & Wilkins 2003, 85-132.
4. Daly-Jones E, John A, Leahy A, McKenna C, Sepulveda W. Vasa praevia; a preventable
tragedy. Ultrasound 2006; 16: 8-14.
5. Derbala Y, Grochal F, Jeanty P. Vasa previa. J Prenat Med 2007; 1: 2-13.
6. Fung TY, Lau TK. Poor perinatal outcome associated with vasa previa. It is preventable?
A report of three cases and review of the literature. Ultrasound Obstet Gynecol 1998; 12:
430-433.
7. Robert JA, Sepulveda W. Fetal exsanguination from ruptured vasa previa: still a
catastrophic event in modern obstetrics. J Obstet Gynaecol 2003; 23: 574.
8. Cordero DR, Helfgott AW, Landy HJ, et al. A non-hemorrhagic manifestation of vasa
previa: a clinicopathologic case report. Obstet Gynecol 1993; 82: 698-700.
9. Schachter M, Tovbin Y, Arieli S, et al. In vitro fertilization as a risk factor for vasa
previa. Fertil Steril 2002; 78: 642-643.

23.- Recibe usted los resultados histopatológicos de una paciente de 24 años de edad que
acudió a revisión rutinaria, los resultados reportan imagen histológica de coilocitos lo cual
sugiere infección por:

a) Herpes virus tipo 2


b) Citomegalovirus
c) Vaginosis bacteriana.
d) Virus del papiloma humano

El coilocito es un tipo de célula hallada en lesiones precancerosas cervicales. También es


común apreciarla microscópicamente en lesiones reaccionales en la mucosa oral, debido a su
similitud con la mucosa vaginal, en enfermedades como Papilomas, o en Condiloma
acuminado.

El coilocito es la manifestación clásica de la infección por VPH en la célula. Fue descrito por
primera vez por Koss y Durfee en 1956. Esta célula también ha sido llamada “célula en
balón”.
El coilocito es una célula epitelial escamosa, más comúnmente superficial e intermedia,
aunque también puede verse en células parabasales y metaplásicas. Esta célula presenta
cambios típicos tanto en su núcleo como en su citoplasma, pierde los bordes angulados
usuales de la célula escamosa superficial y su forma tiende a ser redondeada y ovoide. El
citoplasma muestra una condensación periférica que le da un aspecto en “asa de alambre”,
es opaco, denso y de aspecto céreo, anfofílico, acidofílico o de color rojo/naranja brillante.
Además se observa una gran cavidad o halo con un margen muy bien definido, de forma oval
o ligeramente festoneado. El núcleo de la célula se localiza de manera excéntrica, lo que lo
convierte en un halo paranuclear, no perinuclear. Ocasionalmente puede encontrarse
material fagocitado dentro del espacio coilocítico.

Lesión Intraepitelial de Bajo Grado. Alteraciones Celulares compatibles con Infección


(Coilocitos).

Schlecht, N.F., Kulaga, S., Robitaille, J., Ferreira, S., Santos, M., Miyamura, R.A.,
Duarte-Franco, E., Rohan, T.E., Ferenczy, A., Villa, L.L., & Franco, E.L. (2002) Persistent
Human Papillomavirus Infection as a Predictor of Cervical Intraepithelial Neoplasia.
JAMA, 286, 3106-3114

24.- Mujer de 24 años de edad atendida de parto eutócico en tococirugía , al encontrase


reparando episiotomía media, aprecia un marcado incremento en el sangrado transvaginal.
La causa más probable de este fenómeno es:

a) Retención de restos placentarios


b) Laceración vaginal
c) Laceración cervical
d) Atonía uterina
La mortalidad materna es un indicador de disparidad social y económica. Cada año en todo
el mundo mueren cerca de 600,000 mujeres, entre 15 y 49 años. Como resultado de
complicaciones relacionadas con el embarazo, el parto y el puerperio. Entre las causas
principales destacan: hemorragia postparto (25 %), se psis (15%), eclampsia (12 %) y labor
prolongada o detenida (8 %).1
1

DEFINICIÓN
Se define la hemorragia posparto (HPP) como la pérdida sanguínea de 500 mL. o más en las
primeras 24 horas después del parto o el descenso del hematocrito en un 10 % o más.
CLASIFICACIÓN
HPP Inmediata.- Pérdida sanguínea de 500 mL. o más originada en el canal del parto
dentro de las 24 horas posteriores al parto.
HPP Tardía.- Sangrado anormal o excesivo originado en el canal del parto que se presenta
entre las 24 horas posteriores al parto y el final del puerperio (42 días).
FACTORES DE RIESGO
Se han descrito los siguientes factores de riesgo para la HPP:
• Embarazo múltiple
• Polihidramnios
• Macrosomía
• Trabajo de parto disfuncional
• Gran multiparidad
• Corioamnionitis
• Uso inadecuado de oxitócicos
• Endometritis en el puerperio
• Púrpura trombocitopénica
• Anestesia general
• Administración previa y reciente de inhibidores uterinos
• Placenta previa
• Enfermedad de von Willebrand
• Desprendimiento prematuro de placenta
• Acretismo placentario
CAUSAS DE HEMORRAGIA POSPARTO
A continuación se lisian las causas más frecuentes de HPP. Según su origen, se dividen en
uterinas y no uterinas:
2
Uterinas - No Uterinas
• Hipotonía o atonía uterina
• Retención de placenta o restosplacentarios o membranas
• Placentación anormal (acretismo)
• Inversión uterina
• Traumatismo uterino (rotura uterina, desgarro cervical)
• Laceraciones del canal del parto, incluyendo la episiotomía
• Coagulopatías
• Hematomas
CUADRO CLÍNICO
La HPP se caracteriza por los siguientes signos y síntomas:
• Sangrado transvaginal de moderado a grave.
• Útero flácido (no contraído) o desgarros.
• Alteraciones hemodinámicas que se manifiestan como: mareos, sudoración, náuseas,
taquicardia y/o hipotensión arterial.
• Oliguria.
• Útero nacido (no contraído).
Al considerar la HPP, deben diferenciarse dos tipos de situaciones: la prevención orientada
a minimizar la probabilidad de que una mujer presente hemorragia tras el parto y el
manejo o tratamiento de la hemorragia, cuando ésta ya se haya producido

Referencias Bibliográficas

1. AbdaRabbo SA: Stepwise uterine devascularization: A novel technique for


management of uncontrollable postpartum hemorrhage with preservation of the
uterus. Am J Obstet Gynecol 1994;171:694-700.

2. Bakri YN, Linjawi T: Angiographic embolization for control of pelvic genital tract
hemorrhage. Report of 14 cases. Acta Obstet Gynecol Scand 1992;71:17-21.

3. Bick RL: Disseminated intravascular coagulation. Objetive criteria for diagnosis


and manegement.- Med Clin N Am Vol 1994;78(3):511-43.

4. B-Lynch C, Coker A, Lawal A II, Abu J, Cowen MC: The B-Lynch surgical technique
for the control of massive postpartum haemorrhage: an alternative to? Five cases
reported. Br J Obstet Gynaecol 1997;104:372-5.

5. Brahaems D: Unwanted hysterectomies. Lancet 1993;342-61.

6. Braithwaite JL: Variations in origin of the parietal branches of the internal iliac
artery. J Anatomy 1952;1:423-30.
25.- Femenimno de 26 años , G-1, que cursa con embarazo de 37 SDG, presenta pérdida del
estado de alerta posterior a crisis convulsivas tónico-clónicas, signos vitales con T-A
170.120mmhg Fc 95x´, reflejos osteotendinosos aumentados, se aprecia una Fc fetal de
132x´ y edema importante de miembros inferiores, no se aprecian datos de trabajo de
parto ni modificaciones cervicales, ¿El diagnóstico más probable es?

a) Pre eclampsia severa


b) Crisis epiléptica de gran mal
c) Hipertensión inducida por el embarazo
d) Eclampsia

CUADRO 1. DIAGNÓSTICO*
Preeclampsia Leve: Se presenta después de la semana 20 de gestación, durante el parto, o
en las primeras 6 semanas después de éste.
Presión sistólica ≥ a 140 mm Hg o presión diastólica ≥ 90 mm Hg
Proteinuria ≥ a 300 mg / orina de 24 hrs o su equivalente en tira reactiva
Preeclampsia Severa: Se presenta después de la semana 20 de gestación, durante el
parto, o en las primeras 6 semanas después de éste
Presión sistólica ≥ a 160 mm Hg o presión diastólica ≥ 110 mm Hg
Proteinuria ≥ a 2 gr en orina de 24 horas o su equivalente en tira reactiva
Creatinina sérica > a 1.2 mg/dl
Trombocitopenia ≤ 150 000 cel/mm3
Incremento de la deshidrogenasa láctica ≥ a 600 UI
Elevación al doble de los valores de TGO/AST o TGP/ALT
Cefalea, alteraciones visuales o auditivas
Epigastralgia
Oliguria ≤ a 500 ml en 24 horas 7
Edema agudo de pulmón
Dolor en hipocondrio derecho
Restricción en el crecimiento intrauterino
Oligohidramnios
Eclampsia Preeclampsia mas convulsiones sin otra causa. Se presenta después de la semana
20 de gestación, durante el parto, o en las primeras 6 semanas después de éste.
Síndrome de HELLP Criterios para establecer el diagnóstico del síndrome de HELLP:
Plaquetas < 100 000/mm3 TGO/AST ≥ 70U/L DHL ≥ 600U/LBilirrubina total > 1.2 mg/dl
Se presenta después de la semana 20 de gestación, durante el parto, o en las primeras 6
semanas después de éste.
Hipertensión Crónica: Se diagnostica cuando existe hipertensión arterial ≥ a 140/90 mm
Hg antes de la semana 20 de gestación o si persiste después de doce semanas posteriores
al parto.
Las pacientes con hipertensión crónica deben ser evaluadas antes del embarazo para
determinar la severidad de la hipertensión y facilitar la planeación de un embarazo
mediante el cambio de medicamentos y de hábitos higiénicos y dietéticos para evitar
complicaciones.

Hipertensión Gestacional: Presencia de hipertensión arterial ≥ a 140/90 mm Hg después


de la semana 20 de gestación y se mantiene hasta las doce semanas después del parto
Ausencia de proteinuria
Presencia o no de cefalea, acúfenos y fosfenos
Después de 12 semanas de la interrupción del embarazo se revalorará la presencia de
hipertensión, si continúa, se reclasifica como hipertensión crónica: es un diagnóstico
retrospectivo. 8 Si no hay, se clasifica como hipertensión transitoria.

1. Aagaard-Tillery KM, Belfort MA. Eclampsia: morbidity, mortality, and management.


Clin Obstet Gynecol 48:12-23, 2005.

2. Atallah AN, Hofmeyr GJ, Duley L. Calcium supplementation during pregnancy for
preventing hypertensive disorders and related problems. Cochrane Database Syst Rev
1:CD001059, 2001.

3. Barton JR, Sibai BM. Diagnosis and management of hemolysis, elevated liver
enzymes, and low platelets syndrome. Clin Perinatol 31:807-33, 2004.

4. Baxter JK, Weinstein L. HELLP syndrome: the state of the art. Obstet Gynecol Surv
59:838-45, 2004.

5. Cetin A. Eclampsia. In Mohler III ER, Townsend RR. Advanced therapy in


hypertension and vascular disease. Ontario: B.C. Decker Inc. pp. 407-15, 2006.

6. Cetin A. Hemolysis, elevated liver enzymes, and low platelets (HELLP). In Mohler III
ER, Townsend RR. Advanced therapy in hypertension and vascular disease. Ontario: B.C.
Decker Inc. pp. 416-20, 2006.

26.- Femenino de 36 años nulípara tras 2 años de relaciones sexuales sin contracepción, que
desde hace 1 año presenta dismenorrea, dispareunia y sangrado vaginal intermenstrual. ¿Su
primer orientación diagnóstica es?

a) Insuficiencia luteínica.
b) Enfermedad inflamatoria pélvica.
c) Dismenorrea funcional.
d) Endometriosis.
La endometriosis consiste en la aparición y crecimiento de tejido endometrial fuera del
útero, sobre todo en la cavidad pélvica como en los ovarios, detrás del útero, en los
ligamentos uterinos, en la vejiga urinaria o en el intestino. Es menos frecuente que la
endometriosis aparezca fuera del abdomen como en los pulmones o en otras partes del
cuerpo.

La endometriosis es una enfermedad relativamente frecuente, que puede afectar a


cualquier mujer en edad fértil, desde la menarquia hasta la menopausia, aunque algunas
veces, la endometriosis puede durar hasta después de la menopausia. La endometriosis
altera la calidad de vida de las mujeres que la padecen, afectando a sus relaciones de
pareja, familiares, laborales y de reproducción.

Síntomas

Los síntomas clásicos son la dismenorrea, dolor pélvico, dispareunia, sangrados

intermestruales y en muchos casos, esterilidad.

El dolor no tiene que ver con el tamaño y la severidad de la lesión; generalmente cuanto

menor es la lesión mayor dolor produce. El dolor se agrava con las menstruaciones y en los

casos en que la lesión ocupa el fondo de saco de Douglas, puede dar dispareunia. Existe un

aumento de la PGF2 alfa y PGE2 y un aumento de las contracciones uterinas que podría

deberse a un depósito de endometrio en la cavidad peritoneal.

La esterilidad debido a la endometriosis podría deberse a distintas causas de acuerdo a la

severidad de la patología. En los casos de endometriosis severa puede haber un factor

tuboperitoneal con adherencias y alteración en la anatomía de la pelvis que interfiera con el

transporte del esperma y el óvulo. En los casos de endometriosis leve hay varios

mecanismos propuestos que justifican su relación con la infertilidad: foliculogénesis

alterada, fase lútea inadecuada, fagocitosis espermática, mala calidad ovocitaria,

embriotoxicidad y alteración a nivel de la implantación.. La producción de prostaglandinas

por el endometrio ectópico puede afectar la motilidad tubaria, la foliculogénesis y la

función del cuerpo lúteo. Puede haber un aumento de la activación de los macrófagos

peritoneales en la endometriosis que cause la fagocitosis de los espermas o la secreción de

citoquinas que pueden ser tóxicas para el embrión. Según algunos investigadores habría un

60% de las mujeres con endometriosis que presentan un síndrome de Folículo Luteinizado

no roto (LUF) en el cual el folículo no se rompe en la ovulación y el óvulo queda atrapado.


Referencias bibliográficas

1. Ruiz V. Endometriosis y fertilidad. Ed. Acosta y Warman, pp. 99.

2. Lópes,VH. Palomo E. Incidencias de endometriosis en una población infértil. XXI


Congreso nacional de Ginecología y Obtetricia. Guatemala, 1993.

3. El-Eoley, et al. Danazol but not ginadotropin releasing hormone agonists suppresses
autoantibodies in endomeriosis. Fertil Steril 1990; 54:725.

4. Acosta AA. Buttram VC Jr. Besch PK, Malinak LR, Van Der Heyden J. A.proposed
classfication of pelvic endometriosis. Obstet Gynecol 1973;42:19.

5. Buttran VC Jr. Evolution of the revised American Fertility classification of


endometriosis. Fert. Steril 1985; 43: 347.

6. López VH. Tratamiento médico-quirúrgico de la endometriosis. Simposio El rostro


cambiante de la endometriosis panamá 3. 12. 1993.

7. Steinleitner A. Heterolous transplation of activated murine peritonel macrophages


inhibitis gamete interaction in vivo; A paradigm fo endometriosis associted subfertility.
Fertil Steril 1990; 54:725.

8. Damewood M. Effect of serum from patients with minimal to mild endometriosis on


mouse embryo growth. Fertil Steril 1990; 54: 917.

9. Proug S. Peritoneal fluid fracctions from patients with endometriosis do not promote
two-cell mouse embryo growth. Fertil Steril 1990; 54: 927.

27.- Femenino de 19 años, gesta 1, tuvo un parto con un producto único masculino de 3,600
g. la calificación de Apgar en el RN fue de 9 al primer minuto y 9 a los 5 minutos. La
revisión de sus registros de trabajo de parto mostró que tuvo ruptura de membranas 7
horas antes del parto. 40 horas después del parto la paciente presenta: temperatura 38.2
° C, FC 105 x’, TA 110/70, FR 16x’; E.F. dolor leve a la palpación del útero, las mamas se
encuentran sin eritema, ni dolor a la palpación, no dolor a la compresión de las pantorrillas.
¿Cuál de los siguientes es el mejor tratamiento antes de iniciar los antibióticos?

a) Hemocultivo
b) Cultivo de secreción vaginal
c) Examen general de orina y cultivo
d) Espirometría por incentivo
El vaciamiento incompleto ocasiona orina residual, distensión vesical excesiva y estasis,
además del cateterismo intermitente con sonda vesical durante el trabajo de parto. Por lo
tanto, la vejiga en el puerperio está predispuesta a infecciones. El dolor leve a la palpación
del útero puede ser normal en el puerperio y no se debe suponer de inmediato endometritis
puerperal. Cuando se sospecha endometritis, los cultivos de secreción vaginal tienen poca
utilidad porque se encuentran los mimos microorganismos que en mujeres puérperas sanas.
Los hemocultivos son apropiados para la valoración diagnóstica de la fiebre puerperal, pero
no son el paso inicial. La espirometría por incentivo se utiliza en el posoperatorio de
inmediato para fomentar la expansión pulmonar y disminuir las atelectasias. El legrado
uterino se utiliza para tratar la hemorragia.

Morgan M, Siddighi S. Ginecología y obstetricia, National Medical Series. 5° edición. Mc


Graw Hill. Pp. 29.

28.- Femenino que cursa con 36. 5 semanas de gestación acude al servicio por referir
malestar general, fosfenos, nausea y vómito, aprecia moderada ictericia, usted sospecha
de un síndrome de HELLP ¿Que alteraciones de laboratorio espera encontrar al
confirmar el diagnóstico?

a) Trombocitosis, Enzimas hepática elevadas, Anemia hemolítica.


b) Anemia hemolitica, Trombocitosis, Fosfatasa Alcalina elevada.
c) Anemia Hemolítica, trombocitopenia, enzimas hepáticas elevadas.
d) Trombocitopenia, Leucopenia, Hipertensión Arterial.

DEFINICIÓN:

• Es una complicación de la preeclampsia en la cual además de la Hipertensión Arterial


y proteinuria hay presencia de anemia hemolítica, enzimas hepáticas elevadas y
recuento bajo de plaquetas

EPIDEMIOLOGIA:

• Se presenta en un 4 a 10% de las preeclámpticas, diagnosticándose anteparto en un


70% de los casos preferentemente antes de las 37 semanas, mientras que el 30%
de los casos restantes enferma en los primeros 7 días del puerperio, sobre todo en
las 48 h iniciales.

• La proteinuria e hipertensión pueden estar ausentes en un 15 al 20% de los casos.

• Incidencia mayor en multigestantes y en edades avanzadas.


• Ocurre más frecuentemente cuando se demora la salida del feto y cuando se presenta
desprendimiento de la placenta.

• Mortalidad materna del 24% y mortalidad perinatal del 30-40%.

CLASIFICACION:
Síndrome de HELLP. Clasificación de Mississipi.
CLASE Plaquetopenia LDH AST-ALT
1 Severa <50000 >600 IU/L >70 IU/L
2 Moderada >600 IU/L >70 IU/L
>50000
<100000
3 Ligera >100000 >600 IU/L >40 IU/L
<150000 <70 IU/L
PE severa >150000 <400 IU/L <40IU/L
Eclampsia (sin
HELLP)

MANIFESTACIONES CLINICAS:

• Malestar general, fatiga y molestias inespecíficas 90%

• Cefalea 70%

• Epigastralgia 64%

• Vómito 22%

• Fosfenos 15%

• Visión Borrosa 11%

• Acúfenos 3%

• Ictericia

• Anemia no explicada

• Oliguria
]]Si se añade una HEMORRAGIA HEPÁTICA, el paciente puede quejarse de dolor en el
HOMBRO DERECHO y EL CUELLO, además de las molestias abdominales.

• Equimosis en los sitios de punciones venosas, petequias en los sitios de presión del
brazo, pero pueden tener pruebas de Rumpel Leed negativas.

• En casos severos se pude presentar ascitis como causa de hipertensión portal.

DIAGNOSTICO:
El diagnóstico clínico del síndrome de HELLP se plantea en gestantes o puérperas con
preeclampsia severa-eclampsia, excepto en el 15-20%, en las cuales esta asociación no
puede ser demostrada, en tanto se cumplan los criterios de Sibai:

MANIFESTACIONES CLINICAS:

• Malestar general, fatiga y molestias inespecíficas 90%

• Cefalea 70%

• Epigastralgia 64%

• Vómito 22%

• Fosfenos 15%

• Visión Borrosa 11%

• Acúfenos 3%

• Ictericia

• Anemia no explicada

• Oliguria

• Si se añade una HEMORRAGIA HEPÁTICA, el paciente puede quejarse de dolor en el


HOMBRO DERECHO y EL CUELLO, además de las molestias abdominales.

• Equimosis en los sitios de punciones venosas, petequias en los sitios de presión del
brazo, pero pueden tener pruebas de Rumpel Leed negativas.
• En casos severos se pude presentar ascitis como causa de hipertensión portal.

DIAGNOSTICO:
El diagnóstico clínico del síndrome de HELLP se plantea en gestantes o puérperas con
preeclampsia severa-eclampsia, excepto en el 15-20%, en las cuales esta asociación no
puede ser demostrada, en tanto se cumplan los criterios de Sibai:

HEMOLISIS
• Frotis periférico anormal (eritrocitos fragmentados)

• Hematocrito (>24%)

• Bilirrubina indirecta (>1.2mg/dL)

• Deshidrogenasa láctica (>218 UI/L)

ENZIMAS HEPÁTICAS ELEVADAS

• LDH >218UI/L

• AST >30UI/L

• ALT >37UI/L

PLAQUETAS BAJAS

<100.000/mm3

BIBLIOGRAFIA:

Sibai baha, El síndrome HELLP. Universidad de Valencia , revista quincenal de Obstetricia


clínica y ginecología, Octubre 2003.
V. Cararach, Síndrome de HELLP y Repercusiones maternas. X curso intensivo de
formación continuada materno fetal. Enero de 2003.

Toirac, Abelardo. Síndrome de Weistein HELLP Hospital Ginecoobstetrico Tamara Bunke.


Junio 2002.
De la Fuente, David. Síndrome HELLP. Medicina Universitária 2003; 5 (19): 101 -9
Andrea G. Witlin, DO, Baha M. Sibai, MD. Diagnosis and Management of women with
Hemolysis Elevate Liver Enzymes, and Pletelet Count (HELLP) syndrome. Hospital Physician.
Febrero 1999.
CIFUENTES B, Rodrigo. Ginecología y obstetricia basadas en las evidencias. Bogotá:
Distribuna, 2006. Sexta edición. 447 - 283 p.

29.- Femenino de 28 años acude al servicio de consulta externa refiriendo presentar


baches amenorreicos, acné, hirsutismo y esterilidad de aproximados 2 años de evolución
E.F. con índice de masa corporal 31 KG/M2, es diagnóstico más probable es:

a) Hipotiroidismo.
b) Fallo ovárico precoz.
c) Síndrome del ovario poliquístico.
d) Amenorrea de causa uterina.

El síndrome de ovarios poliquísticos (SOPQ) afecta aproximadamente a un 4% de mujeres


en edad reproductiva y se caracteriza por anovulación crónica e hiperandrogenismo. Es la
causa más común de infertilidad en mujeres.
Se caracteriza clínicamente por acné, alopecia, hirsutismo, irregularidades menstruales
e infertilidad.
Los hallazgos de laboratorio más frecuentes son: aumento de la hormona luteinizante
(LH), aumento de la relación LH/FSH (hormona folículoestimulante), aumento de
andrógenos (tanto ováricos como adrenales) y de estrógenos circulantes. Otros hallazgos
de laboratorio habituales son una prueba tolerancia oral a la glucosa anormal y alteraciones
en el perfil lipídico.
Todo esto junto con las imágenes ecocardiográficas características definen al síndrome.
La terapéutica permite dos grandes enfoques que pueden superponerse: la corrección de
las manifestaciones de hiperandrogenismo y el tratamiento de las alteraciones del eje
reproductivo (anovulación, esterilidad).
Los antiandrógenos están fundamentalmente indicados para tratar los síntomas
virilizantes.
Las alternativas para inducir la ovulación son numerosas: al citrato de clomifeno y a la
antigua resección en cuña se agregan las gonadotrofinas humanas, pulsos de GnRH (hormona
liberadora de gonadotrofinas), medidas o fármacos para modificar los niveles de insulina, y
finalmente técnicas quirúrgicas endoscópicas para reducir la masa ovárica.

Revista de Posgrado de la VIa Cátedra de Medicina - N° 125 – Marzo 2003


Pág. 37-40
SINDROME DE OVARIOS POLIQUISTICOS
Dra. Sandra Beneyto, Dra. María Andrea Ferreyra, Dr. Andrés Galfrascoli,
Dr. Andrés González, Dra. Susana Sosa
30.- Femenino de 34 años, es atendida en sala de partos secundario a eutocia, durante la
reparación de la episiotomía media hay un marcado incremento en el sangrado transvaginal.
La medida terapéutica inmediata en esta paciente es:

a) Masaje y compresión del fondo uterino


b) 20 unidades IV de oxcitocina
c) 0.2mg Im de metilergonovina
d) Empaquetar con gasas

El tratamiento clásico de la atonía uterina está constituido por la reposición volumétrica


(sangre, coloides y cristaloides), la utilización de masajes y por la administración de drogas
que promueven la contracción del músculo uterino. Cuando estas
Medidas no son eficientes, se procede casi invariablemente a la histerectomía de
hemostasia. Es de notar, que la atonía uterina puede aparecer en el primer embarazo y sin
ningún antecedente previo. La atonía uterina constituye, en casi todas las series mundiales,
más del 50% de las hemorragias graves del posparto.

Patología de Urgencia, Año 9, Nro. 3, Septiembre de 2001

31.- Mujer de 35 años que acude a consulta con antecedentes de G3 C2 A1, refiere que
ha presentado durante el primero y segundo trimestres de su embarazo manchado con
frecuencia intermitente ,a las 34 SDG inicia con hemorragia abundante, repentina e
indolora, su principal sospecha es:

a) Coriocarcinoma
b) Ruptura uterina
c) Placenta previa
d) Desprendimiento grave de placenta normoinserta

PLACENTA PREVIA
DEFINICIÓN:
Es cuando la placenta se implanta sobre o muy cerca del orificio cervical interno y una
parte de la placenta precede a la parte fetal que se presenta.
INCIDENCIA:
Esta es difícil determinar ya que muchos casos pasan desapercibidos, sobre todo cuando
ocurren los abortos en embarazos tempranos.
La prevalencia varía de 1 en 100 a 1 en 850 nacidos vivos, pero solo el 20% total.
Etiología:
Edad avanzada, multíparas, paciente con cesáreas previas, paciente con aborto de
repetición, esto debido a las gestaciones previas.
TIPOS:
Inserción baja.- Es cuando el borde placentario se encuentra en el segmento inferior a
menos de 6 cm del orifico cervical interno.
Marginal.- Es cuando el borde placentario alcanza los márgenes del orificio cervical interno.
Parcial.- Es esta la placenta cubre parcialmente el orificio cervical interno.
Total.- La placenta cubre la totalidad del orificio cervical interno aún con dilatación
cervical avanzada.
DIAGNOSTICO:
La característica es el STV de aparición brusca en forma indolora en el segundo o tercer
trimestre. Frecuentemente hay ausencia de dolor a actividad uterina que son parámetros
para hacer el diagnóstico.
La mayor incidencia de sangrado aparece a las 33-34 sdg.
ESTUDIOS DE GABINETE:
El estudio mas utilizado es la ultrasonografía obstétrica.
TRATAMIENTO:
El manejo va a depender de factores como son:
Edad gestacional, magnitud del sangrado, si hay trabajo de parto, variedad de placenta
previa y complicaciones materna.

BIBLIOGRAFÍA:

1.- Waxler P, Gottesfeld KR. Early diagnosis of placenta previa. Obstet Gynecol
1979;54:231-32.
2.- Cabrero-Roura L. Riesgo elevado obstétrico. Ed. Masson 1996; pp; 109-118.
3.- Patrick J, Placenta Previa, Clinical Obst and Gynecology 1990;33(3): 414-421.
4.- Chapman M, Furtenes ET, Significance of ultrasound in location of placenta in early
pregnancy Br J Obst Gynecol 197;86: 846.

32.- Se trata de femenino de 20 años con menarca a los 13 años ritmo menstrual 45x4.
Refiere vida sexual activa desde los 17 años con frecuencia de 4 veces por semana. A la
exploración se encuentra acné intenso en la frente, mejillas y mentón. Acude a consulta
por que desea adoptar un tratamiento anticonceptivo por vía oral, el tratamiento más
adecuado es:

a) Norgestimato
b) Gestodeno
c) Levonorgestrel
d) Ciproterona
El efecto antiandrogénico específico del acetato de ciproterona actúa por inhibición
competitiva de la unión de la 5 - alfa - dihidrotestosterona con el receptor citosólico de las
células blanco, que disminuye la producción y la excreción de sebo y el aumento y el
desarrollo del vello.
Es un derivado de la 17 - alfa - hidroxiprogesterona que posee acción progestágena. Su
acción antigonadotrófica se suma a la del etinilestradiol. El acetato de ciproterona no
posee acción estrogénica sino un efecto antiestrogénico, y tampoco posee acción nociva
sobre la función de la corteza suprarrenal;

Indicaciones en la mujer: Manifestaciones de androgenización de grado severo, por


ejemplo, hirsutismo grave, alopecia androgenética de tipo grave, a menudo acompañados por
manifestaciones graves de acné y/o seborrea.
Indicaciones en el hombre: Atenuación del impulso en las desviaciones sexuales.
Tratamiento antiandrógeno del carcinoma de próstata inoperable.

BIBLIOGRAFÍA:
1. Swift S. Current opinion on the classification and definition of genital tract prolapse.
Curr Opin Obstet Gynecol 2002; 14: 503-7.
2. De Caro R, Aragona F, Herms A, Guidolin D, Bizzi E, Pagano F. Morphometric analysis of
the fibroadipose tissue of the female pelvis. J Urol 1998; 160: 707-13.
3. Gill E, Hurt W. Pathophysiology of pelvic organ prolapse. Clin Obstet Gynecol 1998;
25(4): 757-69.
4. DeLancey, J. Anatomic aspects of vaginal eversion after Hysterectomy. Am J Obstet
Gynecol. 1992; 166(6 pt 1): 1717-24.

33.- Acude a consulta mujer de 23 años la cual presenta un nódulo mamario palpable de
aparición brusca. La ecografía revela un nódulo anecogénico, de limites muy precisos,
morfología regular y refuerzo posterior, único de 3.5 cms. de diámetro. ¿El diagnóstico más
probable es?

a) Cáncer.
b) Displasia fibrosa.
c) Fibroadenoma.
d) Quiste

Quistes. Los quistes mamarios son fáciles de detectar con la ecosonografía. Pueden ser
lesiones únicas o múltiples que se observan como imágenes redondeadas, anecogénicas, de
paredes delgadas, contornos bien definidos, con importante reforzamiento acústico
posterior y sombras laterales delgadas. Pueden presentar septos intraquísticos y, en
ocasiones, se pueden observar ecos internos que sugieren detritus celulares o proceso
inflamatorio. Se debe descartar la presencia de lesiones intraquísticas o la coexistencia de
otras alteraciones benignas o malignas.
En caso de ser sintomáticos, el tratamiento adecuado es la punción y aspiración de la lesión
con aguja guiada por palpación o ecosonografía de acuerdo con ell tamaño, profundidad y
características del contenido. El uso del ultrasonido garantiza el vaciamiento completo.

REFERENCIAS:

• Barth V, Prechtel K. Mama normal. En: Barth V, Prechtel K, editores. Atlas de


patología de la glándula mamaria. 2da ed. Madrid: Editorial Médica
Panamericana, 1991.
• Bush H, McCredie A. Carcinoma of the breast during pregnancy and lactation.
In: Allen HH, Nisker JA. Cancer in pregnancy. New York: Futura Publishing Co.
Inc., 1986.
• Byrd BF, Bayer DS, Robertson JC, Stephenson JE Jr.
34.- Femenino de 35 años de edad, a quien se realiza diagnóstico de placenta percreta, el
tratamiento e elección en esta patología es:

a) Ergonovina a dosis altas.


b) Histerectomía.
c) Hemostasia con puntos transfictivos.
d) Taponamiento uterino.

La placenta anormalmente adherida es poco común y tiene importancia clínica por su


morbimortalidad, a consecuencia de hemorragia, perforación, invasión y lesión de las vías
urinarias. Esta adherencia anormal está asociada con la implantación placentaria sobre
cicatrices de cesárea previa, incisiones uterinas o legrados. La placenta percreta consiste
en la penetración del tejido placetario a través de toda la pared uterina, traspasando la
serosa de la misma. La identificación de esta anormalidad antes del parto es posible
mediante métodos de imagen (escala de grises por ultrasonido, ecografía Doppler color
pulsado o resonancia magnética nuclear). El tratamiento conservador se acompaña de
elevada morbilidad en muchos casos, por lo que el tratamiento quirúrgico se convierte en
el definitivo. La literatura sugiere un aumento previsto en la incidencia de esta condición
con base en el incremento del número de cesáreas, por lo que la histerectomía postcesárea
será una decisión que enfrentarán los especialistas con mayor frecuencia.

1. Perucca E, Domínguez C, Yahng Ch, García R. Placenta previa percreta con invasión
vesical. Rev Chil Obstet Ginecol 1997; 62(3): 206-10.

2. Abbas F, Talati J, Wasti S et al. Placenta percreta with bladder invasion as a cause of
life threatening hemorrhage. J Urol 2000; 164: 1270-4.

3. Perucca E, Cazenave H, Barra A, Ochoa N, Villagrán G, Espinoza R, Estay R, Bustamante


R, Siebert A. Placenta previa percreta con invasión vesical. Rev Chil Obstet Ginecol 2002;
67(5): 364-7.

4. Price F, Resnik E, Heller K, Christopherson W. Placenta previa percreta involving de


urinary bladder. A report of two cases and review of the literature. Obstet Gynecol 1991;
78(3):508-11.
35.- Femenino de 29 años con antecedentes de G/3, P/2, C/1 acude al servicio de consulta
externa, refiere que presenta una secreción transvaginal bastante líquida, de baja
viscosidad, maloliente de color amarillo y gris, espumoso. Esta entidad es propia de
infección por:

a) Cándida albicans
b) Gardenerella
c) Gonococos
d) Tricomonas

• El protozoario Thricomona Vaginalis es el responsable del 25% de las vaginitis.


• 20-50% de las mujeres cursan asintomáticas.
• La tricominiasis es predominantemente una infección transmitida sexualmente.
• Debe sospecharse de abuso sexual en caso de encontrar Trichomona en pacientes
pediátricos.

DIAGNOSTICO:
• Flujo vaginal amarillo-verdoso con burbujas, muy fétido, irritación vulvo-vaginal,
disuria.
• El pH suele ser mayor de 4.5.
• Estudio en fresco en donde se observa al microscopio el organismo flagelado
característico.
• 75% se diagnostican con el estudio del Papanicolaou.

Referencias bibliográficas:

1. Secretaría de Salud. Norma Oficial Mexicana NOM -039-SSA2-2002, Para la prevención


y control de las infecciones de transmisión sexual. D.O.F. 19 de Septiembre 2003.
2. Kettler H, White K, Hawkes S. Mapping the landscape for sexually transmitted
infections: key findings and recommendations. Geneva, TDR (TDR/STI/ IDE/04.1).
3. CDC. Trends in Reportable Sexually Transmitted Diseases in the United States. CDC,
National Report. 2004.
4. Distribución de los casos nuevos de enfermedades por mes Estados Unidos Mexicanos
2004. Sistema Único de Información para la Vigilancia Epidemiológica/Dirección General
de Epidemiología/SSA.
5. Aral S O. Sexual risk behaviour and infection: epidemiological considerations. Sex.
Transm. Inf. 2004;80:8-12
36.- A 71 years-old man complains of occasional lower back pain. His blood pressures
150/85 mmHg and his pulse is 80/min. Cardiac examination reveals an S4 gallop. Abdominal
examination reveals a pulsatile mass approximately 5.0 cm in diameter palpable in the
epigastric area. Peripheral pulses are normal. Which of the following is the most likely
diagnosis?

a) Abdomninal aortic aneurysm


b) Cancer of the proximal colon
c) Chronic pancreatitis
d) Lipoma of the abdominal wall

Epidemiología:
Los aneurismas abdominales, afectan al 2-5% de los mayores de 60 años y más en
hombres que en mujeres, en una proporción 4/1 (1).

La mayoría son de etiología arterioesclerótica y de ubicación infrarrenal (90%); y pueden


extenderse a una o ambas arterias ilíacas (70% de los casos) (1). Según la forma, los
aneurismas son fusiformes (75%) cuando afecta toda la circunsferencia del vaso, o
saculares (con cuello) cuando solo está englobado una porción de dicha circunsferencia.
Estos últimos son infrecuentes de observar en la aorta abdominal, y por lo general se
ubican proximales al origen de las arterias renales.

Clínica:
El 75% de los pacientes son asintomáticos, y se descubren por hallazgos en exámenes de
rutina o por una masa pulsátil en determinadas posiciones corporales. Ocasionalmente el
diagnóstico se hace por laparotomía por otras patologías (1).

Los pacientes sintomáticos consultan por:

• Dolor epigástrico o lumbar.


• Masa pulsátil dolorosa a la palpación.
• Síntomas gastrointestinales como nauseas, vómitos y pérdida de peso, en
aneurismas de gran tamaño.
• Asociado a episodios isquémicos en miembro inferior (raro).

Lo más frecuente es el hallazgo de una masa pulsátil umbilical o supraumbilical (se palpa
cuando su diámetro es > a 4,5 cm).
BIBLIOGRAFIA:

1. Ferraina P, Oria A. Cirugía de Michans. Buenos Aires: El Ateneo, 2000: 949-955


2. Farreras R. Medicina Interna. Madrid: Mosby/ Doyma Libros,1995: Vol 2: 650-651
3. Frutos Ortiz E, Moirano J, Fassi J. Cirugía. Buenos Aires: El Ateneo, 1993: 67.11
4. Chang JB, Stein TA, Liu JP, Dunne ME. Risk factors associated with rapid growt of
small abdominal aortic aneuryms. Surgery 1997, 121: 117-122
5. Matsushita M, Nishikimi N, Sakurai T, Nimura Y. Relationship betwen aortic
calcification and atherosclerotic disease in patients with abdominal aortic aneurym:
Int Angiol 2000, 19: 276- 279
6. Du Toit DF, Louwrens H, Klompje J, Grenewald JH. Ruptured abdominal aortic
aneurym and horseshoe kidney. A Afr Med J 1983, 64: 750-751

37.- En su jurisdicción se reportaron 44 casos de hepatitis A entre los alumnos de segundo


grado de una escuela secundaria durante la primera quincena del mes de septiembre. La
tasa de ataque es de 26.4%. Esta situación nos indica que estamos ante un(a):

a) Brote
b) Endemia
c) Epidemia
d) Pandemia

De acuerdo a la NOM- 017 un brote se define como la ocurrencia de dos o más casos
asociados epidemiológicamente (tiempo, lugar y persona) entre sí. La medida cuantitativa
de la extensión de un brote es la Tasa de Ataque (TA) que se calcula dividiendo el número
de casos nuevos entre el total de personas expuestas por 100.
Greenberg R. S; Epidemiología médica, Manual Moderno, 2ª. Ed. Págs. 77-79

38.- Masculino de 44 años de ead con antecedentes de DM de 5 años de evolución así como
dislipidemia mixta. Actualmente controlado con sulfonilureas y estatinas .La primera lesión
que se observa en la retinopatía diabética temprana es:

a) Microaneurismas
b) Exudados blandos
c) Exudados duros
d) Hemorragias intrarretinianas
La retinopatía diabética tiene cuatro etapas:

1. Retinopatía no proliferativa ligera. Esta es la etapa más temprana de la enfermedad en


la que aparecen los microaneurismas. Estas son pequeñas áreas de inflamación, que parecen
ampollas, en los pequeños vasos sanguíneos de la retina.

2. Retinopatía no proliferativa moderada. Según avanza la enfermedad, algunos vasos


sanguíneos que alimentan la retina se obstruyen.

3. Retinopatía no proliferativa severa. En esta etapa muchos más vasos sanguíneos se


bloquean, haciendo que varias partes de la retina dejen de recibir sangre. Entonces estas
áreas de la retina envían señales al cuerpo para que haga crecer nuevos vasos sanguíneos.

4. Retinopatía proliferativa. En esta etapa avanzada, las señales enviadas por la retina
para alimentarse causan el crecimiento de nuevos vasos sanguíneos. Esto se llama la
retinopatía proliferativa. Estos nuevos vasos sanguíneos son anormales y frágiles. Crecen a
lo largo de la retina y de la superficie del gel vítreo, el gel incoloro que llena el interior del
ojo.

Retinopatía diabética. Forma leve de retinopatía diabética, en la que destacan


microaneurismas /microhemorragias de predominio en arcada temporal superior, junto con
exudados duros que se acercan a la mácula.

1. The Diabetes Control and Compications Trial Research Grop. The Effect of Intensive
treatment of Diabetes on the development and progression of long term complications in
insulin dependent Diabetes Mellitus N Engl J. Med; 1993 ; 329 : 977-986.
2. Early treatment Diabetic Retinopathy Study Research Group Early photocoagulation for
diabetic Retinopathy ETDRS Report 9. Ophthalmology, 1991 ; 98 : 1316-26.
3. Kahn HA Hiller R. Blindness caused by diabetic retinopathy Am. J. Ophthalmol, 1974 ; 78
; 58-67.
4. Ladas ID. Theossiadis GA Long term efectiveness of modified grid photocoagulation for
diffuse Macular edema Acta Ophthalmol, 1993 ; 71(3) ; 393-7.
5. Klein R, Klein BEK , Moss SE , et al The Eisconsin Epidemiology Group Ophthalmology,
1984 ; 91 : 1464-74

39.- Se presenta ante usted femenino de 34 años con referencia por diagnóstico de lupus
eritematoso generalizado, ¿Cuál es el síntoma dentro de los criterios de clasificación de
de éste padecimiento?

a) Fotosensibilidad
b) Alopecia
c) Artralgias
d) Pérdida de peso

Criterios de Clasificación para el Diagnóstico de Lupus Eritematoso Sistémico (LES)


Erupción malar: Eritema fijo, plano o alto, sobre las eminencias malares, que no suele
afectar los surcos nasogenianos.
Erupción discoide: Placas eritematosas altas, con descamación queratósica adherente y
tapones foliculares; puede haber cicatrices atróficas en las lesiones más antiguas.
Fotosensibilidad: Erupción cutánea a causa de una reacción insólita a la luz solar,
referida por el paciente u observada por el médico.
Úlceras bucales: Ulceración nasofaríngea, por lo común indolora, observada por un médico.
Artritis: Artritis no erosiva que afecta dos o más articulaciones periféricas, caracterizada
por dolor a la palpación, tumefacción o derrame. Serositis: Pleuritis o pericarditis
documentada por electrocardiograma o frote o evidencia de derrame pericárdico.
Enfermedad renal: Proteinuria persistente mayor a 0,5g/día o 3+ o cilindros celulares.
Transtorno neurológico: Convulsiones o psicosis en ausencia de otra causa conocida.
Transtorno hematológico: Anemia hemolítica o leucopenia (< 4.000/mm3) o linfopenia:
(<1.500/mm3) o trombocitopenia (< 100.000/mm3) en ausencia de fármacos que produzcan
esta alteración.
Trastorno inmunológico: Anti-DNA, anti-Sm, y/o Anticuerpos antifosofolipídicos (AFL).
Anticuerpo antinuclear: Un título anormal de ANA por inmunofluorescencia o análisis
equivalente en cualquier momento y en ausencia de medicamentos relacionados con el
síndrome de lupus de origen farmacológico.
Cualquier combinación de 4 o más de los 11 criterios, bien documentado durante cualquier
intervalo de la historia del paciente, hace el diagnósticos de LES (especificidad y
sensibilidad son del 95% y 75%, respectivamente).

Petri M. Review of classification criteria for systemic lupus erythematosus. Rheum


Dis Clin North Am. 2005 May;31(2):245-54.
40.- Masculino de 44 años, acude a consulta por presentar de manera espontánea una
erupción de lesiones pruriginosas, habonosas, diseminadas por toda la superficie corporal,
de evolución fugaz así como una amplia placa edematosa en hemicara izquierda. El
diagnóstico más probable es:

a) Angioedema hereditario.
b) Erisipela.
c) Vasculitis.
d) Urticaria aguda.

La urticaria se define como un síndrome reaccional de piel y mucosas caracterizado por


edema y ronchas pruriginosas ocasionadas por edema vasomotor transitorio y circunscrito
de la dermis que dura algunas horas; puede ser recidivante y de origen inmunológico, no
inmunológico o desconocido.1 La lesión elemental es una pápula edematosa dérmica
(roncha).2
El angioedema se manifiesta típicamente como un edema asimétrico causado por la
presencia de plasma dentro de tejido celular subcutáneo y mucosas.1
Clasificación
La urticaria se puede clasificar de acuerdo con diferentes parámetros: 1) según la
evolución: en aguda o crónica; 2) según el cuadro clínico: en urticaria ordinaria (urticaria
propiamente dicha), urticaria física (por estímulo detonador), urticaria por contacto
(inducida por un contacto químico o biológico) y angioedema (sin ronchas), en el cual el
espectro de las manifestaciones clínicas de los diferentes tipos es muy amplio; y 3) según
el mecanismo potencial de su desarrollo: inmunológico, no inmunológico, mediada por el
complemento, o bien urticaria autoinmune.3
Clasificación por evolución
La urticaria aguda se define tradicionalmente por la presencia de ronchas de forma
espontánea, casi la mayoría de los días, por menos de seis semanas.4 La urticaria crónica se
define como la presencia de ronchas de forma espontánea por más de seis semanas,
diariamente o casi la mayoría de los días de la semana.

BIBLIOGRAFIA:

1- Yadav S, Upadhyay A, Bajaj A. Chronic urticaria: An overview. IJD [en línea] 2006
[fecha de acceso 4 de marzo de 2007];51-3:171-177. Disponible en:
2- Woscoff A, Kaminsky A, Marini M, Allevato M. Dermatología en Medicina Interna.
Buenos Aires, Edición de los Autores, 2003: 24-27.
3- Criado PR, Fachini Jardim Criado R, Maruta C, Costa Martins JE, Rivitti E. Urticaria.
An.Bras.Dermatol 2005,v.80 n.6: 183-185.
41.- Femenino de 34 años de edad G4 P3, acude a consulta en busca de información acerca
de los métodos anticonceptivos. No tiene APP de importancia, ni toma medicamentos. Ha
mantenido una relación monógama con su pareja por los últimos 9 años. No quiere volver a
embarazarse, pero se niega a que se le practique una salpingoclasia. Su médico recomienda
un DIU. ¿Con éste método anticonceptivo existe mayor probabilidad de?

a) Amenorrea
b) Embarazo ectópico
c) Embarazo intrauterino
d) Aumento de peso

ETIOLOGIA
III.1 Factores que dificultan el camino del huevo. Este se implanta allí donde se encuentra
en el 6º-7º día postfecundación.
A/ Procesos inflamatorios, causa más frecuente de embarazo ectópico. Las salpingitis
deterioran la actividad ciliar y la motilidad tubárica. Hay otros procesos inflamatorios,
como, apendicitis que afectan secundariamente a las trompas, con producción de
adherencias y acodaduras.
B/ Alteraciones de la motilidad tubárica, como ocurre con la administración de gestágenos
a bajas dosis de forma continua (minipildora) o el empleo de contracepción postcoital con
estrógenos.
C/ DIUs. 9-17% de embarazos en portadoras de DIU son ectópicos, lo que se explica
porque el DIU protege más eficazmente frente a la gestación intrauterina (99’5% de
seguridad) que frente al tubárico (95% de seguridad) con lo que aumenta el riesgo relativo
de E.E.
D/ Tratamientos de esterilidad, sobre todo los quirúrgicos, cirugia conservadora de E.E
tubáricos, ligadura tubárica o técnicas de fecundación asistida (in vitro y transferencia
embrionaria intrauterina)
III.2 Factores que favorecen la nidación en la trompa: endometriosis tubárica, por un
cierto quimiotactismo del endometrio ectópico, así como, por la estenosis y adherencias que
se producen incluso con órganos vecinos.
III.3 Factores dependientes del propio huevo. Las anomalías genéticas del huevo, mediante
alteración en el momento de implantación, o quizá por un mayor volumen, puede favorecer la
anidación ectópica.
BIBLIOGRAFÍA EMBARAZO ECTOPICO:

]Cabero Roura, Ll y cols. Protocolos de Medicina Materno-fetal (Perinatología), 2ª edición.


Ed. Ergon, S.A. Madrid. 2000. 120-122.
]Bajo Arenas, J.M, Castellanos Bolado, P. Embarazo ectópico. En: Manual de asistencia a la
patología obstétrica. Fabre E, Ed. INO: Zaragoza. 1997. 89 -107.
]Carrera Macia, J.M y cols. En: Protocolos de obstetricia y Medicina perinatal del I. U.
Dexeus. 3ª Edición. Masson. Barcelona. 2000. 135-139.

42.- Una mujer de 67 años refiere intenso prurito vulvar y sensación quemante, al examen
el introito vaginal se encuentra estenótico. ¿Cuál de los siguientes es el tratamiento
apropiado?

a) 5-fluoracilo
b) Estrógeno tópico
c) Testosterona tópica
d) Corticoesteroides fluorados

Vulvovaginitis atrófica

El hipoestrogenismo conduce a atrofia de la vagina y el vestíbulo vulvar, que los hace


fácilmente irritables y susceptibles a infecciones secundarias. Las pacientes refieren
sensación de quemadura, prurito, disuria, hipersensibilidad y dispareunia. Puede
encontrarse al examen físico atrofia, fisuras superficiales, y un flujo vaginal acuoso1. Hay
disminución del tamaño del introito2, pérdida de la rugosidad y la vagina toma una apariencia
lisa y brillante.

Los hallazgos histológicos revelan un epitelio vaginal delgado, disminución de los lechos
capilares, y la citología muestra, a medida que la atrofia progresa, aumento de las células
basales y disminución o ausencia de las células superficiales2.

Se aconseja evitar el uso de jabones y demás irritantes de la piel. Se pueden utilizar


lubricantes simultáneamente con los estrógenos o como terapia única, si hay alguna
contraindicación a las hormonas.

El tratamiento con estrógenos por vía sistémica o transvaginal mejora y restaura los signos
y síntomas, y una a dos semanas después de iniciar el tratamiento los cambios de atrofia
empiezan a mejorar rápidamente, se reduce el pH y se induce maduración vaginal y de la
mucosa uretral, reduciendo la frecuencia de las infecciones urinarias3. La dosis y vía de
administración debe ser debidamente individualizada4. Contraindicaciones al tratamiento
con estrógenos, incluyen: la presencia de tumores estrógenosensibles, falla hepática
terminal y antecedentes de tromboembolización relacionada con ellos.
Menopausia y Piel. Parte II: Manifestaciones clínicas dermatológicas durante la
menopausia MARÍA ISABEL BARONA C. Docente adjunto. Dermatóloga Universidad
del Valle-Cali.

43.- Femenino de 27 años, es atendida en consulta en la clínica de displasias por papanicolau


con lesión NIC I. Antecedentes: menarca 14 años, ritmo 30x5 eumenorreica, inicio de vida
sexual a los 15 años, 2 parejas sexuales, método de planificación familiar oclusión tubaria
bilateral, gestas 3 partos 3, cérvix con lesión acetoblanca con extensión lineal de 2 cm.
Para confirmar el diagnóstico se debe realizar:

a) Crioterapia de lesión.
b) Captura de híbridos.
c) Repetir colposcopía
d) Biopsia de la lesión
9.5.2 Las pacientes a quienes se les realizó citología cervical, cuyo resultado es LEIBG
(infección por VPH, displasia leve o NIC 1); LEIAG (displasia moderada y grave o NIC 2 y 3)
o cáncer deben enviarse a una clínica de colposcopía, para realizar estudio colposcópico.
9.5.3 Si el resultado de la citología es LEIBG, la colposcopía es satisfactoria y sin
evidencia de LEIBG, se realizará control citológico en un año (Apéndice Normativo A)
9.5.4 Si la citología es de LEIBG, la colposcopía es satisfactoria y existe evidencia de
lesión, se debe tomar una biopsia dirigida.
9.5.4.1 Si la biopsia dirigida es negativa, se realizará nueva colposcopía para verificar el
diagnóstico y en caso necesario, tomar nueva biopsia dirigida y revalorar.
9.5.4.2 Si la biopsia dirigida es reportada como LEIBG se podrá dar tratamiento
conservador: criocirugía, electrocirugía o laserterapia (sólo si cumple con las condiciones
referidas en el Apéndice 1) o se podrá mantener a la paciente en vigilancia en la clínica de
colposcopía, con colposcopía y estudio citológico cada seis meses, durante 24 meses.
Jueves 31 de mayo de 2007 DIARIO OFICIAL (Primera Sección)
9.5.4.3 Si la biopsia dirigida es reportada como LEIAG (Lesión Intraepitelial Escamosa de
Alto Grado) se realizará tratamiento conservador (electrocirugía o laserterapia). En las
mujeres posmenopáusicas, dependiendo de las condiciones anatómicas del cérvix, se
realizará tratamiento conservador en la clínica de colposcopía o tratamiento quirúrgico
(histerectomía extrafascial) en el servicio que corresponda.
9.5.4.4 Si la biopsia dirigida reporta cáncer microinvasor o invasor, la paciente se
transferirá a un Servicio o Centro Oncológico para su tratamiento correspondiente.
9.5.4.5 Si la citología reporta LEIBG y la colposcopía es no satisfactoria, se tomará
cepillado endocervical (Apéndice Normativo A)
9.6 En caso de colposcopía no satisfactoria, negativa a LEIBG y con cepillado endocervical
negativo, se continuará su control en la clínica de colposcopía en seis meses, con colposcopía
y citología.
9.6.1.1 Si el cepillado endocervical reporta LEIBG se tratará a la paciente como LEIAG,
con métodos conservadores escisionales.
Jueves 31 de mayo de 2007 DIARIO OFICIAL (Primera Sección)
Modificación a la Norma Oficial Mexicana NOM-014-SSA2-1994, Para la prevención,
detección, diagnóstico, tratamiento, control y vigilancia epidemiológica del cáncer
cérvico uterino.

Al margen un sello con el Escudo Nacional, que dice: Estados Unidos Mexicanos.- Secretaría
de Salud.
MODIFICACION A LA NORMA OFICIAL MEXICANA NOM-014-SSA2-1994, PARA LA
PREVENCION,
DETECCION, DIAGNOSTICO, TRATAMIENTO, CONTROL Y VIGILANCIA
EPIDEMIOLOGICA DEL CANCER CERVICO UTERINO.

7.3 El resultado del estudio citológico es descriptivo y debe ser informado de la siguiente
manera:

a.- Negativo a cáncer.

b.- Negativo con proceso inflamatorio.

c.- Displasia leve (NIC 1).

d.- Displasia moderada (NIC 2).

e.- Displasia grave (NIC 3).

f.- Cáncer del cuello del útero in situ (NIC 3).

g.- Cáncer microinvasor e invasor.

h.- Adenocarcinoma.

i.- Maligno no especificado.


44.- Acude a consulta femenino de 19 años con diagnóstico de amenorrea, se acompaña de
resultados de laboratorio. Reportan los siguientes niveles hormonales: GnRH elevada, FSH
y LH elevadas, hormonas ováricas (estrógenos y progesterona) bajas. El defecto está a
nivel de:

a) Hipotálamo.
b) Hipófisis.
c) Ovario.
d) Endometrio.

Evaluación de la paciente con amenorrea secundaria

La mayoría de las pacientes con AS que no están embarazadas o no entraron en el


climaterio tienen una alteración en algún nivel de la cascada reguladora del ciclo menstrual
femenino. A continuación, esquematizamos la evaluación de la AS en pasos. Cada médico
deberá adaptar los tiempos de la evaluación a la situación particular de cada paciente. No
obstante, recomendamos no saltear ninguno de estos pasos ya que un diagnóstico preciso
permitirá realizar un tratamiento racional y dar un pronóstico respecto de la futura
función menstrual y de la fertilidad.

Primer paso (descartar el embarazo)

Su prueba se basa en el dosaje cuali o cuantitativo de la subunidad beta de la


gonadotrofina coriónica humana. Si se certifica la ausencia de embarazo podrá avanzarse al
siguiente paso, pero teniendo en cuenta siempre que si la probabilidad de embarazo es muy
alta se deberá repetir la prueba nuevamente. Se recomienda no obviar esta prueba aunque
la paciente asegure que no ha mantenido relaciones sexuales.

Segundo paso (prueba de progesterona)

Debe realizarse sólo si el test de embarazo es negativo y consiste en dar


medroxiprogesterona entre 30 a 50mg por vía oral (un comprimido de 10mg durante 5 días)
o progesterona oleosa 100 a 200mg por vía intramuscular en una sola dosis. El resultado de
esta prueba provee información acerca de si se produjeron estrógenos. La prueba se
considera positiva si se produce un sangrado luego de 2 a 14 días de la suspensión de la
progesterona. La respuesta positiva indica que existe integridad anatómica del aparato
genital femenino y que el ovario produce estrógenos. En estos casos, la AS se debe a que no
ha habido ovulación. La causa más frecuente de AS con prueba de progesterona positiva es
la disfunción hipotalámica leve. En este caso, no se dispara el pico de LH necesario para
producir la ovulación, no se ovula (ciclo anovulatorio), no hay cuerpo lúteo y no hay
producción de progesterona.
El sistema reproductor queda en un estado folicular, con grados variables de desarrollo
folicular, acompañados de concentraciones cambiantes de estradiol. Puede producirse
hemorragia en ausencia de ovulación dado que el endometrio proliferativo sufrirá
ocasionalmente un desprendimiento parcial en respuesta a estos niveles cambiantes de
estradiol. Estos sangrados luego de un ciclo anovulatorio se denominan sangrados uterinos
disfuncionales (SUD). Como dijimos, ejemplos frecuentes de situaciones de anovulación son
el estrés, la pérdida brusca de peso y el ejercicio intenso. Otras causas menos frecuentes
son la hiperprolactinemia, el hipotiroidismo y el sindrome del ovario poliquístico. Otras
veces no hay una causa clara (idiopática) de este trastorno.

La prueba se considera negativa si NO se produce un sangrado luego de 2 a 14 días de la


suspensión de la progesterona, lo que ocurre cuando los niveles de estrógenos son bajos
(menores a 40 pg/ml). La causa más frecuente de prueba negativa es la insuficiencia
gonadal de la menopausia.

Tercer paso

Para avanzar al tercer paso es preciso distinguir si la prueba de progesterona fue positiva
o negativa.

Pacientes con prueba de progesterona positiva

Una prueba de progesterona positiva hace diagnóstico de ciclos anovulatorios. Según los
antecedentes, el examen clínico y la probabilidad previa el médico podrá orientarse hacia
cuál es el diagnóstico. Hay situaciones de estrés, crisis vitales, cambios de peso brusco o
ejercicio extremo que ocasionan un trastorno del ciclo aislado. En estos casos, la prueba de
progesterona es diagnóstica y terapéutica. Es decir, la paciente no requiere estudios
posteriores. Es conveniente reasegurarla y explicarle que su periodo se normalizará una vez
solucionado el problema. En una paciente con trastornos anovulatorios frecuentes, que
presenta obesidad, acné e hirsutismo, existe alta sospecha de síndrome de ovario
poliquístico (SOP). En este caso, debemos solicitar una ecografía pelviana y un dosaje de
LH/FSH o de hormonas masculinas: testosterona libre (To), dehidroepiandrosterona
sulfato (DHEA-S) y 17OH progesterona.

Si la paciente tiene trastornos del ciclo y galactorrea, se debe solicitar un dosaje de


prolactina (PRL). El valor normal de PRL para la mujer no embarazada es de 20 a 25ng/ml.
Cuando el valor de PRL es menor de 100ng/ml, generalmente es de causa idiopática o
farmacológica; si está entre 100 y 200ng/ml, puede ser por causa farmacológica o por
enfermedad hipotalámica; si es mayor de 200ng/ml, la causa más probable es el adenoma
hipofisario. Cuando el valor de la PRL es mayor de 100ng/ml, se debe solicitar un estudio
por imágenes que puede ser una tomografía computada de cerebro con contraste o una
resonancia magnética nuclear con gadolinio.
Si se observa una imagen menor de 10mm, se trata de un microadenoma de hipófisis y si es
mayor de 10mm, se hace diagnóstico de macroadenoma. Si la paciente consulta por
trastornos del ciclo frecuentes y no presenta galactorrea o signos o síntomas de
hipotiroidismo, igualmente deberá solicitarse un dosaje de tirotrofina (TSH) y de
prolactina (PRL) séricas.

Pacientes con prueba de progesterona negativa

Deberá dosarse el nivel sérico de FSH cuyo valor refleja los niveles de estrógenos
circulantes (por el fenómeno de retroalimentación negativa). Si la FSH es mayor de 40
UI/ml, los niveles de estrógenos son bajos. En este caso, se asume que existe una
insuficiencia ovárica. El valor de la FSH puede hacer el diagnóstico diferencial entre la
falla ovárica (castración temprana, menopausia precoz o menopausia normal) y la disfunción
hipotálamo-hipofisaria severa. En la primera, la ausencia de retroalimentación negativa
estimula la liberación de las gonadotrofinas y, en consecuencia, éstas se encuentran
elevadas en la sangre. En la segunda, los valores de gonadotrofinas son bajos debido a un
defecto en su producción. El dosaje de FSH es más sensible que el de LH para este
propósito, por lo tanto, si la prueba de progesterona es negativa y la FSH es baja, lo más
probable es que la causa sea una disfunción hipotálamo-hipofisaria severa. En este caso
debería solicitarse un estudio de diagnóstico por imagen del cerebro (RMN con gadolinio o
TAC con contraste). En el cuadro 4 se muestran los valores normales de las hormonas
mencionadas.

Cuadro 4: valores hormonales normales

PRL (prolactina): 20 a 25ng/ml en mujeres no embarazadas.

TSH: de 0.5 a 5mUI/ml.

FSH: 5 a 30mUI/ml.

LH: 5 a 20mUI/ml (en el pico ovulatorio este valor se debe multiplicar por dos o tres).

Falla ovárica: FSH mayor de 40mUI/ml.

Disfunción hipotálamo hipofisaria: FSH menor a 5mUI/ml y LH menor a 5mUI/ml.

En general, una paciente con disfunción hipotalámica con gonadotrofinas bajas está
expresando una gravedad mayor que la disfunción hipotalámica leve con prueba de
progesterona positiva.
En los casos en los que la prueba de progesterona es negativa, la FSH es normal, la paciente
no está en una edad cercana a la menopausia y existe alta sospecha de que exista una
enfermedad uterina, se recomienda realizar una “prueba de estrógenos y progesterona”.

Esta prueba sirve para conocer si el efector (el útero) responde a los estímulos
hormonales. Consiste en administrar estrógenos (1.25mg por día durante 20 días) más
progesterona (10mg por día durante los últimos 5 días en que se administran los
estrógenos). Si no hay sangrado, se debe repetir la prueba y, si nuevamente no hay
sangrado, la prueba se considera negativa. Esto indica que hay una falla en el efector, o
sea, en el útero.

En este caso la paciente debe ser derivada al ginecólogo. Las causas más probables de falla
uterina son la endometritis por abortos o partos sépticos y las sinequias por curetajes
vigorosos. Cuando falla el efector, la ecografía transvaginal puede ser normal. Por eso debe
realizarse una prueba de estrógenos y progesterona para evaluar directamente la
funcionalidad del útero. La prueba se considera positiva si hay sangrado (aunque sean sólo
gotitas). Esto significa que el útero está indemne para responder a estímulos hormonales y
el problema es de origen hipotálamo- hipofisario.

1. Jonathan R, Pletcher, Gail B. Slap. Menstrual Disorders Amenorrhea. Pediatric


Clinics of North America. June 1999; 46: Issue 3.
2. Bryan McIver, Susan A Romanski; Todd B Nippoldt. Evaluation and Management of
Amenorrhea. Mayo Clinic Proceedings Dec 1997; 72: 1161-1169.
3. Tarannun Master Hunter, Diana L.Heiman. Amenorrhea: Evaluation and Treatment.
Am Fam Physician 2006; 73: 1374-1382.
4. The Practice Committee of the American Society for Reproductive Medicine.
Current Evaluation of Amenorrhea. Fertility and Sterility Nov 2006: 86 Supl 4:
148-155.

45.- Se trata de mujer de 54 años G.2 P.2, a quien se ha diagnosticado prolapso uterino
acompañado de incontinencia urinaria de esfuerzo, el procedimiento de elección en esta
paciente es:

a) Histerectomia total abdominal.


b) Histerectomía vaginal reconstructiva
c) Colpoperineoplastía.
d) Uterosuspensión.

El prolapso genital y su tratamiento ha sido siempre un importante capítulo de la


ginecología. No existe consenso clínico del concepto que define al prolapso genital como
patológico. Cierto grado de descenso y relajación de la pared vaginal es considerado normal
en la gran mayoría de las mujeres, siendo más frecuente en mujeres de mayor edad.
En la población general solo un 3% presenta prolapso genital severo, entendiendo como tal
al de III° y IV° grado. Es probable que alrededor de 3 a 6% de la población femenina
desarrolle un prolapso de esas características en algún momento de su vida.

La intervención quirúrgica por excelencia para el tratamiento quirúrgico del prolapso


uterino es la histerectomía vaginal. Esta intervención se clasifica como "limpia-contaminada

INDICACIONES PARA LA HISTERECTOMÍA VAGINAL

Prolapso uterino
Hemorragia uterina disfuncional
Carcinoma cervico-uterino "in situ"
Miomatosis uterina poco voluminosa
Hiperplasia endometrial
Piometra

VENTAJAS DE LA HISTERECTOMÍA VAGINAL

No deja cicatriz abdominal


Mínimo trauma abdominal
Escasa hemorragia transoperatoria
Mínima manipulacion intestinal
Menos dolor postoperatorio

CUADRO
CONTRAINDICACIONES PARA LA HISTERECTOMÍA VAGINAL

Impericia
Útero muy voluminoso
Tumores ováricos
Endometriosis
Enfermedad pélvica inflamatoria
Cirugía previa en útero, trompas y ovarios
Cáncer de endometrio
Histerectomía obstétrica
46.- Femenino de 22 años de edad, cursa con 12 semanas de gestación, lleva tres días
sangrando por genitales, con náuseas continuas y constantes, útero mayor aumentado de
tamaño y valores de beta HCG muy elevados. El diagnóstico más probable es:

a) Amenaza de aborto.
b) Aborto diferido.
c) Mola hidatídica.
d) Amenaza de aborto en un útero con miomas.

La enfermedad trofoblástica gestacional agrupa a diferentes entidades interrelacionadas:


mola completa, generalmente diploide con origen cromosómico paterno, mola parcial
generalmente triploide, tumor trofoblástico del lecho placentario y coriocarcinoma, con
tendencias variables a la invasión local y a las metástasis, cuyo denominador común es la
hipersecreción de hCG. El coriocarcinoma es diploide y proviene de ambos progenitores,
excluyendo probablemente su origen directo en la mola completa. El tumor trofoblástico
del lecho placentario está constituido por trofoblasto mononuclear intermedio no
conteniendo vellosidades coriónicas e inmunohistoquimicamente caracterizado por expresar
muchas de sus células hPL y unas pocas hCG.

Cuadro clínico

Tras un periodo de amenorrea, y a partir del segundo mes, hay un aumento de los síntomas
subjetivos del embarazo, sobre todo náuseas y vómitos o hiperémesis en un 30%. Hay
metrorragias irregulares en el 96% de los casos, en principio de escasa cantidad, pero que
se va incrementando. Dicha hemorragia no es continua sino que se repite cada dos o tres
días, de color roja o negruzca, que se produce por la ruptura de vasos maternos al
separarse las vesículas de la decidua.

Ocasionalmente, hay expulsión de restos molares, que lo refiere la paciente como expulsión
de vesículas en el 11 % de los casos y que es patognomónico pero aparece tardíamente.
También puede presentar mal estado general, dolor difuso en hipogastrio y anemia. Esta
última en relación con las metrorragias.

Pueden haber signos y síntomas de hipertiroidismo, como taquicardia, sudoración, y


temblores, en el 7%, y es debido a que la fracción β-hCG es similar a la hormona TSH.
Existen signos de preeclampsia o hipertensión gestacional del primer trimestre hasta en el
50% de los casos, y signos de insuficiencia respiratoria aguda en el 2% por embolismo
pulmonar de las células trofoblásticas, o por la asociación entre hipertiroidismo e
hipertensión arterial. Como complicaciones pueden aparecer coagulopatías y metástasis.
A la exploración el útero esta aumentado de tamaño, en el 50% de los casos, por encima de
lo que correspondería a la edad gestacional,3 de consistencia blanda, sin signos de actividad
fetal a partir de la semana 12 y siempre que se trate de una mola total. También puede que
la paciente presente un tamaño uterino menor al esperado para la edad gestacional.3 El
cuello está cerrado, con metrorragia en cantidad variable, y raramente se observa la
expulsión de vesículas. Pueden haber quistes teca-luteínicos bilaterales en ovario en cerca
del 20% de los casos,3 debido al estímulo de la β-hCG.

La enfermedad trofoblástica maligna va a cursar con metrorragias por lo general intensas,


hay una elevación de la β-hCG y en la ecografía se objetiva la cavidad uterina con signos de
ocupación atípica.

Diagnóstico:

Por la clínica, y pruebas complementarias como la determinación de la β-hCG y la ecografía.


La determinación de la β-hCG se basa en que el trofoblasto produce la hormona
gonadotropina coriónica, presentando cifras elevadas, y su cuantificación va a servir para
diagnóstico, valorar el pronóstico, y el seguimiento postratamiento. La ecografía revela un
útero aumentado de tamaño que no corresponde con la amenorrea, con ecos en su interior,
puntiformes que corresponderían a las vesículas y que asemejan copos de nieve o panal de
abeja. No se aprecia saco gestacional ni estructuras fetales y, en ambos ovarios se
aprecian quistes teca-luteínicos como formaciones ováricas redondas, econegativas, con
múltiples tabiques en su interior
BIBLIOGRAFÍA

1. Mazur MT, Kurman RJ. Gestational trophoblastic disease and related lesions. En:
Kurman RJ editor. Blaunstein’s pathology of the female genital tract. 4th ed. New
York: Springer-Verlag. 1994, p. 1049-93.
2. Kurman RJ, Young RH, Norris HJ, Main CS, Lawrence WD, Scully RE.
Immunocytochemical localization of placental lactogen and chorionic gonadotrophin
in the normal placenta and trophoblastic tumors, with emphasis on intermediate
trophoblast and the placental site trophoblastic tumor. Int J Gynecol Pathol 1984;
3: 101-21.
3. Berkowitz RS, Golstein DP. The management of molar pregnancy and gestational
trophoblastic tumours. En Knapp RC, Berkowitz RS, editores. Gynecologic Oncology,
2nd ed. New York: Mc Graw-Hill 1992, p. 328-38.
4. De Agustín P, Ruiz A, López F, Contreras F. Patología de la enfermedad
trofoblástica. Simposio Enfermedad Trofoblástica 1972; 79-98.
5. Salem S. Ultrasound diagnosis of trophoblastic disease. En: Sanders RC, James
AE(Jr) editores. Ultrasonography in Obstetrics and Gynaecology. New York:
Appleton-Century Crofts: 1977; p. 255-66.
6. Silverberg SG, Kurman RJ. Tumors of the uterine corpus and gestational
trophoblastic disease. En: Rosai J, Sobin LJ, editores. Atlas of tumor pathology:
tumors of the uterine corpus and gestational trophoblastic disease, fasc. 3, ser. 3.
Washington DC: Armed Forces Institute of Pathology; 1992, p. 219-85.
7. Szulman AE, Surti U The syndromes of hydatiform mole II. Morphologic evolution
of the complete and partial mole. Am J Obstet Gynecol. 1978; 132: 20-7.

47.- Femenino de 29 años. Acude al servicio de urgencias por presentar salida de líquido
vaginal. Antecedentes: G2, P1, cursa embarazo de 36 semanas de gestación exploración
física: cervix con 10% de borramiento, 1 cm de dilatación y Tarnier positivo.
¿La complicación más frecuente en esta paciente es?

a) Parto pre término.


b) Sepsis neonatal.
c) Corioamnioitis.
d) Endometritis.

Corioamnioitis: El diagnóstico de la infección intraamniótica (IIA) es básicamente clínico.


La corioamnionitis se debe descartar en toda gestante que presente fiebre sin foco
aparente, sobre todo si se sospecha o se ha confirmado una rotura de membranas. Los
criterios más empleados para el diagnóstico son: fiebre materna y, al menos, 2 de los
siguientes signos: taquicardia materna, taquicardia fetal, irritabilidad uterina, leucocitosis
materna o líquido amniótico purulento o maloliente.
Progresos de obstetricia y ginecología: revista oficial de la Sociedad española de
ginecología y obstetricia, ISSN 0304-5013, Vol. 48, Nº. 6, 2005 , pags. 316-317.
48.- Femenino de 22 años de edad, acude al servicio de urgencias por referir malestar
general, fosfenos, náusea y vómito. Antecedentes: cursa con embarazo de 36.5 SDG, G2,
A1, C0. Exploración física: TA 185/110 mmHg, FC 120 lpm, FR 35 x’, T37.9°, alerta,
inquieta, aprecia ictericia, cardiopulmonar sin compromiso, abdomen globoso a expensas de
útero grávido, se detecta PUVI, longitudinal, cefálico, dorso a la derecha, FCF 142 x’, al
tacto genital cérvix, central, formado, cerrado, extracción de guantes sin evidencia de
pérdidas genitales, extremidades inferiores edema +++.

El dato de laboratorio que apoya el diagnóstico en esta paciente es:

a) Hematuria.
b) Proteinuria.
c) Coluria.
d) Piuria.

Referencia:
La preeclampsia se define por aumento de la presión arterial y la presencia de proteinuria
durante el embarazo.
1) Leve: las pacientes suelen tener unas cuantas manifestaciones, y su presión arterial
diastólica es menor de 110 mmHg. En ocasiones hay edema. La cifra de plaquetas es
mayor de 100 000/µl.
2) Grave: los síntomas son más notorios y persistentes. La presión arterial casi
siempre es con niveles mayores de 160/110 mmHg. Puede haber trombocitopenia
que avance hasta coagulación intravascular diseminada.
Se requiere hospitalización para las mujeres con preeclampsia; debe obtenerse una
biometría hemática completa con cifra de plaquetas y determinación de electrolitos, que
además incluya enzimas hepáticas. Se obtiene una muestra de orina de 24 horas para
determinar la depuración de creatinina y proteínas totales al ingreso hospitalario.
Se debe controlar la hipertensión arterial, para evitar sufrimiento fetal, así como
empeoramiento de la paciente.

Bibliografía:

1. McPhee S, Papadakis M, et. al. Diagnóstico Clínico y Tratamiento 2010. Lange, McGraw
Hill,
49ª edición, México, 2010.
2. Sibai BM, Diagnosis, prevention, and management of eclampsia. Obstet Gynecol. 2005.
Feb; 105;: 402 – 410.
49.- Femenino de 33 años gestante, acude a consulta para realizarse ultrasonido de
control. Reportando: Placenta marginal, embarazo de 37 SDG. Antecedentes: G/5 P/4.
La complicación esperada en esta paciente es:

a) Desprendimiento de placenta normoinserta.


b) Hemorragia materna.
c) Parto pretérmino.
d) Restricción en el crecimiento intrauterino.

Placenta previa: La placenta en lugar de estar implantada en el cuerpo del útero por fuera
del orificio cervical interno, se localizaa por encima del orificio interno o cerca de él. En la
placenta marginal el borde la placenta está en el margen del orificio. La multiparidad y la
edad avanzada parecen favorecer la placenta previa. El hecho más carácterístico de la
placenta previa es la hemorragia no dolorosa, la cual no suele aparecer hasta casi del final
de segundo trimestre o más tarde. Complicaciones maternas: Hemorragia y muerte.
Pritchard, Williams Obstetricia, 3ªEdición, Salvat editores, pag. 395-39

50.- Acude femenino de 55 años, se queja de presión pélvica y una masa en la entrada
vaginal, hace 3 años tuvo su último periodo menstrual. No tomo terapia de reemplazo, tiene
dificultad para evacuar. Tiene una tos crónica y una historia de tabaquismo positivo a razón
de 30 cajetillas año. Ha tenido 3 partos y el ultimo bebe pesó 4,500 gramos. Tiene una
orina de 60 cc. ¿Cuál de los siguientes hallazgos es más probable que encuentres en la
exploración pélvica?

a) Rectocele
b) Cistocele
c) Enterocele
d) Uretrocele

El Rectocele es una hernia de la pared anterior del Recto hacia la porción posterior de la
vagina. La incidencia real de esta “deficiencia anatómica” es desconocida y en muchas
ocasiones es un resultado del paso del tiempo. Es un hallazgo muy frecuente del examen
perineal, siendo en múltiples ocasiones asintomático.
El rectocele puede ser un hallazgo importante del síndrome de Obstrucción Defecatoria
(SOD). No debe tomarse como una deficiencia anatómica única, sino como parte importante
de un problema anatomo-fisiológico complejo.
Un principio importante es la etiología y anatomo-patología del rectocele. Existen varias
teorías sin consenso principal. La existencia, deficiencia o alteraciones del septo
rectovaginal son controversias importantes. No existe una fascia visceral que separe el
recto de la vagina o que forme un septo específico. Existe frontera entre donde termina la
pared anterior del recto y donde comienza la pared de la vagina, pero múltiples estudios no
han encontrado un septo rectovaginal específico. El septo puede estar formado de una
película casi transparente hasta una pared de consistencia fibromuscular fuerte.
El rectocele es un hallazgo común. Se presenta en el 80% de las pacientes femeninas y
13% de los masculinos en una defeco grafía (> a 1cm.) Entre mas grande es el rectocele,
mayor son los síntomas asociados, Dificultad en la evacuación, constipación crónica, dolor
rectal y perineal, sensación de masa y en ocasiones sangrado. La necesidad de presión
manual para ayudar a la evacuación o para vaciar el rectocele es común en más del 50% de
los pacientes.
El diagnostico se hace con un simple tacto rectal e inspección vaginal, pero debe recordarse
que rara vez es el rectocele un hallazgo aislado. La presencia cistocele, peritoneocele,
enterocele u otros prolapsos perineales y problemas funcionales deben ser descartados
antes de proponer la reparación quirúrgica del rectocele. Defeco grafía,
Pruebas de función fisiológica del piso pélvico, pruebas de función urinaria, evaluación del
esfínter anorectal por ultrasonido y hasta la resonancia magnética han sido propuestos
antes de la cirugía.

Bibliografía:

Rectocele: Pathogenesis and surgical managment. Zbar AP, Linemann A, Fritsch H,


Beer-Gabel M, Pescatori M. Int J Colorectal Dis. (2003) 18:369-384.
Evaluation and Treatment of Women with rectocele. Cundiff GW, Fenner D, Obstetrics
and Ginecology 104(6): 1403-1416.
Stapled transanal rectal resection to treat obstructed defecation caused by rectal
intussusseption and rectocele. Renzi A, Izzo D, Di Sanrno (2—6) 21:661-667.
Rectocele repair using biomaterial augmentation. Altman D, Melgren A, Zetterstrom J.
Obstet Gynecol (2005) 60(11)753-760.

51.- Mujer de 26 años, con tumor anexial de 6 cm líquido, dolor abdominal, fiebre,
leucorrea, con historia de cervicovaginitis de repetición y dispareunia crónica, última
menstruación hace una semana.
El manejo más adecuado es:

a) Histerocopía
b) Colposcopía
c) Histerosalpingografía
d) Laparotomía exploradora
El avance de la laparoscopia en los últimos años ha hecho que se convierta en una técnica
fundamental en el diagnóstico y en la estadificación de la EIP, pero además nos ofrece la
posibilidad de realizar endoscopia quirúrgica por medio de una serie de procedimientos que
nos permitirían abreviar y mejorar la evolución natural de la enfermedad. Los
procedimientos quirúrgicos que podemos realizar durante la laparoscopia son:

1. Liberación de adherencias: Es indispensable en muchos casos para realizar el diagnóstico


y puede ser dificultosa en función de la antigüedad del proceso y de la laxitud de dichas
adherencias. En los casos de adherencias perihepática se ha descrito mejoría del cuadro
clínico al liberarlas mediante láser.

2. Drenaje de los abscesos: Son diversos los procedimientos que podemos realizar
laparoscópicamente pero el más frecuente es la apertura de la trompa cerca de la fimbria
con aspiración del contenido purulento y posteriores lavados.

3. Extirpación de la enfermedad.

4. Lavados peritoneales: Con sistemas de aspiración-irrigación con povidona yodada diluida


han sido recomendados o con suero salino.

5. Establecer el diagnóstico diferencial con otras entidades.

Biopsia de endometrio:

La presencia de células plasmáticas en biopsia endometrial sugiere EIP.

CLASIFICACION LAPAROSCOPICA DE LA ENFERMEDAD PELVICA INFLAMATORIA

El stándar de oro para el diagnóstico de EPI es la laparoscopia, ya que además de visualizar


directamente los órganos pélvicos, permite la toma de muestras para estudios
bacteriológicos.

Los criterios laparoscópicos para el diagnóstico de EPI, se describen en la tabla No.3.

Tabla No. 3

CLASIFICACION LAPAROSCOPICA DE LA EPI

Leve Eritema, edema, las trompas se mueven


libremente. No hay exudado purulento.

Moderada Eritema, edema más marcado, material


purulento evidente. No hay movimiento libre
de las trompas. La fimbria puede no ser
evidente.
Severa Presencia de piosálpinx y/o absceso

REFERENCIAS BIBLIOGRÁFICAS

1. Barie P, Hydo L, Eachempati S. Longitudinal outcomes of intra-abdominal infection


complicated by critical illness. Surg Infect. 2004;5:365-73.

2. Boucher H, Talbot G, Bradley J, Edwards J, Gilbert D, Rice L, et al. Bad Bugs, No Drugs:
No ESKAPE! An Update from the Infectious Diseases Society of America. Clin Infect Dis.
2009;48:1-12.

3. Washington AE, Katz P. Cost and payment source for pelvic inflammatory disease.
JAMA. 1991;266:2565-9.

4. Center for Disease Control. Guidelines for treatment of sexually transmited diseases.
MMWR Morb Mortal Wkly Rep. 2006;55 (RR-11):56.

5. Jossens MO, Shachter J, Sweet RL. Risk factors associated with pelvic inflammatory
disease of diflering microbial etiologies. Obstet Gynecol. 1994;83:989-97.

6. Botella Llusiá J, Clavero Núñez JA. Tratado de Ginecología. 14ta ed. Madrid: Ediciones
Díaz de Santos; 1993.

7. Berek J, Berek S. Novak's Gynecology. 14th ed. Philadelphia,Pa: Lippincott Williams &
Wilkins; 2007.

8. Adimora A, Hamilton H, Holmes K, Sparling PF. Pelvic Inflammatory disease in sexually


transmitted diseases. 2nd International Edition. New York: Mc Graw-Hill Inc. 1994. p. 254-
270.

52.- Femenino de 25 años, con embarazo de término, sin antecedentes de control prenatal.
G 3. C-1. Se ingresa al servicio de obstetricia por presentar actividad uterina regular y
dolorosa. Ef.: Deambulante, tranquila, adecuada coloración de tegumentos, abdomen con
fondo uterino a 32 cm. con producto único vivo en situación transversa dorso inferior FCF
144, al tacto vaginal cérvix dilatado a 3 cm. y membranas íntegras. Se realiza cesárea con
retención de placenta e invasión a vejiga.
La alteración placentaria que presenta esta paciente es:

a) Placenta increta
b) Placenta acreta
c) Placenta percreta
d) Placenta marginal
 Es la penetración y adherencia anormal de la placenta en la pared uterina.

 Se divide en:
 Placenta acreta.
 Placenta increta.
 Placenta percreta.

 ACRETA: Las vellosidades se adhieren al miometrio.

 INCRETA: Penetran más de la mitad del espesor del miometrio.

 PERCRETA: Atraviesa todo el espesor del miometrio, llegando a la


serosa, incluso atravesándola y adhiriéndose a órganos vecinos.

Factores:
 Endometriósis previa.
 Tumores submucosos. (Miomas)
 Cicatríz uterina previa. (Cesárea, miomectomía)
 Implantación baja. (Placenta previa)
 Malformaciones placentarias. (Placenta extracorial)
 Legrado enérgico previo.
 Extracción manual previa de una placenta.

Diagnóstico transparto-:
 Placenta retenida por más de 20 minutos.
 Imposibilidad para encontrar un plano de separación placentaria cuando se intenta
su extracción manual.
 Hemorragia incontrolable después de la pseudoextracción.
 El diagnóstico histopatológico corrobora el diagnóstico clínico.
 Escenario menos deseable.

Tratamiento:
 Histerectomía Obstétrica.
 Constituye una cirugía no planeada y secundaria al hallazgo del acretismo
placentario con sangrado incohercible.

 Cesárea-Histerectomía. (Con diagnóstico previo)


 Cirugía planificada ante un correcto diagnóstico prenatal.
 Recomendación ACOG:
 Maduración pulmonar intrauterina.
 Inyectar al cordón umbilical 50 mg de metrotexate.
 Ligar el cordón en el nacimiento placentario y dejar la placenta in-situ.
 Embolización inmediata de arterias uterinas bilaterales, así como de ramas
de la división anterior de la arteria iliaca interna con alcohol polivinílico.
 Continuar con 5 dosis I.M. de 50 mg de metrotexate y cuantificar niveles
de βhCG.
 Programar Histerectomía Total Radical Abdominal y/o Cistectomía parcial
y/o resección pared anterior recto.

Lee et al. Conservative Management of Placenta Percreta. Obstet Gynecol, 112(2):421-424

53.- Se presenta paciente de 40 años de edad a su consultorio refiriendo mastalgia que


es más severa antes de la menstruación. A la palpación hay nodularidad excesiva,
hiperestesia y áreas quísticas que la paciente refiere disminuyen en tamaño después de la
menstruación. El diagnóstico más probable es:

a) Fibroadenomas
b) Papiloma intraductal
c) Mastopatía fibroquística
d) Cáncer de mama

Es raro encontrar una mujer mayor de 35 años a quien no le hayan dicho, en un examen
físico mamario, ecográfico o mamográfico, que tiene quistes en la mama o que su mama es
mastopática.

Es el trastorno benigno de la mama más frecuente y consiste en un aumento del tejido


mamario, especialmente en las zonas superiores y externas de las mamas, hacia las axilas,
que las hace más densas.

La mastopatía fibroquística suele presentarse en ambas mamas, aunque puede ser de


diferente intensidad en una que en otra.

Puede presentarse a cualquier edad después del inicio de la menstruación, pero es más
probable que aparezca entre los 30 años y la menopausia. Raramente se presenta más tarde
de esa edad.
El origen de este trastorno es funcional y responde a desequilibrios de las hormonas
sexuales femeninas y puede condicionar la aparición de quistes mamarios.

Los síntomas pueden fluctuar de leves a severos en una mastopatía fibroquistica mamaria,
se acentúan típicamente antes de cada período menstrual y desaparecen inmediatamente
después.

Los síntomas abarcan:

• Consistencia de protuberancias (como de "guijarros"), irregular y densa del tejido


mamario
o generalmente más notoria en la parte superior externa de la mama
• Molestia en las mamas
o generalmente en ambas mamas
o puede ser persistente o puede aparecer y desaparecer
• Sensación de llenura en las mamas
• Sensibilidad y dolor sordo e intenso
• Sensibilidad y edema premenstrual
• Secreción ocasional del pezón

Bibliografía:

• Jones III HW, Wentz AC. Tratado de Ginecología de Novak. Editorial


Interamericana-McGraw Hill. Undécima Edición 1994.
• DiSaia-Creasman. Oncología Ginecológica Clínica. Editorial Mosby. Cuarta Edición
1994.
• Van Dinh T. Sumario de Patología Ginecológica. Editorial La Prensa Médica
Mexicana. 1992.
• Pernoll ML. Diagnóstico y Tratamiento Ginecoobstétricos. El Manual Moderno.
México. Sexta Edición 1991.
• Alvarez-Bravo A. Diagnóstico de los trastornos menstruales y hemorrágicos. En:
Alfonso Alvarez Bravo y su obra. Editorial Marketing y Publicidad SA. Tomo I.
1993.
• Vázquez E. Aspectos histoquímicos del endometrio humano después del tratamiento
con progestágenos sintéticos. Gac Méd Méx 1966; 96: 1277-93.
• Huerta MR, Malacara JM, Rivera-Cisneros A, Díaz Cisneros FJ. Síntomas en
adolescentes de dos ciudades de México y su asociación con el ciclo menstual. Ginec
Obstet Méx 1994; 62: 146-50.
54.- Se trata de femenino de 32 años, acude a consulta por elevación de T/A AGO. G/2
P/1 se diagnostica preclampsia leve, el fármaco de elección que se administra en esta
patología es:

a) Nifedipina.
b) Inhibidores de la enzima convertidora de angiotensina.
c) Clonidinas.
d) Alfametildopa.

• Prevenir complicaciones a corto plazo de las mujeres con PA elevada que


comprometa el bienestar fetal
• Cuando la PAS es mayor o igual a 150 mmHg y la PAD mayor o igual a 100 mmHg.

 El propósito es alcanzar cifras de TA alrededor de 140/90.


 La medicación antihipertensiva se reserva para los casos en que la PAD ≥ 100 mmHg.
 Se recomienda continuar el tratamiento antihipertensivo previo al embarazo,
exceptuando el uso de IECA.
 La alfametildopa y la hidralazina vía oral son los fármacos de elección dado su uso
extensivo con seguridad y eficacia y sin efectos colaterales para el feto (excepto
hidralazina en lupus).

• ALFA METILDOPA
500-2000 MG/DÍA
• HIDRALAZINA
50-200 MG/DÍA
• LABETALOL
100-400 MG/DIA
• ATENOLOL
50-200 MG/DÍA
• NIFEDIPINA
10-30 MG/DÍA
1. Aagard K, Belfort M. Eclampsia: Morbility, mortality, and management. Clin
Obstet Gynecolol. 2005; 48: 12-23.
2. Oyarzún E. Síndrome hipertensivo del embarazo en Oyarzún E. Ed. Embarazo de alto
riesgo. Ediciones Universidad Católica de Chile. Santiago. 1997: 157-175.
3. Roberts J, Redman C. Pre-eclamsia: More than pregnancy induced hypertens

55.- Paciente de 25 años, Gesta 1, Para 1. Con dos citologías “lesión de alto grado”, prueba
de Schiller positiva y biopsia de cérvix que demuestra carcinoma “In Situ”. La conducta es:

a) Conización.
b) Histerectomía total abdominal.
c) Histerectomía y salpingooforectomía bilateral.
d) Electrocauterización del cérvix.

La conización cervical es el tratamiento de elección en pacientes con cáncer cervicouterino


microinvasor y más si existedeseo de fertilidad. Asimismo, la histerectomía extrafasciales
un método adecuado en lesiones de 0.5 a 3 mm de invasión.Además se propone que, para
pacientes con lesiones de 3.1 a 5 mm de invasión, a partir de la membrana inicial sinfactores
de mal pronóstico como invasión vascular y linfática, sean tratadas con histerectomía
extrafascial, ya que en aquellasa las que se realizó linfadenectomía pélvica, con este tipode
lesión, no se encontró metástasis a ganglios linfáticos.

Resultados del tratamiento en cáncer cervicouterino microinvasor en el Instituto


Nacional de Cancerología de México (1980-1999)

1.- Mestwerdt G. Fruhdiagnose des Kollumkarzinoms. Zentralb Gynaekol, 1947 ;69 :326.
2. - Morrow CP, Curtin JP. Surgery for cervical neoplasia. In Gynecologic Cancer Surgery.
New York, Churchill Livingstone, 1996, p 472.3.
3. - Burghardt E, Holzer E. Diagnosis and treatment of microinvasive carcinoma of the
cervix uteri. J Obstet and Gynecol 1977; 49:641-653.
4.- Sedlis A, Sall S, Tsukada Y, et al. Microinvasive carcinoma of the uterine cervix: a
clinical-pathologic study. Am J. Obstet Gynecol. 1979;133:64.
56.- The Virchow triad of thrombus in the etiology of pulmonary embolism is constituted
by:

a) Venous stasis, endothelial injury and hypoxemia.


b) Hypoxemia, hypercoagulability and cough
c) Venous stasis, hypercoagulability, and endothelial damage.
d) Dry Cough

Tromboembolismo pulmonar

Etiología:

El 90% de los casos de tromboembolismo pulmonar tienen su origen en venas de las


extremidades. Es favorecida por la triada de Virchow: éstasis venosa, daño de la íntima,
aumento de la coagulabilidad (puerperio, cirugía mayor, cáncer, uso de anticonceptivos,
policitemia vera, síndrome de hipercoagulabilidad (deficiencia de PC, PS, ATIII, resistencia
a la proteína C).

Fisiopatología.

Efectos respiratorios: aumento del espacio muerto alveolar, broncoconstricción,


taquipnea e hipoxemia, alteración ventilación/perfusión (V/Q) por redistribución de flujo.

Efectos hemodinámicos: La reducción mecánica leve o moderada no produce


aumento de la resistencia en forma significativa. Sobre el 50% de los casos presentan
incremento brusco de la resistencia y presión. Al efecto del émbolo se suman las aminas
liberadas por las plaquetas. En el 60-70% se desencadena cor pulmonale agudo con
disminución brusca del gasto cardiaco.

Manifestaciones clínicas del tromboembolismo pulmonar:

Disnea, Taquipnea, Dolor pleurítico, Crepitantes, Tos, Taquicardia, Hemoptisis,


broncoespasmo, cianosis, síncope, palpitaciones. Síndromes: disnea aguda de causa
desconocida, hemoptisis y/o dolor pleurítico, shock cardiogénico.

British Thoracic Society, Standards of Care Committee. Suspected acute pulmonary


embolism: A practical approach. Thorax 1.997; 52 (suppl 4): S1-S23.
The PIOPED Investigators. Value of the ventilation/perfusion scan in acute
pulmonary embolism: results of the Prospective Investigation of Pulmonary
Embolism (PIOPED). JAMA 1.990; 263: 2.753-9.

Goodman PC. Spiral CT for pulmonary embolism Sem Resp Crit Care Med 2.000;
21(6): 503-10.

57.- Masculino de 47 años de edad que cursa con un ataque agudo de gota, la articulación
que se afecta con mayor frecuencia en éste caso es:

b) La rodilla
c) La muñeca
a) La primera metatarso-falángica
d) El codo

El American College of Rheumatology tiene 11 criterios, y la presencia más o menos de seis


sugiere la presencia de gota. Los 11 criterios son:

1. Más de un ataque de artritis activa


2. Inflamación máxima desarrollada a lo largo de un día.
3. Ataque de oligoartritis.
4. Enrojecimiento observado en la articulación.
5. Primera articulación metatarsofalángica dolorosa o inflamada
6. Ataque unilateral de la primera articulación metatarsofalángica
7. Ataque unilateral de la articulación del tarso
8. Tofo (probado o sospechado)
9. Hiperuricemia
10. Hinchazón asimétrica en la radiografía dentro de una articulación
11. Terminación completa de un ataque.

Bibliografía:

Coll JM, Blanch J. Hiperuricemia y gota. Aspectos terapéuticos. Jano 1997;1201:47-


52.
González Barber A, Carlavilla AB. Guía en Hiperuricemia y Gota. Madrid:
EDIMSA, 2003.
Nadal A. Farmacovigilancia. Alopurinol: aumento en la incidencia de episodios agudos de
gota. Offarm 2000;19(1):128.
Pérez Ruiz F, Calabozo Raluy M, Ugalde Espiñeria J, Herrero Beites AM.
Artropatías microcristalinas I. Hiperuricemia y gota. Medicine
2001;8(34):1765-72.
Sánchez Pozo A, Faus MJ. Hiperuricemia y gota. Pharm Care Esp 2003;5:105-9.
Sancho Bueso T, Bernardino de la Serna I, García Puig J. Consulta diaria.
¿Qué haría usted ante un paciente con hiperuricemia? Medicina Integral 2000;3(35):100-
11.
Sancho Bueso T, García Puig J. Criterios terapéuticos ante la hiperuricemia.
Revista Clínica Española 2001;2(201):85-7.

58.- Cuando se observa una imagen histológica de acantosis con elongación de las crestas
interpapìlares que incluso se fusionan entre sí, hiperparaqueratosis y acúmulos epidérmicos
de leucocitos polimorfonucleares, estamos hablando de:

a) Psoriasis.
b) Icitiosis.
c) Dermatitis (eccema).
d) Epidermólisis.

Psoriasis:

La psoriasis es una enfermedad cutánea caracterizada por un curso crónico, que cursa a
brotes y tiene hallazgos clínicos variables. Las manifestaciones cutáneas de esta
enfermedad son tan características (Tabla 1) que el diagnóstico suele realizarse con
facilidad. En la actualidad se contempla la psoriasis como una enfermedad de base
inmunológica, mediada por los linfocitos T, que asocia inflamación dérmica y
secundariamente hiperplasia epidérmica.
Es probablemente la enfermedad dermatológica mediada inmunológicamente más
prevalente.

Tabla 1
Características clínicas de la psoriasis

1. Placa eritematosa
2. Escamas blanquecinas
3. Buena delimitación
4. Signo de Auspitz
5. Fenómeno de Koebner
Psoriasis. Hiperparaqueratosis con acúmulos intracórneos de neutrófilos (microabscesos de
Munro). HE, x 400

Cambios histológicos de la psoriasis:

Infiltración de células mononucleares


Hiperplasia epidermica
Acantosis epidermica regular, con hipogranulosis, hiperqueratosis y
paraqueratosis
Atrofia epidermica suprapapilar
Infiltracióninflamatoria
Pustula espongiforme de Kogog
Microabscesos de Munro
Infiltrado de linfocitos CD4 y CD8 en dermis
Cambios vasculares
Proliferación de vasos capilares dilatados ocupando las papilas

Bibliografía:

Arnold HL, James WD, Odom RB. Andrews : tratado de dermatología. 1993. 4ª ed.
Barcelona: Masson; 1993
Ashcroft DM, Po AL, Williams HC, Griffiths CE. Systematic review of comparative
efficacy and tolerability of Calcipotriolin treating chronic plaque psoriasis. BMJ 2000;
320:963-967. [Medline][Texto completo]
Chalmers RJG, O'Sullivan T, Owen CM, Griffiths CEM. Intervenciones para la psoriasis
guttata (Revisión Cochrane). En: La Cochrane Library Plus, Número 2, 2002. Oxford:
Update Software
Ferrándiz Foraster C. Dermatosis eritematoescamosas (I). Psoriasis. Eritrodermias. En:
Ferrándiz C, ed. Dermatología Clínica.2ª ed. Madrid: Harcourt; 2001. p. 165-175.
Freedberg I, Eisen A, Katz SI, Wolff K, Fitzpatrick TB, Goldsmith LA et al, eds.
Dermatology in general medicine. 5th ed. New York: McGraw-Hill; 1999
Riffiths C E, Clark C M, Chalmers R J, Li Wan Po A, Williams H C. A systematic review of
treatments of severe psoriasis. Health Technology Assessment, 2000;4(40):1-125.
Naldi L, Rzany B. Chronic plaque psoriasis. Clin Evid 2002; 7: 1488-1507.
59.- Masculino de 66 años que cursa con postinfarto agudo de miocardio una medicación
generalmente indicada, por disminuir la mortalidad, es:

a) Nitritos.
b) Betabloqueadores.
c) Anticoagulación oral.
d) Antiarrítmicos.

• Los BB son recomendados en todos los pacientes que han presentado un IAM,
siempre que no tengan contraindicaciones para su uso, y de modo permanente
(indefinidamente): clase de recomendación I, nivel de evidencia A.

• Se ha puesto en evidencia que los BB son infrautilizados en esta indicación.

• Más de 35.000 pacientes han sido incluidos en estudios postinfarto con BB.

• Se ha demostrado una reducción de la mortalidad total, muerte súbita y reinfarto


del orden del 20-25%.

Clase de Nivel de
Situación clínica/indicación
recomendación evidencia

Todos los pacientes sin


contraindicaciones, indefinidamente
Para mejorar supervivencia I A
Para prevenir reinfarto I A
Prevención primaria de la muerte súbita I A
Prevención/tratamiento de arritmias
IIb B
ventriculares tardías

• Los BB siguen estando indicados en el tratamiento de la HTA, aunque existen


críticas a su empleo en el primer escalón.

Pese a ello, son sin duda de primera elección si el/la paciente presenta angina, cardiopatía
isquémica en general, insuficiencia cardíaca, taquiarritmias, glaucoma o embarazo.

• Los BB son fármacos de primera elección en la insuficiencia cardíaca con disfunción


sistólica. Sólo el carvedilol, el bisoprolol o el nebivolol son BB que puedan ser empleados a
las dosis disponibles en nuestro medio para dicho tratamiento.
• Todos los pacientes sin contraindicaciones deben recibir BB si presentan
cardiopatía isquémica y, muy especialmente, si existe angina o han presentado un IAM.

ESC Expert consensus document on b-adrenergic receptor blockers. The Task Force on
Beta-Blockers of the European Society of Cardiology.
Eur Heart J 2004; 25: 1341–1362.

60.- Ante un paciente al que se ha determinado una deficiencia deficiencia de factor


intrínseco, se puede traducir en el siguiente tipo de anemia:

a) Micorcítica
b) Drepanocítica
c) Talasémica
d) Perniciosa

Carencia del factor intrínseco


El factor intrínseco es una sustancia natural que normalmente se encuentra en el estómago
y es necesaria para absorber la vitamina B12 de los alimentos.

Una carencia del factor intrínseco ocasiona anemia perniciosa y deficiencia de vitamina
B12, lo cual puede causar anemia y problemas del sistema nervioso y del cerebro
(neurológicos).

Las causas más comunes de anemia perniciosa abarcan:

• Debilitamiento del revestimiento del estómago (atrofia de la mucosa gástrica)


• El sistema inmunitario ataca las células que producen el factor intrínseco
(autoinmunidad contra las células parietales gástricas)
• Autoinmunidad contra el factor intrínseco en sí

El comienzo de la enfermedad es lento y puede tomar décadas para establecerse por


completo. Aunque la forma congénita ocurre en niños, la anemia perniciosa por lo general no
aparece antes de los 30 años en adultos y la edad promedio del diagnóstico es a los 60 años.

Referencias
Antony AC. Megaloblastic anemias. In: Goldman L, Ausiello D, eds. Cecil Medicine. 23rd ed.
Philadelphia, Pa: Saunders Elsevier; 2007: chap 170.
61.- Femenino de 33 años que cursa con 38.5 semanas de gestación que ingresa al servicio
con trabajo de parto. En el transcurso de trabajo de parto durante la dilatación presenta
dolor intenso y brusco. A la exploración usted observa metrorragia escasa y aumento del
tono uterino a la palpación abdominal que resulta muy doloroso. El diagnóstico más
probable es:

a) Placenta previa.
b) Desprendimiento de placenta.
c) Rotura de vasos previos.
d) Crioamnionitis hemorrágica.

Fisiopatología de la hemorragia
La hemorragia es el signo fundamental que domina el cuadro clínico de la placenta previa.
Para explicar su mecanismo existen distintas teorías:

Mecanismo de Jacquemier: Se produce crecimiento armónico de la placenta y del útero


hasta la semana 26, 28. Después el segmento inferior crece más deprisa y favorece el
despegamiento lo que origina la hemorragia en el embarazo.
Mecanismo de Schroeder: Las contracciones uterinas en el parto traccionan del segmento
inferior hacia arriba y empujan al feto hacia abajo despegando la placenta.
Mecanismo de Pinard: Explica las hemorragias gestacionales y del parto. El estiramiento de
las membranas de la zona de menor radio (orificio interno cervical) como consecuencia de
las contracciones tira de la placenta y la desprenden.
Mecanismo de Bartholomew: Explica la hemorragia en los casos de placenta previa central.
La zona placentaria que reviste el orificio interno es un área isquémica ya que no recibe
vasos deciduales. A este nivel disminuye la presión sanguínea, por lo que la sangre tiende a
dirigirse hacia esta zona y escapa por la cara materna.

En el alumbramiento también puede haber una hemorragia importante producida por un


doble mecanismo:
-desprendimiento parcial antes de la expulsión en los casos de placenta oclusiva.
-atonía uterina en la zona de inserción después de expulsada la placenta y vascularización
anómala.

Manifestaciones clínicas de la placenta previa en el embarazo

- Síntomas: Principalmente la hemorragia. Toda hemorragia vaginal acontecida en el tercer


trimestre debe hacer pensar en una placenta previa. Las hemorragias suelen ser
espontáneas, no acompañadas de dolor, de sangre roja y se presentan de forma
intermitente, con intervalos variables entre las mismas. Progresivamente se van haciendo
más frecuentes y más graves. La primera hemorragia suele aparecer en forma inesperada
generalmente nocturna, cesando en menos de media hora. Las hemorragias ulteriores son
más graves y más precoces.
- Signos: La consecuencia fundamental es la anemia materna que depende de la cuantía de la
hemorragia (la sangre es de origen materna ya que procede de espacios intervellosos).

- Exploración:
Exploración general para valorar la existencia de signos de anemia.
Exploración obstétrica: valorar el tamaño del útero (adecuado para la edad gestacional), es
blando e indoloro. A menudo la estética fetal está alterada (transverso, oblicuo, nalgas). No
debe efectuarse tacto vaginal cuando haya existido hemorragia en embarazo avanzado por
el riesgo de infección y de despegamiento y aumentar así la hemorragia.
Auscultación fetal normal.
Exploración ecográfica: es una técnica fundamental en el diagnóstico de la placenta previa.
Permite determinar la localización placentaria y la variedad de la placenta previa. En
general, la placenta puede identificarse a partir de la 9ª semana. No obstante, a lo largo de
la gestación por crecimiento uterino se produce un cambio en sus relaciones con el útero
("emigración placentaria", imagen de desplazamiento). Así el diagnóstico de certeza de
límites placentarios solo puede establecerse hacia la semana 34. Siempre será necesario
hacer una adecuada identificación del orificio cervical interno (más fácil con sonda
transvaginal). Añadiendo al estudio Doppler color se observa la vascularización y las zonas
que sangran.

62.- Una mujer de 23 años de edad G/1, tuvo un parto vaginal espontáneo con un producto
con peso de 4,350 g. después de 5 minutos de tracción suave del cordón umbilical se
expulsó la placenta, que parece estar intacta. Se inició el masaje del fondo uterino y se
pidió a la enfermera que administrara 20 unidades de oxitocina en 100 ml de solución
Ringer lactato. Después de una inspección cuidadosa del canal del parto se observa una
laceración de segundo grado y una laceración de 2 cm en la pared vaginal izquierda que se
intentó reparar. En la E.F.se encuentra un fonso uterino blando y atónico. Los signos vitales
son: temperatura 37.1°C, TA 164/92, FC 130x’, FR 18 X’. ¿Cuál de los siguientes es el mejor
tratamiento?

a) Oxitocina 10 unidades directas en goteo intravenoso

b) Metilergonovina 0.2 mg IM

c) Prostaglandina F 0.25 mg IM

d) Legrado
Morgan M, Siddighi S. Ginecología y obstetricia, National Medical Series. 5° edición. Mc
Graw Hill. Pp. 28. La atonía uterina es la causa más común de hemorragia puerperal. El
masaje energético y la oxitocina diluida no han sido útiles para interrumpir la hemorragia y
por tanto el siguiente paso es agregar un fármaco uterotónico. La metilergonovina está
contraindicada porque la paciente se encuentra hipertensa a pesar de la hemorragia
intensa, el siguiente fármaco es la prostaglandina. La administración de oxitocina no diluida,
10 UI por vía IV podría causar hipotensión grave. La exploración manual podría ser
apropiada si se sospecha laceración como causa de hemorragia. El legrado es apropiado
para la hemorragia puerperal tardía, cuando se sospecha retención de los productos de la
concepción.

63.- Femenino de 37 años de edad, G-3, C-2. Es ingresada a hospital presentando cefalea,
acúfenos, fosfenos y epigastralgia en barra con embarazo de 34 semanas. E.F T/A
160/110, FC 84 x´, FR 18 x´, no presenta fiebre, somnolienta, sin agregados
cardioventilatorios, hepatalgia. F.U. de 25 cm. Producto único vivo. FCF 110 lpm, genitales
sin pérdidas ni modificaciones cervicales. Laboratorio: hb 9.8 g/dl, plaquetas de 54 mil, TP
11 seg TPT 27, TGO 160 ng/dl TGP 160 ng/dl, hiperbilirrubinemia indirecta, albuminuria
300 mg/dl, Acido Úrico de 8.1 mg/dl, creatinina de 1.5 mg/dl.
El diagnóstico más probable es:

a) Sx anticuerpos antifisfolípidos
b) Sindrome de hellp
c) Púrpura trombocitopénica trombótica
d) Hígado graso

DEFINICIÓN:
• Es una complicación de la preeclampsia en la cual además de la Hipertensión Arterial y
proteinuria hay presencia de anemia hemolítica, enzimas hepáticas elevadas y recuento
bajo de plaquetas

MANIFESTACIONES CLINICAS:
• Malestar general, fatiga y molestias inespecíficas 90%
• Cefalea 70%
• Epigastralgia 64%
• Vómito 22%
• Fosfenos 15%
• Visión Borrosa 11%
• Acùfenos 3%
• Ictericia
• Anemia no explicada
• Oliguria
BIBLIOGRAFIA:

Sibai baha, El síndrome HELLP. Universidad de Valencia , revista quincenal de Obstetricia


clínica y ginecología, Octubre 2003.
V. Cararach, Síndrome de HELLP y Repercusiones maternas. X curso intensivo de
formación continuada materno fetal. Enero de 2003.
Toirac, Abelardo. Síndrome de Weistein HELLP Hospital Ginecoobstetrico Tamara Bunke.
Junio 2002

De la Fuente, David. Síndrome HELLP. Medicina Universitária 2003; 5 (19): 101 -9


Andrea G. Witlin, DO, Baha M. Sibai, MD. Diagnosis and Management of women with
Hemolysis Elevate Liver Enzymes, and Pletelet Count (HELLP) syndrome. Hospital Physician.
Febrero 1999.
CIFUENTES B, Rodrigo. Ginecología y obstetricia.

64.- Se trata de paciente femenino de 29 años de edad que cursa con 34 SDG, G2. C.1 inicia
con datos compatibles de amenaza de parto pre-término y ruptura prematura de
membranas La principal causa de esta patología está relacionada estrechamente al
siguiente diagnóstico:

a) Traumatismo.
b) Idiopático.
c) Infección de vías urinarias.
d) Infección vaginal por cándida albicans.

La infección de vías urinarias es la complicación infecciosa más frecuente del embarazo.


De hecho, las mujeres son más susceptibles a la infección de vías urinarias debido a los
siguientes factores:

• Una uretra más corta.


• Fácil contaminación de la uretra por bacterias de la vagina y el recto.
• Posibilidad de la que la mujer no vacíe por completo la vejiga cada vez que
orina.
• Movimiento de bacterias al interior de la vejiga con cada relación sexual.

Se suman además, los cambios que el propio embarazo produce en el aparato urinario como
son la relajación del músculo liso de los uréteres que impide que la orina llegue
adecuadamente a la vejiga para ser eliminada, y la compresión que ejerce la matriz sobre la
vejiga lastimando su cubierta interna y dejándola incapacitada para vaciarse por completo.
Bibliografía: 1. Velasco MV. Prevención y tratamiento del parto pretérmino. Lo nuevo
acerca del viejo problema Rev Med IMSS 2001, 39 (%) 417-42.
2. Lastra ELG El parto pretérmino como problema de salud pública .Perinatol Reprodud.
Human.
Vol 15 No 2.Abril-junio 2001 113-14.

65.- Femenino de 28 años G/4, P/0 con 6 SDG acude a su primera visita prenatal. Su
historia obstétrica pasada es importante porque tiene tres pérdidas de producto en el
segundo trimestre. Refiere que en las tres ocasiones al presentarse al hospital presentaba
dilación cervical completa. No recuerda haber tenido contracciones dolorosas. Niega
antecedentes médicos y quirúrgicos. El examen físico es normal incluyendo un examen
pélvico que muestra un cervix largo y cerrado. Después de una larga discusión con la
paciente ella pide que se le practique un cerclaje durante este embarazo. Cual de los
siguientes es el momento más apropiado para realizarlo.

a) Inmediatamente
b) 12 a 16 semana
c) 24 a 28 semanas
d) 32 a 36 semanas

El cerclaje cervical tiene sus indicaciones en la profilaxis y tratamiento de la incompetencia


cervical.
La incompetencia o insuficiencia cervical representa un 10% de las causas de parto
pretérmino y está asociada a una importante morbimortalidad neonatal.
Las modificaciones cervicales en el segundo trimestre de gestación son causa de parto
prematuro y pueden deberse a:
1) Incompetencia cervical.
2) Pérdida de tejido conectivo tras una cirugía cervical (conización).
3) Defectos congénitos como la hipoplasia cervical tras exposición a dietilestilbestrol.
4) Infección intrauterina. Hasta un 51.5% de las pacientes con clínica compatible con
incompetencia cervical enmascaran un cuadro de infección intraamniótica subclínica.
Diferenciamos tres tipos de cerclaje:
1. El cerclaje se considera profiláctico o electivo (o primario) cuando se realiza de forma
electiva por historia previa de incompetencia cervical antes de evidenciar cambios en el
cerviz y generalmente suele realizarse entre las 13 y 16 semanas de gestación.
2. El cerclaje terapéutico secundario que se realiza tras la detección, en el seguimiento
obstétrico, de modificaciones en el cérvix antes de las 26 semanas de gestación. Se realiza
en pacientes con un riesgo potencial de parto pretérmino.
3. El cerclaje terapéutico terciario, en caliente, de rescate o “emergent cerclage” que
se realiza en pacientes que presentan la membrana amniótica visible a través del orificio
cervical externo o en vagina.

GUIA CLÍNICA:
INDICACIONES DEL CERCLAJE
Unitat de Prematuritat. Servei de Medicina Maternofetal.
Institut Clínic de Ginecologia, Obstetrícia i Neonatologia, Hospital Clínic de Barcelona
Responsables del protocolo: T.Cobo, M. López, M. Palacio
Creación: 24/01/07
Modificaciones: 05/09/07
Última actualización: 17/01/10

66.- Mujer de 26 años, con tumor anexial de 6 cm líquido, dolor abdominal, fiebre,
leucorrea, con historia de cervicovaginitis de repetición y dispareunia crónica, última
menstruación hace una semana. El diagnóstico más probable es:

a) Cistadenoma
b) Embarazo ectópico
c) Quiste de ovario
d) Enfermedad pélvica inflamatoria

La EIP puede cursar con los siguientes síntomas:

• Dolor abdominal bajo (incluyendo dolor anexial, dispareunia). Es el síntoma más


frecuente (95%)
• Aumento del flujo vaginal, flujo de características anormales (74%)
• Sangrado anormal (intermestrual, poscoital) (45%)
• Síntomas urinarios (35%)
• Vómitos (14%)
• Es posible la ausencia de síntomas
Y en ella podemos encontrar estos signos:

• Dolor a la movilización del cuello, dolor anexial en la exploración vaginal bimanual


(99%)
• En el examen con espéculo observamos cervicitis y descarga endocervical purulenta
(74%)
• Fiebre (> 38º C) (menos del 47%).
• Masa pélvica: sugiere abceso tuboovárico (ATO)
• Peritonitis

CRITERIOS CLINICOS PARA EL DIAGNOSTICO DE SALPINGITIS

a. Dolor abdominal con o sin rebote.


b. Sensibilidad a la movilización del cérvix.
c. Sensibilidad anexial.

Los tres criterios anteriores son necesarios para establecer el diagnóstico, con uno o más
de los siguientes:

a. Extendido de Gram de endocérvix positivo, para diplococos gram negativos


intracelulares
b. Temperatura mayor de 38°C
c. Leucocitosis (mayor de 10.000 por c.c.)
d. Material purulento (positivo para leucocitos) en la cavidad peritoneal
obtenido por culdocentesis o laparoscopia.

Establecido el diagnóstico clínico de EPI, se debe hacer la definición del estado clínico y
anatómico de la patología pélvica:

a) No complicada (limitada a trompas u ovarios)

1) Sin peritonitis pélvica

2) Con peritonitis pélvica

b) Complicada (masa inflamatoria o absceso que compromete trompa (s) u ovario (s)

1) Sin peritonitis pélvica

2) Con peritonitis pélvica


Bibliografía:

• Beigi RH, Wiesenfeld HC. Pelvic inflammatory disease: new diagnostic criteria and
treatment. Obstet Gynecol Clin Norh Am. 2003; 30 (4): 777 – 93
• Center for Disease Control. Guidelines for treatment of sexually transmited diseases.
MMWR Recomm Rep. 2002 May 10;51(RR-6):1-78
• Center for Disease Control. Guidelines for prevention and management (MMWR. 40: 1 -
25 1991) Pelvic inflammatory disease: guidelines for prevention and management.
MMWR Recomm Rep. 1991 Apr 26;40(RR-5):1-25.
• Hager WD, Eschenbach DA, Spence MR, Sweet RL. Criteria for diagnosis and grading
of salpingitis. Obstet Gynecol. 1983 Jan;61(1):113-4.
• Prodigy Guidance. Pelvic inflammatory disease. [Internet]. UK : NHS, Department of
Health; 2003. [Acceso 18 de Junio de 2005]. Disponible en:

Ross J. Pelvic inflammatory disease. Clin Evid. 2004 Dec;(12):2259-65.

• Royal College of Obstetricians and Gynaecologists. Pelvic Inflammatory Disease.


Guideline nº 32. [Internet]. RCOG; Mayo 2003. [Acceso 18 de Junio de 2005].

67.- Una mujer de 21 años, nuligrávida, acude a consulta para hablar sobre anticoncepción.
Es sexualmente activa desde hace 2 semanas y actualmente utiliza condón. Tiene
antecedente de asma, la cual se ha mantenido inactiva por 2 años. No toma medicamentos y
niega alergias. No hay AHF de cáncer. Su EF es normal. Después de una plática con su
médico, escoge tomar anticonceptivos orales combinados, y continúa tomándola por 6 años.
Ahora ha disminuido su riesgo de desarrollar:

a) Cáncer de mama
b) Cáncer cervical
c) Cáncer hepático
d) Cáncer ovárico

PROTECCION CONTRA EL CANCER EPITELIAL OVARICO.

Debido a la falta de estrategias eficaces para el diagnostico y tratamiento temprano del


cáncer de ovario, es de capital importancia la prevención (76). Esta patología es una
importante causa de morbilidad y mortalidad. Se estimó que para 1980 ocurrieron 137.600
casos nuevos en el mundo (86). Estudios a gran escala realizados por el Centro para el
Control de las Enfermedades de EE.UU. y el Royal Collage of General Practitioners del
Reino Unido (RCGP) indican que la supresión de la ovulación causada por los anticonceptivos
orales protege contra el desarrollo del cáncer epitelial ovárico (20,76). Este efecto
benéfico es directamente proporcional al tiempo de uso y persiste muchos años después de
suspendida la planificación con este método (87). La evaluación norteamericana denominada:
Estudio sobre cáncer y hormonas (CASH) demostró que el uso de uno a cinco años de
anovulatorios orales disminuye en un 50 a 70% el riesgo de cáncer ovárico (88).
Este efecto protector aumenta entre más sea el tiempo de uso y se extiende por lo menos
hasta diez años después de interrumpido (70,89).

La Organización Mundial de la Salud también realizó un estudio multicéntrico confirmando


el efecto protector de los anticonceptivos orales contra el cáncer epitelial ovárico (90).
Dos estudios de Cohortes realizados en Gran Bretaña, confirmaron el efecto protector de
la píldora al encontrar riesgos relativos de 0.3 y o.6 en mujeres que habían usado el método
en algún momento (83,91). El efecto protector es tanto para tumores malignos como para
Bordenline (92) y cada uno de los principales subtipos histológicos de cáncer epitelial
(70,93).

TABLA Nº 3

BENEFICIOS NO CONTRACEPTIVOS DE LOS ANTICONCEPTIVOS ORALES


COMBINADOS
MEJORIA DE LA DISMENORREA
CORRECCION DE LOS CICLOS MENSTRUALES IRREGULARES
PREVENCION DE QUISTES OVARICOS FUNCIONALES
PROTECCION CONTRA EL CANCER EPITELIAL OVARICO
MEJORIA DEL MITTELSCHMERZ
PROTECCION CONTRA EL CANCER ENDOMETRIAL
PROTECCION CONTRA TUMORES BENIGNOS MAMARIOS
DISMINUCION DE LA ENFERMEDAD PELVICA INFLAMATORIA
DISMINUCION EN LA INCIDENCIA DE EMBARAZO ECTOPICO
PREVENCION DE ANEMIA FERROPENICA
MENOR INCIDENCIA DE ARTRITIS REUMATOIDEA
MENOR INCIDENCIA DE OSTEOPOROSIS POST-MENOPAUSICA
MEJORIA DEL SINDROME PREMENSTRUAL
PREVENCION DE LA MIOMATOSIS UTERINA
MEJORIA DEL ACNE

1. Bagshaw S. the combined oral contraceptives. Risk and adverse effects in


perspective. Drug-Saf 1995; 12 (2): 91 - 96.
2. American Collage of Obstetricians and Gynecologist. Hormonal contraception.
ACOG technical bulletin Nº. 198 - October de 1994 Int J Gynaecol Obstet 1995; 48 (1):
115 - 126.
3. Mishell Jr DR. Oral contraception: past, present and future perspectives. Int J
Fertil 1992; 37 (1) Suppl: 7 - 18.
4. Melo NR, Pinotti J.Advances in hormonal contraception. Adv. in contraception 1994;
10 (suppl 1): 33 - 39.
5. Winkler UH, Schindler AE, Endrikat J, et al. A comparative study of the effects
of the hemostatic system of two monophasic Gestodene oral contraceptive containing 20
ug and 30 ug Etinil-Estradiol. Contraception 1996; 53: 75 - 84.
6. Coenen CMH, Thomas CMG, Borm GF, et al. Changes in androgens during treatment
with four low-dose contraceptives. Contraception 1996; 53: 171 - 176
7. Wilde MI, Balfour JA. Gestodeno. A review of its pharmacology, efficacy and
tolerability in combined contraceptive preparation. Drug 1995; 50 (2): 364 - 395.
8. Monterrosa A. Anticoncepción hormonal. EN : Caraballo J, Parra E, Taylor H.
Memorias del 1º Curso de actualización en Ginecología y Pediatría. Imprenta U. de
Cartagena. Cartagena.1994; 241 - 250.
9. Hannaford PC, Combined oral contraceptives : do we know all of their effects.
Contraception 1995; 51: 325-327.
10. Rosenberg MJ, Waugh MS, Meehan T. Use and misuse of oral contraceptives : risk
indicators for poor pill taking and discontinuation. Contraception 1995; 51: 283- 288.

68.- Acude a consulta una mujer de 25 años, cursando su 14ª semana de gestación, por
tenesmo vesical, disuria y escalofrío. Por su estado actual, ¿cuál de los siguientes
antimicrobianos recomendaría?

a) Metronidazol
b) Tetraciclina
c) Ampicilina
d) Levofloxacina

El Metronidazol no ha mostrado efectos tóxicos en humanos, pero es teratogénico en


modelo animal. Las tetraciclinas ocasionan coloración anormal de los dientes,
hepatotoxicidad y alteración en el desarrollo de huesos. Las sulfas podrían tener un efecto
deletéreo en el primer trimestre dada su actividad como antimetabolitos, y en los últimos
meses pueden favorecer kernicterus en el recién nacido si es que tiene alteraciones
metabólicas que favorezcan anemia hemolítica. Las quinolonas se han asociado a
malformaciones óseas en modelos animales, y se recomienda evitarlas si existen mejores
opciones.

Kasper DL, Braunwald E, Fauci AS, Hauser SL, Longo DL, Jameson JL. Harrison´s
Principles of Internal Medicine. McGraw Hill. 16 Ed. 789-806 pp.
69.- Femenino de 23 años acude al servicio l servicio de ginecología, por referir ciclos opso-
menorreicos desde el inicio de su menarquia, en los últimos 7 días ha incrementado 15 Kg.
de lo que pesaba habitualmente, se aprecia una gran cantidad de acne, pero además refiere
depilarse el área del bigote cada semana, y cree que esto le sensibiliza la piel para que
aumente el acne.
En el caso de ovario poliquístico el dato clínico que con más frecuencia les acompaña es:

a) Anovulación y esterilidad
b) Hirsutismo
c) Amenorrea
d) Obesidad

El síndrome de ovarios poliquísticos (SOPQ) afecta aproximadamente a un 4% de mujeres


en edad reproductiva y se caracteriza por anovulación crónica e hiperandrogenismo. Es la
causa más común de infertilidad en mujeres.
Se caracteriza clínicamente por acné, alopecia, hirsutismo, irregularidades menstruales e
infertilidad.
Los hallazgos de laboratorio más frecuentes son: aumento de la hormona luteinizante (LH),
aumento de la relación LH/FSH (hormona folículoestimulante), aumento de andrógenos
(tanto ováricos como adrenales) y de estrógenos circulantes. Otros hallazgos de
laboratorio habituales son una prueba tolerancia oral a la glucosa anormal y alteraciones en
el perfil lipídico.
Todo esto junto con las imágenes ecocardiográficas características define al síndrome.
La terapéutica permite dos grandes enfoques que pueden superponerse: la corrección de
las manifestaciones de hiperandrogenismo y el tratamiento de las alteraciones del eje
reproductivo (anovulación, esterilidad). Los antiandrógenos están fundamentalmente
indicados para tratar los síntomas virilizantes.
Las alternativas para inducir la ovulación son numerosas: al citrato de clomifeno y a la
antigua resección en cuña se agregan las gonadotrofinas humanas, pulsos de GnRH (hormona
liberadora de gonadotrofinas), medidas o fármacos para modificar los niveles de insulina, y
finalmente técnicas quirúrgicas endoscópicas para reducir la masa ovárica.

BIBLIOGRAFIA
1. Guzick D.Polycystic ovary syndrome: Symptomatology, pathophysiology, and
epidemiology. Am J Ostetric Gynecol 1998; 179 (6): 89-93.
2. Stephen Franks. Polycystic ovary syndrome. N Engl J Med 1995; 333(13): 853-861.
3. Gori J.R., Larusso A. Ginecología de Gori. 2ª Edición. Buenos Aires, Argentina.
Editorial El Ateneo. 2001.
4. Adams J., Polson D. W., Franks S. Prevalence of polycystic ovaries in women with
anovulation and idiopathic hirsutism. Br Med J 1986; 293: 355-9.
5. Copeland L. J . Ginecología. Buenos Aires, Argentina. Editorial Panamericana. 1ª
Edición. 1994.
6. Ehrmann D.A., Rosenfield R.L., Barnes R.B., Brigell D.F., Sheikh Z. Detection of
functional ovarian hyperandrogenism in women with androgen excess. N Engl J Med
1992; 327:157-162.
7. Kahasar-Miller M., Conway Myers B., Boots L., Azziz R. Steroidogenic acute
regulatory protein (StAR) in the ovaries of healthy women and those with
polycystic ovary syndrome. Am J Obstet Gynecol 2001; 185(6): 1381-7.
8. Pérez Sánchez A. Ginecología. Santiago de Chile. Publicaciones Técnicas
Mediterráneo. 3ª Edición. 1995.
9. Velázquez E., Mendoza S., Hamer T., Sosa F., Glucck C. Metformin therapy in
women with polycistic ovary syndrome reduces hiperinsulinemia, insulin resistence,
hyperandrogenemia, and systolic blood pressure, while facilitating menstrual
regularity and pregnancy. Metabolism 1994 ; 43: 647-655.

70.- Se trata de femenino de 31 años, con un cuadro clínico de sangrado menstrual


irregular, dismenorrea, dispareunia, y una esterilidad de 3 años de evolución, con
resultados de laboratorio hormonal normal y un estudio ecográfico transvaginal que informa
de un útero normal y sendas formaciones quística ováricas bilaterales de 4 cms . ¿Cuál sería
la orientación diagnóstica?

a) Hemorragia uterina disfuncional.


b) Síndrome del ovario poliquístico.
c) Endometriosis.
d) Quistes dermoides bilaterales.

Causas y síntomas de endometriosis

Las causas de la endometriosis aún no se conocen. Las células del revestimiento


interno del útero de alguna manera se desplazan hasta zonas externas al mismo y
siguen creciendo. Este desplazamiento podría quizás deberse a que pequeños
fragmentos del revestimiento uterino, desprendidos durante la menstruación,
retrocedan hacia las trompas de Falopio en dirección a los ovarios hasta entrar
en la cavidad abdominal, en lugar de salir con el flujo menstrual a través de la
vagina.
La endometriosis causa dolor en la parte inferior del abdomen y la zona pélvica,
irregularidades menstruales (como manchar antes de la menstruación) e
infertilidad. Algunas mujeres con endometriosis grave no presentan síntomas,
mientras que otras con la enfermedad en grado mínimo sufren un dolor
invalidante. Con frecuencia, el dolor menstrual debido a la endometriosis no
aparece hasta años después de desarrollar la enfermedad. En algunos casos, se
constata dolor durante el coito (dispareunia), antes o durante la menstruación.
El tejido endometrial adherido al intestino grueso o a la vejiga urinaria puede
provocar hinchazón abdominal, dolor durante las deposiciones, hemorragia rectal
durante la menstruación o dolor en la parte inferior del abdomen durante la
micción. Así mismo, cuando el tejido se localiza en un ovario o una estructura
cercana puede dar lugar a la formación de una masa llena de sangre
(endometrioma). En ocasiones, el endometrioma se rompe bruscamente o se
escapa algo de su contenido, lo que causa un agudo y repentino dolor abdominal.

71.- Paciente de 40 años con nódulo mamario indoloro, de bordes imprecisos. La


mamo¬grafía revela imagen nodular, con espículas en todos sus márgenes, y 10
microcalcificaciones finas, agrupadas en el interior. El diagnóstico más probable, entre los
que se citan, es:

a) Fibroadenoma.
b) Quiste
c) Mamografía normal para la edad de la paciente.
d) Carcinoma

Técnicas diagnósticas
Exploración
Masa palpable o engrosamiento unilateral. La posibilidad de que una masa palpable en la
mama sea maligna está en relación con mayor edad, postmenopausia y con las siguientes
características en el examen físico: consistencia firme, aspecto sólido, bordes irregulares,
escaso desplazamiento sobre la piel, la región costal o los tejidos que le rodean, unilateral,
no dolorosa y la presencia de adenopatías axilares. Sin embargo, aún en ausencia de estos
factores un 10% pueden ser malignas, algunas veces una zona de engrosamiento que no llega
a masa puede ser cáncer. La coexistencia de masa y adenopatía axilar palpable debe
considerarse cáncer mientras no se demuestre lo contrario. El 90 % de las masas suelen
ser lesiones benignas. Las masas de superficie lisa y consistencia elástica están asociadas
a fibroadenoma en mujeres entre 20-30 años y a quistes en las mujeres de 30 a 40. La
exploración a realizar ante esta situación es una mamografía si hay antecedentes de cáncer
de mama y una ecografía sobre todo si existe dolor (ICSI, 2005).
Secreción por el pezón. Siempre se debe estudiar. Hay mayor riesgo de lesión maligna en el
caso de que la secreción contenga restos hemáticos y esté asociado a masa. La citología del
líquido expulsado sólo puede ser tenida en cuenta si es positiva. Está indicado realizar
mamografía y galactografía en el caso de que el exudado se presente en un solo conducto.
La presencia de secreción lechosa bilateral orienta a causa endocrinológica se ha de
realizar el diagnóstico diferencial de galactorrea (ICSI, 2005).

Dolor. Es uno de los motivos de consulta mas frecuente. En ausencia de masa otros
síntomas de sospecha suele ser debida a tensión premenstrual, dolor condrocostal y a otras
causas (ICSI, 2005). Está asociado con mayor frecuencia a cambios fibroquísticos en la
mama premenopáusica.
Síntomas cutáneos. La Enfermedad de Paget afecta al pezón y areola de forma unilateral,
clínicamente muy similar a la dermatitis crónica crónica eccematosa se asocia a un
carcinoma mamario intraductal subyacente. (Fitzpatrick, 2001)
La retracción del pezón o de la piel de presentación reciente se debe evaluar
cuidadosamente. Los fenómenos inflamatorios del tipo de eritema, induración, aumento de
temperatura y dolor pueden ser indicativos de un tumor inflamatorio de mal pronóstico. En
ocasiones un tumor evolucionado puede dar lugar a un cáncer ulcerado.

Imágenes
Mamografía
Tiene una sensibilidad y especificidad del 90%, siendo el método aislado de diagnóstico más
eficaz, aunque en mamas densas pierde sensibilidad. Utilizada para el screening puede
reducir la mortalidad del cáncer de mama en un 33%. Nos puede dar el diagnostico, la
presencia de multicentricidad o de lesiones sincrónicas.
La mamografía nos va a valorar distintos tipos de imágenes:
1: Signos primarios:
A. Masa dominante: Valorando tamaño, densidad, forma nitidez y estabilidad en el
seguimiento. Es la lesión más frecuentemente hallada.
Considerando la clínica y los datos mamográficos, ecográficos, etc. se establece la
probabilidad de malignidad del nódulo, pudiendo expresarla en las siguientes cuatro
categorías:

o Benigno
o Probablemente benigno
o Probablemente maligno
o Maligno

NÓDULO BENIGNO

NÓDULO PROBABLEMENTE BENIGNO


NÓDULO PROBABLEMENTE MALIGNO

NÓDULO MALIGNO

B. Lesiones estrelladas (o de alteración de la arquitectura): Representadas por áreas


de distorsión de la arquitectura mamaria, de bordes irregulares y que adoptan una
morfología radiada. Suele ser un signo temprano en el carcinoma de mama y es de difícil
interpretación. Posee menos valor predictivo positivo que el nódulo o las
microcalcificaciones, por lo que se recomienda biopsia quirúrgica en todos los casos
excepto en los que los antecedentes de traumatismo, cirugía previa o inflamación permitan
optar por el seguimiento de la lesión. En estos casos siempre es conveniente haber
realizado una citología con resultado negativo.
C. Microcalcificaciones: Son hallazgos frecuentes y el análisis de sus características
nos puede ayudar a diferenciar las benignas de las sospechosas y de las claramente
malignas.
Las benignas no requieren mas pruebas diagnósticas complementarias, las probablemente
benignas precisas de un seguimiento mamográfico no inferior a los dos años y en las
sugestivas de malignidad, la biopsia es preceptiva. Hay que analizar las siguientes
características:
• Tamaño: Las superiores a 2 mms. se clasifican de macrocalcificaciones y suelen ser
benignas. Por debajo de los 2 mms. se denominan microcalcificaciones y cuanto más
pequeñas y agrupadas más sospechosas son de malignidad
• Morfología: Las calcificaciones malignas suelen ser heterogéneas en forma y
tamaño, puntiagudas, anguladas, irregulares, en "coma", ramificadas y con forma de punto y
raya. Las benignas suelen ser homogéneas, redondas y en ocasiones anulares y de centro
claro.
• Número: Se considera que cuando hay cinco o más calcificaciones menores de 1 mm.
En un área de 1x1 cm. de mamografía, existe sospecha de malignidad. Cuanto mayor es el nº
de calcificaciones en esa área, más sospechosas son.
• Distribución: Las calcificaciones distribuidas de forma segmentaria, no al azar, son
sospechosas e indicativas de biopsia.
• Variación en el tiempo de las calcificaciones: Las calcificaciones malignas varían con
el tiempo. La estabilidad de las calcificaciones durante año y medio - dos años, se
consideran como benignas.
• Calcificaciones asociadas a mama: Los carcinomas de mama calcifican en un 50 %.
Cuando hay calcificaciones internas en lesiones con signos de malignidad, aumentan las
posibilidades de malignidad. Se hallan en un 75% de los cánceres ocultos y suponen el 30-
47% de hallazgo aislado en los cánceres de mama. Son el primer marcador de cáncer de
mama en las mujeres jóvenes.

2: Signos secundarios:

1. Engrosamiento de la piel.
2. Permeabilidad linfática.
3. Aumento de la vascularización.
4. Afectación linfática.
5. Dilatación ductal.
Bibliografía:

Apantaku LM. Breast cancer diagnosis and screening. [Internet]. American Family
Physician; 2000 [ acceso 28/6/2007]. Disponible en:

http://www.aafp.org/afp/20000801/596.html
Barratt A, Howard K, Irwig L, Salkeld G and Houssami N. Model of outcomes of
screening mammography: information to support informed choices. BMJ 2005;330;936-
940.[Texto completo]
Brewer NT, Salz T, Lillie SE. Systematic review: the long-term effects of false-
positive mammograms. Ann Intern Med. 2007;146(7):502-10 [PubMed] [Texto completo]
Breast Imaging reporting and data system (BI-RADS). 2nd ed. Reston (VA):
American College of Radiology; 2007 [Reseña]
Cantin J, Scart H, Levine M, Hugi M. Clinical practice guidelines for the care and
treatment of breast cancer: 13. Sentinel lymph node biopsy. Can. Med. Assoc. J. 2001; 165:
166 – 173 [Texto completo]
Dynamed. Breast cancer (female9) [Internet]. Ebsco industries ;2007 [acceso
3/7/22007]. Disponible en http://dynamed102.ebscohost.com/Detail.aspx?id=114433
Eberl MM, Fox ChH, Edge SB, Carter CA, Mahoney MC. BI-RADS Classification for
Management of abnormal Mammograms. J Am Board Fam Med 2006;19:161-4 [PubMed]
[Texto completo]
Fitzpatrick TB, Johnson RA, Wolff K, Suurmod D. Atlas color y sinopsis de
dermatología clínica. Madrid: McGraw Hill Interamericana; 2001. p.494-5

72.- Femenino de 32 años de edad es llevada a quirófano por una laparoscopía diagnóstica
debido a dolor pélvico y en cuadrante superior izquierdo crónicos (durante los últimos 2
años). No tiene alteraciones funcionales vesicales o intestinales. Tiene antecedente de 2
episodios de gonorrea previos. Bebe una cerveza al día. Labs: HCG urinaria negativa; Hto
39%; Leuc. T 8 000; Plt 200 000; AST 12; ALT 14. Intraoperatoriamente se observan
adhesiones densas que involucran los oviductos, ovarios y útero. También se observan
adhesiones perihepáticas que se extienden desde la superficie hepática hacia el diafragma.
¿Cual de los siguientes es el diagnóstico más probable?

a) Sx de Fitz-Hugh-Curtis
b) Hepatitis
c) Carcinoma hepatocelular
d) Sx Wolff-Parkinson-White
El síndrome de Fitz-Hugh-Curtis se define como la presencia de una perihepatitis asociada
a salpingitis. Los agentes etiológicos reconocidos hasta la fecha son Chlamydia trachomatis
y Neisseria gonorrhoeae. El cuadro clínico de este síndrome es inespecífico y puede ser
confundido con procesos inflamatorios o infecciosos del tubo digestivo, aparato urinario y
respiratorio, en los cuales la manifestación sintomática fundamental es el dolor en
hipocondrio derecho. El diagnóstico debe de sospecharse en aquella mujer joven con vida
sexual activa que tenga antecedentes de promiscuidad en ella o en su pareja, que se queje
de dolor subcostal derecho. Es más probable el diagnóstico si se cuenta con el antecedente
de enfermedad pélvica inflamatoria y más aún, si se tiene evidencia de que ésta sea
causada por Neisseria gonorrehoeae y/o Chlamydia trachomatis. El diagnóstico definitivo
se realiza con la visualización directa de la adherencia perihepática por laparoscopía o
laparotomía. Se recomienda la primera. El tratamiento médico es a base de cefalosporinas y
dicloxacilina y en algunos casos se requiere de la extirpación quirúrgica del proceso
adherencial para mitigar el dolor.

El Síndrome de Fitz-Hugh-Curtis. Causa frecuente de error de diagnóstico en hepatología y


gastroenterología / The Fitz-Hugh-Curtis Syndrome. a frequent misdiagnosis in hepatology
and gastroenterology Rev. gastroenterol. Méx;60(4):223-8, oct.-dic. 1995.

73.- Femenino de 20 años de edad refiere irregularidades menstruales tipo hipo-opso-


oligomenorrea desde hace 3 años. Niega tener vida sexual activa y no recuerda su fecha de
última menstruación. No hay antecedente de galactorrea ni de uso de hormonales exógenos.
Mide 164cm y pesa 60kg. Sin datos de hirsutismo, las mamas, útero y anexos son normales.
El ultrasonido pélvico es normal, así como el perfil hormonal. El diagnóstico clínico más
probable es:

a) Hiperplasia del endometrio


b) Sangrado uterino disfuncional
c) Alteración menstrual fisiológica
d) Endometriosis

El ciclo menstrual normal ha cambiado en el curso de los siglos, dependiendo de las


modificaciones en el patrón reproductivo y el estado nutricional de la población.
El debut del sangrado menstrual, definido como menarquia, marca una etapa importante en
la madurez biológica durante la pubertad.

En el Programa Nacional de Atención Integral a la Salud de los Adolescentes se mencionan


los trastornos menstruales como una de las primeras causas de consulta o de urgencia en
servicios clínicos de primero y segundo nivel.

Diversas alteraciones menstruales, ya sea las que inducen aumento en la cantidad o


frecuencia o las que se presentan como episodios infrecuentes e irregulares se observan en
la adolescencia y pueden impactar la calidad de vida de las pacientes, con repercusiones
significativas en el ámbito reproductivo y metabólico.

Por esas razones podemos afirmar que los trastornos menstruales constituyen un problema
de salud pública y requieren un enfoque más integral en aspectos preventivos y curativos en
la etapa de la adolescencia.

Aproximadamente la mitad de todas las adolescentes tienen períodos irregulares durante


el primer año después de la menarquia. Estos periodos irregulares pueden persistir hasta
cinco años después de la menarquia en 20 % de estas adolescentes. La principal
complicación es la anemia, que puede ser severa y raramente tiene consecuencias fatales.

PATRÓN MENSTRUAL NORMAL

La Federación Internacional de Ginecología y Obstetricia considera el patrón menstrual


normal entre tres y cuatro días, aunque fluctúa entre dos y siete días. El intervalo entre
menstruaciones es de veintiocho días, considerándose como límites de veintiuno a treinta y
cinco días.

El volumen de sangre menstrual es de sesenta a ochenta mililitros; no obstante, se


consideran normales los rangos de sangrado entre cincuenta y ciento cincuenta mililitros. El
aspecto de la sangre es rojo oscuro incoagulable.

En la práctica médica se utiliza un grupo de términos para hacer referencia a las diversas
alteraciones del ciclo menstrual, que requieren precisión por la frecuencia en que son
diagnosticados.

Según Schiavon (2000), las alteraciones menstruales más frecuentes son:


• oligoamenorrea: episodios de sangrado infrecuentes, irregulares, con intervalo de más de
cuarenta días;

• Polimenorrea: episodios frecuentes pero regulares de sangrado uterino, que ocurren a


intervalos menores de veintiún días;
• Menorragia: sangrado excesivo, tanto en cantidad como en duración, que ocurre con
regularidad y es sinónimo de hipermenorrea;

• Metrorragia: sangrado generalmente no excesivo, que ocurre a intervalos irregulares;


• Menometrorragia: sangramiento generalmente excesivo y prolongado, que ocurre a
intervalos frecuentes e irregulares;
• Hipomenorrea: sangrado uterino regular, pero disminuido en cantidad;
• Sangrado intermenstrual: sangrado uterino generalmente no excesivo, que ocurre entre
períodos menstruales regulares.

Las irregularidades menstruales son causa frecuente de consulta en las adolescentes,


siendo 95 % de las veces de naturaleza disfuncional, por inmadurez del eje hipotálamo–
hipofiso–ovárico (HHO).

Uno de los primeros problemas que hay que plantear ante estas irregularidades
menstruales, es la hemorragia uterina disfuncional (HUD). Su definición guarda relación con
las características en cantidad y frecuencia que difieren del sangrado menstrual normal.

Con mayor frecuencia se encuentra en forma de sangrados excesivos y prolongados,


asociados a ciclos anaovulatorios, en ausencia de una patología o enfermedad existente,
aunque raramente la HUD puede presentarse con ciclos ovulatorios.

De forma práctica, consideramos una hemorragia uterina (HU) como anormal cuando el
sangrado es excesivo, con cualquier desvío o alteración de su duración, cantidad o intervalo.

El diagnóstico de HUD supone una alteración de origen endocrino (eje HHO); por lo tanto,
su diagnóstico impone haber descartado cualquier patología orgánica y sistémica que
produzca hemorragia genital. Es un diagnóstico por exclusión.

ETIOPATOGENIA DE LA HUD EN LA ADOLESCENCIA

Los ciclos anovulatorios son más frecuentes en las adolescentes por la inmadurez del eje
HHO en el primer año tras la menarquia.

En esos casos de HUD, como ya se mencionó, se producen ciclos anovulatorios que se


traducen en una proliferación desorganizada del endometrio por falta de efecto
progestagénico. Una vez que el endometrio alcanza un grosor crítico, comienza a
descamarse en forma irregular, traduciéndose en un sangrado permanente de cuantía
variable.
Las manifestaciones clínicas de la HUD son:

• Fases de amenorrea de dos a cuatro meses, seguidas de salida de sangre abundante


durante tres o cuatro semanas; en oportunidades existe irregularidad completa en el
sangrado;
• sangrado de más de seis compresas (bien empapadas) al día;

• Presencia de coágulos;

• Suele ser indolora;

• Menstruaciones de más de siete días de duración;

• Ciclos de menos de veintiún días.

La gravedad de esta hemorragia se clasifica, de acuerdo con el grado de anemia que


produzca, en metrorragia leve, moderada o grave:
Leve:
• metrorragia leve y prolongada,
• ciclo menstrual acortado,
• hemoglobina y hematocrito normales.
Moderada:
• metrorragia copiosa prolongada,
• ciclo menstrual acortado,
• anemia leve (cifras de hemoglobina inferior a diez gramos por litro).
Grave:
• metrorragia copiosa prolongada,
• ciclo acortado e irregular,
• anemia grave (cifras de hemoglobina de ocho gramos por litro o menos).

Ante un sangramiento uterino en estas edades se debe realizar el diagnóstico diferencial


con:
a) Gestación y problemas relacionados con ésta, tales como abortos y gravidez ectópica;
b) Coagulopatía: 20 % de las adolescentes con hemorragia uterina tienen un defecto de la
coagulación. La manifestación más precoz de alteraciones de la coagulación sanguínea puede
ser evidenciada por un sangramiento genital anormal, lo cual puede estar relacionado con
deficiencias de plaquetas, leucemias, púrpuras, enfermedad de Von Willebrand, deficiencia
de protrombina u otros factores de la coagulación;
c) Malformaciones del aparato genital, traumatismosgenitales, presencia de cuerpos
extraños;
d) Dispositivos intrauterinos;
e) Tumores uterinos, sarcoma botroides o tumores anexiales;
f) Hipo o hipertiroidismo;
g) Insuficiencia renal o hepática.

Como el diagnóstico de HUD es de exclusión, hay que hacer una historia clínica minuciosa,
exámenes complementarios y sin falta descartar las otras causas de sangramiento
transvaginal.

Se debe precisar con detalle el nivel de desarrollo puberal, la actividad sexual y la


presencia de situaciones concomitantes como: a) contacto sexual sin protección
contraceptiva;
b) uso irregular de anticonceptivos orales o antecedentes de inserción de dispositivos
intrauterinos;
c) ejercicios físicos extenuantes;
d) historia previa de sangrado excesivo, asociado a extracciones dentarias, pequeñas
heridas, epistaxis y otras;
e) dolencias renales u hepáticas preexistentes.

Ante cualquier demanda de atención por adolescentes con sangramiento genital con las
características descritas, se requiere de un examen físico general que incluya exploración
general completa, toma de tensión arterial y pulso, búsqueda de exoftalmia, fascie Cushing,
visceromegalias o presencia de masas abdominales palpables, edemas parpebrales y de
miembros superiores, así como puntos hemorrágicos en epidermis y otros signos de
coagulopatía.

El examen ginecológico debe realizarse en todas las adolescentes, con excepción de las que
no han tenido actividad sexual y presentan sangramiento leve. Además del examen de sus
genitales, hay que efectuar una valoración citológica y microbiológica en particular en
quienes presenten manifestaciones clínicas.

En la inspección de los genitales durante el examen de la paciente, es importante evaluar


que el sangramiento se origine en lesiones ubicadas en los genitales externos, uretra u
hemorroides, así como indagar acerca de la posibilidad de abuso sexual. Si después del
tratamiento de la HUD leve continúa el sangrado, se recomienda la realización de ecografía
abdominal para precisar el diagnóstico.

En pacientes que ya han tenido relaciones sexuales se debe buscar si el útero tiene
características gravídicas, la posibilidad de un aborto en curso, así como la presencia de
una masa anexial que permita corroborar la existencia de embarazo o alguna neoplasia
benigna o maligna. El examen con espéculo podrá demostrar un cuello hiperémico, sangrante
o gravídico; de encontrarse estos hallazgos, se descartaría la etiología disfuncional del
sangrado.
Exámenes complementarios básicos que no pueden faltar:

• Hemograma completo,
• Coagulograma completo,
• Orina,
• Ultrasonido ginecológico abdominal, transvaginal o transrectal según proceda,
• Ecografía abdominal. Si fuese necesario por los signos identificados en el examen de la
paciente, se deben realizar:
• Dosificación de FSH, LH, T3, T4, TSH y prolactina si hay sospechas clínicas de otras
enfermedades endocrinas concomitantes,
• Laparoscopia en casos seleccionados por patología de base,
• Otros, según hallazgos de la historia clínica y la exploración.

74.- Femenino 40 años, G-3 P-2 A-1, se detecta de anemia ferropénica, de 9.5 g/dl,
refiere ciclos menstruales de 31,32 x 8,9 días de duración, acompañados de coágulos, los
cuales aparecieron después del nacimiento de su segundo hijo hace 12 años. e.f.: con ligera
palides de tegumentos, S/V dentro de los parámetros normales, , genitales con evidencia de
sangrado activo, al tacto vaginal bimanual se detecta útero de consistencia firme
voluminoso, irregular, aproximadamente de 12 cm. anexos libres. El tratamiento para esta
paciente es:

a) Histerectomía total sin conservar anexos.


b) Histerectomía total conservando anexos.
c) Histerectomía vaginal.
d) Histerectomía radical.

Tratamiento: La cirugía es el tratamiento más común del mioma uterino:


Miomectomia: Extirpar sólo el mioma, los tumores únicos y accesibles, la resección
histeroscópica de miomas submucosos también es posible realizarlo con electrocoagulación
en pacientes con hemorragias (>90%).
Histerectomía:
La cirugía puede ser abdominal o laparoscópica, la decisión final de la técnica dependerá de
la elección del cirujano, en base al caso individual y a su experiencia, el procedimiento
puede ser:
-Histerectomía subtotal.
-Histerectomía total (remoción cervical). Técnicamente más sencillo en su realización. La
incidencia de carcinoma con origen en el muñón cervical es menor al 1% en nuestros días.
Bibliografía:

1. Hanafi m. predictors of leiomyoma recurrence after myomectomy. am coll obstet ginecol


2005;
105: 877-880.
2. Inclan j, mojarra j. miomectomía histeroscópica. abordaje actual para el manejo de los
miomas submucosos. reporte de un caso y revisión de la literatura. bol clin hosp. infant edo
son 2001;18: 29-34.
3. goldrath mh, husain m. the hysteroscopic management of endometrial leiomyomatosis. j
am assoc gynecol laparosc 1997; 4: 263-267.
4. clement pb, scully re. mullerin adenofibroma of the uterus with invasion of myometrium
and pelvic veins. int j gynecol pathol 1990; 9: 363-371.
5. ravina jh, herbreteau d, ciraru-vigneron n, bouret jm, houdart e, aymard a, merland jj.
arterial embolization to treat uterine myomata. lancet 1995; 346: 671-672.
6. kunhardt-urquiza e, cruz si, fernández-martínez rl, hernández-zúñiga ve: miomatosis de
localización poco frecuente. ginecol obstet mex 1997; 65: 541-544.
75.- Femenino de 23 años, acude a su consultorio refiriendo secreción vaginal fétida.
Exploración Física: especuloscopía se observa secreción gris que rodea la cúpula vaginal. En
la preparación en fresco se observan cocobacilos que rodean a las células epiteliales.
¿La medida terapéutica más apropiada en esta paciente es?

a) Clindamicina
b) Clotrimazol
c) Metronidazol
d) Tratamiento para ella y su pareja.

Metronidazol: Tratamiento específico en vaginosis, la evidencia apoya en primer lugar el


uso de:
metronidazol por vía oral, en dosis de 500mg cada 12 horas durante siete días, con una tasa
de curación de 80 a 90 %. Para el tratamiento local no se ha demostrado una diferencia
significativa en la efectividad para curarla vaginosis, al comparar clindamicina en crema a 2
% y metronidazol vaginal, ni entre éstos y el tratamiento con metronidazol por vía bucal (Ia
y III).4,8 En vaginosis tampoco se ha demostrado la utilidad de tratar a la pareja sexual
(Ia).
Trejo y PJA, Hernández LB , Carrasco RJR , Ducoing DDLR
Guía clínica para el diagnóstico, tratamiento y prevención de cervicovaginitis por bacterias,
Trichomonas y Candida
Rev Med IMSS 2003; 41 (Supl 1):71-76

76.- A 23-year-old nulligravid female has not menstruated in the past 4 months. Previously,
her menstrual cycles were regular. She is otherwise well and denies recent onset of
stress, change in exercise routine, headaches, visual field alterations, or galactorrhea..
She has a body mass index of 24, blood pressure of 120/78 mm Hg, and does not appear
hirsute. No adnexal masses can be palpated. Laboratory investigations reveal a negative
beta human chorionic gonadotropin (β-hCG), normal thyroid-stimulating hormone (TSH),
and prolactin levels.
What is the next best step in the management of this patient?

a) Preescribe 7 days of medroxyprgesterona and reevaluate.

b) Measurement of luteinizing hormone.

c) CT scan of the sella turca.

d) Prescribe oral estrogen for 21 days followed by 7 days of medroxyprgesterona and


reevaluate.
La Prueba de respuesta a progestágenos se basa en la observación de que el tratamiento
con progestágeno (acetato de medroxiprogesterona 10 mg por 5 a 6 días) solo induce la
menstruación en las mujeres con concentraciones normales de estrógenos circulantes. Una
prueba positiva (hemorragia después de concluir el tratamiento con progestágenos) señala
cifras normales de producción de estrógenos y una prueba negativa (sin hemorragia por
privación), hipogonadismo franco.
Danforth, Tratado de Obstetricia y Ginecologia, 9ª Edición, Ed. Mc Graw Hill
Interamericana, Pág 668.

77.- El cáncer de tiroides que puede producir un síndrome paraneoplásico y que se asocia a
elevaciones de calcitonina es:

a) Cáncer anaplásico
b) Cáncer de células de Hürttle
c) Cáncer medular
d) Cáncer papilar

El cáncer medular de tiroides surge de las células parafoliculares de la tiroides, que


normalmente producen calcitonina. La medición de calcitonina es importante sobre todo en
el seguimiento de los pacientes para detectar enfermedad residual o recidivante.

Jiménez RSA, Gómez VE, Bolaños GF. Tiroides. En Flores JF, Cabeza A, Calarco Z
(eds): Endocrinología. 5ª ed. México. Méndez Oteo México, 2005: 584-92.

78.- ¿Cual de las medidas siguientes es más adecuada para combatir la hiperbilirrubinemia
(l0mg/dl) de un lactante de 3 semanas, con un desarrollo y crecimiento normales, que
recibe lactancia materna?

a) Fototerapia
b) Exanguineotransfusión
c) Fenobarbital
d) Esperar un par de días y repetir la prueba.
La fototerapia es el empleo de luz visible para el tratamiento de hyperbilirubinemia en

el recién nacido (RN). Esta terapia relativamente común baja el nivel de bilirrubina en

el suero por transformación de la bilirrubina en isómeros solubles en agua que pueden

ser eliminados sin la conjugación en el hígado

La fototerapia convierte la bilirrubina que está presente en los capilares superficiales y

espacio intersticial a isómeros solubles en agua que son excretables sin pasar por el

metabolismo del hígado (Fig. 4). Maisels, un notable experto en bilirrubina, sugiere que

la fototerapia se parece mucho a una droga percutanea. Cuando la fototerapia ilumina la

piel, una infusión de fotones de energía, como moléculas de una medicina, es absorbida

por la bilirrubina de la misma manera que una molécula de medicina se une a un receptor.

Las moléculas de bilirrubina en la piel expuestas a la luz sufren las reacciones

fotoquímicas relativamente rápido, configurational isomerization, isomerización

estructural, y la forma de fotooxidación no tóxica, isómeros excretables. Estos

isómeros de bilirrubina tienen formas diferentes del isómero natal, son más polares, y

pueden ser excretados del hígado en la bilis sin sufrir la conjugación o requerir

transporte especial para su excreción. La eliminación urinaria y gastrointestinal son

ambas importantes en reducir la carga de bilirrubina.


Figura 4. El mecanismo de fototerapia. Cuando las moléculas de bilirrubina absorben la

luz, 2 reacciones fotoquímicas principales ocurren: el natural 4Z, 15Z-bilirubin se

convierte a 4Z, 15E bilirubin (también conocido como photobilirrubina) y a lumirrubina.

A diferencia de 4Z, 15Z la bilirrubina, photobilirrubina puede ser excretado vía

hepática sin la conjugación, pero su clearance es muy lento, y su conversión es

reversible. En el intestino (lejos de la luz), photobilirrubina es convertida atrás a

bilirubina natal.

La lumirrubina no es reversible. Aunque mucho menos lumirrubina que photobilirrubina

es formado, lumirrubina es eliminado del suero mucho más rápidamente, y es probable

que la formación de lumirrubina es principalmente responsable de la disminución en el

suero de la bilirrubina. Las pequeñas cantidades de bilirrubina natal también son

oxidadas a monopyrroles y dipyrroles que pueden ser excretados en la orina. Esto es un

proceso lento y sólo un contribuidor menor a la eliminación de bilirrubina durante la

fototerapia. Cortesía de diagrama de María Puchalski.

El objetivo de la fototerapia es disminuir la bilirrubina sérica y prevenir su acumulación

tóxica en el cerebro, donde puede causar serias complicaciones neurológicas

permanente conocido como kernicterus. La fototerapia ha reducido enormemente la

necesidad de exanguíneo transfusión para tratar la hiperbilirrubinemia.


La fototerapia es usada de 2 modos principales: profiláctica y terapéuticamente.

• En RN prematuros o aquellos con un conocido proceso hemolítico, a menudo

es usado profilacticamente, para prevenir un rápido aumento de la

bilirrubina sérica.

• En pretérminos pequeños o RN de término, es administrada en dosis

terapéuticas para reducir niveles de bilirrubina excesivos y evitar el

desarrollo de kernicterus.

La fotoisomerización de bilirrubina comienza casi al instante cuando la piel es expuesta

a la luz. A diferencia de la bilirrubina no conjugada, los fotoproductos de estos procesos

no son neurotóxicos. Por lo tanto, ante una hiperbilirrubinemia severa del RN, es

importante comenzar la fototerapia sin retraso.

Referencias:

Stokowski LA. Early recognition of jaundice and kernicterus. Adv Neonatal Care

2002;2:101-114.

Maisels MJ. A primer on phototherapy for the jaundiced newborn. Contemp Pediatr.

2005; 22(6): passim. (OR, 38, 40, 44, 47, 48, 53, 54,57).

1. McDonagh AF. Phototherapy: from ancient Egypt to the new millenium. J

Perinatol 2001;21:S7-S12.
Maisels MJ. Phototherapy-traditional and nontraditional. J Perinatol 2001; 21(Suppl
1):S93-S97.

79.- Cuando nos enfrentamos ante un paciente que presenta un cuadro clínico caracterizado
por amenorrea, galactorrea y pérdida de campo visual el diagnóstico más probable es:

a) Adenoma hipofisario no funcionante.


b) Intoxicación por benzodiacepinas.
c) Prolactinoma.
d) Meningioma del tubérculo solar.
El prolactinoma es el tumor hipofisario más frecuente, es de naturaleza benigna y pequeño
en 90 % de los casos. El cuadro clínico típico en la mujer se compone de trastornos
menstruales, galactorrea y/o esterilidad; se acompaña de síntomas neurológicos sólo
cuando se extiende por arriba de la silla turca. Niveles de prolactina superiores a 100
ng/mL son prácticamente diagnósticos de prolactinoma, siempre y cuando no existan
embarazo y/o hipotiroidismo. La primera opción terapéutica del prolactinoma es la
farmacológica con dopaminérgicos, lo que prácticamente ha eliminado la cirugía. Los
dopaminérgicos suprimen la síntesis y secreción de prolactina con la consecuente
normalización del eje hipotálamo-gonadotrópico. Los dopaminérgicos son efectivos para
inducir la ovulación y favorecen la consecución de embarazo. Para cualquier dimensión del
prolactinoma se usan los dopaminérgicos durante uno a dos años y generalmente después de
suspenderlos se puede esperar que el tumor se reduzca de tamaño y se corrija la
hiperprolactinemia. En contraste, en el hombre generalmente se encuentran
macroprolactinomas, mayores de 10 milímetros con extensión extraselar acompañándose de
síntomas neurológicos; sin embargo, también responden favorablemente a los
dopaminérgicos.

Gac Méd Méx Vol. 140 No. 5, 2004

Referencias:

1. Schlechte JA. Prolactinoma. N Engl J Med 2003;349:2035-2041.


2. Zárate A, Canales ES, Jacobs LS, Soria J, Daughaday WH. Restoration of ovarian
function in patients with the amenorrhea-galactorrhea syndrome after long-term therapy
with L-Dopa. Fertil Steril 1973;24:340.
3. Tyson JE, Carter JN, Andreassen B, Huth J, Smith B. Nursing mediated prolactin
and luteinizing hormone secretion during puerperal lactation. Fertil Steril 1978;30:154.
4. Schlechte JA, Sherman BM, Chapler FK, VanGilder J. Long-term followup of women
with surgically treated prolactin-secreting pituitary tumors. J Clin Endocrinol Metab
1986;62:1296-301.
5. Losa M, Mortini P, Barzaghi R, Gioia L, Giovanelli M. Surgical treatment of
prolactin-secreting pituitary adenomas: early results and long-term outcome. J Clin
Endocrinol Metab 2002;87:3180-3186.
6. Zárate A, Canales ES, Cano C, Pilonieta CJ. Follow-up of patients with prolactinomas
after discontinuation of long-term therapy with bromocriptine. Acta Endocrinol
1983;104:139-42.
7. Zárate A, Canales ES, Alger M. The effect of pregnancy and lactation on pituitary
prolactin secreting tumors. Acta Endocrinol 1979;92:407-11.
8. Bevan JS, Webster J, Hburke J, Scanlon MF. Dopamine agonists and pituitary tumor
shrinkage. Endocr Rev 1992;13:220-240.
80.- Masculino de 3 años, preescolar. Es atendido en consulta. desde los 2 años de edad, al
llorar presenta cianosis labial y peribucal, cuando se golpea o al regañarlo, no pierde el
conocimiento, la cianosis desaparece al ceder el llanto, le ocurre casi a diario, no hay
antecedente familiar de enfermedad neurológica o cardiovascular. Examen físico peso 13.6
kg., talla 93 cm., resto sin datos patológicos. En este paciente el diagnóstico más probable
es:

a) Epilepsia.
b) Espasmo del sollozo.
c) Tetralogia de fallot.
d) Enfermedad por reflujo gastroesofágico.

Espasmo del sollozo


Es la retención de la respiración posterior a un evento que disguste tal como una caída, el
estar frustrado o enojado, o por estar asustado. -El niño da uno o dos gritos largos y
posteriormente retiene su respiración en expiración hasta que sus labios se ponen azules.
-El niño posteriormente se desmaya (algunos llegan a tener algunos espasmos musculares). -
después el niño respira normalmente y permanece completamente alerta en menos de 1
minuto. -El inicio ocurre entre 6 meses y 2 años. Esto sucede solamente cuando el niño está
despierto.

Se define al espasmo del sollozo (Breath-holding spells), como un evento caracterizado por
crisis recurrentes de apnea transitoria, pérdida del conocimiento y cambios en el tono
muscular normal. De inicio súbito, no seconsidera secundario a patología orgánica ni
resultado de una manifestación psiquiátrica.

Entre el 5 y el 7% de la población infantil sana presenta crisis de espasmos de sollozos, sin


embargo, algunos autores coinciden en que esta entidad es mucho más frecuente. Por lo
general dichos eventos inician entre los 6 a 12 meses de edad con un pico de incidencia
entre los 12 y los 24 meses de edad.

Es poco frecuente que se presente en niños mayores de 6 años de edad por lo que su
aparición en niños mayores de 4 años amerita especial atención. El 25% de los niños que los
presentan tiene un familiar directo que lo padeció en la infancia. Se presenta más
frecuentemente en varones.

Son causas frecuentes de síncopes y convulsiones anóxicas secundarias a isquemia o


hipoxia, con la consecuente depresión súbita de la función neuronal, en contraste con las
convulsiones epilépticas, secundaria a una descarga excesiva de las neuronas.

El espasmo del sollozo a sido dividido en el espasmo del sollozo pálido y cianótico,
dependiendo de la coloración de la piel durante el evento. La fisiopatología, en cada caso en
particular, es diferente. Entender las características del espasmo del sollozo y poder
diferenciarlo de otras entidades, puede ayudar al pediatra a tranquilizar a los padres.
Bibliografía:

1- Berman, RE., Kliegman, RM., Jenson, HB: Nelson Textbook of Pediatrics. 16th edition,
Philadelphia, Pennsylvania, W.B. Saunders Company U.S.A.., 2,000; 1829
2- Anderson JE, Bluestone D: espasmos del sollozo. Contemp Pediatr 2,000;17(1):61-72
3- Macan H, et al: Espasmo del sollozo en 91 niños y respuesta al tratamiento con hierro.
Arch Dis Child 1999;81:261-262.
4- Breningstall GN: Breath-holding Spells . Pediatr Neurol 1996;14:91-97
5- DiMario FJ: Breath-holding spell in childhood. Am J Dis Child 1992;146:125-131
6- Lombroso CT, Lei-man P: Breath-holding spell (cyanotic and pallid in fantile syncope).
Pediatrics 1967;38:563-581.
7- Gauk EW,Kidd L, Prichard JS: Mechanism of seizures associated with breath-holding
spell. N Engl J Med 1963; 268: 1436-1441.
Universidad La Salle.
Facultad Mexicana de Medicina.
Curso de Extensión Universitaria para la Preparación del Examen Nacional para
Aspirantes a Residencias Médicas.

Examen de Pediatría

1.- Se trata de masculino de 7 años de edad, cursa con odinofagia y fiebre no cuantificada
desde hace más de tres días. E.F. orofaringe con exudado membranoso y petequias en
paladar blando y pilares anteriores. Lengua de fresa roja. To. 39.5 Oc. Usted elige el siguiente
fármaco por ser el medicamento de elección en esta patología.

a) Cefuroxime.
b) Amoxicilina.
c) Claritromicina
d) Penicilina benzatínica.

ESCARLATINA
Tratamiento: 1) Reducir las complicaciones, 2) erradicar al
S. pyogenes.
Penicilina benzatínica: 20,000-50,000 U/kg/do (única).
< 20 kg: 600,000 U IM.
> 20 kg: 1,200,000 U IM.
Penicilina V (fenoximetilpenicilina): 25-50 mg/kg/día cada 6
hrs. ó 250-500 mg cada 6 hrs. por 10 días. Separado de los
alimentos mejora absorción.

Eritromicina: 30-50 mg/kg/día cada 6 hrs. ó 250-500 mg


cada 6 hrs. por 10 días. Alergia a la penicilina.

Streptococcus pyogenes (hemolítico del grupo A) es el agente etiológico de la escarlatina, no


ofrece resistencia a la penicilina benzatínica y con dosis única ofrece un buen índice de
curación, erradicación y reducción de complicaciones.

González-Saldaña N, Infectología Clínica Pediátrica, 7ª edición, páginas 403-407

2.- Se trata de masculino de 2 años y medio inicia con fiebre elevada de 5 días de evolución,
con afectación del estado general y ligera irritabilidad. A la exploración destaca la presencia de
un exantema maculopapuloso discreto en tronco e hiperemia conjuntival bilateral sin secreción.
Presenta además enrojecimiento bucal con lengua aframbuesada e hiperemia faríngea sin
exudados amigdalares, además de adenopatías laterocervicales rodaderas de unos 1,5 cm de
tamaño. Ha recibido 3 dosis de Azitromicina. El diagnóstico más probable del paciente es:

a) Mononucleosis infecciosa.
b) Rubeola
c) Escarlatina.
d) Síndrome de Kawasaki.

Definición: Vasculitis generalizada


Descrita por el Dr. Tomisaku Kawasaki en 1967. Ese año él publica 50 casos, presentados entre
1961 y 1967, con el nombre de Síndrome Linfomucocutáneo. Al principio se pensó que era una
enfermedad benigna, pero ya en 1970 se habían publicado 10 casos de muerte por esta
enfermedad.
Recientemente se examinó en Londres el corazón de un niño de seis años que había muerto en
1870, con diagnóstico de “escarlatina”, encontrándose tres aneurismas coronarios trombosados.
Antes de la publicación de Kawasaki, los casos de esta enfermedad identificados post mortem
fueron denominados periarteritis nodosa infantil.

Etiología
Continúa siendo desconocida, pero se acepta que tiene etiología infecciosa, por el cuadro clínico
y la epidemiología. Existiría, además, una predisposición genética e inmunológica con formación
de un súper antígeno, el que desencadenaría la enfermedad. Las células T serían las receptoras
del súper antígeno.

Criterios diagnósticos
No existen exámenes patognomónicos, ya que la etiología continúa siendo desconocida, por lo
que el diagnóstico se basa en hecho clínicos.
Fiebre de más de cinco días, sin causa conocida, con al menos cuatro de los signos siguientes:
— Inyección conjuntival bilateral, no exudativa.
— Cambios en la mucosa bucal, como: labios rojos, secos y fisurados; eritema de la faringe y
lengua de frutilla.
— Cambios en las manos y pies: enrojecimiento y edema en la etapa aguda y en la etapa
subaguda, descamación periungueal
— Linfoadenopatías cervicales con más de 1,5 cm de diámetro cada una, indoloros y móviles.
Si existe compromiso coronario, sólo se exige fiebre de más de cinco días de evolución y tres de
los otros criterios clínicos.
Un hecho que llama la atención es la gran irritabilidad que presentan estos enfermos.
Como hechos asociados están: compromiso cardiaco, meningitis aséptica, artralgia o artri tis,
disfunción hepática leve, hídrops vesicular, diarrea, neumonitis radiológica, otitis media e
inflamación de la vacuna BCG.
Desgraciadamente, no todos los pacientes llenan los criterios para el diagnóstico. Existe el
síndrome de Kawasaki atípico o incompleto, con menos signos que los expuestos y con
compromiso coronario. Por esto el síndrome de Kawasaki debe estar presente en el diagnóstico
diferencial de todo cuadro febril prolongado sin causa etiológica demostrable, en niños.

BIBLIOGRAFÍA Apuy. JJ. Enfermedad de Kawasaki. 1. 2003; 564:109-113. Brogan P, et al.


Kawasaki disease: 2. an evidence based approach to diagnosis, treatment, and proposal for
future research. Arch Dis Child 2002;86:286-292.Burns JC, et al. Kawasaki Disease: a 3. Brief
history. Pediatrics 2000; 106(2): 27.Burns JC, Glodé M. kawasaki syndrome. 4. The Lancet 2004;
364: 533-44.
3. – Se trata de masculino de un año de edad que acude al servicio con antecedentes de
aumento de temperatura hasta de 39.5°C de dos días de evolución actualmente controlada con
paracetamol, el día de hoy presenta datos de exantema. Se realiza diagnóstico de roséola ya
que el exantema en ésta patología se caracteriza por su siguiente presentación:

a) Máculopapular violáceo, que no desaparece a la dígito presión.


b) Máculopapular eritemotoso, con descamación.
c) Máculopapular eritemotoso, de inicio súbito al desaparecer la fiebre.
d) Máculas, vesículas y costras, pruriginoso.

Exantema súbito

Manifestaciones clínicas: conocida también como “roséola infantum” o “fiebre de los tres días”,
comienza con fiebre alta – en ocasiones hasta de 40,5°C – siendo característico en estos niños
que conservan un relativo buen estado general, pese a la fiebre. No es infrecuente que consulten
en el servicio de urgencia por convulsiones febriles. Actualmente se sabe que el virus tiene
tropismo hacia el sistema nervioso central y las convulsiones pueden representar un efecto
directo del virus sobre el tejido cerebral. Después de 3 a 4 días de fiebre, aparece el exantema
que es macular o papular y se palpa granuloso; es de color rosado pálido y no es pruriginoso. Se
observa primero en el tronco, cuello y en la región retroauricular; su generalización es
infrecuente. El exantema dura alrededor de 2 a 3 días.

Agente etiológico: virus herpes tipo 6 (HHV6), virus ADN, miembro de la familia Herpesviridae.

Epidemiología: afecta a los lactantes entre los 6 a 18 meses de vida, y su mecanismo de


transmisión se presume por la vía respiratoria y contacto directo con algún huésped que esté
excretando el virus. El período de incubación es de 9 a 10 días.

Diagnóstico: se hace por el cuadro clínico y la exclusión de otras enfermedades febriles agudas.
Existen estudios serológicos específicos aún en evaluación para diagnóstico de infección aguda.

Aislamiento del paciente hospitalizado: precauciones estándar.

Tratamiento: manejo sintomático de la fiebre y eventualmente de episodios convulsivos.

Bibliografía:

 González-Saldaña N, Macías-Parra M. Vacunas en pediatría. 1er ed. México. McGraw-Hill


Interamericana; 1998.
 Comittee on Infectious Diseases 1997-2000. Red Book 2000, 25 ed. American Academy of
pediatrics; 2000.

4.- Se trata de masculino de 13 años de edad, ingresa a la sala de urgencias por presentar
dificultad respiratoria y tos hace 14 horas. E.F.: Fr: 45 rpm Fc: 120 lpm, tórax con tiraje
intercostal, sibilancias que se aprecian a distancia, a la auscultación se escucha disminución del
murmullo vesicular con sibilancias espiratorias bilaterales, a la percusión aumento en la
resonancia pulmonar.
El estudio más sensible y accesible para establecer la gravedad del cuadro clínico es:

a) Espirometria.
b) Flujo espiratorio máximo.
c) Gasometria venosa.
d) Teleradiografia de torax.
La necesidad de contar con un instrumento de medición más simple que el espirómetro, condujo
al desarrollo del flujímetro, instrumento simple y económico que puede ser empleado en los
consultorios, en los servicios de urgencia e incluso por los pacientes en sus domicilios (Figura
4.1).

Figura 4.1
El flujímetro permite medir el flujo espiratorio máximo, más conocido como PEF (peak expiratory
flow). Este índice funcional es similar al VEF1, con el cual se correlaciona muy bien.
Aun cuando el PEF necesita bastante colaboración del sujeto, su reproducibilidad, una vez
aprendida la maniobra, es habitualmente muy buena. Por su mediana sensibilidad, no permite
detectar obstrucciones leves.
El PEF se emplea preferentemente en el control seriado de pacientes con asma, que hace
posible evaluar objetivamente las variaciones de la obstrucción de las vías aéreas producidas
por la enfermedad o por su tratamiento.
Técnica de medición del PEF
Esta prueba debe ser efectuada por el médico general, por lo que éste debe conocer la técnica
correcta de realizarla.
El paciente debe tomar el flujímetro, cuidando de no interferir con el movimiento del cursor y sin
apretarlo. Debe tomar una inspiración máxima, hasta CPT, e inmediatamente después efectuar
una espiración forzada, máxima. A diferencia de la espirometría, la espiración puede ser muy
corta, lo que causa menos disconfort al paciente.
El enfermo debe efectuar tres maniobras satisfactorias y se debe registrar el mejor PEF.
Posteriormente se puede administrar un aerosol de salbutamol, generalmente 200 microgramos,
preferentemente mediante una cámara de inhalación.
Después de 5 minutos se efectúa una nueva medición de PEF (con tres maniobras) y se evalúa
la respuesta al broncodilatador.

5.- R/N masculino de término, en la sala de parto, a los cinco minutos de vida, tiene fc 140 lpm,
acrocianosis, tono muscular con cierta flexión de extremidades, tose y estornuda, con respiración
irregular. Antecedentes: producto de gesta 1, sin control prenatal, parto distósico, ameritó
utilización de fórceps, requirió 2 ciclos de presión positiva con bolsa válvula mascarilla en el
primer minuto de vida.
La calificación según la puntuación de apgar a los 5 minutos es:

a) 3.
b) 5.
c) 7.
d) 9.

Este test lleva el nombre por virginia apgar, anestesióloga, especializada en obstetricia, quien
ideó el examen en 1952 en el columbia university´s babies hospital
Los cinco criterios del Apgar son:

0 puntos 1 punto 2 puntos Acrónimo

Color de la piel todo azul extremidades azules normal Apariencia

Frecuencia
no posee <100 >100 Pulso
cardíaca

Reflejos sin respuesta amueca / llanto débil al se estornudos / tos / pataleo a


Gesto
e irritabilidad estimulación estimulado ser estimulado

Tono muscular ninguna alguna flexión movimiento activo Actividad

Respiración ausente débil o irregular fuerte Respiración

Literatura

 Apgar, Virginia (1953). «A proposal for a new method of evaluation of the newborn
infant» Curr. Res. Anesth. Analg.. Vol. 32. n.º 4. pp. 260–267. PMID 13083014.

 Finster M; Wood M. (abril de 2005). «The Apgar score has survived the test of
time» Anesthesiology. Vol. 102. n.º 4. pp. 855-857. PMID 15791116.

 Casey BM; McIntire DD, Leveno KJ (15 de febrero, 2001). «The continuing value of the
Apgar score for the assessment of newborn infants» N Engl J Med.. Vol. 344. n.º
7. pp. 467-471. PMID 11172187.

6.- Femenino de 6 años, escolar. Es atendida en consulta externa por dolor faríngeo, erupción
cutánea y fiebre de dos días de evolución. e.f.: adenomegalias cervicales de 1.5 mm., faringe
hiperémica, hepatomegalia de 3 cm. por debajo del borde costal derecho, esplenomegalia de 2
cm., exantema maculopapular de predominio en tórax y abdomen. Recibió tratamiento con
amoxicilina.

El agente etiológico más probable es éste caso es:

a) Streptococcus pneumoniae.
b) Virus epstein barr.
c) Streptococcus pyogenes.
d) Adenovirus.

Mononucleosis infecciosa
Introducción
El término de mononucleosis infecciosa fue introducido en 1920 cuando se describió un
sindrome caracterizado por fiebre, linfadenomegalias, fatiga y linfocitosis en 6 pacientes. Pero
recién en 1968 Henle demostró que el VEB era el agente etiológico de los SMN asociados con la
presencia de anticuerpos heterófilos.
El VEB es la causa más común de SMN. Este virus también se le asocia a ciertas neoplasias
como linfoma de Burkitt africano, carcinoma de nasofaringe indiferenciado y enfermedades
linfoproliferativas. Datos epidemiológicos y serológicos sugieren la asociación entre VEB y
enfermedad de Hodgkin, aunque la exacta contribución del virus al desarrollo de este tumor no
es conocida. Está en investigación el rol que pueda jugar el VEB como cofactor en la
patogénesis del cáncer de cuello uterino.

Etiología
El VEB pertenece a la familia Herpesviridae (DNA). Comparte con los otros virus de esta familia
las características de ser un virus persistente y de ocasionar infección latente. El VEB tiene
tropismo por los linfocitos B y las células del epitelio oral. Se replica en el sistema linforreticular y
provoca una intensa respuesta inmune. La infección latente ocurre en los linfocitos B. Otro foco
potencial de infección por VEB es el tracto genital. Puede reactivarse en períodos de
inmunodepresión.
Clínica
El período de incubación es de 30 a 45 días. Le sigue el período prodrómico que dura de 7 a 14
días y está caracterizado por astenia, mialgia y cefalea. Posteriormente se asiste al período de
estado, que suele ser de comienzo insidioso, pero en algunos casos se inicia en forma brusca
con fiebre alta, escalosfríos, sudores, malestar general, cefalea, mialgias, edema periorbitario,
anorexia, malestar abdominal y odinofagia, siendo este último el motivo de consulta más
frecuente. La faringitis reviste las características descritas para el SMN. Las amígdalas están
aumentadas de tamaño y pueden observarse petequias en la unión del paladar duro con el
blando. Si bien la erupción cutánea propiamente viral se describe solo en 5% de los casos, es
muy frecuente la erupción máculopapular pruriginosa (90%) en aquellos que recibieron
aminopenicilinas.
Se han descrito úlceras genitales en personas con MNI, especulándose que el VEB pudo haber
sido transmitido por contacto orogenital. La infección por VEB podría ser incluido en el
diagnóstico diferencial de úlceras genitales cuando otras causas más comunes fueron excluidas.

Las manifestaciones clínicas y la frecuencia con que se observan, según estadísticas


extranjeras, son:

Síntomas Signos
Dolor de garganta 82%Adenomegalia 94%
Malestar febril 57%
Fiebre 76%
Faringitis 84%
Cefalea 51%Fiebre 76%

Anorexia 21%
Esplenomegalia 52%

Mialgias 20%
Hepatomegalia 12%

Escalofríos 16%Enantema palatino 11%


Ictericia 9%
Náuseas 12%
Exantema 10%
Dolor abdominal 9%

Tos 5%

Vómitos 5%

Artralgias 2%
Síndrome Mononucleosido,
Mononucleosis Infecciosa y Diagnósticos diferenciales

Bibliografía
Alfieri C., Rousseau E., Tanner J. Chronic-active Epstein-Barr virus infection. J IHMF.
1998;5(1):12-14.
Andersson J. Infectious mononucleosis: clinical Characteristics, complicationa and
management.J IHMF. 1998;5(1):15-19.

Fischer T.M et al. "Temas de Enfermedades Infecciosas".


Cap. XII " El síndrome Mononucleósico": Edic. Libreria Med. Mont. Urug. 1978: Alvarez J, López
T y Muñoz María J, P 209-241.

Ho M. Cytomegalovirus en: Mandell Gl, Bennet JE, Dolin eds. 1995.

Jenson H.B. Infection during pregnancy and congenital infection with Epstein-Barr virus. J IHMF.
1998;5(1):20-23.

Losa García J.E., Miró Meda J.M., García Alcaide F. et al. Sindrome mononucleósico. Medicine.
1998; 7(82): 3813-7.

Löwhagen G.B., Ricksten A. Genital Epstein-Barr virus infection. J IHMF. 1998;5(1):4-7.

Mandell, Douglas and Bennet´s: Principles and Practice of Infections Deseases (4 th ed),
Nueva York; Churchill Livington, 1995; 1364-1377.

Marra C.M. Neurological disorders associated with Epstein-Barr virus. J IHMF. 1998;5(1):8-11.

Tomkinson B.E., Sullivan J.L. Epstein-Barr virus infection and infectious mononucleosis.
Gorbach, Bartlett, Blacklow. Infectious diseases. Saunders company. 1992:1348-56.

Schooley RT, Epstein Barr (Infections mononucleosis) En: Mandell Gl, Bennet JE, Dolin R y eds.

Wolff M. Mononucleose infecciosa. Tratado de infectología. Veronesi R. Atheneu. 1997: 424-8.

7.- Masculino en edad preescolar. Acude al servicio de consulta externa. Antecedentes: historia
de geofagia. Hace 2 semanas presenta tos húmeda y "silbido del pecho". E.f.: temp. :37.2 ºc.,
mala higiene personal, tórax con sibilancias espiratorias bilaterales. Se reportan labs. BH con
anemia y eosinofilia.

En la radiografía de tórax de éste paciente espera encontrar datos de:

a) Neumonía lobar.
b) Infiltrado intersticial unilateral.
c) Infiltrado miliar.
d) Infiltrados migratorios.

Figura 1: Placa PA de tórax del paciente., infiltrado intersticial


Difuso abigarrado bilateral compatible con neumonitis.
Los hallazgos radiológicos en los síndromes PIE generalmente son inespecíficos, los infiltrados
en la radiografía (Rx) tórax pueden ser intersticiales, alveolares o mixtos, habitualmente son
bilaterales y difusos. Radiológicamente la eosinofilia inducida por parásitos se manifiesta
por infiltrados fugaces y migratorios. En la NEC se observa la clásica imagen de “fotografía
negativa de edema pulmonar”, caracterizada por compromiso de las zonas apicales y periféricas,
con indemnidad de las zonas centrales e inferiores. La Rx tórax en la ABPA muestra infiltrados
fugaces, mediante la tomografía computada (TC) de tórax se pueden evidenciar bronquiectasias
centrales. En la NEA puede encontrarse un derrame pleural de escasa cuantía, que contiene un
alto porcentaje de eosinófilos. La imagenología además es útil en determinar la extensión del
compromiso pulmonar, elegir los sitios óptimos para obtener la biopsia pulmonar y la respuesta
al tratamiento.

8.- Se trata de masculino de 8 años. Es atendido en consulta por aumento de peso del escolar.
Antecedentes H.F. : Madre Con obesidad grado II portadora de DM tipo II. El niño no desayuna
al acudir a la escuela y al regresar permanece solo en casa consumiendo alimentos grasos y
poca fibra. Tiene coloración oscura en cuello. Pesa 45 kg., talla: 1.30 m., imc: 27 kg/m2 y se
encuentra en el percentil 78 para su edad y sexo.
En este caso, el factor pronóstico más importante para desarrollar complicaciones metabólicas
en la vida adulta es:

a) La obesidad materna.
b) La hiperpigmentacion del cuello.
c) Los habitos alimentarios.
d) El sedentarismo.

ACANTOSIS NIGRICANS EN LA OBESIDAD DE LA NIÑEZ


Escrito por Dres. Tulay Guran, Serap Turan, Teoman Akcay y Abdullah Bereket
23.02.2009
La acantosis nigricans (AN) es un engrosamiento cutáneo con papilomatosis e
hiperpigmentación que puede aparecer en cualquier parte del cuerpo: axila, cuello, ingle
superficie antecubital y poplítea, área umbilical y superficies mucosas

Está frecuentemente asociada a obesidad, endocrinopatías, malignidad, síndromes genéticos y


al uso de algunas drogas. La diabetes mellitus (DM), dislipemia, síndrome del ovario poliquístico
e hipertensión arterial también pueden asociarse con AN, lo que significa insulinoresistencia e
hiperinsulinismo independiente de la obesidad.

Se reportó que la AN estuvo relacionada con hiperinsulinemia y obesidad en niños. No


obstante sólo pocos estudios han considerado el rol del peso corporal en el desarrollo de AN
como un marcador independiente de insulinoresistencia en niños y adolescentes.
9.- Se trata de masculino de 11 años de edad. Doce días después de una infección de vías
aéreas altas desarrolla debilidad de miembros inferiores la cual es progresiva en días hasta
afectar el tronco, a la exploración física se encuentra arreflexia atrofia muscular y dolor en
miembros inferiores. El LCR solo muestra proteinorráquea, el diagnóstico más probable es:

a) Parálisis de Bell.
b) Distrofia muscular
c) Síndrome de Guillian Barré.
d) Enfermedad de Charcot Marie Tooth.

Se conoce como síndrome de Guillain- Barré a una serie heterogénea de neuropatías periféricas
de alivio espontáneo mediadas inmunológicamente.
El hallazgo común en ellas es la polirradiculoneuropatía de evolución rápida que se
desencadena casi siempre después de un proceso de tipo infeccioso. Se manifiesta más
frecuentemente con parálisis motora simétrica, con o sin pérdida de la sensibilidad, y en
ocasiones con alteraciones de tipo autonómico.
La debilidad de los músculos se agrava al máximo en las dos o tres semanas posteriores al inicio
del cuadro y la recuperación parcial o total ocurre en semanas o meses.
Por lo general evoluciona hacia la curación sin dejar secuelas clínicas evidentes, aunque pueden
surgir complicaciones riesgosas para la vida del enfermo. Hasta hoy constituye un mal sin
remedio, pues las terapias aplicadas se limitan a disminuir la gravedad del trastorno y a acelerar
la recuperación de la mayor parte de los pacientes.

Etiopatogenia
La gran mayoría de los pacientes se recoge el antecedente de una infección respiratoria o
gastrointestinal, bacteriana o viral, varias semanas antes del comienzo de los síntomas
neurológicos. Dentro de los antecedentes infecciosos en este síndrome se encontró una mayor
frecuencia de infección por Campylobacter jejuni, Cytomegalovirus y virus de Epstein
Barr, también detectó infecciones por Mycoplasma pneumoniae, virus de la hepatitis, herpes
simple, mononucleosis infecciosa y SIDA (HIV). También se ha asociado con vacunación
(influenza, antirrábica, etc.), enfermedades sistémicas (enfermedad de Hodgkin, lupus
eritematoso sistémico, sarcoidosis) y cirugía.

Diagnóstico
Se basa en los criterios clínicos considerando los antecedentes, hallazgos en el líquido
cefalorraquídeo (LCR), serología para anticuerpos específicos y criterios electromiográficos.

Criterios diagnósticos para Síndrome de Guillain-Barré son:


1. Hallazgos necesarios para hacer el diagnóstico:
• Debilidad progresiva en varias extremidades
• Arreflexia
2. Hallazgos que apoyan fuertemente el diagnóstico
a) Datos clínicos en orden de importancia
• Progresión desde unos días a 4 semanas
• Relativa simetría
• Alteraciones sensoriales leves
• Compromiso de pares craneales incluyendo el facial
• Recuperación que comienza 2 a 4 semanas después de detenerse la progresión
• Disfunción autonómica
• Ausencia de fiebre una vez instalado el síndrome
b) Estudio del líquido cefalorraquídeo
• Proteínas elevadas después de una semana
• Menos de 10 linfocitos /mm3
c) Pruebas electrofisiológicas
• Conducción nerviosa lenta
• Latencias distales prolongadas
• Respuestas tardías anormales
3. Hallazgos que hacen el diagnóstico dudoso
• Existencia de un nivel sensorial
• Marcada asimetría de síntomas y signos
• Disfunción severa y persistente de vejiga e intestino
• Más de 50 células/mm3 en LCR
4. Hallazgos que excluyen el diagnóstico
• Diagnóstico de botulismo, miastenia, poliomielitis o neuropatía tóxica
• Metabolismo alterado de las porfirinas
• Difteria reciente
• Síndrome sensorial puro sin fatiga

BIBLIOGRAFIA
1. Valls JS, Casademont JP, Berciano BD. Enfermedades de los nervios periféricos. En: Farreras
VP, Rozman
CR y col. Medicina Interna. 14 ed. Madrid: Ed. Harcourt SA; 2000: Vol 2: 1753-70.
2. Newswanger DL. Guillain-Barré Syndrome. Am FammPhysician [online] mayo 2004 [fecha de
acceso 15 de febrero 2007]; 69(10) URL
3. Melano Carranza E, Carrillo Maravilla E, Gulias Herrero A y col. Síndrome de Guillain-Barré en
el anciano: un estudio retrospectivo. Arch. Neurocien. (Mex DF)

10.- Masculino de 6 años presenta edema generalizado y proteinuria de 8 gramos al día, sin
datos de hematuria, hipertensión ni disminución de la función renal. La actitud más adecuada en
este caso es:

a) Confirmar la existencia de proteinuria


b) Realizar biopsia renal
c) Estudiar a fondo su estado inmunológico
d) Administrar esteroides

Este es un caso de síndrome nefrótico (proteinuria > 3.5g al día). La causa más frecuente en los
niños es la Glomerulonefritis de cambios mínimos (70% de los casos); si se tratara de un adulto,
deberíamos pensar que de tratarse de una glomerulonefritis membranosa (la de cambios
mínimos ocasiona sólo un 20% d los síndromes nefróticos de los adultos).
Es más frecuente en los varones, existiendo con frecuencia antecedentes de infección de la vía
respiratoria alta.
Clínicamente se manifiestan con aparición de proteinuria, con edemas en la región periorbitaria,
derrames, ascitis llegando incluso a la anasarca. No suele existir hipertensión. En la analítica
predomina una tendencia a la hipercolesterolemia y a la hiperlipidemia.
En el estudio histológico, lo más característico es la ausencia de alteraciones en la microscopía
óptica (únicamente se encuentra un aumento de la matriz mesangial y acúmulo de gotas lipídicas
como consecuencia de la excesiva reabsorción en el túbulo). La inmunofluorescencia es
negativa, mientras que en la microscopía electrónica se demuestra la ausencia o fusión de los
podocitos (dato característico de la nefrosis lipoidea).
En general, ante un síndrome nefrótico en un niño debemos iniciar el tratamiento sin necesidad
de la biopsia, ya que en la mayoría de los casos es consecuencia de una glomerulonefritis de
cambios mínimos. Sin embargo, en un adulto, es necesario realizar previamente una biopsia
antes de aplicar el tratamiento.
Masson, Tratado de Medicina Interna., págs. 2368-72
11.- Femenino de 9 y medio años presenta astenia, adinamia, decaimiento y pérdida de 3
kilogramos de su peso habitual. Desde hace 15 días ha padecido fiebre de 39°c, disfagia y
epistaxis. Al explorarla se encuentra pálida, con adenomegalia cervical y axilar; se palpa
hepatoesplenomegalia y se observan equimosis en las piernas.

El estudio que se debe practicar para establecer el diagnóstico de certeza es :

a) determinación de Igm.
b) biopsia ganglionar.
c) prueba de Elisa para mycobacterium tuberculosis.
d) biopsia de medula ósea.

Linfoma de Hodgkin

1.*** Fernández-Teijeiro A. Enfermedad de


Hodgkin. En: Madero López L y Muñoz
Villa A editores. Hematología y Oncología pediátricas. Madrid: Ergon; 1997.
p. 467-82.
El autor hace un recuerdo histórico de EH, señalandoaspectos epidemiológicos, etiológicos e
histológicos, resaltando la presentación clínica y el diagnóstico diferencial, así como los estudios
de diagnóstico. Describe con amplitud aspectos terapéuticos de radioterapia, quimioterapia y de
efectos secundarios del tratamiento.
2.* Ferrís Tortajada J, García Castell J, Berbel
Tornero O, Clar Gimeno S. Factores de riesgo para los linfomas no hodgkinianos.
An Esp Pediatr 2001; 55: 230-8.
Exposición de factores de riesgo para LNH, revisados de la literatura médica, con el objetivo,
según los autores, de divulgar entre los pediatras dichos factores de riesgo.
3.* Ferrís Tortajada J, García Castell J, López
Andreu JA, Berbel Tornero O, Clar
Gimeno S. Factores de riesgo para los linfomas de Hodgkin. An Esp Pediatr
2001; 55: 239-43.
Exposición de factores de riesgo para EH, revisados de la literatura médica, con el objetivo,
según los autores, de divulgar entre los pediatras dichos factores de riesgo.

12.- Se trata de RN 16 meses de edad, inició con tos seca, rinorrea hialina, temperatura
axilar de 37.8°C. Por la tarde muestra tos intensa y en accesos, dolor subesternal al toser,
estridor inspiratorio; al explorarle se le observa pálido, con aleteo nasal, hundimiento intercostal
bilateral a la inspiración y taquicardia. Faringe hiperémica roja; en tórax se escuchan algunos
estertores gruesos diseminados en ambos hemitorax y disminución del murmullo vesicular.
El agente causal que se asocia más frecuentemente a este padecimiento es:

a) b) Staphylococcus aureus.
b) Diplococcus pneumoniae.
c) Streptococcus viridans.
d) Virus parainfluenza

La laringotraqueobronquitis también conocida como laringitis subglótica o crup vírico, es una


enfermedad infecciosa aguda de etiología viral que afecta fundamentalmente a la laringe y a las
estructuras que se localizan por debajo de las cuerdas vocales, en donde los tejidos blandos
subglóticos inflamados producen obstrucción de la vía aérea en grado variable. Este
padecimiento afecta con mayor frecuencia al grupo de edad comprendido entre seis meses y tres
años, con un pico de incidencia a los dos años de edad; no hay predominio de sexo, y se
presenta más frecuentemente en otoño e invierno.

ETIOLOGÍA

Los virus parainfluenza tipos 1, 2 y 3 son los principales agentes causales de la


laringotraqueobronquitis, y con menor frecuencia, los virus sincitial respiratorio, parainfluenza tipo
2 y adenovirus. En brotes epidémicos, los virus influenza y parainfluenza tipo 1 son los más
frecuentes. Los cuadros más graves de la enfermedad se han asociado a infección por
parainfluenza tipo 3 e influenza tipo A.

FISIOPATOGENIA

A partir de las superficies epiteliales nasales y faríngeas, el virus se extiende por contigüidad
hasta afectar la laringe y la tráquea, que son las regiones de mayor importancia clínica; se sabe
que 1 mm de edema puede disminuir el diámetro de la subglotis hasta un 65%. Desde estos
sitios, la infección puede progresar hacia los bronquios, y hasta en un 15% de los casos hacia el
intersticio pulmonar.

CUADRO CLÍNICO

El cuadro inicial es el de una rinofaringitis e incluye irritación nasal, coriza, fiebre generalmente
menor de 39 °C, tos seca y odinofagia, con poca o nula afección del estado general; 24 a 48
horas después aparece disfonía, la tos se hace "crupal" (traqueal, perruna o en ladrido) y se
presentan en forma gradual estridor inspiratorio creciente (estridor laríngeo) y signos de dificultad
respiratoria de intensidad variable. En la exploración física son evidentes la disfonía, el estridor
laríngeo que en ocasiones es audible a distancia, la disminución del murmullo vesicular y datos
de dificultad respiratoria.1-6

Forbes describe la progresión del cuadro clínico de la siguiente manera:

Etapa 1: Fiebre, ronquera, tos crupal y estridor inspiratorio al molestar al paciente.

Etapa 2: Estridor respiratorio continuo, participación de músculos accesorios de la


respiración con retracción de costillas inferiores y de tejidos blandos del cuello.

Etapa 3: Signos de hipoxia e hipercapnia, inquietud, ansiedad, palidez, diaforesis y


taquipnea.

Etapa 4: Cianosis intermitente, cianosis permanente, paro respiratorio.

Por lo general la enfermedad es leve, en pocas ocasiones progresa más allá de la etapa 1, más
del 95% reciben tratamiento ambulatorio; del 5% que requiere hospitalización sólo del 1 al 1.5%
requieren medidas de apoyo ventilatorio (intubación endotraqueal o traqueostomía).

La duración del padecimiento es muy variable, desde tres a siete días en casos leves, hasta
siete a catorce días en casos graves.

DIAGNÓSTICO

1. Clínico. Presenta las características ya mencionadas.


2. Paraclínico. Aun cuando es de poca utilidad, comprende lo siguiente:

a) Biometría hemática. Generalmente está normal o presenta leve linfocitosis.

b) Radiografía lateral de cuello. Con frecuencia muestra sobredistensión de


hipofaringe y estrechamiento de la columna aérea de la laringe y la tráquea
cervical. En estudios dinámicos durante la espiración se puede observar que la
luz traqueal se reduce.

c) Radiografía posteroanterior de tórax. Ésta puede mostrar estrechamiento de


la luz traqueal en la región subglótica.

3. Etiológico. Comprende cultivo para virus y serología para la determinación de


anticuerpos, sólo con fines epidemiológicos

DIAGNÓSTICO DIFERENCIAL
1. Epiglotitis aguda. No existe período prodrómico, se presenta con inicio súbito y
progresión rápida de los signos de dificultad respiratoria, estado de toxiinfección,
ausencia de tos crupal y leucocitosis con neutrofilia.

2. Aspiración de cuerpo extraño.

3. Absceso retrofaríngeo.

4. Crup espasmódico.

5. Crup diftérico.

TRATAMIENTO

El tratamiento se debe individualizar en cada caso de acuerdo con la intensidad de las


manifestaciones de dificultad respiratoria. Es muy importante mantener la calma tanto de los
padres como del personal médico. La terapéutica comprende: 8,9

6. Medidas generales: Reposo, ambiente tranquilo, control de la temperatura e hidratación


adecuada.

7. Permeabilidad de la vía aérea. Ésta se logra a través de las siguientes medidas:

a) Ambiente húmedo. En ocasiones con esta medida es suficiente; más del 90% de los casos se
pueden tratar en casa al proporcionar humedad por medio de una regadera caliente o por un
humidificador fresco en tienda en cama. En el hospital es posible proporcionar nebulizaciones
frescas en una tienda o croupette mediante un aparato humidificador con 30 o 40% de oxígeno y
temperatura de 21 a 24 °C.

b) Epinefrina racémica. Estimula los receptores alfa adrenérgicos en la mucosa subglótica,


produciendo vasoconstricción y por lo tanto disminución del edema de la mucosa. Se administra
al 2.25% en 3 ml de solución salina al 0.9%, en nebulizaciones y/o sesiones de presión positiva
intermitente (RPPI) durante 15 o 20 minutos; la dosis depende del peso y es de 0.25 ml en niños
con peso menor de 20 kg, 0.5 ml entre 20 y 40 kg y 0.75 ml para niños con peso mayor de 40 kg;
el máximo efecto se obtiene a los 60 minutos después del tratamiento; si existe persistencia del
estridor y/o datos de dificultad respiratoria a las dos horas siguientes se debe hospitalizar el
paciente. 8-14

c) Traqueostomía o intubación endotraqueal. Estas medidas son


necesarias en raras ocasiones; sólo se utilizan cuando a pesar de
las medidas previas se incrementan los signos de dificultad respiratoria. Si se presenta el caso,
se prefiere la intubación endotraqueal ya que reduce el tiempo de tratamiento y hospitalización y
tiene menor morbilidad. Cuando se requiere la intubación endotraqueal debe permitirse que
transcurra tiempo suficiente aproximadamente de tres a cinco días para que remitan la
inflamación y el edema; para realizar la extubación con mayor éxito se mencionan los siguientes
criterios: que el niño esté afebril, secreciones escasas, que exista una fuga de aire alrededor del
tubo endotraqueal (presencia de tos o vocalización alrededor del tubo edotraqueal o una fuga
audible de aire con una presión positiva de 30 cm de H2O); se considera que si no existen fugas
de aire al cabo de cinco a siete días y se cumplen los otros criterios, deberá extubarse.

3. Esteroides. Su utilidad aún es discutible, sin embargo, el uso de una sola dosis intramuscular
o intravenosa de dexametasona a dosis de 0.6 mg/kg ha mostrado disminución en la severidad y
duración de los síntomas, con máximo efecto a las seis horas después de su administración y no
existe justificación para repetir la dosis. Si se utilizan pocos minutos antes de administrar
epinefrina racémica se logra potenciar el efecto de esta última. Se ha observado que cuando un
paciente requiere intubación endotraqueal se logra extubar en menor tiempo si recibe esteroides
que si no se le administran

4. Antibióticos. No tienen utilidad y no se deben usar como profilácticos

13.- Se trata de masculino de 6 años acude al servicio de urgencias por salida de áscaris por el
ano. A la exploración física presenta dificultad y sibilancia respiratoria, abdomen con
hepatomegalia y distensión abdominal, se confirma el diagnóstico por USG.
El tratamiento de elección en este paciente es:

a) Secnidazol vía oral


b) Metronidazol vía oral
c) Salbutamol Inhalado
d) Albendazol vía oral

Tratamiento

Se realizará tratamiento farmacológico por vía oral, siendo las drogas de elección el albendazol,
el pamoato de pirantel y oxantel y la clásica piperacina

La ascaridiasis intestinal es una patología ocasionada por un helminto que parasita el tubo
digestivo del ser humano. Es la parasitosis más frecuente y cosmopolita de todas las helmintiasis
humanas, se calcula más 1.5 billones de portadores en el mundo de los cuales 51 millones de
afectados son niños.

La gran mayoría de los casos cursa en forma asintomático o pueden producirse signos
inespecíficos como dolor abdominal difuso o ir acompañado en los casos crónicos por signos de
desnutrición, déficit de crecimiento y retardo del aprendizaje.

Los efectos patológicos producidos por Áscaris en el organismo humano, se


presentan en varios sitios de acuerdo a la localización de las diversas formas evolutivas. Las
larvas al pasar por el pulmón producen ruptura de los capilares y de la pared alveolar. Cuando
ocurre en forma masiva da origen al síndrome de Löeffler que se caracteriza por lesiones
múltiples de los alvéolos donde producen granulomas de cuerpo extraño, el cual se observa a los
rayos X como opacidades diseminadas.

Cuando el número de parásitos es grande puede ocasionar complicaciones tales como


obstrucción intestinal, vólvulos, y perforación visceral con peritonitis, complicaciones que deben
ser resueltas en forma quirúrgica.

Las mayores complicaciones se presentan por las migraciones de Áscaris adultos a diferentes
sitios del organismo. La invasión del parásito al árbol biliar, puede causar ictericia obstructiva,
colecistitis acalculosa, colangitis, pancreatitis, abscesos hepáticos.
Se han descrito diversas modalidades de tratamiento, La migración masiva de parásitos hacia el
árbol biliar se asocia con procesos infecciosos, para lo cual deben administrarse antibióticos aun
en forma profiláctica. La administración de albendazol se considera el tratamiento
antihelmíntico de elección por su mayor excreción biliar. En un bajo porcentaje (21.2%) se
logra expulsar los parásitos de la vesícula biliar. Si no se logra expulsarlo, si persiste la
sintomatología o si el paciente empeora, es necesario someterlo a una laparotomía exploradora
para realizar colecistectomía.

Los pacientes pueden caer en una septicemia fatal, si no llevamos a cabo la terapéutica en
forma adecuada, lo que hace que la evolución y el pronóstico empeoren.

La parasitosis es uno de los grandes problemas de la salud pública. La OMS la considera una de
las principales causas de morbilidad; estrechamente ligada a la pobreza y relacionada con
inadecuada higiene personal y de los alimentos crudos, falta de servicios sanitarios, falta de
provisión de agua potable y contaminación fecal del ambiente. Infecta a personas de todas las
edades, pero la sufren principalmente los niños, a lo que les causa trastornos del crecimiento y
desarrollo.

Desde hace años la OMS propone como solución, aunque sea parcial, el uso de tratamientos
antihelmínticos masivos y reiterados, sin diagnóstico parasitológico previo individual, en aquellas
comunidades que tengan una elevada prevalencia de geohelmintos (>50%).

Bibliografía
Cook GC, Zumla AI (edit). Manson´s Tropical Diseases. 21th ed. London: Elsevier Science; 2002
García LS. Diagnostic Medical Parasitology. 4th. Ed. Washington: American Society for
Microbiology; 2002
Gill G. Lectura Notes on Tropical Medicine. 5th ed. Blackwell Scientific Pub; 2004
Mensa JM, Gatell MT, Jiménez de Anta G, Prats A, Dominguez-Gil A. Guía terapeútica
antimicrobiana. 14ª ed. Barcelona: MASSON; 2004.
Murray PR, Baron EJ (Edit.)Manual of clinical microbiology. 8th edit. Washington: ASM Press;
2003
Villa Luis F. Guía de Terapia Farmacológica. Medimecum. Barcelona: ADIS Internacional; 2005.

14.- Se trata de femenino de 6 meses de edad, atendida en consulta externa por presentar
rinorrea hialina y malestar general de cinco días de evolución. El día de hoy inicia con febrícula,
respiración ruidosa, tos productiva en accesos, disnea y dificultad para deglución. Antecedentes:
alimentada al seno materno, tienen un gato en casa, su madre padece rinitis alérgica desde la
infancia. Exploración física: TEMP 37.5º C, FC: 110 LPM, FR: 60 RPM, faringe hiperémica, sin
adenomegalias cervicales, tórax con sibilancias inspiratorias y espiratorias a distancia.

El agente causal más probable en este caso es:

a) Alérgenos del polvo de casa.


b) Haemophilus influenza.
c) Pelo de gato.
d) Virus sincitial respiratorio. (bronquilitis)

La bronquiolitis es una enfermedad respiratoria aguda que afecta a la vía aérea pequeña y, más
concretamente a los bronquiolos; frecuente durante la lactancia, especialmente en los menores
de doce meses.
Constituye la primera causa de hospitalizaciones del lactante; se calcula que durante una
epidemia puede contraer la enfermedad sobre el 10% de los lactantes de una comunidad, de los
cuales aproximadamente el 15% requerirán ingreso hospitalario.
Quizá, la primera definición de bronquiolitis la hizo Holt en 1898, cuando se refería a una forma
grave de bronquitis catarral que afectaba a la pequeña vía aérea y que denominó “bronquitis
capilar”.
Pero fue Mc Connochie, en 1983, quién estableció los criterios que definen la bronquiolitis.
Según Mc Connochie, la bronquiolitis se define como un primer episodio agudo de sibilancias, en
el contexto de un cuadro respiratorio de origen viral, que afecta a lactantes menores de 24
meses.

El VRS es el principal agente etiológico de la bronquiolitis, especialmente durante las epidemias


y en los casos que requieren ingreso hospitalario.

TABLA I.
Escala de Wood- Downes modificada por Ferrés

El puntaje de Tal modificado (cuadro 2) es un puntaje clínico que incluye frecuencia cardíaca,
frecuencia respiratoria, sibilancias y uso de músculos accesorios, asignándose a cada ítem un
valor de 0 a 3. Tiene como máximo un valor 12, que corresponde a una severa enfermedad

Revista de Posgrado de la VIa Cátedra de Medicina - N° 167 – Marzo 2007

15.- En el cunero, se observa que un neonato presenta asimetría de pliegues glúteos. A la EF


la cadera, una de ellas puede ser fácilmente luxada posteriormente con un “click” y regresada a
su posición normal con un sonido parecido. La familia se encuentra preocupada porque el primer
hijo tuvo el mismo problema. El diagnóstico más probable ES:

a) Displasia de cadera
b) Enfermedad de Legg-Perthes
c) Artritis séptica
d) Parto traumático

La falta de relación normal en las estructuras que forman una articulación se conoce como
luxación. En el caso de la cadera, la cabeza del fémur no encaja debidamente en su cavidad
(acetábulo).
El desarrollo del acetábulo será normal siempre que la cabeza del fémur permanezca en
posición correcta y el esfuerzo muscular sea adecuado.
Siempre que esto no se cumpla, se producirá una alteración entre el cótilo y la cabeza femoral,
dando lugar a una alteración con el resultado de luxación de la cadera.

DIAGNÓSTICO: EXPLORACIÓN FÍSICA


a) Maniobras de Ortolani y Barlow - click audible-.
Ortolani: el click se produce cuando la fóvea de la cabeza del fémur encuentra la cresta
cotiloidea caminando sobre ella. La limitación a la abducción desaparece, esto es signo de
reducción.
Barlow: se realiza la aducción de la cadera, que tras una ligera presión longitudinal sobre el
fémur, produce una luxación.
La prueba de Ortolani traduce una luxación y la de Barlow una cadera luxable.
b) Asimetría de pliegues -no valorable en luxaciones bilaterales-.
c) Limitación a la abducción.
d) Dismetría.

• García Portabella, M.
Luxación congénita de cadera antes de los tres meses de edad. 2001.
• Garcia-Siso Pardo, J. M.
Displasia del desarrollo de la cadera.
(Parte I). Peditría Rural y Extrahospitalaria.
Vol. 32. Nº 304 Págs. 481-491. 2002.
• Graf, R.
Classification of hip joint dysplasia by means of sonography.
Arch Orthop Trauma Surg 102:248-255, 1984.
• Edeiken, J.
Luxación congénita de cadera.
Diagnóstico Radiológico de las Enfermedades de los Huesos.
Ed. Médica Panamericana. Págs. 388-392. 1977.

16.- A 6 year old girl who presented skin lesions such macules and papules that evolve into
blisters within hours. The mother states that introduced upper respiratory infection 2 weeks ago.
The diagnosis you do it?

a) Herpes Zoster.
b) Rubella
c) Varicella
d) Scarlet fever

Varicela

Manifestaciones clínicas: la infección primaria produce fiebre moderada, y un exantema


vesicular genereralizado y pruriginoso. Las lesiones se observan en distintos estados (mácula,
pápula, vesícula y costras) y duran alrededor de una semana. Es posible también apreciar
vesículas o úlceras en la mucosa oral. La reactivación del virus es responsable del herpes
zoster.
Agente etiológico: virus varicella zoster (ADN) perteneciente a la familia Herpesviridae

Epidemiología: los humanos son la única fuente de contagio para este agente. La transmisión
es a través del contacto persona a persona y por la ruta respiratoria. Los brotes aparecen hacia
fines del invierno y en primavera. El período de incubación varía entre 10 a 21 días, y
usualmente es de dos semanas. El momento de mayor contagiosidad ocurre desde dos días
antes de que el exantema aparezca, hasta que las lesiones se encuentren en etapa de costra.
En niños inmunodeprimidos, los períodos de incubación, de contagiosidad y de erupción
pueden ser más largos.

Diagnóstico: el diagnóstico es fundamentalmente clínico; sin embargo, existen situaciones de


diagnóstico diferencial de lesiones vesiculares en el huésped inmunodeprimido, en que la
inmunofluorescencia directa permite hacer el diagnóstico rápido de infección por virus varicela
y así, tomar decisiones terapéuticas precoces.

Aislamiento del paciente hospitalizado: Aislamiento respiratorio y de contacto por al menos


cinco días desde que comienza el exantema o hasta que todas las vesículas estén en estado
de costra.

Tratamiento: en el huésped inmunocompetente el manejo del paciente con infección aguda es


sintomático. Si se requiere uso de antipiréticos se recomienda usar sólo paracetamol, por la
posible riesgo de desarrollo del Síndrome de Reye con el uso de ácido acetilsalicílico. Debe
prevenirse la sobreinfección bacteriana de las lesiones, evitando el prurito y el grataje,
manteniendo las uñas cortas, baño diario y usando antihistamínicos.

El uso de aciclovir está indicado en sujetos con riesgo de desarrollar complicaciones


(inmunodeprimidos, adolescentes, adultos, pacientes con terapia crónica con salicílicos y
esteroides) y debe iniciarse en las primeras 24 hrs del exantema. El uso de aciclovir puede
también considerarse en el segundo caso intrafamiliar ya que en estos pacientes se ha
observado una evolución más severa de la enfermedad.

Bibliografía:

Report of the Committee on Infectious Diseases, 25h edition, Red Book 2000. American
Academy of Pediatrics.
Fifth (human parvovirus) and sixth (herpesvirus 6) diseases. Koch WC. Curr Opin Infect Dis
2001, Jun; 14 (3): 343-356.

Primary human herpesvirus 8 in immunocompetent children. Andreoni M, Sarmati L,


Nicastri E, El Sawaf G, El Zalabani M, Uccella I, et al. JAMA 2002 Mar 13; 287 (10): 1295-
300.

Varicella vaccine update. AAP. Pediatrics 2000, Jan 105: 136-141.

17.- Recién nacido de 32 semanas de gestación, que presenta de manera súbita hipotensión,
anemia y abombamiento fontanelar. A al EF presenta una FC de 100 x’, una FR de 20 x’ y
comienza a convulsionar. El diagnóstico más probable del paciente es:

a) Hemorragia de la matriz germinal


b) Leucomalacia periventricular
c) Hidrocefalia
d) Hipotiroidismo congénito

Manual CTO pediatría, 7° edición p.1295


La matriz germinal es una estructura del cerebro inmaduro hasta la semana 34 de gestación. Su
lesión produce hemorragia. Se manifiesta con deterioro del estado general, aparición de pausas
de apnea, bradicardia, hipoventilación, convulsiones, disminución del reflejo del Moro y
fontanelas a tensión. Debe sospecharse ante un paciente pretérmino que presenta de forma
súbita anemia, hipotensión y abombamiento fontanelar. Puede derivar en la aparición de
leucomalacia periventricular.

18.- Se trata de masculino de 2 años con otitis media aguda recurrente quien recibió
tratamiento con amoxicilina en el mes previo, el tratamiento de elección en éste paciente es:

a) Amoxacilina+ clavulanato a 25 mg/kgdía


b) Amoxacilina+ clavulanato a 90 mgkgdía
c) Azitromicina v.o 10mg kg día
d) Ceftriaxona IM 50 mgKgdía

En niños mayores de 2 años, con OMA moderad o leve y sin factores de riesgo, el
tratamiento inicial debería ser sintomático. Si a las 48-72 horas persiste o empeora la clínica se
debe iniciar antibioterapia dirigida frente al S. pneumoniae, por su elevada frecuencia en nuestro
medio y la baja tasa de curación espontánea. Se administrará amoxicilina a las dosis estándar
recomendadas (40-50 mg/kg/día), ya que tienen una buena actividad frente a S.
pneumoniaesensible o con resistencia intermedia a la penicilina. La duración del tratamiento
antibiótico no está claramente establecida, aunque se ha observado que con 5-7 días se logra la
resolución de la infección. Con una pauta corta se consigue, además, disminuir el riesgo de
resistencias bacterianas, del número total de antibióticos pautados y del coste económico.
En caso de fracaso terapéutico, es decir, reaparición o persistencia de signos y síntomas de
OMA, se debe administrar un antibiótico no sólo efectivo frente al S. pneumoniaeresistente a
penicilina sino también frente a patógenos productores de betalactamasa: amoxicilina-
clavulánico a dosis altas (80-90 mg/kg/dia de amoxicilina y 10 mg/kg/dia de ácido clavulánico).
Hasta que no se disponga de formas comerciales adecuadas (relación amoxicilina/clavulánico=
8/1), esta dosificación se puede alcanzar mezclando amoxicilina y amoxicilina-clavulánico

BIBLIOGRAFÍA
1. Rosenfeld RM. Método basado en la evidencia para tratar la otitis media. Clin Ped Nort(ed
esp) 1996; 6: 1075-1092.
2. Arnold JE. El oído. En: Behrman RE, Kliegman RM, Arvin AM, ed. Nelson. Tratado de
Pediatría.l5ª edición.
McGraw-Hill-Interamericana de España, 1997; 2258-2270.
3. Pitkaranta A, Virolainen A, Jero J, Arruda E, Hayden F. Detección de infecciones por rinovirus,
virus respiratorio sincitial y coronavirus en la otitis media aguda mediante la reacción en cadena
de la polimerasa de transcriptasa inversa. Pediatrics (ed esp.)1998; 6: 85-89.
4. Palva T, Pulkkinen K. Mastoiditis. J Laryngol Otol 1991; 105: 765-766.

19.- Se trata de masculino de 7 años, es traído a urgencias por cuadro diarreico desde hace 24
horas. La diarrea es líquida, profusa, sin moco ni sangre, se acompaña de vómito de contenido
gástrico. Refiere la madre que varios de los compañeros de la primaria han presentado el mismo
cuadro.
El agente causal responsable del cuadro clínico de este paciente es:

a) Escherichia coli
b) Campylobacter jejuni
c) Virus Norwalk.
d) Bacillus cereus

El virus del Norwalk pertenece al género de virus ARN de la familia Calciviridae, la transmisión
ocurre vía fecal-oral. Agente infeccioso más común de las gastroenteritis benignas de la
comunidad. El promedio de incubación es de 24 horas. El cuadro clínico por lo común dura de 12
a 60 horas y se caracteriza por uno o más de los síntomas siguientes: náusea, vómito, cólicos
abdominales y diarrea. El vómito es más prevalente en niños, en tanto que una proporción mayor
de los adultos mostrará diarrea.
Harrison, Principios de Medicina Interna, 16ª Edición, Mc Graw Hill, p.p. 1266-1267

20.- Se trata de femenino de 19 años con menarca a los 12 años ritmo menstrual 45x4.
Refiere vida sexual activa desde los 17 años con frecuencia de 4 veces por semana. A la
exploración se encuentra acné intenso en la frente, mejillas y mentón. Acude a consulta por
que desea adoptar un tratamiento anticonceptivo por vía oral, el más adecuado es:

a) Norgestimato
b) Ciproterona
c) Gestodeno
d) Levonorgestrel

El efecto antiandrogénico específico del acetato de ciproterona actúa por inhibición competitiva
de la unión de la 5 - alfa - dihidrotestosterona con el receptor citosólico de las células blanco,
que disminuye la producción y la excreción de sebo y el aumento y el desarrollo del vello.
Es un derivado de la 17 - alfa - hidroxiprogesterona que posee acción progestágena. Su acción
antigonadotrófica se suma a la del etinilestradiol. El acetato de ciproterona no posee acción
estrogénica sino un efecto antiestrogénico, y tampoco posee acción nociva sobre la función de
la corteza suprarrenal;

Indicaciones en la mujer: Manifestaciones de androgenización de grado severo, por ejemplo,


hirsutismo grave, alopecia androgenética de tipo grave, a menudo acompañados por
manifestaciones graves de acné y/o seborrea.
Indicaciones en el hombre: Atenuación del impulso en las desviaciones sexuales.
Tratamiento antiandrógeno del carcinoma de próstata inoperable.

BIBLIOGRAFÍA
1. Swift S. Current opinion on the classification and definition of genital tract prolapse. Curr Opin
Obstet
Gynecol 2002; 14: 503-7.
2. De Caro R, Aragona F, Herms A, Guidolin D, Bizzi E, Pagano F. Morphometric analysis of
the fibroadipose tissue of the female pelvis. J Urol 1998; 160: 707-13.
3. Gill E, Hurt W. Pathophysiology of pelvic organ prolapse. Clin Obstet Gynecol 1998; 25(4):
757-69.
4. DeLancey, J. Anatomic aspects of vaginal eversion after Hysterectomy. Am J Obstet
Gynecol. 1992;
166(6 pt 1): 1717-24.

21.- Se trata de masculino de 7 meses de edad presenta una historia de estreñimiento que ha
venido en incremento con antecedente de dos impactaciones fecales, durante éste mes expulsa
heces duras una vez a la semana. Su exploración física con mal incremento ponderal. El
diagnóstico más probable del menor es:

a) Hipotiroidismo
b) Enfermedad de Hirschprung
c) Envenenmiento por plomo
d) Estreñimiento funcional

La enfermedad de Hirschsprung (EH) es considerada una enfermedad congénita caracterizada


por una ausencia de células ganglionares en el plexo mientérico de Auerbach y en el submucoso
de Meissner, en el recto y otros segmentos del colon en forma ascendente (de caudal a cefálico)
(1). Esta alteración produce una anormalidad de la motilidad intestinal, que se manifiesta más
frecuentemente como una obstrucción intestinal (2).
La EH puede ser clasificada según el segmento intestinal comprometido. Así puede dividirse en:
1.- Segmento corto: cuando no compromete más allá de la unión rectosigmoidea;
2.- Ultracorto: si sólo afecta esfínter interno o algunos centímetros próximos a dicho esfínter;
3.-Segmento largo cuando el segmento agangliónico afecta más allá de la unión rectosigmoidea.
Otros autores sólo distinguen dos tipos: segmento corto y segmento largo, siendo controversial la
existencia de EH de segmento ultracorto, ya que esta compromete menos de 5 cm del recto
distal .
La mayoría de los casos corresponde a EH de segmento corto (75 a 80%), una décima parte de
ellos sería un aganglionismo ultracorto. El 20% restante pertenecería a EH de segmento largo,
incluyéndose en este grupo aquellos que afectan los segmentos proximales al ángulo esplénico.
La EH forma parte de los trastornos conocidos como disganglionismos que incluyen también el
hipoganglionismo y la displasia neuronal intestinal.

Expresión Clínica
Dentro de los síntomas que permiten una sospecha precoz, se encuentra el estreñimiento o
constipación, definida en el recién nacido como el retraso en la eliminación de meconio mayor a
48 horas asociada a distensión abdominal, y en los niños mayores como deposiciones
infrecuentes de consistencia aumentada (. El 98% de los lactantes elimina el meconio en las
primeras 48 horas de vida. Los prematuros eliminan más tardíamente el meconio, pero la EH es
rara en prematuros. De los pacientes con EH, sólo el 60% elimina el meconio después de las 48
horas, por lo que este signo no es patognomónico de la enfermedad.
La mayoría de los niños que presentan aganglionosis congénita, son sintomáticos los primeros
días o las primeras semanas luego del nacimiento (2). Alrededor de dos tercios de los pacientes
presenta síntomas dentro de los tres primeros meses de vida y 80% desarrolla síntomas dentro
del primer año de vida. Sólo un 10% de los pacientes inicia síntomas entre los 3 y 14 años de
edad y en general se trata de pacientes con enfermedad de segmento ultracorto. (13)
Los recién nacidos y lactantes pequeños presentan con frecuencia signos de obstrucción
intestinal, distensión abdominal, vómitos biliosos e intolerancia a la alimentación. La inspección
anal y la radiografía pueden orientarnos hacia una causa mecánica de obstrucción, pero no
descarta EH. Si la obstrucción no tiene una causa mecánica, además de pensar en una EH,
debe plantearse el diagnóstico diferencial con hipotiroidismo, insuficiencia suprarrenal,
hipokalemia, hipercalcemia, hipomagnesemia, y en casos excepcionales alteraciones
neuromusculares.
Cuando la sintomatología es poco evidente,
Puede presentarse como un cuadro de constipación crónica, con historia de dificultad en la
eliminación de deposiciones, masas fecales palpables en fosa ilíaca izquierda y un tacto rectal
en que no se encuentran deposiciones en la ampolla rectal y esfínter anal hipertónico. En
muchas ocasiones la estimulación rectal provoca salida explosiva de heces líquidas de olor fétido
(17). Por lo tanto, frente a pacientes con constipación crónica, en los cuales se ha descartado
causa mecánica de obstrucción intestinal, que no cede a las medidas dietéticas ni
farmacológicas, debe plantearse el diagnóstico de
EH. También puede encontrarse dilatación de asas intestinales, adelgazamiento de la pared
abdominal, alteraciones de la nutrición y el crecimiento.
En niños mayores, los síntomas más comunes incluyen constipación crónica progresiva,
impactación fecal recurrente, mal incremento ponderal y malnutrición.

Rev. Ped. Elec. 2008, Vol 5, N° 1. ISSN 0718-0918


Servicio Salud Metropolitano Norte
Facultad de Medicina Hospital Clínico de Niños
Departamento de Pediatría y Cirugía Infantil Roberto Del Río
Referencias
1. De Manueles J. Enfermedad de Hirschsprung.
Protocolos diagnósticos y terapéuticos en pediatría. Sociedad Española de Pediatría. Pag. 56-60.
2. Feldmon T., Wershil B. Hirschsprung Disease. Pediatrics in review. Vol 23. N 11, August 2003.
3. Luis L.A., Encinas J.L., Avila L.F., et cols.
Enfermedad de Hirschsprung: enseñanzas de los últimos 100 casos. Cir Pediatr 2006; 19:177-
181.
4. J.M. Gil-Vener y cols. Diagnóstico dieferncial de Hirschsprung-neurodisplasia intestinal.
Fiabilidad de las pruebas diagnósticas. Cir Pediatr 2006; 19: 91-94.
5. M. López, y cols. Índices de fiabilidad de la manometría anorrectal para el diagnóstico de la
enfermedad de Hirschsprung en cualquier edad. Cir Pediatr 2005; 18:13-16
6. Hernández F., Rivas S., Ávila L.F., Díaz M., ET cols. Aganglionismos extensos. Tratamiento y
resultados a largo plazo. Cir Pediatr 2003; 16: 54-57.
7. Goulet O. y cols. Intestinal transplantation in children: preliminary experience en Paris. JPEN
J Parenter Enteral Nutr 1999; 23 (5 Suppl)
8. Peña A. Enfermedad de Hirschsprung. Los avances y las preguntas no contestadas. Cir
Pediatr 2002: 15:46-47
9. Polliotto S, Heinen F, Anduna G, Korman R. Evaluación de resultado a tres años de nuestra
primera experiencia en el tratamiento laparoscópico de la enfermedad de
Hirschsprung. Cir Pediatr 2001; 14: 85-87
22.- Recién nacido a término, obtenido por cesárea. Presenta dificultad respiratoria durante las
primeras horas de vida, con frecuencias respiratorias de 90 por minuto y cianosis que mejora con
el oxígeno. Los campos pulmonares están bien ventilados. La radiografía de tórax muestra líneas
de líquido en las cisuras, y marcas vasculares prominentes. El diagnóstico más probable es:

a) Aspiración de meconio
b) Taquipnea transitoria del recién nacido
c) Cardiopatía congénita cianógena
d) Enfermedad de membrana hialina

La Taquipnea Transitoria del Recién Nacido (TTRN), se define como su nombre lo dice, como un
aumento de la frecuencia respiratoria en el neonato de manera transitoria, es una enfermedad
benigna y autolimitada que afecta principalmente el neonato a término, aunque puede afectar a
los neonatos pretérmino limite nacidos por cesárea.

CLÍNICA
El cuadro clínico se inicia desde el nacimiento, se caracteriza por taquicardia, frecuencia
respiratoria aumentada (60-160 rpm), retracciones esternales y subcostales, quejido espiratorio y
cianosis leve que mejora al administrar cantidades mínimas de oxígeno. La clínica puede
agravarse en las primeras 6-8 horas, para estabilizarse posteriormente y, a partir de las 12-14
horas, experimentar una rápida mejoría de todos los síntomas, la totalidad de los signos y
síntomas suelen autolimitarse a los 3-4 días después del nacimiento.
Los gases sanguíneos pueden revelar acidosis respiratoria que se resuelve en las siguientes 8 a
24 horas.
Al examen físico no se suelen encontrar ruidos pulmonares patológicos, no estertores ni roncus.
Los Rx de tórax pueden revelar hiperinsuflación, una trama vascular pulmonar prominente,
fisuras interlobares aumentadas de tamaño por la presencia de líquido, aplanamiento y
depresión del diafragma, aumento de la sombra cardiotímica (cardiomegalia) y en ocasiones
derrame pleural.
El diagnóstico diferencial incluye enfermedad de la membrana hialina, la cual se diferencia de la
TTRN por que ésta última presenta una brusca recuperación del recién nacido y tiene ausente un
patrón reticulogranular con broncograma aéreo en la Rx de tórax; otros son: neumotórax,
aspiración meconial, insuficiencia cardiaca congestiva, neumonías bacterianas o víricas,
trastornos metabólicos, policitemia e hiperviscosidad y hernia diafragmática asociada a
hipoplasia pulmonar; las características radiológicas y la mayor gravedad de la dificultad
respiratoria permiten diferenciar estos trastornos del TTRN.
El diagnóstico se confirma por la evolución favorable de los Rx a las 24-48 horas en asociación
con el mejoramiento del cuadro clínico.
En el manejo de la TTRN hay controversia, algunos textos(2) recomiendan, debido a que los
síntomas son inespecíficos y compatibles con sepsis neonatal o neumonía, tomar los exámenes
( PCR, hemograma y cultivos) necesarios para confirmar su etiología y tratar con antibióticos de
amplio espectro hasta establecer el diagnóstico definitivo; de otra parte hay literatura(3) que
teniendo en cuenta el carácter autolimitado de la enfermedad solo recomienda administración de
oxigeno con el objetivo de mantener la presión parcial de oxigeno normal.
No se recomienda el uso de diuréticos (furosemida) en el edema pulmonar ya que no se ha
demostrado su utilidad y pueden llevar a un desequilibrio hidroelectrolítico.
En cuanto al manejo nutricional del neonato con TTRN se recomienda tener en cuenta la
frecuencia respiratoria (FR) del neonato para decidir si dar vía oral o parenteral, es así como: si
la FR es menor de 60 rpm se permite vía oral, si la FR esta entre 60-80 rpm la alimentación debe
ser por sonda nasogástrica y si la FR es mayor de 80 rpm se recomienda dar nutrición
parenteral(4).
NOTAS
1. W. Taeusch, R. Ballard, C. Gleason: Avery's Diseases of the Newborn. Octava Edición.
Elsevier Saunders. 2005. pag
697-699.
2. G. Avery, M. Fletcher, M. MacDonald: Neonatología, fisiopatología y manejo del recién nacido.
Quinta Edición. Editorial
Médica Panamericana. 2001. Página 507.
3. A. Rudoklpph. R. Collin: Pediatría de Rudolph. Mc Graw Hill Interamericana. Edición 21.
España 2004.

23.- En época invernal es referido al servicio un lactante de 6 meses que acude a guardería,
inicia sintomatología hace 48 hrs, con febrícula y algún vómito ocasional, el día de hoy en un
principio presenta evacuaciones blandas y en el transcurso del día se presentan de consistencia
líquidas, incrementándose los vómitos. Refiere la madre que en la guardería ha habido otros
niños con un cuadro similar. La etiología más probable en este caso es:

a) Yersinia enterocolítica.
b) Campylobacter yeyuni.
c) Rotavirus.
d) Adenovirus.

Rotavirus es un género de virus perteneciente a la familia Reoviridae. Se han identificado siete


grupos, tres de los cuales (Grupo A, B y C) infectan a los humanos. El grupo A es el más común
y el más esparcido, causando el 90% de las infecciones.

Estos causan vómito y diarrea y son los más comúnmente causantes de diarrea severa en los
infantes, ocasiona aproximadamente 55,000 hospitalizaciones cada año en los Estados Unidos y
mata a cerca de 600,000 niños cada año en países en vías de desarrollo. Nuevas vacunas han
mostrado ser efectivas y seguras en el 2006. Virtualmente todos los niños menores de cinco
años han sido infectados por algún rotavirus. Rotavirus es la causa más frecuente de
gastroenteritis en niños de guardería y de gastroenteritis nosocomial en niños pequeños. En
líneas generales es el motivo más frecuente de gastroenteritis en niños menores de dos años (4
a 24 meses de vida) y de deshidratación y muerte en los países en vías de desarrollo

Se transmiten por la ruta fecal oral, infectando células del intestino delgado y produciendo una
enterotoxina, provocando una gastroenteritis que puede llevar a una diarrea e incluso
deshidratación. Aunque fueron descubiertos en 1973 y son responsables de más del 50% de los
ingresos hospitalarios de niños con diarrea severa, siguen siendo subestimados por la
comunidad médica, sobre todo en los países en vías de desarrollo..

Los rotavirus propician gastroenteritis aguda y fuerte dolor abdominal. "Diarrea infantil", "diarrea
invernal", "infección no bacterial aguda" y "gastroenteritis viral aguda" son los otros nombres con
los que se denomina a este padecimiento. La dosis infectante se presume que es de 10-100
partículas virales infecciosas, ya que una persona con rotavirus frecuentemente excreta una gran
cantidad de partículas virales: en el orden de (108-1010 partículas infecciosas /ml de heces). La
vía de contagio se da a través del contacto con manos, objetos o utensilios contaminados. El
período de incubación de la enfermedad por rotavirus es de aproximadamente 2 días. La
enfermedad está caracterizada por vómito y diarrea acuosa de 3 a 8 días, y fiebre con dolor
abdominal ocurre con frecuencia. La inmunidad se produce después de la infección. Infecciones
posteriores tienden a ser menos severas que la infección original.
Una micrografía electrónica de Rotavirus: obsérvese el aspecto de «rueda» del virión

BIBLIOGRAFÍA RECOMENDADA

American Academy of Pediatrics, Committee on Infectious Diseases. Prevention of rotavirus


disease: guidelines for use of rotavirus vaccine. Pediatrics 1998; 102: 1483-91.

Bernstein D. J Infect Dis 2002; 186: 1487-1489.

Centers for Diseases Control and Prevention. Rotavirus vaccine for the prevention of rotavirus
gastroenteritis among children: recommendations of the Advisory Committee on Immunization
Practices (ACIP). MMWR 1999; 48: 1-23.

Chang EJ, Zangwill KM, Lee H, Ward JI. Lack of association between rotavirus infection and
intussusception: implications for use of attenuated rotavirus vaccines. Pediatr Infect Dis J 2002;
21: 97-102.

24.- Se trata de masculino de 3 y medio años que acude al servicio de pediatría referido para
valoración por masa y distención abdominal, a la E.F. con buen estado general y
hemihipertrofia corporal, se corrobora una masa abdominal; se envía para estudio urográfico i.v.
el cual muestra una masa voluminosa sin calcificación en el riñón izquierdo que distorsiona el
sistema pielocalicial, la ecografía abdominal determina el carácter sólido de la masa, así como la
existencia de trombosis en la vena renal. El diagnóstico más probable es:

a) Neuroblastoma.
b) Nefroma mesoblástico congénito.
c) Carcinoma renal.
d) Tumor de Wilms.

TUMOR DE WILMS

• Es el tumor maligno primario renal más frecuente en la edad pediátrica


• Tumor sólido que se origina en el riñón.

Etiología

• Varias regiones cromosómicas se han asociado al Tumor de Wilms


– Banda 11p13 (contiene el gen supresor del tumor de Wilms WT1)
– Banda 11p15(sitio donde se coloca el gen WT2 del tumor de Wilms)
– Brazo cromosómico 17q (contiene el locus familiar FWT1)
– Brazo cromosómico 19q (contiene el locus familiar FWT2)
• p53
– Gen supresor de tumor
– Se asocia a anaplasia, estadios avanzados de la enfermedad
– Recurrencia

• Otros
– Bcl-2, TrkB
Cuadro clínico

• Masa abdominal
– Localizada en el flanco
– No se mueve con respiración

• Distensión abdominal
• Dolor abdominal
• Hematuria macroscópica
• Fiebre
• Anemia (asociada a hemorragia subcapsular repentina)

• Hipertensión (por aumento de la actividad de la renina): 25%

Diagnóstico

• Ultrasonido abdominal
– Masa sólida o quística
– Identificación del órgano donde se origina la masa
– Medición

• Radiografía simple
– Metástasis pulmonares
• Tomografía axial computarizada contrastada
• Evalua la naturaleza y extensión de la neoplasia
– Sugiere extensión aparente a otras estructuras

Bibliografía:

1. Grundy P, Breslow N, Green DM, et al.: Prognostic factors for children with recurrent
Wilms' tumor: results from the second and third National Wilms' Tumor Study. Journal of
Clinical Oncology 7(5): 638-647, 1989.
2. Shamberger RC, Guthrie KA, Ritchey ML, et al.: Surgery-related factors and local
recurrence of Wilms tumor in National Wilms Study 4. Annals of Surgery 229(2): 292-297,
1999.
3. Abu-Ghosh A, Goldman S, Krailo M, et al.: Excellent response rate (91%) to ifosfamide,
carboplatin, and etoposide (ICE) in children with advanced and/or relapsed Wilms' tumor.
Proceedings of the American Society of Clinical Oncology A2157, 559a, 1999.
4. Kung FH, Bernstein ML, Camitta BM, et al.: Ifosfamide/carboplatin/etoposide (ICE) in the
treatment of advanced, recurrent Wilms tumor. Proceedings of the American Society of
Clinical Oncology A2156, 559a, 1999.
5. Garaventa A, Hartmann O, Bernard JL, et al.: Autologous bone marrow transplantation
for pediatric Wilms' tumor: the experience of the European Bone Marrow Transplantation
Solid Tumor Registry. Medical and Pediatric Oncology 22(1): 11-14, 1994.
6. Pein F, Michon J, Valteau-Couanet D, et al.: High-dose melphalan, etoposide, and
carboplatin followed by autologous stem-cell rescue in pediatric high-risk recurrent Wilms'
Tumor: a French Society of Pediatric Oncology study. Journal of Clinical Oncology
16(10): 3295-3301, 1998.
7. Tannous R, Giller R, Holmes E, et al.: Intensive therapy for high risk (HR) relapsed
Wilms' tumor (WT): a CCG-4921/POG-9445 study report. Proceedings of the American
Society of Clinical Oncology 19: A2315, 2000.
25.- Se trata de femenino de 11 años quien es sometida a una prueba cutánea de Mantoux
como parte de una exploración física de control. No ha sufrido exposiciones conocidas a
tuberculosis ni tiene factores de riesgo. A las 48 horas aparece una zona de eritema de 20mm y
un área de induración de 7mm. La interpretación del hallazgo presentado en la paciente es:

a) Es un resultado positivo de exposición a tuberculosis.


b) Es un resultado negativo de exposición a tuberculosis.
c) Conviene hacer una radiografía de tórax.
d) Deben obtenerse lavados gástricos de Mycobacterium tuberculosis.

Para determinar si el hallazgo de una prueba cutánea de Mantoux es positiva o no, se usa la
zona de induración, y no la de eritema. En ausencia de factores de riesgo conocidos o de
exposición una zona de induración menor a 10 mm, es un resultado de Mantoux negativo. No
están indicados ni radiografía de tórax ni lavado gástrico.
Bibliografía:
1. Grupo de trabajo de Tuberculosis de la Sociedad Española de Infectología Pediátrica.
Interpretación de la prueba de tuberculina.An Pediatr (Barc) 2003;59(6):582-5
2. López-Hermosa, P. Papel actual del Mantoux en la población Infantil. Rev.Ped. Atención
Primaria.Vol I. Nº5, Jul/Sep 1999.
3. Alcaide Megías J et al. Epidemiologia de la tuberculosis. An Esp Pediat. 2000;53: 449-457
4. Alonso Moreno FJ et al. Prevalencia de la infección tuberculosa en las personas inmigrantes
del Área de salud de Toledo. Rev Esp Salud pública 2004; 78: 593-600.
5. Rodríguez Valvín E. Situación actual de la tuberculosis en España: incidencia y mortalidad
desde 1995.Características de los casos de tuberculosis y meningitis tuberculosa declarados
en 2000. SEMERGEN 2002; 28(7):395-400.
6. Rodrigo T, Caylà JA. Efectividad de los programas de control de la tuberculosis en España.
Med Clin (Barc) 2003;121(10):375-7.
7. De Paula F et al. Prevalencia de la infección tuberculosa entre los inmigrantes magrebies.
Med Clin (Barc) 2000;114(7):245-9.
8. Fernández Sanfrancisco MT et al. Prevalencia de la infección tuberculosa en la población de
inmigrantes de Ceuta, España. Rev Esp Salud Pública 2001; 75:551-8.

26.- Se trata de femenino de 7 años atendido en consulta por presentar hace 3 días, fiebre de
38.2 ºC, malestar general, congestión nasal, cefalea. Refiere la madre que el día de ayer inició
con erupción cutánea en varias partes del cuerpo. E.F.: To. 38.5 ºC, Fc 100 LPM, Fr 44 RPM, TA
80/50. Se observan lesiones eritematosas, máculas, pápulas y algunas lesiones vesiculares, con
tendencia a la distribución generalizada, huellas de rascado de predomino en pliegues.
El diagnóstico más probable es:

a) Varicela.
b) Erisipela.
c) Sarampion.
d) Escarlatina.

Cuadro clínico
Período de Incubación de 14 -21días (que puede extenderse hasta 28 días en los pacientes que
recibieron inmunización pasiva o en huéspedes inmunocomprometidos) Período prodrómico:
Precede al inicio del rash.
Dura 1 -2 días donde el paciente presenta febrícula o malestar, con un rash atípico.
En la infancia este período puede estar ausente.
Período de estado: Rash generalizado y rápidamente progresivo desde mácula a pápula y luego
a vesícula, seguida de una pústula y finalmente costra. Las lesiones son de 1-4 mm de diámetro.
Un chico sano tiene usualmente entre 200-500 lesiones.
El rash se produce en brotes sucesivos (son 3 brotes generalmente) iniciándose en la cabeza,
luego en el tronco, y de allí se extiende a las extremidades, es de distribución cefalocaudal y
centrífuga. Esto hace que el rash sea polimorfo, encontrándose en una misma zona del cuerpo
lesiones en distintos estadíos (máculas, vesículas y costras por ejemplo). Suele ser muy
pruriginoso.
Se pueden comprometer mucosas (orofaríngea, vagina, conjuntiva).
Puede haber adenopatías regionales. La fiebre, si está presente, suele ser leve y aparece con
los brotes, durando dos a tres días. El adulto, a diferencia del niño suele hacer un cuadro severo
con más incidencia de complicaciones.
En general la infección primaria por VZV, la varicela, deja inmunidad de por vida, pero se pueden
ocasionalmente presentar segundos episodios, sobre todo en huéspedes inmunocomprometidos.

Bibliografía
-Pediatrics 1999;103:783-90; Pediatr Infect Dis J, 2002; 21: 166-168.
- Int j Clin Pract Suppl.2003 Apr; 135:50-3.
- Pediatr Infect Dis J. 2000;19:648-657.
- Pediatrics. 1975;56:388-397.
- J Infect Dis. 1985;6:1172-1181.
- Ann Intern Med. 1992;117:358-363.
- Pickering L,ed 2000 Red book: Report of the committee on Infectious Diseases . 25th
ed.American Academy of Pediatrics,2000.
-Pediatrics 2002; 109:1068-73.
-Nueva forma de tratamiento de varicela zoster en pacientes pediátricos con transplante
hepático. Marcó del pont, Duca, P; De Cicco L et al. - Abstract A8 del 3er Congreso Argentino de
Infectología Pediátrica, Rosario 1999.

27.- Femenino de 25 años con antecedentes de G/2, P/1, C/1 acude al servicio de consulta
externa, refiere que presenta una secreción transvaginal bastante líquida, de baja viscosidad,
maloliente de color amarillo y gris, espumoso. Esta entidad es propia de infección por:

a) Cándida albicans
b) Gardenerella
c) Tricomonas
d) Gonococos

• El protozoario Thricomona Vaginalis es el responsable del 25% de las vaginitis.


• 20-50% de las mujeres cursan asintomáticas.
• La tricominiasis es predominantemente una infección transmitida sexualmente.
• Debe sospecharse de abuso sexual en caso de encontrar Trichomona en pacientes
pediátricos.

• DIAGNÓSTICO:
• Flujo vaginal amarillo-verdoso con burbujas, muy fétido, irritación vulvo-vaginal, disuria.
• El pH suele ser mayor de 4.5.
• Estudio en fresco en donde se observa al microscopio el organismo flagelado
característico.
• 75% se diagnostican con el estudio del Papanicolaou.
Referencias bibliográficas:
1. Secretaría de Salud. Norma Oficial Mexicana NOM -039-SSA2-2002, Para la prevención y
control de las infecciones de transmisión sexual. D.O.F. 19 de Septiembre 2003.
2. Kettler H, White K, Hawkes S. Mapping the landscape for sexually transmitted infections:
key findings and recommendations. Geneva, TDR (TDR/STI/ IDE/04.1).
3. CDC. Trends in Reportable Sexually Transmitted Diseases in the United States. CDC,
National Report. 2004
4. Distribución de los casos nuevos de enfermedades por mes Estados Unidos Mexicanos
2004. Sistema Único de Información para la Vigilancia Epidemiológica/Dirección General
de Epidemiología/SSA
5. Aral S O. Sexual risk behaviour and infection: epidemiological considerations. Sex. Transm.
Inf. 2004;80:8-12

28.- Recién nacido de 38 semanas de gestación en su segundo día de vida, que presenta una
dermatosis diseminada a cara y tronco caracterizada por vesículas y pústulas, que respeta
palmas y plantas. El diagnóstico más probable es:

a) Melanosis pustulosa
b) Dermatitis atópica
c) Rubéola congénita
d) Eritema Tóxico

El eritema tóxico es una entidad sin significado patológico que se manifiesta con vesículas y
pústulas sobre una base eritematosa, respeta palmas y plantas. Aparece entre el primero y
tercer día de vida. Están formadas por un infiltrado de eosinófilos, el cultivo es estéril y
desaparece en la primera semana. En cambio, la melanosis pustulosa se presenta al nacimiento,
se localiza en palmas y plantas, las lesiones están formadas por PMN, el cultivo es estéril y
desaparecen en varias semanas.
Bibliografía

1. Askin DF. Complications in the transition from fetal to neonatal life. J Obstet Gynecol
Neonatal Nurs 2002 May-Jun;31(3):318-27.
2. Bondas-Salonen T. New mothers' experiences of postpartum care--a phenomenological
follow-up study.:J Clin Nurs. 1998 Mar;7(2):165-74.
3. Desmond M. Franklin R, Vallbona C. et al: The clinical behavior of the newly born. Infant.
The term baby. J Pediatr 1965; 62:307.

Desmond M , Rudilph A, Phitaksphraiwan P.: The transitional Care Nursery. 1966 Pediatr Clin
North Am 13:651

29.- Masculino de tres meses de edad el cual desde hace aproximadamente 3 semanas
presenta episodios intermitentes de distensión abdominal, dolores de tipo cólico y algunos
vómitos. Tendencia al estreñimiento. Entre sus antecedentes personales hay que destacar que
fue prematuro, pesó 900gr. al nacimiento y tuvo dificultad respiratoria importante que precisó
ventilación asistida durante 15 días. El diagnóstico más probable en este paciente es:

a) Estenosis cólica secundaria a Enterocolitis necrotizante.


b) Megacolon congénito.
c) Enteritis crónica por rotavirus.
d) Vólvulo intestinal intermitente.

La enterocolitis necrotizante (ECN) es una enfermedad grave que afecta a recién nacidos, en
especial prematuros, con una incidencia y morbimortalidad elevados. Constituye la urgencia
gastrointestinal más frecuente en las UCI neonatales.
Se presenta como un síndrome gastrointestinal y sistémico que comprende síntomas variados y
variables, como distensión e hipersensibilidad abdominal, sangre en heces, intolerancia a la
alimentación, apnea, letargia, y en casos avanzados acidosis, sepsis, CID y shock.
El síndrome clínico ha sido clasificado en estadios por Bell y col. (1978) y modificado por
Walsh y Klegman (1986) para incluir hallazgos sistémicos, intestinales y radiológicos.
 A. Estadio I : sospecha de enterocolitis necrotizante
 Los hallazgos sistémicos son inespecíficos.
 Los hallazgos intestinales incluye el residuo gástrico y heces guayaco –positivas.
 Los hallazgos radiológicos son normales e inespecíficos.
 B. Estadio II A: enterocolitis necrotizante leve
 Los hallazgos sistémicos son similares al estadio I.
 Los hallazgos intestinales incluyen distensión abdominal prominente con
hipersensibilidad a la palpación o sin ella, ruidos hidroaéreos ausentes, sangre macroscópica
en materia fecal.
 Los hallazgos radiológicos, íleo con asas dilatadas con áreas focales de neumatosis
intestinal.
 C. Estadio II B: enterocolitis necrotizante moderada
 Los hallazgos sistémicos incluyen acidosis leve y trombocitopenia
 Los hallazgos intestinales incluyen edema de la pared abdominal e hipersensibilidad a la
palpación con una masa palpable o sin ella.
 Los hallazgos radiológicos incluyen neumatosis extensa y ascitis temprana.
 Puede haber gas en la vena porta intrahepática.
 D. Estadio IIIA: enterocolitis necrotizante avanzada:
 Los hallazgos sistémicos incluyen acidosis respiratoria y metabólica, ventilación asistida
por apnea, hipotensión arterial, oliguria, neutropenia y coagulación intravascular diseminada.
 Los hallazgos intestinales incluyen edema que disemina, eritema e induración del
abdomen.
 Los hallazgos radiológicos incluyen ascitis prominente y asa centinela persistente sin
perforación.
 E. Estadio IIIB: enterocolitis necrotizante avanzada:
 Los hallazgos sistémicos revelan signos vitales e índices de laboratorio en deterioro,
síndrome de shock y desequilibrio electrolítico.
 Los hallazgos intestinales y radiológicos muestran evidencias de perforación.

Hallazgos radiológicos en la radiografía de abdomen


 Distensión abdominal generalizada
 Íleo paralítico
 Neumatosis intestinal quistoide (patognomónico)
La neumatosis intestinal quistoide se la puede visualizar de varias maneras, cúmulos de gas
lineales, curvilíneos, esponjosos y espumosos.
Este último debe distinguirse de materia fecal o meconio mezclado con aire.
Cualquier lactante con sospecha de enterocolitis necrotizante en el que se encuentren
radiográficamente colecciones de aire lineales, curvilíneas esponjosas o espumosas debe
considerarse que tiene neumatosis intestinal quistoide hasta que se demuestre lo
contrario.
La neumatosis intestinal quistoide suele verse con mayor frecuencia en el colon, pero puede
ocurrir desde estomago hasta recto.
Se observa neonato con marcada distensión abdominal, con acidosis metabólica, letargo
y edema de la pared abdominal (a)

1. Hartmann G. E., Drugas G. T., Shochat S. J. Post-necrotizing enterocolitis strictures


presenting with sepsis of perforation: risk of clinical observation. J. Pediatr. Surg. 1988;
23: 562-6.
2. Kosloske A. M., Burstein J., Bartow S. A. Intestinal obstruction due to colonic stricture
following neonatal necrotizing enterocolitis. Ann Surg. 1980 Aug;192 (2): 202-7.
3. Schwartz M. Z., Hayden C. K., Richardson C. J., Tyson K. R., Lobe T. E. A prospective
evaluation of intestinal stenosis following necrotizing enterocolitis. J. Pediatr. Surg. 1982
Dec; 17 (6): 764-70.
4. Bell M. J., Ternberg J. L., Askin F. B. Intestinal stricture in necroting enterocolitis. J.
Pediatr. Surg. 1976; 11: 319-27.
5. Pokorny W. J., Harr V. L., McGill, C. W., et al; Intestinal stenosis resulting from
necrotizing enterocolitis. Am J. Surg 1981 42: 721-724.
6. Schimpl G., Hollwarth M. E., Fotter R., Becker H. Late intestinal strictures following
successful treatment of necrotizing enterocolitis. Acta Paediatr. Suppl. 1994; 396: 80-3.
7. Bütter A., Flageole H., Laberge J. M. The Changing face of Surgical Indication for
Necrotizing Enterocolitis J. Pediatr. Surg. 2002; 37: 469- 499.
8. Gobet R. , Sacher P. , Schwobel M. G. Surgical procedures in colonic strictures after
necrotizing enterocolitis. Acta Paediatr. Suppl. 1994;396:77-9.

30.- Femenino de 4 meses de edad, residente de campo agrícola; con antecedente de anemia,
refiere la madre que actualmente lo encuentra irritable, caliente, no quiere comer, no fija la
mirada, como que no ha orinado bien, indica que ha administrado gotas para los cólicos. E, F.
llama la atención Fc 94x´ y mucosas secas. Por las características clínicas existe alta
probabilidad de intoxicación por:

a) Hierro
b) Ácido acetil salicílico
c) Órgano fosforado
d) Anticolinérgicos

En el mundo existen alrededor de 13 millones de químicos naturales y sintéticos, y menos de


3000 causan el 95% de las intoxicaciones. Un veneno (tóxico) es una substancia capaz de
producir efectos adversos en un organismo viviente. Existen distintos tipos, aquellos de uso
humano (comidas y sus aditivos, medicamentos y cosméticos) y aquellos que no lo son
(productos de limpieza, industriales, químicos, plantas y hongos no comestibles). Una sobredosis
implica exposición a cantidades excesivas de los primeros y a cualquier cantidad de los últimos.

DIAGNÓSTICO

Para diagnosticar una intoxicación es vital tener una historia clínica y examen físico lo más
detallado posible dentro del contexto de urgencia de cada paciente.

Historia

1. Si es posible, nombre y cantidad de cada substancia.


2. Tiempo, ruta, duración y circunstancias de la exposición.
3. Tiempo de inicio, naturaleza y severidad de los síntomas
4. Medidas de ayuda administradas.
5. Historia médica y psiquiátrica, incluyendo medicamentos que el paciente
habitualmente.

Examen físico

1. Signos vitales, signos de estimulación o depresión, Glasgow modificado para


niños.
2. Examen físico, buscando lugar de entrada del tóxico (Ej: punciones venosas,
quemaduras por ácidos o cáusticos) o signos de intoxicación crónica (Ej: líneas
de Mees en el lecho ungueal, en la intoxicación por arsénico).
3. La presencia de ciertos síntomas y signos pueden clasificarse en síndromes
tóxicos, dentro de los cuales los principales están:
o Anticolinérgico: Midriasis, fiebre, íleo, taquicardia, rubor, mucosas
secas, visión borrosa, retención urinaria, mioclonus, psicosis tóxica,
agitación, convulsiones y coma.
Causado por: Atropina, antihistamínicos, fenotiazinas, antidepresivos
tricíclicos, floripondio (planta alucinógena).
Acercamiento terapéutico: Fisostigmina sólo en casos graves con riesgo
vital.
o Colinérgico: Miosis, salivación, epífora, defecación, emesis, bradicardia,
broncoconstricción.
Causado por: Insecticidas organofosforados y carbamatos, pilocarpina.
Acercamiento terapéutico: Atropina, pralidoxima en intoxicación por
organofosforados.
o Extrapiramidal: Coreoatetosis, hiperreflexia, trismus, opistótonos,
rigidez y temblor.
Causado por: Haloperidol, fenotiazinas.
Acercamiento terapéutico: Difenhidramina y benztropina
o Alucinógeno: Alucinaciones, despersonalización, desrealización.
Causado por: Anfetaminas, canabinoides, cocaína, fenciclidina,
alcaloides indol.
Acercamiento terapéutico: Benzodiazepinas.

Bibliografía:

1. Beers, Mark; Berkow, Robert. The Merck Manual, 17th Ed., 1999. Section 23: Poisoning.
2. Isselbacher, Kurt. Harrison´s Principles of Internal Medicine, 13th Ed., 1994. Chapter 395:
Acute poison and drug overdosage.
3. Goldfrank, Lewis et al.; Goldfrank´s Toxicologic Emergencies, 6th Ed., 1998. Chapter 9:
Identifying the Nontoxic Exposure.
Leikin, Jerrold; Paloucek, Frank. Poisoning & Toxicology Compendium 1st Ed., 1998. Approach to
Toxicology.

31.- Se trata de masculino de 32 semanas de gestación que presenta ruptura de membranas


de más de 18 horas de evolución, sin progreso de trabajo de parto por lo que se decide realizar
cesárea, recibe maniobras habituales de reanimación y posteriormente presenta apnea, por lo
que se da presión positiva intermitente (PPI) durante 30 segundos, al evaluarlo presenta pobre
esfuerzo respiratorio por lo que requiere PPI nuevamente por 5 minutos sin presentar esfuerzo
respiratorio efectivo.
El hallazgo radiológico más probable en éste paciente es:

a) Infiltrados cotonosos
b) Rx. Normal
c) Broncograma aéreo, disminución del volumen pulmonar, infiltrado reticulonodular
de diferente grado.
d) Atelectasia apical derecha

SINDROME DE DISTRES RESPIRATORIO POR DEFICIENCIA DE FACTOR SURFACTANTE.


(ENFERMEDAD DE LA MEMBRANA HIALINA)
Hallazgos radiológicos
Pulmones pequeños Patrón granular fino Broncograma aéreo que se extiende a la periferia.
El patrón granular refleja los hallazgos histológicos de dilatación de conductos alveolares y
bronquiolos terminales sobre un colapso alveolar generalizado.
Al sobredistenderse los conductos alveolares y bronquiolos terminales, aparecen burbujas
pequeñas, redondeadas de 1 a 2 mm de diámetro.
Durante la fase espiratoria desaparecen el broncograma aéreo y el patrón granular, viéndose los
pulmones totalmente opacos.
Diagnóstico diferencial.- Pueden verse opacidades pulmonares similares en: Neumonía neonatal,
linfangiectasia pulmonar, síndrome de retención de líquido y cardiopatías congénitas asociadas
con obstrucción venosa pulmonar.
Sin embargo a diferencia de los pacientes con EMH, los volúmenes pulmonares en estas
patologías están normales o aumentados

Clasificación radiológica de la enfermedad de membrana hialina


GRADO I. Granularidad pulmonar fina, broncograma aéreo confinado a los bordes de la silueta
cardiotímica claramente definidos.
GRADO II. Pulmones ligeramente menos radiolúcidos, broncogramas aéreos proyectados por
encima de los bordes cardiotímicos.
GRADO III. Las densidades son más confluentes, broncogramas más extensos.
GRADO IV. Completa opacificación pulmonar, ausencia de broncograma aéreo.
Figura 8. Enfermedad de Membrana Hialina
32.- Masculino de 2 años, se encuentra en sala de espera de consulta con, fiebre, presenta
crisis convulsiva caracterizada por sacudidas de brazos y piernas que duran 3 minutos cede
espontáneamente. Antecedente: cuadro catarral de tres días de evolución. E.F.: FC: 110 LPM.
FR: 28 RPM, TA 85/70, TEMP: 39.0 ºC. PESO 14 KG., actualmente se observa somnolienta pero
se puede despertar, nariz con rinorrea cristalina, oídos normales, faringe hiperémica, buen
esfuerzo respiratorio y buena perfusión.
La acción terapéutica inicial en este caso es:

a) Administrar diacepam vía rectal para mitigar la crisis.


b) Obtener un acceso vascular y pasar líquidos iv.
c) Ceftriaxona I.M.
d) Mantener la vía aérea abierta y administrar oxígeno.

La Crisis Convulsiva (CC) es un frecuente motivo de consulta pediátrica en urgencias que


representa una situación de riesgo inmediato o potencial. La valoración y el tratamiento inicial del
paciente en fase aguda, tiene como objetivos el mantenimiento de la vía aérea, asegurar
oxigenación, ventilación y circulación adecuadas. Simultáneamente se intenta suprimir la
actividad convulsiva, prevenir las recurrencias e identificar y tratar la causa. En el paciente
pediátrico es esencial considerar el origen metabólico (hipoglucemia, alteraciones iónicas y
metabolopatías). Se describe un algoritmo de tratamiento inicial que incluye la administración de
diazepam inicialmente rectal, seguido de intravenoso. Excepto en neonato y lactante pequeño,
como fármaco de segunda línea se propone el uso de Valproato
IV, si no existen contraindicaciones o el paciente estaba ya utilizando éste, empleando
difenilhidantoína en caso de fallo de éste. El uso de midazolam por diversas vías puede ser una
alternativa útil, frente a tiopental en pacientes con crisis rebeldes que no han respondido a
tratamientos previos. Se analiza la presencia de CC en el contexto de fiebre, que obliga a
diferenciar entre procesos benignos propios de la edad pediátrica como las crisis convulsivas
febriles típicas y las crisis febriles atípicas, éstas con mayor posibilidad de estar relacionados
con procesos patológicos. Se describen las características de ambas y se propone un algoritmo
para orientar su manejo adecuado en la urgencia pediátrica. Las crisis comiciales en pacientes
afebriles representan un problema diferente y en pediatría se relacionan generalmente con
desajustes de la medicación en pacientes con antecedentes previos o con procesos de origen
epiléptico que efectúan su comienzo. Sin embargo, en estos pacientes es imprescindible
descartar la presencia de intoxicación y lesiones estructurales mediante una cuidadosa
exploración y pruebas de imagen inmediatas o diferidas según los casos.

En la crisis
ABC (Oxigeno y vía venosa)
Anticonvulsivantes IV o rectal (Diazepam 0.3-0.5 mg/kg), Lorazepam 0.05 mg/kg, menor
depresión respiratoria y acción prolongada
Uso de Diazepam rectal en domicilio (disminuir riesgo de crisis prolongada)
Actuación en urgencias ante una crisis convulsiva en niños
S. García García1, M. Rubio Sánchez-Tirado2, F. Ruza Tarrio3
1UNIDAD DE URGENCIAS PEDIÁTRICAS. HOSPITAL UNIVERSITARIO LA PAZ. MADRID.
2CENTRO COORDINADOR. SUMMA 112.
COMUNIDAD AUTÓNOMA DE MADRID. 3SERVICIO CUIDADOS INTENSIVOS
PEDIÁTRICOS. HOSPITAL UNIVERSITARIO LA PAZ. MADRID.

33.- Lactante menor próximo a cumplir 2 meses, atendido en consulta para control de niño sano.
Se refiere asintomático. E.F.: dentro de límites normales. Antecedente: esquema de vacunación
completo para la edad.
De acuerdo a la cartilla nacional de vacunación, que vacunas ya recibió:

a) Polio y bcg.
b) Hepatitis b y bcg.
c) Solo bcg.
d) Polio y hepatitis b.
34.- Masculino de 7 años, ingresa al servicio de urgencias, refiere la madre que la menor ha
presentado tos, ardor retroesternal y silbido del pecho. . Tiene tos irritativa, no productiva,
hipertermia no cuantificada. Antecedentes: Originaria de Veracruz, nivel socioeconómico bajo,
habita en un rancho. Malos hábitos de higiene. Exploración física: febril 39 °c, con dificultad para
respirar, aleteo nasal, tiraje intercostal y subcostal. Laboratorio: eosinofilia marcada. Radiografía
de tórax muestra infiltrados redondos u ovalados de unos milímetros hasta varios cm. de
diámetro, bilateral.

El probable diagnóstico de este paciente es síndrome de:

a) Atelectasia
b) Loeffer.
c) Derrame pleural.
d) Rarefaccion pulmonar.

En 1932 Loeffler describe un síndrome caracterizado por sintomatología respiratoria compatible


con "pulmón sibilante", radiografía de tórax con infiltrado intersticial bilateral, difuso, abigarrado y
transitorio compatible con neumonitis asociada de modo obligatorio a eosinofilia periférica. Este
síndrome clásicamente se presenta cuando existe migración parasitaria alveolo capilar pulmonar
y es causado por larvas de parásitos con ciclo hístico tisular como Ascaris l, Strongyloides s,
Ancylostoma duodenale, Necator americanus y Filarias (1-2-3) excepcionalmente está asociado
al síndrome de larva migrans cutánea (1).
La invasión por larvas de helmintos puede originar infiltración pulmonar, con marcada dificultad
respiratoria por el espasmo bronco-alveolar como respuesta a la invasión parasitaria (pulmón
sibilante), cuadro pulmonar inflamatorio transitorio, con respuesta alérgica local que explica y
exige para su diagnóstico, una intensa eosinofilía periférica(5-6-7) Este síndrome es más
frecuente en personas que se infectan por primera vez o que viven en zonas no endémicas; por
hipersensibilidad a las larvas presentándose como una neumonía eosinofílica, aguda, benigna y
con infiltrados pulmonares migratorios y transitorios(7), no existiendo restricción para ninguna
edad, describiéndose aun en neonatos ya desde el año
2001 con cuadro clínico similar al de niños de mayor edad (4).

Cuadro I. Principales causas de eosinofilia


Parasitosis
Helmintiasis con fase tisular
Infecciones micóticas
Coccidioidomicosis
Enfermedades alérgicas
Asma bronquial
Rinitis alérgica
Alergia a medicamentos
Edema angioneurótico
Fiebre del heno
Neoplasias
Enfermedad de Hodking
Carcinomas
Sarcomas
Tumores óseos
Tumores de ovario
Leucemia mieloide crónica
Leucemia eosinofílica
Enfermedades gastrointestinales
Gastritis eosinofílica
Enfermedades inmunológicas
Artritis reumatoide
Vasculitis
Síndrome de Wiskott-Aldrich
Síndrome de Hiper-IgE
Déficit selectivo de IgA
Injerto contra huésped
Enfermedades respiratorias
Aspergilosis broncopulmonar alérgica
Sinusitis aspergilar alérgica
Neumonía crónica eosinófila
Intoxicaciones
Intoxicación con fósforo
Inhalación de humos
Enfermedades dermatológicas
Urticaria aguda
Pénfigo
Penfigoide ampolloso
Síndrome de Wells
Dermatitis atópica
Hiperplasia angiolinfoide con eosinofilia
Herpes gestationis
Convalecencia de infecciones
Escarlatina
Fiebre reumática
Otros
Eosinofilia post-irradiación
Síndrome hipereosinofílico
Los hallazgos radiológicos en los síndromes PIE generalmente son inespecíficos, los infiltrados
en la radiografía (Rx) tórax pueden ser intersticiales, alveolares o mixtos, habitualmente son
bilaterales y difusos. Radiológicamente la eosinofilia inducida por parásitos se manifiesta
por infiltrados fugaces y migratorios. En la NEC se observa la clásica imagen de “fotografía
negativa de edema pulmonar”, caracterizada por compromiso de las zonas apicales y periféricas,
con indemnidad de las zonas centrales e inferiores. La Rx tórax en la ABPA muestra infiltrados
fugaces, mediante la tomografía computada (TC) de tórax se pueden evidenciar bronquiectasias
centrales. En la NEA puede encontrarse un derrame pleural de escasa cuantía, que contiene un
alto porcentaje de eosinófilos. La imagenología además es útil en determinar la extensión del
compromiso pulmonar, elegir los sitios óptimos para obtener la biopsia pulmonar y la respuesta
al tratamiento.

REFERENCIAS
1.- Del Giudice P, Desalvador F, Bernard E, Caumes E, Vandenbos F, et al. 2002. Löffler's
syndrome and cutaneous larva migrans: a rare association. British J Dermatol 2002; 147: 385- 7
2. Botero D, Restrepo M. Parasitosis Humanas. 3° Ed., Medellín Colombia: Corporación para
Investigaciones
Biológicas 1998.
3.- Noemí I, Atias A. Eosinofilia y parasitosis. En Atias A. Parasitologia Médica. Mediterraneo,
Santiago-Chile.
2000.
4.- Fujimura J, Murakami Y, Tsuda A, Chiba T, Migita M, Fukunaga Y. 2001.A neonate with
Loeffler syndrome.Journal
of Perinatology 2001; 21: 207-8
5.- Hunninghake GW, RichersonHB. Neumonitis por hipersensibilidad y neumonias eosinófilas.
En Harrison TR.
Principios de Medicina Interna. 15°Ed. McGraw-Hill -Interamericana de España, Madrid- España.
2001
6.- Chusid MJ. Eosinophilia in childhood. Immunol and Allergy Clinics North America 1999; 19:
327-46
7.- Rothenberg ME. Eosinophilia N Engl J Med 1998; 338: 1592
8.- Noemi IH. Eosinofilia y parasitosis. Rev. Chil. Pediatr. 1999; 70: 1-7
9.- Hotez PJ, Broker S, Bethony JM, Bottazzi ME, Loukas A. Hookworm Infection. N Engl. J Med
2004; 351(8):
799-808
10.- Yilderan A, Ikinciogullari A. In the light of recent advances: eosinophil, eosinophilia and
idiopathic hypereosinophilic syndrome. Turk Haematol 2005; 22(3): 107-16

35.- Masculino de 2 meses y medio. Atendido en consulta por ictericia que inició a los 16 días
de vida y ha sido progresiva, asociada a falta de pigmentación de las evacuaciones.
Antecedentes: producto de gesta 1, obtenido a término sin complicaciones perinatales.
Exploración física: Se aprecia ictericia generalizada y hepatomegalia de consistencia dura. Se
corrobora acolia.
El diagnóstico más probable es:

a) Hepatitis a.
b) Atresia de vías biliares.
c) Quiste de coledoco.
d) Galactosemia.

Clínica
La atresia de vías biliares (AVB) se presenta como la principal causa de colestasia neonatal,
tanto en series nacionales como internacionales (9), sin embargo su diagnóstico no es fácil. Esto
dado, entre otras cosas, por la alta incidencia de ictericia fisiológica, siendo ésta, algunas veces,
sobre-diagnosticada en desmedro de patologías como la AVB.
Sin embargo, existen diferencias sustanciales en la forma de presentación de ella que deben
tenerse siempre en consideración, con el fin de realizar el diagnóstico y tratamiento en forma
oportuna y favorece el buen pronóstico.
La edad de diagnóstico de la AVB varía en entre los 60 y 120 días según las distintas series (10,
11).
Lo más frecuente es que se manifieste con la presencia de ictericia tardía, generalmente
después de las dos semanas de vida (12), razón por la cual no debería considerarse como
fisiológica.
Además, se puede acompañar de signos de obstrucción de la vía biliar tales como acolia y
coluria. Si bien estos últimos pueden presentarse desde el nacimiento, lo más frecuente es que
se observen desde la segunda semana de vida.
Al examen físico, la hepatomegalia constituye el signo más precoz y constante
(1), junto con la consistencia dura y firme del hígado. En el caso de la AVB asociada a otras
malformaciones, como la poliesplenia, se puede pesquisar esplenomegalia al examen, siendo
ésta, un signo de hipertensión portal de aparición más tardía.
Si el diagnóstico y restablecimiento del flujo biliar no es precoz, se hace presente una marcada
disfunción hepática y progresión a cirrosis. En esta etapa destacan el retraso del crecimiento
pondoestatural y la tendencia a una coloración ictérico-verdínica. Pueden además presentar
prurito, a veces refractario a tratamiento convencional.

Atresia de vías biliares en pediatría:


Una Revisión de la Literatura
Biliary Atresia in pediatrics: A Review of the Literature
Javiera Benavides T. 1, Carolina Espinoza G. 1, Nicolás Pereira C. 1 y Carmen Gloria Rostion A.
21 Chile

36.- Infant, 33 weeks of gestation presenting in the first day of life, tachypnea, xiphoid
retraction, intercostal retractions, nasal flaring, deep moan and cyanosis. On physical
examination, crackles are audible bibasilar. The chest radiograph shows reticulonodular
infiltrates with air bronchogram. What is the most likely diagnosis?

a) Pneumonia
b) hyaline membrane disease.
c) Transient tachypnea of the newborn.
d) meconium aspiration syndrome.

La enfermedad de membrana hialina se debe a un déficit de surfactante, afecta a recién


nacidos prematuros, su frecuencia aumenta en hijos de madres diabéticas y en embarazos
múltiples. El cuadro clínico se caracteriza por datos de dificultad respiratoria de inicio precoz
como: taquipnea, quejido intenso, aleteo nasal, retracciones xifoideas e inter y subcostales y
cianosis parcialmente refractaria al oxígeno. Los síntomas progresan hasta alcanzar un
máximo al tercer día. En la auscultación aparecen crepitantes en ambas bases. En la
radiografía de tórax se presenta un infiltrado reticulonodular con broncograma aéreo en unos
pulmones poco ventilados, se pueden presentar atelectasias. En la gasometría se observa
hipoxemia, hipercapnia y acidosis respiratoria. El tratamiento consiste en soporte respiratorio,
administración endotraqueal de surfactante y antibióticos. Por otro lado, la taquipnea transitoria
del recién nacido se presenta en recién nacidos de término, que nacen por cesárea o parto
vaginal rápido, debido a un retraso en la absoprción del líquido; manifestándose con un distrés
respiratorio de inicio precoz, la auscultación es normal.

Manual CTO pediatría, 7° edición p. 1302.


Guía clínica SDR neonatalMarzo 2006

37.- Se trata de masculino R/N prematuro 33 sdg, hijo de madre de 29 años de edad gesta 2,
con control prenatal regular, diabética controlada y nace por parto. Apgar 8/9. Inicia con
dificultad respiratoria la cual se presenta progresiva. Peso 1.900KG.
El tratamiento indicado para éste paciente es:

a) Surfactante de rescate
b) Surfactante profiláctico
c) Toracocentesis
d) Antibióticos

ENFERMEDAD DE MEMBRANA HIALINA


Definición:
El Síndrome de distress respiratorio idiopático (SDRI), conocido también como enfermedad
de membrana hialina, se origina en la deficiencia de surfactante pulmonar. Éste último,
mezcla de fosfolípidos, principalmente dipalmitoil fosfatidil colina, es el responsable de la
estabilización distal del alvéolo a volúmenes pulmonares bajos al final de la espiración,
gracias a que reduce la tensión superficial. Cuando existe déficit de surfactante, el recién
nacido puede no ser capaz de generar el aumento de la presión inspiratoria requerido para
insuflar las unidades alveolares, lo que resulta en el desarrollo de atelectasia progresiva (1).
EMH
 Conjunto de signos y síntomas clínicos, radiológicos y gasométricos atribuibles a la
deficiencia o ausencia de surfactante pulmonar .
 Incidencia inversamente proporcional con edad gestacional (60% a 80% en menores de
28 SDG, 15% A 30% de 32 a 36 SDG, 5% de 36 a 37 SDG)
MAYOR RIESGO:
 Hijo de madre diabética sin compromiso vascular (macrosómico)

 Asfixia (induce vasoconstricción pulmonar, persistencia de circulación fetal)

 Hermanos afectados previamente

Fisiopatología:
La hipoxemia se produce inicialmente por una alteración de la relación ventilación perfusión
debido al colapso difuso del pulmón, contribuyendo posteriormente al desarrollo de shunts
intra y extrapulmonares.
El déficit de surfactante también origina inflamación pulmonar y daño del epitelio respiratorio
conduciendo a edema pulmonar y aumento de la resistencia de la vía aérea. Esto último
contribuye al daño pulmonar con mayor deterioro de la función pulmonar. Paralelamente, el
pulmón dañado disminuye su capacidad de reabsorción de líquido, la que resulta
ineficiente, contribuyendo al edema pulmonar. Finalmente, éste último puede estar,
además, exacerbado por las respuestas sistémicas al síndrome de distress respiratorio que
contribuyen a la retención de líquidos.

El déficit de surfactante y el edema pulmonar conducen a anormalidades en la función


pulmonar que llevan a hipoxemia. Las anormalidades primarias en la mecánica pulmonar
son la disminución de la compliance y la disminución del volumen pulmonar que se refleja
en la disminución de la capacidad residual funcional.

Cuadro clínico:
Los recién nacidos con este síndrome casi siempre son prematuros. Las manifestaciones
clínicas son el resultado de la función pulmonar anormal y la hipoxemia. El síndrome de
dificultad respiratoria y la cianosis se presentan rápidamente después del nacimiento. Los
recién nacidos afectados presentan taquipnea y distintos grados de dificultad respiratoria.
La taquipnea, al acortar el tiempo espiratorio, reduce la pérdida del volumen pulmonar en el
pulmón con déficit de surfactante. Otros signos típicos son el quejido, un esfuerzo
compensatorio para prevenir el colapso alveolar al final de la espiración; aleteo nasal que
reduce la resistencia nasal y refleja la utilización de musculatura respiratoria accesoria; y
retracción esternal, subcostal e intercostal debido a la disminución de la compliance
pulmonar asociado a una pared torácica muy complaciente. A la auscultación, los sonidos
respiratorios estarán disminuidos. Los recién nacidos aparecerán pálidos con disminución
de los pulsos periféricos. La diuresis habitualmente es baja durante las primeras 24 a 48
horas y es común el edema.

Historia natural:
Dado que corresponde a un trastorno primario del desarrollo de la producción de
surfactante, el síndrome de dificultad respiratoria se presenta típicamente al nacer. Si no se
trata empeora progresivamente durante las primeras 48 horas de vida. En algunos casos,
los recién nacidos pueden no manifestar enfermedad inmediatamente después de nacer y
desarrollan dificultad respiratoria y cianosis dentro de las primeras horas de vida. Estos
recién nacidos pueden tener una cantidad límite de surfactante pulmonar que se consume
o se inactiva rápidamente. La historia natural de la enfermedad se modifica enormemente
por el tratamiento con surfactante exógeno. Antes del uso de surfactante, la enfermedad de
membrana hialina no complicada progresaba típicamente durante las primeras 48 a 72
horas, lo que iba seguido de una mejoría de la función respiratoria asociada a la producción
de surfactante endógeno para resolverse alrededor de la semana de vida. La mejoría era
precedida de una diuresis marcada. La administración de surfactante generalmente acorta
el curso clínico de la enfermedad.
Tratamiento:

 El objetivo es ventilar los pulmones atelectasiados y mantener las medidas generales


adecuadas hasta su resolución

MEDIDAS GENERALES
 Colocar en incubadora o cuna de calor radiante. Monitorear con oximetría de pulso
 Control estricto de líquidos y glucosa (60-80 ml/k/d, glucosa al 10%, incrementar a 120-
160 ml/kg/d para el día 5 de vida), muy prematuros pueden requerir hasta 200 ml/k/d
 Manipulación mínima
MANEJO DE LA VÍA AEREA
 Posición de olfateo
 Posición prona mejora el trabajo ventilatorio
 Aspiración gentil de secreciones en fase exudativa (inicia a las 48 horas de vida)
ADMINISTRACION DE OXIGENO
 Debe administrarse humidificado, caliente. Se puede administrar en casco cefálico,
mascarilla, puntas nasales, o cánula orotraqueal.
 Mantener PaO2 entre 50 y 80 Torr y/o saturaciones entre 85% y 95% PCO2 entre 40 y
55 Torr

MENEJO NO VENTILATORIO

 Catéteres umbilicales
 Hematocrito
 Acidosis
 Alimentación
 Infección
 Aminofilina
 Indometacina
MANEJO VENTILATORIO

 Casco cefálico FIO2- 88% y 96%


 CPAP si FIO2 mayor de 30% tomar gasometría y RX de tórax
 Si >1,500g evaluar 30 min. Tomar RX y gasometría
CONSIDERAR INTUBACION
 Hipercarbia (PaCo2 > 55 Torr)
 Hipoxemia (PaO2 < 50 Torr)
 Pobre esfuerzo ventilatorio o apnea
 Incremento de la dificultad respiratoria
 Plan de administrar surfactante
 Acidosis metabólica persistente (<7,25)

ESTRATEGIA VENTILATORIA MANEJO CON SIMV

 PEEP entre 4 a 6 cm H2O


 Tiempo Inspiratorio entre 0.25 y 0.35 segundos
 PIM adecuada para asegurar un volumen corriente entre 4 y 7 ml/kg
 Ajustar los ciclados para mantener la PaCo2 en límites fijados, no exceder 60 ciclados
por minuto
 Flujo necesario para llevar al PIM fijado

SUFRACTACTE.
Existen dos tipos de surfactantes:
1) El derivado de animales o “natural”
2) Los sintéticos

 SURFACTANTES NATURALES
Pueden obtenerse de pulmones porcinos o bovinos. Los bovinos son: BERACTANT
(SURVANTA), SURFACTANT TA (SURFACTEN), CALFACTAN T (INFASURF), SF-RI1
(ALVEOFACT). Los porcinos son: PORACTANT (CUROSURF)

 SURFACTANTES SINTÉTICOS
COLFOSCERIL (EXOSURF), COMPONENTE ARTIFICIAL DE EXPANSIÓN PULMONAR
(PULMACTANT), LUCINACTANT

38.- Se trata de recién nacido pretermino de 30 semanas de gestación, peso de 1200 gr. Cursó
con enfermedad de membranas hialinas por lo que se manejó con una dosis de surfactante y
manejo con ventilación convencional por 3 días. Posteriormente inició con alimentación enteral
por sonda orogástrica con leche humana, sin tolerancia a la misma (residuo gástrico y distensión
abdominal) existe la sospecha diagnóstica de enterocolitis necrotizante. Los signos y síntomas
esperados son:

a) Sangre macroscópica en heces, cólico intestinal, diarrea


b) Sangre microscópica en heces, ictericia, neumatosis intestinal
c) Distensión abdominal, doble burbuja gástrica
d) Distensión abdominal, radiografía normal

La enterocolitis necrotizante (ECN) es una enfermedad grave que afecta a recién nacidos, en
especial prematuros, con una incidencia y morbimortalidad elevados. Constituye la urgencia
gastrointestinal más frecuente en las UCI neonatales.
Se presenta como un síndrome gastrointestinal y sistémico que comprende síntomas variados y
variables, como distensión e hipersensibilidad abdominal, sangre en heces, intolerancia a la
alimentación, apnea, letargia, y en casos avanzados acidosis, sepsis, CID y shock.

El síndrome clínico ha sido clasificado en estadios por Bell y col. (1978) y modificado por
Walsh y Klegman (1986) para incluir hallazgos sistémicos, intestinales y radiológicos.
 A. Estadio I : sospecha de enterocolitis necrotizante
 Los hallazgos sistémicos son inespecíficos.
 Los hallazgos intestinales incluye el residuo gástrico y heces guayaco –positivas.
 Los hallazgos radiológicos son normales e inespecíficos.
 B. Estadio II A: enterocolitis necrotizante leve
 Los hallazgos sistémicos son similares al estadio I.
 Los hallazgos intestinales incluyen distensión abdominal prominente con
hipersensibilidad a la palpación o sin ella, ruidos hidroaéreos ausentes, sangre macroscópica
en materia fecal.
 Los hallazgos radiológicos, íleo con asas dilatadas con áreas focales de neumatosis
intestinal.
 C. Estadio II B: enterocolitis necrotizante moderada
 Los hallazgos sistémicos incluyen acidosis leve y trombocitopenia
 Los hallazgos intestinales incluyen edema de la pared abdominal e hipersensibilidad a la
palpación con una masa palpable o sin ella.
 Los hallazgos radiológicos incluyen neumatosis extensa y ascitis temprana.
 Puede haber gas en la vena porta intrahepática.
 D. Estadio IIIA: enterocolitis necrotizante avanzada:
 Los hallazgos sistémicos incluyen acidosis respiratoria y metabólica, ventilación asistida
por apnea, hipotensión arterial, oliguria, neutropenia y coagulación intravascular diseminada.
 Los hallazgos intestinales incluyen edema que disemina, eritema e induración del
abdomen.
 Los hallazgos radiológicos incluyen ascitis prominente y asa centinela persistente sin
perforación.
 E. Estadio IIIB: enterocolitis necrotizante avanzada:
 Los hallazgos sistémicos revelan signos vitales e índices de laboratorio en deterioro,
síndrome de shock y desequilibrio electrolítico.
 Los hallazgos intestinales y radiológicos muestran evidencias de perforación.

Hallazgos radiológicos en la radiografía de abdomen


 Distensión abdominal generalizada
 Íleo paralítico
 Neumatosis intestinal quistoide (patognomónico)
La neumatosis intestinal quistoide se la puede visualizar de varias maneras, cúmulos de gas
lineales, curvilíneos, esponjosos y espumosos.
Este último debe distinguirse de materia fecal o meconio mezclado con aire.
Cualquier lactante con sospecha de enterocolitis necrotizante en el que se encuentren
radiográficamente colecciones de aire lineales, curvilíneas esponjosas o espumosas debe
considerarse que tiene neumatosis intestinal quistoide hasta que se demuestre lo
contrario.
La neumatosis intestinal quistoide suele verse con mayor frecuencia en el colon, pero puede
ocurrir desde estomago hasta recto.

Hartmann G. E., Drugas G. T., Shochat S. J. Post-necrotizing enterocolitis strictures


presenting with sepsis of perforation: risk of clinical observation. J. Pediatr. Surg. 1988; 23:
562-6.

Kosloske A. M., Burstein J., Bartow S. A. Intestinal obstruction due to colonic stricture
following neonatal necrotizing enterocolitis. Ann Surg. 1980 Aug;192 (2): 202-7.

Schwartz M. Z., Hayden C. K., Richardson C. J., Tyson K. R., Lobe T. E. A prospective
evaluation of intestinal stenosis following necrotizing enterocolitis. J. Pediatr. Surg. 1982 Dec;
17 (6): 764-70.
Bell M. J., Ternberg J. L., Askin F. B. Intestinal stricture in necroting enterocolitis. J. Pediatr.
Surg. 1976; 11: 319-27.

Pokorny W. J., Harr V. L., McGill, C. W., et al; Intestinal stenosis resulting from necrotizing
enterocolitis. Am J. Surg 1981 42: 721-724.

Schimpl G., Hollwarth M. E., Fotter R., Becker H. Late intestinal strictures following
successful treatment of necrotizing enterocolitis. Acta Paediatr. Suppl. 1994; 396: 80-3.

Bütter A., Flageole H., Laberge J. M. The Changing face of Surgical Indication for Necrotizing
Enterocolitis J. Pediatr. Surg. 2002; 37: 469- 499.

Gobet R. , Sacher P. , Schwobel M. G. Surgical procedures in colonic strictures after necrotizing


enterocolitis. Acta Paediatr. Suppl. 1994;396:77-9.

39.- Femenino de 20 años, atendida en sala de urgencias ginecoobstétricas, Antecedente:


cursa embarazo de 38 SDG. Exploración Física: en trabajo de parto. Repentinamente presenta
sangrado profuso transvaginal y dolor abdominal. Se sospecha diagnóstico de abruptio
placentae. El factor de riesgo más frecuente de ésta patología es:

a) Edad materna
b) Elevada paridad
c) Trauma abdominal
d) Hipertensión materna

Se han sugerido numerosos factores que desempeñan un papel causal en el abruptio


placentae, pero no existe una explicación etiológica satisfactoria para cada uno de ellos:
• Traumatismo.
• Malformación o tumoración uterina.
• Brevedad de cordón umbilical (menor de 20 cm).
• Descompresión brusca del útero (RPM, expulsión de un 1º gemelo).
• Compresión de la vena cava inferior (sumamente raro y no demostrado).
• Hipertensión materna: más del 50% de los casos de desprendimientos están asociados a
HTA.
• Deficiencia de ácido fólico. No demostrado.
• Tabaquismo. Asociado por la necrosis de vasos deciduales (anomalías deciduales).
• Paridad y edad materna. En general la asociación de mayor edad y paridad, es no
demostrable,
No se puede descartar, que la paciente, por lógica, que sufre un desprendimiento,
generalmente
Es mayor de 20 años (no excluyente) y multípara. Sin embargo, si hay relación en que la
paciente que sufrió un DPPNI, tiene 5 veces más probabilidades de sufrir otro DPPNI,
independientemente de la edad y paridad.

40.- Masculino de 9 años que acude al servicio refiriendo dolor inguinal y escrotal intenso, que
comenzó de forma brusca posterior a realizar actividad física. El niño está afectado y se observa
tumefacción y dolor en el hemiescroto derecho, no se detecta reflejo cremastérico y el testículo
derecho está elevado y rotado. De los siguientes tratamientos, el más adecuado para este
paciente es:

a) Analgésicos y antibióticos por vía endovenosa.


b) Tratamiento analgésico ambulatorio.
c) Exploración quirúrgica inmediata.
d) Ingreso hospitalario para observación clínica.

Torsión testicular
Aunque la torsión testicular puede ocurrir a cualquier edad hay dos picos de incidencia:
• En el período neonatal: suele tratarse de un edema escrotal asintomático, ya que la torsión
sucede intraútero y sólo veremos los cambios posteriores: aumento del tamaño del teste, que se
ve más oscuro, e hidrocele reactivo.
• En las primeras fases de la pubertad (edad media 14 años). Se debe a la fijación anormal de
los testículos a la cubierta escrotal, pudiendo rotar libremente dentro de la serosa, sufriendo una
torsión, con la isquemia vascular consiguiente.
Clínica
• Dolor testicular súbito e intenso, acudiendo al pediatra habitualmente en las primeras 12 horas.
Con frecuencia se irradia al abdomen.
• Suele asociar naúseas o vómitos.
• En ocasiones está precedida de un traumatismo leve.
Exploración física
• El teste se suele ver más elevado y horizontalizado.
• Reflejo cremastérico. Servirá de comparación el obtenido previamente en el lado sano. Para
ello se roza suavemente la cara interna del muslo superior, constatando la existencia de un
movimiento de ascenso del teste dentro de la bolsa escrotal. En la torsión suele estar ausente,
aunque no es patognomónico: también puede faltar en las hernias inguinales, hidroceles, orquitis
o tumores. Sin embargo, su presencia casi descarta dicho diagnóstico (aunque puede verse en
las torsiones parciales o en torsión de poco tiempo de evolución).

Se debe palpar el teste y el epidídimo normal, y pasar después al afecto. El dolor testicular es
muy intenso, en todo el teste, aumentando al elevarlo. En ocasiones puede haber dolor
abdominal
infraumbilical referido. El hemiescroto, sólo en un tercio de los casos, puede estar edematoso o
hiperémico.
• Transiluminación: se observa escaso líquido libre, con el teste aumentado de tamaño y de
densidad respecto al contralateral.
• Se ha comunicado la existencia de torsión en testes no descendidos, por lo que esta entidad
debe ser sospechada en aquellos dolores abdominales bajos de inicio súbito y de intensidad
elevada.
• Siempre explorar el abdomen. En ocasiones, el dolor testicular puede ser un dolor referido de
otras localizaciones (apendicitis, cólico nefrítico, etc.).
Diferencial
• Otros procesos: hidrocele (poco doloroso, transiluminación demostrando líquido libre); hernias
inguinoescrotales (presencia de bultoma inguinoescrotal reducible); traumatismos; púrpura de
Schölein-
Henoch, varicocele, etc.
Exploraciones complementarias
• Diagnóstico clínico claro: exploración quirúrgica inmediata.
• Diagnóstico clínico no claro: sobre todo si han transcurrido más de
12 horas, está indicada la realización de ecoDoppler-color (sensibilidad
100% y especificidad del 97%): disminución o ausencia de flujo arterial en los casos de torsión
testicular comparado con el teste sano. Aunque en prepúberes el flujo testicular es pequeño y
puede no verse claramente en la ecoDoppler, por lo que en ausencia de flujo visible en el
testículo no doloroso, no podemos confirmar ni descartar la torsión. En estos casos, y sobre todo
si la clínica es muy sugestiva, se hará una exploración quirúrgica.
En torsiones intermitentes o si se ha destorsionado espontáneamente, la ecografía puede
mostrar un flujo aumentado, normal o disminuido.
En las torsiones evolucionadas se puede ver el signo del «halo» (aumento de la vascularización
y flujo alrededor del testículo).
• Otra prueba complementaria útil es la gammagrafía con Tc-99, pero no es superior a la
ecoDoppler.
Tratamiento
Es una urgencia quirúrgica, por cuanto las posibilidades de supervivencia del teste afectado son
del 90% en las primeras 6 horas, disminuyendo rápidamente con posterioridad (menos del 20%
tras 12 horas de evolución). Se procederá a la extirpación u orquidopexia, según la viabilidad del
teste. Es esencial la exploración y fijación del escroto contralateral, ya que la afectación es
bilateral hasta en el 40%. Tras un episodio de torsión testicular, la función endocrina testicular
(formación de testosterona) no se ve afectada. Sin embargo sí se han encontrado alteraciones
en la función exocrina (espermatogénesis) del teste contralateral.

BIBLIOGRAFIA
1. Dogra V, Gottlieb R, Oka M, Rubens D. Sonography of the scrotum. Radiology
2003; 227: 18-36.
2. Jefferson RH, Perez LM, Joseph DB. Critical analysis of the clinical presentation of acute
scrotum: a 9 year experience at a single institution. J Uro 1997; 158: 1198-1200.
3. Kadish HA, Bolte RG. Revisión retrospectiva de pacientes pediátricos con epididimitis, torsión
testicular y torsión de las hidátides. Pediatrics (ed esp) 1998; vol 46 (1):35-39.
4. Kass EJ, Lundak B. In: Pediatric Urology. The acute scrotum. Pediatrics Clinics of North
America. October 1997, Vol 44 (5): 1251-66.
5. Nussbaum Blask A, Bulas D, Shalaby-Rana E, Rushton G, Shao C, Madj M. Color Doppler
sonography and scintigraphy of the testis: a prospective, comparative analysis in children with
acute scrotal pain. Ped Emerg Care 2002; 18(2): 67-71.
6. Perron C. Pain scrotal. Synopsis of Pediatric Emergency Medicine. Fleisher G, Ludhig S,
Silverman B. 4th ed. 2002; 187-191.

41.- Masculino de 5 años no inmunizado acude a consulta con una historia de 2 semanas de
evolución de tos paroxística, fiebre de bajo grado, emesis post-tos y descarga nasal viscosa. El
EF revela otitis media bilateral y conjuntivitis hemorrágica. Se auscultan estertores inspiratorios
bilateralmente. BH con LT 45 000, con 95% de linfocitos. El l diagnóstico más probable de éste
paciente es:

a) Neumonía por Chlamydia


b) Tosferina
c) Bronquiolitis
d) Neumonitis por VSR

La tos ferina es una enfermedad infecciosa altamente contagiosa causad por un bacilo gram (-)
llamado Bordetella pertussis.
El comienzo suele ser insidioso con una fase catarral, con tos irritante que poco a poco se vuelve
paroxística, por lo regular en el término de una a dos semanas, y que dura de uno a dos meses o
más. Los paroxismos se caracterizan por accesos repetidos y violentos de tos; cada serie de
ellos comprende innumerables toses sin inspiración intermedia y puede ser seguida por un
estridor respiratorio de tono alto característico. Los paroxismos con frecuencia culminan con la
expulsión de mucosidades claras y adherentes, a menudo seguida de vómito. Los lactantes
menores de 6 meses de edad, los adolescentes y los adultos frecuentemente no tienen el cuadro
típico de estridores o tos paroxística.
Resumen:
Etiología: Bordetella pertussis.
Otros: Bordetella parapertussis, Bordetella bronchiseptica, Mycoplasma pneumoniae, Chlamydia
trachomatis, Chlamydia pnuemoniae y adenovirus.
Fuentes de contagio adolescentes y adultos.
Infección intradocimiciliaria en el 80 % de los no vacunados.
Contagio: Fase catarral y hasta 2 semanas de iniciada la tos.
Periodo de incubación de 10 a 21 días.
Cultivo nasofaríngeo Bordet-Gengou (Dacrón o alginato de calcio).
Negativo en fase temprana, > 4 semanas sin estaba vacunada y > 5 días si recibió tratamiento.
Leucocitosis con linfocitosis absoluta.

Tratamiento:
• Apoyo: Apnea, hipoxia y otras complicaciones.
• Antibiótico en fase catarral es efectivo.
• Frenar la propagación del germen.
• Eritromicina 40-50 mgkgd cada 6 hrs. x 14 días.
• Claritromicina 15-20 mgkgd cada 12hrs. x 7 días.
• Azitromicina 10-12 mgkgd cada 24 hrs. x 5 días.
• TMP SMZ 8 mgkgd cada 12 hrs. x 14 días.

Bibliografía
• Abul K. Abbas, Andrew H. Lichtman, Jordan S. Pober. Inmunología Celular y Molecular.
McGrall-Hill Interamericana. Cuarta edición 2001.
• Napoleón González Saldaña y Mercedes Macías Parra. Vacunas en Pediatría. McGrall-
Hill
Interamericana. Primera edición 1999.
• Report of the Committee on Infectious Diseases. American Academy of Pediatrics. Red
Book 2000. 25 edición 2000.
• Stanley A. Plotkin, Walter A. Orenstein. Vaccines. W. S. Saunders Company.
Tercera edición 1999

42.- Recién nacido de 6 días, producto de 36 semanas de gestación, que desde el tercer día
presenta una ictericia que ha ido en aumento. Producto de madre primigesta, grupo sanguíneo A
(Rh negativo), R/N con grupo sanguíneo O (Rh positivo). El 6o día tiene una bilirrubina total de
12.5 mg/dL, detectándola en fracción indirecta. E.f del R/N con buen estado genera, los valores
de hematocrito, hemoglobina y reticulocitos son normales. La causa más probable de hiperbili-
rrubinemia es:

a) Hepatitis neonatal.
b) Enfermedad hemolítica Rh.
c) Ictericia fisiológica.
d) Atresia de vías biliares.

Ictericia fisiológica. Ictericia monosintomática de inicio a partir del segundo día de vida, con un
pico máximo de B de 12-15 MG/Dl. en el 3°-5° día, no persistiendo más allá del 7día. No requiere
tratamiento pero sí observación y seguimiento por si se tratase de una ictericia patológica. Se
debe a una limitación del hígado para metabolizar el exceso de B producida en los primeros días
de vida.

Metabolismo de la bilirrubina e ictericia fisiológica.

Durante la etapa fetal, la mayor parte de la bilirrubina atraviesa la placenta y es metabolizada por
el hígado materno. En el momento del nacimiento este proceso se corta bruscamente y al igual
que lo que ocurre con otros procesos fisiológicos, debe ser asumida por los órganos y sistemas
del recién nacido los cuales deben adaptarse y madurar para asumir el proceso en forma
eficiente. La Figura 7.1 muestra las distintas etapas del metabolismo de la bilirrubina. Ellas nos
permitirán comprender las causas que llevan a la elevación de la bilirrubina sérica en los
primeros días de vida.

Metabolismo de la Bilirrubina

Producción de la bilirrubina: En el período neonatal, la producción de bilirrubina está


aumentada debido a la alta masa eritrocitaria del neonato (hematocritos de 61% ± 7,4) y a que la
vida media del glóbulo rojo es más corta, alrededor de 90 días comparada con 120 días en el
adulto.

Transporte de la bilirrubina: La bilirrubina no conjugada (BNC) circula en el plasma unida a la


albúmina. Normalmente en estas condiciones no atraviesa la barrera hematoencefálica. Puede
aparecer BNC libre (no unida a la albúmina) en condiciones en que la cantidad de bilirrubina
supera la capacidad de unión de la albúmina. Esto puede ocurrir porque hay cifras muy altas de
bilirrubina, hipoalbuminemia o presencia de substancias y factores que desplazan o debilitan la
unión de la bilirrubina con la albúmina. La presencia de BNC libre es siempre anormal y resulta
en paso de esta al SNC y eventual daño del cerebro
Captación, conjugación y excreción hepáticas: La bilirrubina es captada por receptores
específicos del polo sinusoidal del hepatocito y luego transportada por proteínas específicas al
retículo endoplasma. Aquí es conjugada con ácido glucurónico por acción de la enzima glucuronil
transferasa. Se obtiene así la llamada bilirrubina conjugada (BC) que se caracteriza por ser
soluble en agua y no difundir a través de las membranas celulares. La actividad de la glucuronil
transferasa es más baja en los primeros días de vida El principal estímulo para aumentar su
actividad son los niveles séricos de bilirrubina. También puede ser estimulada con fenobarbital.

La excreción de la BC es un proceso de transporte activo a través de la membrana del


hepatocito hacia los canalículos biliares. Luego es transportada como un componente de la bilis
al intestino.

Circulación enterohepática de la bilirrubina: La BC que llega al duodeno es en parte


eliminada por las deposiciones, previa transformación en urobilinógeno y similares, por la acción
de las bacterias y en parte reabsorbida pasando nuevamente a la circulación, luego de haber
sido desconjugada del ácido glucurónico en el intestino por acción de la enzima
betaglucuronidasa. En el recién nacido hay varios factores que favorecen la reabsorción
intestinal de la bilirrubina, especialmente en los primeros días de vida:

 ausencia de bacterias.
 menor movilidad especialmente si hay ayuno.
 niveles altos de betaglucuronidasa, enzima que hidroliza la BC en BNC, la cual puede
ser reabsorbida.

En resumen los principales factores que explican la ictericia fisiológica del recién nacido son:

 Aumento de la producción de la bilirrubina


 Limitaciones en la captación y conjugación de la bilirrubina
 Aumento de la reabsorción intestinal de la BNC

Bibliografía

 AAP Subcommittee on Neonatal Hyperbilirubinemia. Neonatal jaundice and kernicterus.


Pediatrics 2001 Sep;108(3):763-5.
 American Academy of Pediatrics. Practice Parametres: Management of
Hiperbilirubinemia in the Healthy Term Newborn. Pediatrics 1994; 94; 558-565.
 Bezerra JA, Balistreri WF. Cholestatic syndromes of infancy and childhood. Semin
Gastrointest Dis 2001 Apr;12(2):54-65.
 Harris MC, Bernbaum JC, Polin JR, Zimmerman R, Polin RA. Developmental follow-up of
breastfed term and near-term infants with marked hyperbilirubinemia, Pediatrics 2001
May;107(5):1075-80
 Connolly AM y Volpe JJ: Clincal Features of Bilirubin Encephalopathy. Clin Perinatol
1990; 17: 371-379
 Gartner LM, Herschel M. Jaundice and breastfeeding.Pediatr Clin North Am. 2001
Apr;48(2):389-99.
 Geiger AM, Petitti DB, Yao JF Rehospitalisation for neonatal jaundice: risk factors and
outcomes. Paediatr Perinat Epidemiol 2001 Oct;15(4):352-8
 Kemper K, Forsyth B, McCarthy P: Jaundice, terminating breast-feeding, and the
vulnerable child. Pediatrics. 1989; 84; 773-778.
 Kramer Ll: Advancement of dermal icterus in the jaundiced newborn. AJDC. 1969; 118:
454.

Hansen TW. Bilirubin brain toxicity. J Perinatol 2001 Dec;21 Suppl 1:S48-51
43.- Se trata de primigesta de 27 años con control prenatal regular, cursa con ruptura de
membranas a las 30 semanas de gestación, se interrumpe el embarazo por cesárea.
Es probable que el recién nacido prematuro disminuya la posibilidad de presentar enfermedad
de membranas hialinas por la siguiente medicación materna:

a) Oxitocina IV
b) Esteroides IM
c) Sulfato de magnesio IV
d) Antibioticos IV

Uso prenatal, un pilar en neonatología

Por su efecto en la maduración fetal, los corticoesteroides se han utilizado prenatalmente desde
hace más de tres décadas. Liggins y Howie en 1972, reportaron por primera vez los efectos
benéficos de los esteroides, administrados prenatalmente, en la maduración pulmonar y en la
disminución de la incidencia de SDR. Desde ese primer reporte a la fecha, se han descrito otros
efectos positivos. Su uso "adecuado" y oportuno ha demostrado tener beneficio en el recién
nacido pretérmino. Ahora se sabe que la administración de esteroides a la madre embarazada se
ha asociado a disminución en la incidencia de SDR, hemorragia intraventricular, displasia
bronco–pulmonar y mortalidad neonatal. Se ha observado que el máximo beneficio en el recién
nacido se obtiene de 24 horas a siete días después de la administración materna de esteroides.
Sin embargo, aún antes de 24 horas de administrados, se ha reportado beneficio. Aunque los
cambios bioquímicos generados con los esteroides desaparecen después de siete días, los
cambios estructurales persisten. Algunos estudios clínicos han demostrado efecto benéfico de
esteroides aún después de siete días de administrados.

Un metanálisis de 12 estudios, realizado por Crowley y col. en 1990, demostró que el uso
prenatal de esteroides disminuía la incidencia de SDR en aproximadamente 50%. Es claro que el
efecto más importante se encuentra en los fetos menores de 34 semanas de edad gestacional;
sin embargo, también se ha demostrado disminución en la incidencia de SDR en neonatos de
mayor edad gestacional.18

Probablemente el efecto benéfico más importante asociado al uso prenatal de esteroides sea la
disminución en la mortalidad neonatal. Crowley y col. demostraron en el metanálisis, con más de
3 000 neonatos, que ésta disminuía considerablemente (razón de momios 0.59, intervalo de
confianza =0.47–0.75). La disminución en mortalidad se ha detectado aún en pacientes menores
de 800 g.

Bol. Med. Hosp. Infant. Mex. v.62 n.5 México sep. /oct. 2005

Esteroides en Neonatología: entusiasmo, uso, abuso y desuso. ¿Dónde está el justo


medio?

Steroids in neonatology: enthusiasm, use, abuse and disuse. Is there an equilibrium?

Liggins GC, Howie RN.A controlled trial of antepartum glucocorticoid treatment for prevention of the
respiratory distress syndrome in premature infants. Pediatrics. 1972; 50: 515–25. [

Crowley P, Chalmers I, Keirse MJNC. The effects of corticosteroid administration before preterm
delivery:An overview of the evidence from controlled trials. Br J Obstet Gynaecol. 1990; 97: 11–
25. [

Van Marter LJ, Leviton A, Kuban KCK. Maternal glucocorticoid therapy and reduced risk of
bronchopulmonary dysplasia. Pediatrics. 1990; 86: 331 –6.
Morales WJ, Diebel D, Lazar AJ, Zadrozny D.The effect of antenatal dexamethasone
administration on the prevention of respiratory distress syndrome in preterm gestations with
premature rupture of membranes. Am J Obstet Gynecol 1986; 154:591–5.

. Gamsu HR, Mullinger BM, Donnai P, Dash CH. Antenatal administration of betamethasone to
prevent respiratory distress syndrome in preterm infants: Report of a UK multicentre trial. Br J
Obstet Gynaecol. 1989; 96:401–10.

. Young BK, Klein SA, Katz M. Intravenous dexamethasone for prevention of neonatal respiratory
distress: A prospective controlled study. Am J Obstet Gynecol. 1980; 138:203–9.

. Doyle LW, Permezel MJ, Kitchen WH. Is there a lower limit for birth–weight/gestational age and
antenatal steroid therapy? Aust NZ J Obstet Gynaecol. 1992; 32: 193–5.

44.- Femenino de 7 años, escolar. Es atendida en consulta externa por dolor faríngeo, erupción
cutánea y fiebre de dos días de evolución. E.F.: adenomegalias cervicales de 1.5 mm., faringe
hiperémica, hepatomegalia de 3 cm. por debajo del borde costal derecho, esplenomegalia de 2
cm., exantema maculopapular de predominio en tórax y abdomen. Recibió tratamiento con acetil
cefuroxima.

El tratamiento de primera elección en este paciente es:

a) Esteroide.
b) Amoxicilina.
c) Paracetamol.
d) Aciclovir.

El tratamiento de la mononucleosis infecciosa consiste en reposo y alivio del malestar. El


paracetamol reduce fiebre y el dolor de garganta. Debe evitarse un exceso de actividad física
durante el primer mes para evitar la posibilidad de una rotura esplénica. Aunque se han usado
corticoides (prednisona 40-60 mg/dia durante 2 ó 3 días con reducción de las dosis en la
semana siguiente) para evitar la obstrucción de las vías respiratorias en los pacientes con
hipertrofia tonsilar, estos no se recomiendan ya que pueden originar superinfecciones.

El aciclovir no ha mostrado ningún impacto significativo sobre la mononucleosis infecciosa


aunque in vitro inhibe la replicación del virus. Esto se debe a que el aciclovir (y otros antivíricos
como el ganciclovir o el foscarnet) actúan sobre la DNA-polimerasa vírica pero no sobre la DNA-
polimerasa celular implicada en la replicación del virus no integrada en su genoma.

45.- Se trata de masculino de 2 meses y medio. Atendido en consulta por ictericia que inició a
los 16 días de vida y ha sido progresiva, asociada a falta de pigmentación de las evacuaciones.
Antecedentes: producto de gesta 1, obtenido a término sin complicaciones perinatales.
Exploración física: Se aprecia ictericia generalizada y hepatomegalia de consistencia dura. Se
corrobora acolia.

El diagnóstico definitivo se establece mediante:


a) Colangiografia y biopsia hepática.
b) Determinación de ig m para hepatitis a.
c) Determinación de galactosa-1-fosfato-uridil-transferasa.
d) Determinación de ggt , tgo y tgp

Pruebas de imagen son útiles en el diagnóstico, tales como la gammagrafía hepatobiliar y la


colangiografía, sin embargo, éstas no han demostrado ser mejores que la ecografía y presentan
otras limitaciones como la disponibilidad (22).
Por último, la biopsia hepática permite confirmar el diagnóstico en 97-98% de los casos (23). En
ella, los hallazgos más típicos son la proliferación de los conductos biliares, fibrosis portal,
estasia biliar canalicular y transformación gigantocelular, pero menos intensa que en la hepatitis
neonatal. En los casos precoces, es posible observar escasos conductos biliares en vez de
proliferación, por lo que ante la sospecha clínica se recomienda la realización de biopsias
seriadas.
Atresia de vías biliares en pediatría:
Una Revisión de la Literatura
Biliary Atresia in pediatrics: A Review of the Literature
Javiera Benavides T. 1, Carolina Espinoza G. 1, Nicolás Pereira C. 1 y Carmen Gloria Rostion A.
2
1 Alumnos VII° Medicina, Facultad de Medicina Universidad de Chile.
2 Tutor Docente, Cirujano Infantil, Servicio Cirugía Infantil Hospital Roberto del Río y Facultadde
Medicina Universidad de Chile.

46.- Se trata de masculino de 5 años llevado a consultar por presentar exantema


máculopapular de inicio en cara, posteriormente en tronco, extremidades y nalgas, con
adenopatía cervical y retroauricular, refiere la madre que ha presentado fiebre no cuantificada y
catarro. El diagnóstico más probable de este paciente es:

a) Varicela.
b) Sarampión.
c) Rubéola
d) Exantema súbito

La también llamada Tercera enfermedad, era conocida antiguamente como Sarampión alemán y
considerada como una variante del Sarampión o la Escarlatina. También se la conoce como
Sarampión rojo.

En 1914 se teorizó sobre su origen viral y recién en 1938 lo confirmaron Hiro y Tosaka. En 1940,
durante una epidemia en Australia, un oftalmólogo reportó 78 casos de cataratas congénitas en
bebés nacidos de madres infectadas en el primer trimestre del embarazo: fue el primer dato
reconocido del Sindrome de Rubeola Congénita (SRC).

Agente etiológico:

Es un rubivirus (RNA) de la familia togavirus, aislado en 1962 por Parkman y Weller.

Se clasifica de dos maneras:


1) Post-natal:

Es una enfermedad infectocontagiosa, común en la infancia y juventud. Da síntomas generales


leves: exantema máculopapular difuso, estado febril y adenopatías suboccipitales,
postauriculares y cervicales posteriores (que son características de esta patología,
antecediéndola entre 5 a 10 días). El 25 a 50% cursa en forma subclínica. En niños mayores y
adultos (mujeres fundamentalmente), puede dar poliartralgias. Aunque en general no deja
secuelas, puede complicarse con encefalitis o trombocitopenia.

Esta forma se transmite por contacto directo o por gotitas de Pflügge. Predomina a fines del
invierno y comienzos de primavera. En comunidades pequeñas o cerradas, durante un brote
suelen padecerla todos los susceptibles. El período de incubación varía entre 14 y 21 días. El
contagio se produciría desde pocos días antes del exantema y hasta 7 días después del inicio
del mismo (aunque estudios hechos en voluntarios, mostraron presencia del virus rubeola en
secreciones nasofaríngeas desde 7 días antes y hasta 14 días después).

Manifestaciones clínicas: es una enfermedad infecciosa leve que evoluciona de manera


subclínica u oligosintomática en alrededor del 50% de los niños pequeños. Los síntomas
prodrómicos, como aumento de volumen doloroso (adenomegalia) en ganglios retroauriculares,
cervicales posteriores y occipitales son más frecuentes entre los adolescentes y adultos jóvenes.
El exantema puede ser también el signo inicial, con máculas y pápulas rosadas pálidas que
aparecen inicialmente en la cara y cuello y se generalizan en 2 a 3 días. El exantema puede ser
intensamente eritematoso y descamarse finamente. Los síntomas generales, como fiebre,
cefalea, mialgias, artralgias y ocasionalmente artritis, son más frecuentes entre las mujeres
adolescentes y adultos jóvenes. Las articulaciones más afectadas son las muñecas y las
interfalángicas.

2) Congénita:

Más del 25% de los fetos cuya madre padeció rubeola durante el primer trimestre del embarazo,
son afectados. Puede producir muerte uterina o aborto espontáneo.
Las patologías asociadas con esta enfermedad son: oftalmológicas (microoftalmía, cataratas,
coriorretinitis), cardiológicas (conducto arterioso persistente, estenosis periférica de la arteria
pulmonar, etc.), auditivas (sordera neurosensorial) y neurológicas (microcefalia,
meningoencefalitis, retraso mental).

Además presentan retardo del crecimiento, hepatoesplenomegalia, ictericia y lesiones similares a


las purpúricas. Muchas de las rubeolas maternas pueden cursar en forma asintomática y los
casos leves del feto, se diagnostican varios meses o años después del nacimiento (sordera
parcial o manifestaciones cardíacas leves). Después de la vigésima semana de gestación, la
infección fetal es casi nula.

Los lactantes con SRC pueden eliminar el virus en sus secreciones y orina durante un año o
más, afectando así a otras personas susceptibles. De todas maneras, esta susceptibilidad se ha
mantenido antes y después de la vacuna, en un 10 a 20%.
Bibliografía

1. Report of the Committee on Infectious Diseases, 25h edition, Red Book 2000. American
Academy of Pediatrics.
2. Fifth (human parvovirus) and sixth (herpesvirus 6) diseases. Koch WC. Curr Opin Infect
Dis 2001, Jun; 14 (3): 343-356.
3. Primary human herpesvirus 8 in immunocompetent children. Andreoni M, Sarmati L,
Nicastri E, El Sawaf G, El Zalabani M, Uccella I, et al. JAMA 2002 Mar 13; 287 (10):
1295-300.
4. Varicella vaccine update. AAP. Pediatrics 2000, Jan 105: 136-141.

47.- Paciente masculino de 5 años con diagnóstico de escarlatina, se indica tratamiento, en


caso de encontrar una complicación tardía o no supurada usted espera encontrar:

a) Adenitis cervical y absceso cervical


b) Otitis media aguda y sinusitis.
c) Fiebre reumática y glomerulonefritis.
d) Celulitis y fascitis necrosante.

Con el tratamiento apropiado, es raro que se presenten complicaciones, pero éstas pueden ser:

 Fiebre reumática aguda


 Problemas óseos o articulares (osteomielitis o artritis)
 Infección del oído (otitis media)
 Inflamación de una glándula (adenitis) o absceso
 Daño renal (glomerulonefritis)
 Daño hepático (hepatitis)
 Meningitis
 Neumonía
 Sinusitis

Algunas cepas de Streptococcus pyogenes ( -hemolítico del grupo A) son cardiogénicas o


nefrogénicas, que mediante mecanismos inmunitarios posterior a la infección desencadenan
fiebre reumática y glomerulonefritis.

González-Saldaña N, Infectología Clínica Pediátrica, 7ª edición, páginas 403-407.


48.- Masculino de año y medio de edad, que acude a su consultorio, Ud. realiza el diagnóstico
de dermatitis de la zona de pañal, la complicación más frecuente en esta patología es:

a) Impétigo
b) Candidosis
c) Granuloma glúteo infantil
d) Dermatofitosis

Se entiende por dermatitis del pañal, en sentido amplio, cualquier enfermedad cutánea que se
manifieste única o primordialmente en la zona cubierta por el pañal. Sin embargo, es más
correcto definir la dermatitis del pañal, o mejor dicho, la dermatitis irritativa del área del pañal,
como el proceso cutáneo irritativo e inflamatorio debido a las especiales condiciones de
humedad, maceración, fricción y contacto con orina, heces y otras sustancias (detergentes,
plásticos, perfumes, etc.) que se producen en la zona cubierta por el pañal durante un periodo
muy concreto de la edad del individuo.
Complicaciones
Las más habituales son de tipo infeccioso, especialmente la sobreinfección por Candida
albicans, que se manifiesta clínicamente como eritema intenso de tinte rojo violáceo con
formación de pápulas y pápulo-pústulas de extensión periférica. El llamado granuloma glúteo
infantil es una complicación de una dermatitis del pañal asociada a la utilización de corticoides
tópicos de alta potencia, caracterizada por la aparición de nódulos violáceos o purpúricos de
hasta 2 ó 3 cm de diámetro. La hipopigmentación o cicatrización son complicaciones de
dermatitis del pañal moderadas o severas.

Piel. 2008;23:460-70.

Bibliografía:

1. Berg RW, Milligan MC, Sarbaugh FC. Association of skin wetness and pH with diaper
dermatitis.
Pediatr Dermatol 1994; 11: 18-20.
2. Chavigny JM, Crépin V, Stalder JF. Ann Dermatol Venereol 1996;123: 681-685.
3. Lane AT, Rehder PA, Helm K. Evaluations of diapers containing absorbent gelling material with
conventional disposable diapers in newborn infants. Am J Dis Child 1990;144: 315-318.
49.- Se trata de paciente masculino de 24 meses de edad que acude al servicio de consulta
externa por presentar otorrea bilateral mucopurulenta de una semana de evolución, como
antecedentes ha presentado cuadros repetitivos de infecciones de vías respiratorias altas en
todos ellos ha recibido diferentes tratamientos. A la exploración física presenta ambas
membranas timpánicas opacas e íntegras a la rinoscopía presenta mucosa pálida y violácea con
moco hialino en forma abundante, orofaringe sin alteraciones, cuello sin adenomegalias, la
radiografía lateral de cuello muestra datos francos de obstrucción nasofaringea. El diagnóstico
más probale del paciente es:

a) Mastoiditis bilateral
b) Otomastoiditis bilateral
c) Timpanitis bulosa
d) Otitis media bilateral

[1] La mayoría de los episodios de OMA (70-90%) coinciden con una infección de vías
respiratorias altas, por lo que los síntomas de la misma suelen estar presentes, pero presentan
escaso valor discrimitativo:
Rinitis.
Tos.
Irritabilidad, llanto.
Fiebre.
Rechazo de la alimentación.
[2] Uno o más de los síntomas siguientes elevan sustancialmente la probabilidad de otitis media
aguda y, por lo tanto, hace preciso realizar un examen otoscópico para corroborarlo:
Otorrea.
Otalgia, tocarse la oreja.
Falta de descanso nocturno, despertar frecuente durante la noche.
La asociación otalgia y falta de descanso nocturno diagnostica el 71% (menores de
2 años) y 75% (mayores de 2 años) de los episodios de OMA.
En menores de 2 años, la asociación de otalgia y conjuntivitis incrementa la probabilidad hasta
el 76% y sugiere etiología por Haemophilus influenzae.
Otros síntomas que en los libros clásicos se relacionan tradicionalmente con los episodios de
otitis no aportan información adicional para el diagnóstico de OMA, dado que se presentan con la
misma frecuencia en niños con otros cuadros:
Fiebre.
Vómitos, molestias abdominales.
Diarrea.
Hipoacusia.
Vértigo.

[3] En el examen otoscópico es necesario valorar la coloración, transparencia y movilidad del


tímpano, siendo altamente sugestivos de otitis media aguda los siguientes hallazgos:
Opacificación.
Abombamiento.
Disminución/ausencia de movilidad.
En ausencia de sintomatología, la disminución de movilidad y opacificación son indicativos de
OME.
El enrojecimiento como signo aislado resulta muy poco específico.

[4] Se considera afectación del estado general:


Presencia de otalgia moderada/severa (irritabilidad, tocarse la oreja en <2 años).
Fiebre >39ºC (oral) o 39.5ºC (rectal).
También quedarían incluidos en este grupo, a efectos del tratamiento a seguir, aquellos niños
cuyo seguimiento y control no pueda garantizarse.
[5] Se consideran factores de riesgo que incrementan la posibilidad de infección por neumococo:
Asistencia a guardería.
Tratamiento antibiótico en el mes previo a la consulta.

50.- Se trata de mujer con embarazo normoevolutivo, RN termino con dificultad respiratoria
severa inmediatamente después de su nacimiento, ruidos respiratorios disminuidos, abdomen
plano. Rx de tórax con masa multiquística izquierda que desplaza mediastino a la derecha. El
diagnóstico más probable es:

a) Hernia diafragmática
b) Fístula traqueo-esofágica
c) Neumonía congénita complicada
d) Atelectasia izquierda

La hernia diafragmática congénita se produce por la falta del cierre de la membrana


pleuroperitoneal en la 8ª semana de gestación con defecto de la porción posterolateral del
diafragme (Hernia de Bochdalek) 80% es izquierda. Puede asociarse a otras anomalías (atresia
esofágica, anomalías del tubo neural, cardiopatías, hipoplasia pulmonar,
malrotación y alteraciones cromosómicas como trisomía 18, 21, 13.
Tiene una mortalidad que va del 25 al 80% y una frecuencia de 1/2000 –5000 recién nacidos,
más frecuente en varones que mujeres
El cuadro clínico puede presentarse como:
• Insuficiencia Respiratoria Severa desde el nacimiento, con polipnea, quejido, aleteo nasal y
retracción costal
• Disminución o ausencia de murmullo vesicular
• Ruidos hidroaéreos (RHA) en tórax
• Desplazamiento de ruidos cardiacos al lado contralateral
Diagnóstico Prenatal: Ecografía rutina
Diagnóstico Postnatal: Radiografía (Rx) tórax AP y cuadro clínico (Imágenes radiográficas donde
se aprecia consolidación en hemitorax izquierdo e imágenes hidroaéreas con
desplazamiento mediastino al lado contralateral)
Diagnóstico Diferencial: quistes congénitos de pulmón – neumotórax

Referencias:

1. Hernia Diafragmática Congénita 2000 – 2006 Dra. Nálit Almuna Fernández


2. Hernia Diafragmática Congénita – Avances en tratamiento Prenatal Clínicas de Perinatología,
ISSN 0186-0208, Nº. 3, 2003 (Ejemplar dedicado a: Cirugía maternofetal), ISBN 8448603508,
págs. 459-473
3. Hernia Diafragmática. Manual de Diagnóstico y Terapéutica en Pediatría Hospital Infantil La
paz Universidad Autónoma de Madrid Pág. 320
2009
4. Hernia Diafragmática Urgencias y tratamiento del Niño Grave J Casado Ana Serrano pág. 627
2000

51.- Se trata de masculino de 12 años referido a consulta externa por dolor mesogástrico
aumento en el número de evacuaciones entre 4 a 5 veces por día de consistencia semilíquida,
fétida, de color verdoso y aspecto grasoso que flotan en el sanitario no refiere sangre. Cuadro de
aproximados 10 días de evolución que se ve alternado con estreñimiento. E.F. Abdomen
distendido, timpánico y con peristalsis aumentada.
El tratamiento de elección en el manejo de este paciente es:

a) Metronidazol 30 mg/kg/dia por 10 días.


b) Metronidazol 15 mg/kg/día por 5 días.
c) Albendazol 400 mg/día dosis única.
d) Ciprofloxacina 20 mg/kg/dìa por 7 días.

Tabla1. Fármacos utilizados en el tratamiento de la giardiosis, y dosis más recomendadas.


Fármaco Dosis del Adulto Dosis pediátrica
Metronidazol 250 mg 3 v/día X 5 días 15 mg/kg. en 3 dosis X 5 días
Tinidazol 2 gr. una vez 50 mg/kg. Una vez (máx. 2 gr.)
Furazolidona 100 mg 4 v/día X 7-10 días 6 mg/kg. 4 dosis X 7-10 días
Secnidazol 2 gr. dosis única oral. 30 mg/kg./día. Una dosis oral.
Albendazol 400 mg/día X 5 días 15 mg/Kg/día (sin pasar de 400mg) X 5 días
INSTITUTO DE MEDICINA TROPICAL “PEDRO KOURÍ”
SUBDIRECCIÓN DE PARASITOLOGÍA
DEPARTAMENTO DE PARASITOLOGÍA
Estudio de factores asociados con la reinfección por
Giardia lamblia en niños de círculos infantiles
Autor: Dr. Fidel Angel Núñez Fernández
Tutor: Dr. Carlos M. Finlay Villalvilla, Dr C Biológicas.
Asesor: Dr. Jorge Sarracent Pérez, Dr C Biológicas.
La Habana
2004
52.- Masculino de 3 años, preescolar. Es atendido en consulta. desde los 2 años de edad, al
llorar presenta cianosis labial y peribucal, cuando se golpea o al regañarlo, no pierde el
conocimiento, la cianosis desaparece al ceder el llanto, le ocurre casi a diario, no hay
antecedente familiar de enfermedad neurológica o cardiovascular. Examen físico peso 13.6 kg.,
talla 93 cm., resto sin datos patológicos. En este paciente el diagnóstico más probable es:

a) Epilepsia.
b) Tetralogia de fallot.
c) Enfermedad por reflujo gastroesofágico.
d) Espasmo del sollozo.

Espasmo del sollozo


Es la retención de la respiración posterior a un evento que disguste tal como una caída, el estar
frustrado o enojado, o por estar asustado. -El niño da uno o dos gritos largos y posteriormente
retiene su respiración en expiración hasta que sus labios se ponen azules.
-El niño posteriormente se desmaya (algunos llegan a tener algunos espasmos musculares). -
después el niño respira normalmente y permanece completamente alerta en menos de 1 minuto.
-El inicio ocurre entre 6 meses y 2 años. Esto sucede solamente cuando el niño está despierto.

Se define al espasmo del sollozo (Breath-holding spells), como un evento caracterizado por crisis
recurrentes de apnea transitoria, pérdida del conocimiento y cambios en el tono muscular
normal. De inicio súbito, no se considera secundario a patología orgánica ni resultado de una
manifestación psiquiátrica.

Entre el 5 y el 7% de la población infantil sana presenta crisis de espasmos de sollozos, sin


embargo, algunos autores coinciden en que esta entidad es mucho más frecuente. Por lo general
dichos eventos inician entre los 6 a 12 meses de edad con un pico de incidencia entre los 12 y
los 24 meses de edad.

Es poco frecuente que se presente en niños mayores de 6 años de edad por lo que su aparición
en niños mayores de 4 años amerita especial atención. El 25% de los niños que los presentan
tiene un familiar directo que lo padeció en la infancia. Se presenta más frecuentemente en
varones.

Son causas frecuentes de síncopes y convulsiones anóxicas secundarias a isquemia o hipoxia,


con la consecuente depresión súbita de la función neuronal, en contraste con las convulsiones
epilépticas, secundaria a una descarga excesiva de las neuronas.

El espasmo del sollozo a sido dividido en el espasmo del sollozo pálido y cianótico, dependiendo
de la coloración de la piel durante el evento. La fisiopatología, en cada caso en particular, es
diferente. Entender las características del espasmo del sollozo y poder diferenciarlo de otras
entidades, puede ayudar al pediatra a tranquilizar a los padres.

Bibliografía:

1- Berman, RE., Kliegman, RM., Jenson, HB: Nelson Textbook of Pediatrics. 16th edition,
Philadelphia, Pennsylvania, W.B. Saunders Company U.S.A.., 2,000; 1829
2- Anderson JE, Bluestone D: espasmos del sollozo. Contemp Pediatr 2,000;17(1):61-72
3- Macan H, et al: Espasmo del sollozo en 91 niños y respuesta al tratamiento con hierro. Arch
Dis Child 1999;81:261-262
4- Breningstall GN: Breath-holding Spells . Pediatr Neurol 1996;14:91-97
5- DiMario FJ: Breath-holding spell in childhood. Am J Dis Child 1992;146:125-131
6- Lombroso CT, Lei-man P: Breath-holding spell (cyanotic and pallid in fantile syncope).
Pediatrics 1967;38:563-581.
7- Gauk EW,Kidd L, Prichard JS: Mechanism of seizures associated with breath-holding spell. N
Engl J Med 1963; 268: 1436-1441
53.- Recién nacido de 2 días, que presenta vómitos gástricos, evacuaciones normales,
abdomen distendido, los rayos x de abdomen demuestran asas muy distendidas, con
presencia de niveles hidroaéreos, el diagnóstico más probable del paciente es:

a) Atresia de duodeno
b) Hipertrofia pilórica
c) Atresia de íleon
d) Reflujo gastroesofágico

De las malformaciones intestinales, la atresia de íleon es la más frecuente y la asociación con


otro tipo de malformaciones o cromosomopatías es elevada; en este caso no existió tal
correlación. Hacer un correcto diagnóstico prenatal mediante ultrasonidos nos permite planificar
el momento idóneo del parto, llevándolo a cabo en un centro con cirugía pediátrica y así mejorar
los resultados perinatales. Se discute cuándo finalizar la gestación, y en principio no hay
contraindicación en prolongarla hasta la madurez fetal siempre que no exista ascitis fetal, que
sería un signo de riesgo inminente de perforación intestinal por íleo meconial; en tal caso finalizar
el embarazo es la postura más prudente por la elevada mortalidad de esta eventualidad; éste no
fue nustro caso. Aunque autores como Schwobel et al propugnan la evacuación mediante
punción de la ascitis fetal en el caso de presentarse con la finalidad de disminuir la presión
abdominal fetal y alargar todo lo posible la duración del embarazo. Nosotros nos vimos abocados
a terminar la gestación por motivos meramente obstétrica

 Clínica e investigación en ginecología y obstetricia, ISSN 0210-573X, Vol. 32, Nº. 1,


2005 , pags. 36-37

54.- Se trata de femenino de 10 años, acude a consulta inicialmente con una historia de fiebre
de 2 días de evolución y una adenopatía cervical anterior izquierda fluctuante, sensible y caliente
de 4 cm. El diagnóstico más probable es:

a) Enfermedad de Hodgkin
b) Leucemia linfoblástica aguda (LLA)
c) Histiocitosis X
d) Linfadenitis bacteriana aguda

La fiebre y los signos de supuración sugieren infección bacteriana aguda. La enfermedad de


Hodgkin, la leucemia linfoblástica aguda, la histiocitosis X, y el neuroblastoma metastático
pueden asociarse a fiebre y adenopatías, sin embargo, su aparición no sería tan aguda como
se aprecia en la linfadenitis bacteriana aguda, y la supuración difícilmente, sería un signo de
presentación de éste problema.

55.- Femenino de 11 meses ingresa por tercera vez en un hospital con sibilancias sin fiebre ni
coriza. La madre dice que el primer episodio ocurrió cuando tenía 2 meses y que siempre que ha
ocurrido ha sido de forma brusca. Había notado que el lactante presento tos nocturna antes del
ingreso. Por otra parte, la niña tiene una causa ponderal ascendente y recibe alimentación con
fórmula enriquecida con proteínas, y aunque babea mucho, este problema parece estar
resolviéndose. En la familia no hay ninguna historia de fiebre del heno, asma o eccema. En las
dos ocasiones anteriores, la niña respondió parcialmente al tratamiento con broncodilatadores,
aunque había seguido teniendo ligeras sibilancias entre los episodios. Los hallazgos de la
exploración física son irrelevantes, a excepción de roncus bilaterales y sibilancias espiratorias.
No existe sangre oculta en las heces, y el recuento sanguíneo completo no muestra eosinofilia
periférica. La radiografía de tórax revela hiperinsuflación leve de ambos campos pulmonares,
pero no infiltrados. El diagnóstico más probable asociado a estas recaídas de sibilancias es:

a) Reflujo gastroesofágico
b) Alergia a las proteínas de leche de vaca
c) Fístula traqueoesofágica
d) Fibrosis quística

Los rasgos típicos del broncoespasmo por reflujo son: aparición en edad temprana, inicio brusco
de las sibilancias sin elemento ambiental que lo desencadene, inexistencia de historia de atopia
familiar, tos nocturna que puede ser señal de reflujo con micro aspiración, respuesta escasa o
incompleta a broncodilatadores. La tele de tórax solo puede mostrar cambios inespecíficos de
broncoespasmo. A los 22 meses de vida, será de esperar enfermedad parenquimatosa
pulmonar, e infiltrados pulmonares en niños con fístula traqueoesofágica así como fibrosis
quística.

56.- A 4-year-old boy presents with a history of constipation since the age of 6 months. His
stools, produced every 3-4 days, are described as large and hard. Physical examination is
normal; rectal examination reveals a large ampulla, poor sphincter tone, and stool in rectal
vault. The next step in the management of this infant would be.

a) Lower GI barium study


b) Parental reassurance an counseling in Hirschprung’s disease
c) Serum electrolyte measurement
d) Upper GI barium study

Expresión Clínica
Dentro de los síntomas que permiten una sospecha precoz, se encuentra el estreñimiento o
constipación, definida en el recién nacido como el retraso en la eliminación de meconio mayor
a 48 horas asociada a distensión abdominal, y en los niños mayores como deposiciones
infrecuentes de consistencia aumentada (. El 98% de los lactantes elimina el meconio en las
primeras 48 horas de vida. Los prematuros eliminan más tardíamente el meconio, pero la EH
es rara en prematuros. De los pacientes con EH, sólo el 60% elimina el meconio después de
las 48 horas, por lo que este signo no es patognomónico de la enfermedad.
La mayoría de los niños que presentan aganglionosis congénita, son sintomáticos los primeros
días o las primeras semanas luego del nacimiento (2). Alrededor de dos tercios de los
pacientes presenta síntomas dentro de los tres primeros meses de vida y 80% desarrolla
síntomas dentro del primer año de vida. Sólo un 10% de los pacientes inicia síntomas entre los
3 y 14 años de edad y en general se trata de pacientes con enfermedad de segmento
ultracorto. (13)
Los recién nacidos y lactantes pequeños presentan con frecuencia signos de obstrucción
intestinal, distensión abdominal, vómitos biliosos e intolerancia a la alimentación. La inspección
anal y la radiografía pueden orientarnos hacia una causa mecánica de obstrucción, pero no
descarta EH. Si la obstrucción no tiene una causa mecánica, además de pensar en una EH,
debe plantearse el diagnóstico diferencial con hipotiroidismo, insuficiencia suprarrenal,
hipokalemia, hipercalcemia, hipomagnesemia, y en casos excepcionales alteraciones
neuromusculares.
Cuando la sintomatología es poco evidente,
Puede presentarse como un cuadro de constipación crónica, con historia de dificultad en la
eliminación de deposiciones, masas fecales palpables en fosa ilíaca izquierda y un tacto rectal
en que no se encuentran deposiciones en la ampolla rectal y esfínter anal hipertónico. En
muchas ocasiones la estimulación rectal provoca salida explosiva de heces líquidas de olor
fétido (17). Por lo tanto, frente a pacientes con constipación crónica, en los cuales se ha
descartado causa mecánica de obstrucción intestinal, que no cede a las medidas dietéticas ni
farmacológicas, debe plantearse el diagnóstico de
EH. También puede encontrarse dilatación de asas intestinales, adelgazamiento de la pared
abdominal, alteraciones de la nutrición y el crecimiento.
En niños mayores, los síntomas más comunes incluyen constipación crónica progresiva,
impactación fecal recurrente, mal incremento ponderal y malnutrición.

Rev. Ped. Elec. 2008, Vol 5, N° 1. ISSN 0718-0918


Servicio Salud Metropolitano Norte
Facultad de Medicina Hospital Clínico de Niños
Departamento de Pediatría y Cirugía Infantil Roberto Del Río
Referencias
1. De Manueles J. Enfermedad de Hirschsprung.
Protocolos diagnósticos y terapéuticos en pediatría. Sociedad Española de Pediatría. Pag. 56-
60.
2. Feldmon T., Wershil B. Hirschsprung Disease. Pediatrics in review. Vol 23. N 11, August
2003.
3. Luis L.A., Encinas J.L., Avila L.F., et cols.
Enfermedad de Hirschsprung: enseñanzas de los últimos 100 casos. Cir Pediatr 2006; 19:177-
181.
4. J.M. Gil-Vener y cols. Diagnóstico dieferncial de Hirschsprung-neurodisplasia intestinal.
Fiabilidad de las pruebas diagnósticas. Cir Pediatr 2006; 19: 91-94.
5. M. López, y cols. Índices de fiabilidad de la manometría anorrectal para el diagnóstico de la
enfermedad de Hirschsprung en cualquier edad. Cir Pediatr 2005; 18:13-16
6. Hernández F., Rivas S., Ávila L.F., Díaz M., ET cols. Aganglionismos extensos. Tratamiento
y resultados a largo plazo. Cir Pediatr 2003; 16: 54-57.
7. Goulet O. y cols. Intestinal transplantation in children: preliminary experience en Paris. JPEN
J Parenter Enteral Nutr 1999; 23 (5 Suppl)
8. Peña A. Enfermedad de Hirschsprung. Los avances y las preguntas no contestadas. Cir
Pediatr 2002: 15:46-47
9. Polliotto S, Heinen F, Anduna G, Korman R. Evaluación de resultado a tres años de nuestra
primera experiencia en el tratamiento laparoscópico de la enfermedad de
Hirschsprung. Cir Pediatr 2001; 14: 85-87

57.- Femenino de 9 años presenta astenia, adinamia, decaimiento y pérdida de 3 kilogramos de


su peso habitual. Desde hace 15 días ha padecido fiebre de 39°c, disfagia y epistaxis. Al
explorarla se encuentra pálida, con adenomegalia cervical y axilar; se palpa
hepatoesplenomegalia y se observan quimosis en las piernas.
El diagnóstico más probable es:

a) linfoma de hodgkin.
b) tuberculosis extrapulmonar.
c) leucemia aguda.
d) lupus eritematoso sistémico.

La EH se presenta habitualmente con adenopatías cervicales o supraclaviculares, indoloras, más


consistentes que las inflamatorias y de lento crecimiento.
Al menos, dos tercios de casos asocian adenopatías mediastínicas.

Síntomas sistémicos
Síntomas sistémicos no específicos pueden ser fatiga, anorexia y ligera pérdida de peso.
Síntomas específicos (fiebre inexplicada, pérdida de peso de 10% en los 6 meses previos y
sudor nocturno abundante) tienen significado pronóstico.
Algunos pacientes tienen prurito.
Exámenes de laboratorio
En el hemograma, se observa leucocitosis con neutrofilia, linfopenia, eosinofilia y monocitosis. La
anemia puede indicar la presencia de enfermedad avanzada.
Se ha observado anemia hemolítica asociada a EH, con test de Coombs positivo, reticulocitosis
e hiperplasia normoblástica normoblástica de médula ósea. La asociación de púrpura
trombopénica idiopática (PTI) también ha sido referida y la respuesta al tratamiento de la PTI va
a depender del estadio en que se encuentre la EH. La elevación de VSG, cupremia y ferritinemia
reflejan la activación del sistema mononuclear fagocítico, que puede servir como test inespecífico
para control de seguimiento de la EH. Las anomalías del sistema inmune en el momento del
diagnóstico de EH pueden persistir durante y después del tratamiento.

1.*** Fernández-Teijeiro A. Enfermedad de Hodgkin. En: Madero López L y Muñoz


Villa A editores. Hematología y Oncología pediátricas. Madrid: Ergon; 1997.
p. 467-82.
2.* Ferrís Tortajada J, García Castell J, Berbel
Tornero O, Clar Gimeno S. Factores de riesgo para los linfomas no hodgkinianos.
An Esp Pediatr 2001; 55: 230-8.
Exposición de factores de riesgo para LNH, revisados de la literatura médica, con el objetivo,
según los autores, de divulgar entre los pediatras dichos factores de riesgo.
3.* Ferrís Tortajada J, García Castell J, López

58.- Lactante de 5 meses que es traído a consulta por presentar desde 3 días antes, fiebre,
rinorrea acuosa y estornudos, comenzando el día de la consulta con tos y dificultad respiratoria.
En la exploración presenta taquipnea, tiraje subcostal, alargamiento de la espiración, sibilancias
diseminadas. De las siguientes pruebas que hay que solicitar para determinar la etiología es:

a) Biometría Hematica
b) Radiografía de tórax
c) Cultivo de aspirado traqueal
d) Búsqueda de virus respiratorio sincitial en exudado nasofaríngeo (IF- IND)

Un cuadro infeccioso en un lactante (fiebre, rinorrea y estornudos), en el que hay localidad


respiratoria (tos y dificultad respiratoria) y signos de dificultad en la espiración, debe hacer
pensar en el diagnóstico de Bronquiolitis.
Edad: Lactantes entre 3 y 6 meses de edad
Etiología: Suele responder a una etiología viral
Lo más frecuente: Virus Sincitial Respiratorio
Otros: Virus parainflueza 3, micoplasmas, adenovirus.
Clínica: Cuadro consistente en:
 Infección de vía aérea alta (rinorrea, estornudos).
 Dificultad respiratoria con tos, sibilancias, disnea, irritabilidad, taquipnea,
tiraje.
 Espiración prolongada

Dado que el agente causal más frecuente es el Virus Sincitial Respiratorio, si se precisa el
diagnóstico etiológico de certeza, se debe buscar al virus en el exudado nasofaríngeo mediante
técnicas de detección de los antígenos o por cultivo. Para la detección de los antígenos en las
células de aspirados se utilizó el método de diagnóstico rápido por Inmunofluorescencia
indirecta recomendado por el CDC con anticuerpos monoclonales que reconocen a los
virus Sincicial Respiratorio, Adenovirus, Influenza A, Influenza B y Parainfluenza serotipos
1, 2, 3, de procedencia comercial Chemicon (USA) cedidos por OMS/CDC.
El diagnóstico diferencial preferente es con el asma; siendo factores que apoyan dicho
diagnóstico:
- Historia familiar de asma
- Repetición de episodios similares en el mismo lactante
- Comienzo brusco no precedido de infección
- Espiración muy prolongada
- Eosinofilia
- Respuesta favorable a la administración de una sola dosis de salbutamol en aerosol
- La frecuente repetición de los síntomas en el asma es un punto importante de forma que
menos de un 5% de los ataques repetidos de bronquiolitis tienen un origen inicial.

Nelson, Tratado de Pediatría 15ª Ed., págs. 1521-1522

59.- Masculino de 4 años de edad presenta un peso y talla en el percentil 3, se refiere por
presentar anorexia, irritabilidad, apatía al juego, se le observa con palidez, el laboratorio reporta
una hemoglobina de 7.3g/dl, hierro sérico de 36mcg/dl y saturación de transferían de 12%. El
tipo de anemia más probable es:

a) Hemolítica
b) Perniciosa
c) Drepanocítica
d) Ferropénica

Es el tipo de anemia más frecuente en la población general. Se trata de una anemia


caracterizada por la disminución o ausencia de los depósitos de hierro, baja concentración de
hierro sérico, baja saturación de transferrina y baja concentración de la Hb o del hematocrito.

La ferropenia se manifiesta con un síndrome característico:

SNC: irritabilidad, labilidad emocional, disminución de la concentración y la memoria, cefalea,


ataxia, parestesias, trastornos del sueño y la alimentación (PICA: apetencia desmedida por
sustancias poco habituales. Es muy característica la pica de hielo o pagofagia), aumento de la
sensibilidad al frío, síndrome de piernas inquietas…
Piel y mucosas: coloración azulada de la esclerótica, fragilidad de uñas y cabello, coiloniquia o
aspecto cóncavo de las uñas, glositis, queilitis angular o rágades, atrofia gástrica, en casos
graves producción de membranas esofágicas (síndrome de Plummer-Vinso).

Etiología:

La anemia ferropénica puede deberse a:

Aumento de la utilización: gestación, lactancia, crecimiento corporal rápido en la infancia


y adolescencia.
Pérdidas fisiológicas: menstruación
Pérdidas patológicas:
Hemorragia digestiva. El sangrado digestivo crónico es la causa más frecuente
en este grupo: hemorragias gástricas por medicamentos (AAS, AINES,
corticoides asociados a AINES…), hernia hiatal, diverticulosis, hemorroides. En
pacientes mayores de 60 años debe considerarse la posibilidad de neoplasia,
sobre todo de colon y la angiodisplasia intestinal.
Genito-urinarias
Aparato respiratorio
Hemólisis intravascular
Alteraciones en la absorción:
Dietas insuficientes: Debe interrogarse siempre acerca de los hábitos dietéticos.
El hierro presente en alimentos de origen vegetal como cereales y legumbres
tiene baja biodisponibilidad (algunos alimentos como las lentejas tienen
excesiva fama de riqueza en hierro). Incluso en carnes y pescados el contenido
en Fe es relativamente bajo. En gestantes, niños, mujeres en edad fértil, la
dieta puede ser un factor  ostmenopáusi o causal de deficiencia de hierro. En
mujeres  ostmenopáusicas y en el varón la dieta sería causa excepcional de
ferropenia.
Absorción defectuosa: gastrectomías parciales o totales, enfermedad
inflamatoria intestinal, enfermedad celíaca (causa relativamente frecuente de
anemia ferropénica); en algunos casos podría estar indicada la detección de
anticuerpos antiendomisio y antigliadina para descartar una enfermedad celíaca
no diagnosticada.

Diagnóstico:
El nivel de ferritina sérico bajo es indicativo de una situación de depleción de hierro y es la
prueba definitiva de la existencia de anemia por déficit de hierro (AF). Los demás parámetros no
permiten diferenciar con seguridad la AF de la anemia por enfermedad crónica. Generalmente
aparece microcitosis, aunque hasta en el 30% de las anemias ferropénicas, en algunas
poblaciones, el VCM puede ser >80; asimismo en el 10% de las microcitosis los parámetros
relacionados con el hierro son normales. Otros datos a tener en cuenta:

El frotis periférico habitualmente muestra anisocitosis y poiquilocitosis.

Cuando la microcitosis se asocia a aumento de glóbulos rojos, sobre todo si es mantenida,


pensar en talasemia.

Puede haber trombocitosis reactiva.

Bibliografía:
Abramson S, Abramson N. Common' Uncommon Anemias. Am Fam Physc 1999; 59 (4). 851-8
Adamson JW. Anemia and Polycytemia. En: Kaasper DL, Braunwald E (Edit.). Harrison´s
principles of internal medicine. 16th. ed. Nwe Cork: McGraw-Hill; 2005. p.329-336
Hernández Nieto L, Hernández García MT, Juncá Piera J, Vives-Corrons JL, Martín-Vega C.
Enfermedades del sistema eritrocitario: anemias. En: Farreras Valentí P, Rozman C. (Dir.).
Medicina Interna. Barcelona: Elsevier; 2004. p. 1644-1669
Glader B. Anemia: General Considerations. En: Wintrobe´s Clinical Hematology. 11th.
Philadelphia: Lippincott Williams & Wilkins; 2004. p.947-978.

60.- Femenino de 36 años, es atendida en consulta externa con reporte de papanicolaou que
reporta un NIC I, la especuloscopía se observa cérvix con ectropión periorificiario.
El método más sensible para corroborar el diagnóstico en esta paciente es:

a) Papanicolaou.
b) Colposcopía
c) Exudado vaginal.
d) Prueba de koh.
9.5.2 Las pacientes a quienes se les realizó citología cervical, cuyo resultado es LEIBG
(infección por VPH, displasia leve o NIC 1); LEIAG (displasia moderada y grave o NIC 2 y 3) o
cáncer deben enviarse a una clínica de colposcopía, para realizar estudio colposcópico.
9.5.3 Si el resultado de la citología es LEIBG, la colposcopía es satisfactoria y sin evidencia de
LEIBG, se realizará control citológico en un año (Apéndice Normativo A)
9.5.4 Si la citología es de LEIBG, la colposcopía es satisfactoria y existe evidencia de lesión, se
debe tomar una biopsia dirigida.
9.5.4.1 Si la biopsia dirigida es negativa, se realizará nueva colposcopía para verificar el
diagnóstico y en caso necesario, tomar nueva biopsia dirigida y revalorar.
9.5.4.2 Si la biopsia dirigida es reportada como LEIBG se podrá dar tratamiento conservador:
criocirugía, electrocirugía o laserterapia (sólo si cumple con las condiciones referidas en el
Apéndice 1) o se podrá mantener a la paciente en vigilancia en la clínica de colposcopía, con
colposcopía y estudio citológico cada seis meses, durante 24 meses.
Jueves 31 de mayo de 2007 DIARIO OFICIAL (Primera Sección)
9.5.4.3 Si la biopsia dirigida es reportada como LEIAG (Lesión Intraepitelial Escamosa de Alto
Grado) se realizará tratamiento conservador (electrocirugía o laserterapia). En las mujeres
posmenopáusicas, dependiendo de las condiciones anatómicas del cérvix, se realizará
tratamiento conservador en la clínica de colposcopía o tratamiento quirúrgico (histerectomía
extrafascial) en el servicio que corresponda.
9.5.4.4 Si la biopsia dirigida reporta cáncer microinvasor o invasor, la paciente se transferirá a
un Servicio o Centro Oncológico para su tratamiento correspondiente.
9.5.4.5 Si la citología reporta LEIBG y la colposcopía es no satisfactoria, se tomará cepillado
endocervical (Apéndice Normativo A)
9.6 En caso de colposcopía no satisfactoria, negativa a LEIBG y con cepillado endocervical
negativo, se continuará su control en la clínica de colposcopía en seis meses, con colposcopía
y citología.
9.6.1.1 Si el cepillado endocervical reporta LEIBG se tratará a la paciente como LEIAG, con
métodos conservadores escisionales.

Jueves 31 de mayo de 2007 DIARIO OFICIAL (Primera Sección)


Modificación a la Norma Oficial Mexicana NOM-014-SSA2-1994, Para la prevención,
detección, diagnóstico, tratamiento, control y vigilancia epidemiológica del cáncer
cérvico uterino.
Al margen un sello con el Escudo Nacional, que dice: Estados Unidos Mexicanos.- Secretaría
de Salud.
MODIFICACION A LA NORMA OFICIAL MEXICANA NOM-014-SSA2-1994, PARA LA
PREVENCION,
DETECCION, DIAGNOSTICO, TRATAMIENTO, CONTROL Y VIGILANCIA EPIDEMIOLOGICA
DEL CANCER CERVICO UTERINO.

61.- Se trata de masculino quien a la exploración física presenta datos de microoftalmía,


microcefalia, coriorretinitis, antecedentes de erupción petequial neonatal, que sufre retraso
mental probablemente. Antecedentes: Sólo una visita de control prenatal con datos de retardo en
el crecimiento intrauterino. El diagnóstico más probable es:

a) Infección por TORCH


b) Síndrome cromosómico
c) Síndrome de alcoholemia fetal
d) Galactosemia

Definición:

El síndrome de TORCH es una infección materna que afecta al feto en gestación. El síndrome
corresponde a un conjunto de síntomas y signos que presenta tanto el feto cono el RN afectado
por la infección congénita y que es producida por una serie de agentes infecciosos: virales,
parásitos y otros, que se han agrupado bajo la sigla TORCH.

Agentes etiológicos del síndrome de TORCH

Citomegalovirus, rubéola, herpes simple, hepatitis B


Virus
C, parvovirus 19, VIH, Enterovirus, varicela zoster
Parásitos Toxoplasma gondii, Tripanosoma cruzi
Treponema pallidum, Mycobacterium tuberculosis
Otros
Ureaplasma urealyticum , Micoplasma hominis, etc

Con todos estos agentes la madre puede tener una infección clínica o subclínica (más frecuente
la última situación). La madre durante el embarazo puede presentar una promoinfección, una
reactivación, reinfección o una infección crónica latente; cada una de estas situaciones puede
ser infectante para el recién nacido, lo que varía de un agente etiológico a otro.
La infección puede ser transmitida vía transplacentaria, en el momento del parto o por leche
materna.
Dependiendo del agente causal y de las semanas de gestación al momento de la infección, el
impacto de ésta se puede traducir en aborto, mortinato, mortineonato, RN con malformaciones,
RN con infección subclínica, clínica o aparentemente sano. Las secuelas afectan principalmente
al SNC.

Tabla 2. TORCH: síntomas y signos


 Retardo del crecimiento intrauterino
 Hidrops no inmune
 Prematurez
 RN pequeño para la edad gestacional
 Hepatomegalia
 Esplenomegalia
 Púrpura
 Ictericia
 Anemia
 Microcefalia
 Hidrocefalia
 Calcificaciones cerebrales
 Coriorretinitis
 Neumonitis
 Alteraciones músculoesqueléticas
El diagnóstico se establece, en general, en el período postnatal y se confirma con la detección
de anticuerpos o de antígenos. Idealmente el diagnóstico debiera ser prenatal, lo que permitiría
en algunos casos, disminuir la transmisión materno –fetal.
Cifras de IgM total sobre 20 mg/dl en el RN, apoyan el diagnóstico de infección congénita pero
valores normales no lo descartan y no excluyen el estudio específico. La detección de IgM
específica debe ser complementada con otras técnicas porque da falsos positivos y falsos
negativos. La presencia de IgG en el RN puede estar dada por traspaso de Ac maternos. Por lo
tanto, se debe realizar una curva serológica, con 2 mediciones separadas por 21 días,
considerando de valor el alza en los títulos en por lo menos 4 veces el valor basal. El estudio
específico se debe efectuar en el binomio madre-hijo e incluye los exámenes enumerados en la
tabla 3:

El estudio complementario comprende un conjunto de exámenes a efectuar en el RN, entre los


que se cuentan: fondo de ojo, Rx de cráneo y huesos largos, pruebas hepáticas, hemograma
completo y eco cerebral.

Bibliografía:
1. Red Book. 2000. Report of the Committee on Infectious Disease 25ª Ed.
2. Villarroel J. y Reyes A. Síndrome de TORCH. Enfermedades Infecciosas en
Pediatría. Banfi. A. Ledermann W., Cofré J., Cohen J., Santolaya M.E. 3ª ed. 2004.
312-324.
Ilabaca G. Síndrome de TORCH, en Neonatología. Nazer J. Ramírez R. Hospital Clínico de la U.
de Chile. 2002. Cap. 26, 301-331.

62.- Se trata de femenino de 3 meses de edad que hace tres días inicia con rinorrea hialina, tos y
estornudos. El día de ayer presenta taquipnea con 55 rpm, tiraje intercostal con aleteo nasal. A
su ingreso se observan distrés respiratorio con sibilancias inspiratorias y espiratorias y algunos
crepitantes bilaterales. Una Rx de tórax muestra hiperinsuflacción bilateral con una atelectasia
laminar y corazón pequeño. Presenta: pH 7,24; pCO2: 58 mmg; CO3H: 21 mEq/L. El
diagnóstico más probable de la menor es:

a) Bronconeumonía bilateral.
b) Crisis asmática de origen infeccioso.
c) Bronquiolitis.
d) Neumonitis intersticial.

La bronquiolitis es la enfermedad del tracto respiratorio inferior más frecuente en los dos
primeros años de vida. Aunque su mortalidad es baja, su elevada morbilidad origina una gran
demanda asistencial y causa un importante número de ingresos hospitalarios. Se presenta de
forma epidémica durante el invierno y principio de la primavera. Su etiología es viral, y el virus
respiratorio sincitial (VRS) es el agente que con más frecuencia se aísla.
El diagnóstico es eminentemente clínico. En la mayoría de los casos, cursa de forma benigna y
autolimitada; las formas más graves se presentan en lactantes más pequeños y en los que
tienen algún factor de riesgo asociado, en estos casos la tasa de hospitalización es alta.

El cuadro comienza como un catarro de vías altas, con estornudo, tos, rinorrea y en ocasiones
fiebre; y en el transcurso de 2 ó 3 días se intensifica la tos, aparecen los síntomas de obstrucción
de la vía respiratoria respiratoria inferior con aumento del trabajo respiratorio, taquipnea e
irritabilidad.
En los casos más graves, la dificultad respiratoria es marcada y presenta rechazo de las tomas
de alimento y postración.
La apnea puede ser la primera manifestación de bronquiolitis en lactantes pequeños.
Dada la evolución del curso clínico de la bronquiolitis, se recomienda reevaluar a las 24-48 horas
a todos los lactantes menores de 12 meses que, en ambiente epidémico consulten por un catarro
de vías altas. Además, es conveniente advertir a los padres de la necesidad de consultar ante la
presencia de dificultad respiratoria, agitación, mal color, vómitos o rechazo de la alimentación.
A la exploración física, podemos encontrar retracciones costales (tiraje subcostal e intercostal)
y la auscultación pulmonar puede ser normal, aunque en la mayoría de los casos presenta
espiración alargada, roncus, sibilantes y crepitantes bilaterales. Los sibilantes pueden
escucharse con el oído desnudo, sin ayuda del fonendoscopio.
No existe un patrón radiológico típico en la bronquiolitis. Los hallazgos radiológicos más
comunes son: el atrapamiento aéreo, engrosamientos peribronquiales, infiltrados intersticiales y
atelectasias laminares o segmentarias.
El hemograma es inespecífico y sólo estaría indicado realizarlo en aquellos casos en los que
sospechemos alguna complicación.
Para el diagnóstico de la hipoxemia se recomienda la pulsioximetría transcutánea, ya que
tiene muy buena correlación con la PaO2 sanguínea. Se considera hipoxia leve del 96-98%;
moderada, del
93-95%, y grave, menos de 93%.
González Caballero D, González Pérez- Yarza E. Bronquiolitis aguda: bases para un protocolo
racional. An Esp Pediatría
2001; 55 (4): 355-64.

63.- Recién nacido al que se decide colocar una sonda orogástrica, al darnos cuenta de que
dicha sonda no avanza se solicita una radiografía toracoabdomial en la que se aprecia
ausencia de aire en el intestino, la sospecha diagnóstica más probable es:

a) Atresia de esófago tipo II y V


b) Atresia de esófago tipo I y II.
c) Hernia diafragmática congénita
d) Atresia de esófago tipo III y IV

. Consiste en la interrupción de la luz esofágica con o sin comunicación con la vía aérea. No
existe aún una definición precisa del término "long gap". De acuerdo a nuestra experiencia se
trata de aquellas AE, independientemente del tipo, caracterizadas por una distancia entre cabos
esofágicos lo suficientemente grande como para imposibilitar la realización de una anastomosis
primaria término-terminal.

De acuerdo a nuestro criterio, basado en la clasificación de Ladd, las atresias de esófago pueden
clasificarse en los tipos I, II, III, IV y V
1). La Tipo I es la atresia de esófago aislada sin fístula traqueoesofágica o “atresia pura”. No
tiene comunicación con la vía aérea. Representa aproximadamente el 5% de los casos
observados y es la segunda en frecuencia. Puede diagnosticarse con más facilidad en el
embarazo, que cursa típicamente con polihidramnios y ausencia de imagen gástrica en las
ecografías prenatales. Al nacer, los neonatos presentan el abdomen excavado por falta de
pasaje de aire al intestino. En todos los casos ambos cabos esofágicos se encuentran muy
separados entre sí.
La Tipo II es una forma muy rara de atresia de esófago que representa del 1 al
3% de los casos. Existe una fístula traqueoesofágica desde el cabo superior del esófago a la
tráquea cervical. Al igual que en las Tipo I, no hay pasaje de aire al intestino distal y ambos
extremos se encuentran muy alejados entre sí.
La Tipo III es la forma observada en el 90% de los pacientes. Comúnmente se la conoce como
sinónimo de atresia de esófago. Se caracteriza por una bolsa esofágica comunicación entre el
extremo distal esofágico y la tráquea (fístula tráqueoesofagica al cabo inferior), que puede nacer
a cualquier altura de la tráquea y excepcionalmente, también en los bronquios. En estos casos el
embarazo no cursa típicamente con polihidramnios y suele verse una imagen gástrica normal en
las ecografías prenatales. Al nacer los neonatos suelen presentar el abdomen distendido por el
gran pasaje de aire que existe desde la vía aérea hacia el estómago e intestino. La distancia
entre ambos cabos es por lo general inferior a dos o tres vértebras. La coexistencia de atresia de
esófago Tipo III con atresia duodenal, intestinal o malformaciones anorrectales constituye un
cuadro de urgencia; el gran flujo de aire desviado hacia el sistema digestivo a través de la fístula
traqueoesofágica inferior provoca distensión abdominal severa y eventualmente, incluso,
perforación intestinal.
La Tipo IV se caracteriza por una fístula tráqueoesofagica en ambos cabos esofágicos. Su
observación es poco frecuente. Representa menos del 2% de los casos. La fístula superior suele
encontrarse por accidente cuando se efectúa la corrección quirúrgica inicial. En ocasiones pasa
inadvertida por el cirujano y su diagnóstico es efectuado en forma tardía. La Tipo V no es en
realidad una atresia de esófago. El esófago se encuentra permeable y es de buen calibre.
En estos pacientes lo que se observa es una fístula traqueoesofágicaaislada, generalmente de
ubicación cervical o torácica alta. El diagnóstico se realiza excepcionalmente en el periodo
neonatal ya que los síntomas suelen aparecer en la infancia.
Bibliografía: Urgencias en Pediatría, Interamericana.McGraw – Hill. Capítulo: Urgencias
Médico Quirúrgicas, Sección XXIII, pág. 774-778.
Operative Pediatric Surgery. Moritz M. Ziegler. International Edition, pág. 349- 354.
Cirugía Pediátrica, Ashcraft - Holder Interamericana.McGraw – Hill pág. 257 a 277.

64.- Masculino de 13 años, ingesa a la sala de urgencias por presentar dificultad respiratoria y
tos hace 14 horas. E.F.: Fr: 45 rpm Fc: 120 lpm, tórax con tiraje intercostal, sibilancias que se
aprecian a distancia, a la auscultación se escucha disminución del murmullo vesicular con
sibilancias espiratorias bilaterales, a la percusión aumento en la resonancia pulmonar.

Se inicia administración de oxígeno, además de ésta medida el tratamiento de primera elección


es:

a) Adrenalina subcutánea.
b) Aminofilina intravenosa.
c) Clenbuterol via oral.
d) Salbutamol inhalado.
Los cuadros obstructivos e se manifiestan por síntomas y signos clínicos caracterizados por tos,
sibilancias, espiración prolongada, aumento de diámetro anteroposterior del tórax, retracción
costal, hipersonoridad a la percusión. Estos hallazgos al examen físico son inespecíficos y no
nos orientan hacia una etiología determinada. La tabla 2 detalla algunas características clínicas
que hacen sospechar algunas etiologías.

TABLA 1.
Condiciones congénitas y adquiridas que se asocian a sibilancias en niños pequeños
Frecuentes Poco frecuentes Raras
 Infecciones virale  Fibrosis quística  Masas mediastínica
(SAIV)  Displasia (tumores, TBC)
 Bronquiolitis: 1e broncopulmonar  Inmunodeficiencias
episodio  Cardiopatías congénita  Disquinesia ciliar
 Episodios recidivantes  Cuerpo extraño en ví  Bronquiolitis
 Asma bronquial aérea obliterante
 Bronquiectasias
 Síndromes aspirativos
 Malformaciones: Anill
vascular Mal
adenomatoide quística
 Quiste broncógenos
TABLA 2
Causas de Síndrome bronquial obstructivo secundario y algunas características
orientadoras en su etiología
Etiología Características Laboratorio
Displasia broncopulmonar prematurez radiografía de tórax
SDRI del RN
ventilación mecánic
período RN
dependencia de oxígeno
28 días
Fibrosis quística desnutrición electrólitos en sudor
síndrome de malabsorció
neumopatías a repetición
Cardiopatía congénita soplo radiología
insuficiencia cardíaca ECG
ecocardiografía, doppler
Aspiración cuerpo extraño episodio asfíctico brusco radiología
signos pulmonare
broncoscopía rígida
asimétricos
Reflujo gastroesofágico e vómitos recurrentes radiología
pacientes predispuestos pH metría
Trastornos de la deglución neumopatía recurrente cintigrafía de aspiració
prolongada pulmonar
daño neurológico
Malformaciones pulmonares hallazgo radiológico ecografía prenatal
radiografía de tórax
TAC
angiografía
resonancia magnética
Malformación vascular estridor radiología,
esofagograma
endoscopia
eco doppler
angiografía
resonancia magnética
Disquinesia ciliar patología sinusal radiología
biopsia epiteli
situs inverso
respiratorio

TABLA 1. Puntaje clínico modificado de Tal y cols. (modificación nacional)


PuntajeFrec. resp. Sibilancias* Cianosis Uso
< 6 m. > 6 musculatura
m.
0 <40 > 30 No No No
1 41-55 31-45 Sólo final espiración Peri oral Con (+)
llanto Subcostal
2 56-70 46-60 Esp Insp. con estetoscopio Peri oral (++)
En reposo Sub e
intercostal
3 > 70 > 60 Esp Insp sin estetoscopio o Generalizada (+++)
ausentes En reposo Supraesternal
Sub e
intercostal
* Sibilancias pueden no auscultarse en obstrucción muy grave.
Obstrucción Leve: 0 a 5 Moderada: 6 a 8 Grave: 9 a 12
------------------------------------------------------------------------------------------------------
La valoración adecuada de los signos y síntomas de gravedad de la crisis obstructiva, permitirá
racionalizar su tratamiento para disminuir la morbimortalidad por este síndrome, así como
programar junto a los padres la prevención y reconocimiento precoz de nuevos episodios
obstructivos.
Se debe considerar en forma especial a los pacientes del grupo de riesgo.
Grupo de Riesgo
1. Menor de 3 meses
2. Hospitalización previa por SBO severo
3. Antecedentes de ventilación mecánica por SBO
4. Prematurez
5. Paciente con SBO secundario
6. Requerimiento previo de curas con esteroides sistemáticos
7. Falta de cumplimiento del tratamiento, o conflicto entre los padres y el equipo médico en
relación al tratamiento.

I.- MANEJO EN EL SERVICIO DE URGENCIA:


Objetivos:
1. Corregir la hipoxemia.
2. Corregir rápidamente la obstrucción bronquial.
3. Indicar claramente el manejo ulterior en domicilio y derivación apropiada.
Tratamiento Inicial
A) Oxigenoterapia: el objetivo es lograr una Sa02 > 95%. El oxígeno debe administrarse desde el
principio con puntaje > 6, incluso cuando no se dispone de oximonitor de pulso, con mascarilla,
naricera o halo según la tolerancia individual.
B) Broncodilatadores:
Salbutamol: en aerosol de dosis medida 2 puffs (200 mcg) cada 10 min. 3 veces o en
nebulización con solución al 0.5%, 0.05 ml/kg (0.25 mg/kg), (mínimo 0.25 ml, máximo 1 ml) cada
20 min. por 3 veces.
En episodios leve y moderado se debe privilegiar la aerosolterapia. Preferir la nebulización en
pacientes graves o con Sa02 < 90% respirando aire.
1.- Técnica de Aerosolterapia
 Debe utilizarse aerocámara de aproximadamente 450 ml de volumen y 18 cms de
longitud.
 Agite el inhalador presurizado y colóquelo en posición vertical en la parte posterior de la
aerocámara.
 Aplique la aerocámara sobre la boca y nariz, con el niño sentado.
 Administre un puff y permita que el niño respire 5 a 6 veces o espere 10 segundos sin
retirar la aerocámara.
 Espere 3 a 5 minutos para aplicar un segundo puff.
2.- Técnica de Nebulización
 Nebulizador tipo Hudson o similar.
 Gas: Oxígeno
 Flujo: 6 8 lt/min
 Duración o volumen total: completar a 4 ml con suero fisiológico.
3.- Esteroides sistémicos: deben prescribirse:
 Si el episodio es grave (hospitalización UCI, Intermedio)
 Si el episodio forma parte de un cuadro obstructivo frecuente.
Se usa prednisona o prednisolona 2 mg/kg/dosis, oral, por 1 vez, dosis máxima 40 mg o
Hidrocortisona 10 mg/kg/dosis o metilprednisolona 2 mg/kg/dosis, por vía intravenosa. Privilegiar
los corticoides por vía oral, ya que son tan eficaces como los administrados por vía intravenosa.
Criterios de hospitalización
 Hospitalizar de inmediato a pacientes con:
 Puntaje > 10
 Insuficiencia respiratoria global
 Compromiso de conciencia
 Convulsiones o sospecha de agotamiento
 Cuadro clínico asociado a apneas
 Persistencia de una obstrucción grave o saturación que no mejora post tratamiento
inicial.
 Respuesta insuficiente después de 2 horas de tratamiento.
 Condiciones adversas en el hogar que determinan falla en la accesibilidad a la atención
médica de urgencia frente a un agravamiento, o falla en el cumplimiento del tratamiento.
BIBLIOGRAFIA:
 Morgan WJ, Martínez FD. Risk factors for developing wheezing and asthma in childhood.
Ped Clin N Am 1992;39:1185-1203.
 Martínez FD, Morgan WJ, Wright AL, et al. Initial airway function is a risk factor for
recurrent wheezing respiratory illnesses during the first three years of life. Am Rev Respir
Dis 1991;143:312-316.
 Wennergren G, Kristjansson S. Relationship between respiratory syncytial virus
bronchiolitis and future airway diseases. Eur Respir J 2001;18:1044-58.
 Panitch HB. Bronchiolitis in infants. Curr Opin Pediatr 2001; 13:256-60.
 Godden DJ, Ross S, Abdalla M, et al. Outcome of wheeze in childhood. Am J Respir Crit
Care Med 1994;149:106-112.

65.- Se trata de mujer de 36 años con embarazo de 32 semanas. Ingresa al servicio de


urgencias por presentar cuadro de 1 día de evolución por sangrado transvaginal leve, rojo
brillante, sin contractilidad uterina. A.G.O.: G-3, P-0, C-2. E.F.: IMC: 30 kg/m2, F.U. de 28 cm,
fcf presente, especuloscopía cérvix cerrado con huellas de sangrado. USG reporta producto
único vivo con fetometría normal, líquido amniótico normal y placenta anterior que cubre
parcialmente el orificio cervical interno.
El factor de riesgo principal para poder desencadenar esta patología es:

a) edad avanzada
b) cirugías uterinas previas
c) multiparidad
d) obesidad

PLACENTA PREVIA
DEFINICIÓN:
Es cuando la placenta se implanta sobre o muy cerca del orificio cervical interno y una parte de
la placenta precede a la parte fetal que se presenta.
INCIDENCIA:
Esta es difícil determinar ya que muchos casos pasan desapercibidos, sobre todo cuando
ocurren los abortos en embarazos tempranos.
La prevalencia varía de 1 en 100 a 1 en 850 nacidos vivos, pero solo el 20% total.
Etiología:
Edad avanzada, multíparas, paciente con cesáreas previas, paciente con aborto de repetición,
esto debido a las gestaciones previas.
TIPOS:
Inserción baja.- Es cuando el borde placentario se encuentra en el segmento inferior a menos
de 6 cm del orifico cervical interno.
Marginal.- Es cuando el borde placentario alcanza los márgenes del orificio cervical interno.
Parcial.- Es esta la placenta cubre parcialmente el orificio cervical interno.
Total.- La placenta cubre la totalidad del orificio cervical interno aún con dilatación cervical
avanzada.
DIAGNOSTICO:
La característica es el STV de aparición brusca en forma indolora en el segundo o tercer
trimestre. Frecuentemente hay ausencia de dolor a actividad uterina que son parámetros para
hacer el diagnóstico.
La mayor incidencia de sangrado aparece a las 33-34 sdg.
ESTUDIOS DE GABINETE:
El estudio más utilizado es la ultrasonografía obstétrica.
TRATAMIENTO:
El manejo va a depender de factores como son:
Edad gestacional, magnitud del sangrado, si hay trabajo de parto, variedad de placenta previa y
complicaciones materna.

BIBLIOGRAFÍA:
1.- Waxler P, Gottesfeld KR. Early diagnosis of placenta previa. Obstet Gynecol 1979;54:231-
32.
2.- Cabrero-Roura L. Riesgo elevado obstétrico. Ed. Masson 1996; pp; 109-118.
3.- Patrick J, Placenta Previa, Clinical Obst and Gynecology 1990;33(3): 414-421.
4.- Chapman M, Furtenes ET, Significance of ultrasound in location of placenta in early
pregnancy Br J Obst Gynecol 197;86: 846.

66.- Masculino de 7 meses, ingresa al servicio de urgencias por presentar en 24 hrs., 7


evacuaciones diarréicas, sin sangre, fiebre no cuantificada y vómito 4 veces. La madre refiere
que en las últimas horas el menor está hiporéxica pero bebe con avidez. E.F. : Inquieto,
fontanela anterior deprimida, ojos hundidos, llanto sin lágrimas, lienzo húmedo negativo, pulsos
normales, llenado capilar de 3 segundos.

El tratamiento inmediato para este paciente es:


a) Plan a de hidratación.
b) Plan c de hidratación.
c) Poluciones calculadas intravenosas.
d) Plan b de hidratación.

Las enfermedades diarreicas, de acuerdo con la evaluación del estado de hidratación, se


clasifican en: casos sin deshidratación, con deshidratación, con choque hipovolémico por
deshidratación.
7.2.3 Caso sin deshidratación, es aquel que presenta generalmente menos de cuatro
evacuaciones líquidas en 24 horas, ausencia de vómito, sin signos clínicos de deshidratación.
7.2.4 Caso con deshidratación, es aquel que presenta dos o más de las manifestaciones
clínicas siguientes:
7.2.4.1 Inquieto o irritable;
7.2.4.2 Ojos hundidos, llanto sin lágrimas;
7.2.4.3 Boca y lengua secas, saliva espesa;
7.2.4.4 Respiración rápida;
7.2.4.5 Sed aumentada, bebe con avidez;
7.2.4.6 Elasticidad de la piel, mayor o igual a dos segundos;
7.2.4.7 Pulso rápido;
7.2.4.8 Llenado capilar de tres a cinco segundos;
7.2.4.9 Fontanela anterior hundida (lactantes);
7.2.5 Caso con choque hipovolémico, es aquel que presenta dos o más de las manifestaciones
clínicas siguientes:
7.2.5.1 Inconsciente o hipotónico;
7.2.5.2 No puede beber;
7.2.5.3 Pulso débil o ausente;
7.2.5.4 Llenado capilar mayor de cinco segundos;
7.2.6 El manejo de los casos de enfermedades diarreicas se basa en tres planes generales
de tratamiento:
7.2.6.1 Plan A: Para pacientes con enfermedad diarreica sin deshidratación con atención en el
hogar:
7.2.6.1.1 Continuar con la alimentación habitual;
7.2.6.1.2 Aumentar la ingesta de los líquidos de uso regular en el hogar así como Vida Suero
Oral: de este último, en los niños menores de un año de edad, ofrecer media taza (75 ml) y en
los mayores de un año, una taza (150 ml) y administrarlo a cucharadas o mediante sorbos
pequeños, después de cada evacuación.
7.2.6.1.3 Capacitar a la madre para reconocer los signos de deshidratación y otros de alarma por
enfermedades diarreicas: (sed intensa, poca ingesta de líquidos y alimentos, numerosas heces
líquidas, fiebre, vómito y sangre en las evacuaciones), con el propósito de que acuda
nuevamente a solicitar atención médica en forma oportuna.
7.2.6.2 Plan B: Para pacientes con diarrea y deshidratación con atención en la unidad de salud:
7.2.6.2.1 Administrar Vida Suero Oral 100 ml por kilogramo de peso, en dosis fraccionadas cada
30 minutos durante cuatro horas;
7.2.6.2.2 Si el paciente presenta vómito, esperar 10 minutos e intentar otra vez la hidratación
oral, más lentamente;
7.2.6.2.3 Al mejorar el estado de hidratación, pasar al Plan A. En caso contrario, repetir el Plan B
por otras cuatro horas, de no existir mejoría pasar al Plan C;
7.2.6.2.4 Si los vómitos persisten, existe rechazo al Vida Suero Oral, o gasto fecal elevado (más
de 10 g/kg/hora o más de tres evacuaciones por hora) se hidratará con sonda nasogástrica, a
razón de 20 a 30 ml de Vida Suero Oral por kilogramo de peso, por hora.

7.2.6.3 Plan C: Para pacientes con choque hipovolémico por deshidratación:


7.2.6.3.1 Inicie inmediatamente administración de líquidos por vía intravenosa, con solución
Hartmann; si no se encuentra disponible, use solución salina isotónica al 0.9%, de acuerdo con
el siguiente esquema:
PRIMERA HORA SEGUNDA HORA TERCERA HORA
50 ml/kg 25 ml/kg 25 ml/kg
Evalúe al paciente continuamente. Si no mejora, aumente la velocidad de infusión.
Cuando pueda beber (usualmente en dos a tres horas), administre VSO, a dosis de 25
ml/kg/hora; mientras sigue líquidos IV.
Al completar la dosis IV, evalúe al paciente para seleccionar Plan A o B, y retirar venoclisis, o
repetir Plan C.
Si selecciona el Plan A, observe durante dos horas para asegurarse de que el responsable
encargado del paciente puede mantenerlo hidratado con VSO y además, alimentarlo en su
domicilio.
NORMA Oficial Mexicana NOM-031-SSA2-1999, Para la atención a la salud del niño.
Evaluación del estado de hidratación de un paciente con diarrea17
Signos Plan A Plan B Plan C

Definición Leve o inaparente Moderada o clínica Grave

Pérdida de agua Menos de 50 ml/kg peso o 50 a 100 ml/kg peso ó 100 ml/kg peso o más
corporal menos de 5% del peso 6 a 9% del peso (10% o más del peso)

Condición
Bien, alerta Irritable Letárgico o inconsciente
general

Globo ocular Normales Algo hundidos Muy hundidos y secos

Lágrimas Presentes Ausentes Ausentes

Mucosa oral Húmeda Seca Muy seca

Paciente bebe con Paciente bebe mal o no


Sed Paciente bebe normalmente
avidez, sediento es capaz de hacerlo

Vuelve a lo norma Se retrae lentamente Se retrae muy lentamente


Pliegue cutáneo
rápidamente <2s >2s

Tratamiento Plan A Plan B Plan C

67.- Masculino de 8 años. Es atendido en consulta por aumento de peso del escolar.
Antecedentes H.F. : Madre Con obesidad grado II portadora de DM tipo II. El niño no desayuna
al acudir a la escuela y al regresar permanece solo en casa consumiendo alimentos grasos y
poca fibra. Tiene coloración oscura en cuello. Pesa 45 kg., talla: 1.30 m., imc: 27 kg/m2 y se
encuentra en el percentil 78 para su edad y sexo.
El paso inicial en el tratamiento de este paciente es:

a) Elaborar un recordatorio nutricional de 24 horas.


b) Disminuir el consumo de grasas.
c) Incrementar consumo de verduras.
d) Programar 3 a 5 comidas diárias.

ABORDAJE TERAPÉUTICO
Para impactar en el problema de obesidad las acciones terapéuticas deberán iniciarse en el
momento de identificar un problema de sobrepeso. El abordaje terapéutico del niño con obesidad
en el primer nivel de atención debe ser preventivo dirigido a la modificación del estilo de vida y a
mejorar el patrón de alimentación. (Ver figura 2)
El sobrepeso y la obesidad por sí mismos, no requieren tratamiento farmacológico.
_ Los niños y adolescentes obesos que presentan alteraciones metabólicas secundarias pueden
ser manejados con tratamiento farmacológico, pero éste debe ser decidido y vigilado por un
médico especializado

Hábitos saludables
1. Hacer de 3 a 5 comidas diarias
2. Establecer horarios de alimentación
3. Aumentar consumo de verduras
4. Disminuir consumo de carbohidratos simples; en particular, disminuir consumo de bebidas
dulces (ej. refrescos y jugos)
5. Disminuir consumo de grasas
Ejemplo: niño en etapa escolar / horarios sugeridos de alimentación
Desayuno Colación Comida Cena
7:00 10:30 15:00 18:00 20:00
Intervenciones temporales en tanto se recibe en el segundo nivel
1. Informar que el objetivo inicial es mantener el peso
2. Explorar los hábitos alimenticios: elaborar recordatorio de alimentación
(Preguntar todo lo que ha comido en 24 horas y en qué horarios)
3. Comenzar a implantar hábitos saludables
4. Informar que los hábitos saludables deberán ser adoptados a nivel familiar para alcanzar el
objetivo inicial de mantener el peso
5. Informar que las “dietas de moda” pueden generar complicaciones y que, dado que los niños
no han completado su crecimiento, deben recibir un plan de alimentación individualizado
6. Mencionar que, a pesar de realizar cambios alimenticios para mantener el peso, es
indispensable incrementar la actividad física diaria
Peso normal Alimentación Sobrepeso y obesidad
Intervenciones temporales en tanto se recibe en el segundo nivel
1. Informar que el objetivo inicial es mantener el peso
2. Explorar los hábitos alimenticios: elaborar recordatorio de alimentación (preguntar todo lo que
ha comido en 24 horas y en qué horarios)
3. Comenzar a implantar hábitos saludables
4. Informar que los hábitos saludables deberán ser adoptados a nivel familiar para alcanzar el
objetivo inicial de mantener el peso
5. Informar que las “dietas de moda” pueden generar complicaciones y que, dado que los niños
no han completado su crecimiento, deben recibir un plan de alimentación individualizado
6. Mencionar que, a pesar de realizar cambios alimenticios para mantener el peso, es
indispensable incrementar la actividad física diaria
Alimentación
No es necesario indicar una dieta de reducción de peso, se sugiere iniciar las siguientes
acciones:
_ Cada paciente será evaluado individualmente.
a) Evaluar los hábitos alimentarios a través de un registro de alimentos, frecuencia de alimentos
y recordatorio de 24 horas.
b) Recordatorio de 24 horas. Inicialmente se entrevistará al paciente, a sus padres o tutores
acerca de los alimentos que consumió el día anterior, indagando si fue o no un día normal en la
vida del paciente (ejemplo: si asistió a una fiesta) que lo haya hecho cambiar sus hábitos en
cuanto a la comida.
c) Es recomendable tener modelos de alimentos o utensilios de medidas conocidas (taza,
cucharas, cucharitas, platos, etc.). Consultar y aplicar el Sistema Mexicano de Equivalentes
(editado por
Fomento de Nutrición y Salud, A.C., web: cuadernos@fns.org.mx), para evaluar el aproximado
de energía consumido.
d) Frecuencia de alimentos. Preguntar cuántos días de la semana consume los diferentes
alimentos para obtener la frecuencia de alimentos, registrar.
e) Analizar, con los datos obtenidos, la dieta habitual del paciente, identificando excesos y/o
carencias de algunos alimentos de los ocho grupos de alimentos (leches, productos de origen
animal, leguminosas, frutas, verduras, cereales, azúcares, grasas) y horarios de alimentación.
f) Dar orientación nutricional a la madre, padre y/o tutor y al paciente, respecto a lo encontrado,
haciéndolo consciente de que disminuya los excesos, que combine los diferentes grupos de
alimentos, que evite un solo grupo de alimentos en una comida (ejemplo: grupo cereales, tiempo
comida, menú arroz, tortas de papa, tortillas y arroz con leche).
G) Indicar cambios en los hábitos alimenticios del paciente, así como en los factores que influyen
en la alimentación, como son: actividad física, horas de sueño, horarios de alimento, compañía
en los horarios de comida, lugar para comer, marcación de límites, premios y castigos.
H) Dar orientación alimentaría, mediante el Plato del bien comer y Sistema Mexicano de
Equivalentes.

GUIA DEL ABORDAJE DIAGNÓSTICO Y DE MANEJO DE LA OBESIDAD INFANTIL

68.- Lactante de 24 meses de edad que acude al servicio de consulta externa por presentar
otorrea bilateral mucopurulenta de una semana de evolución, como antecedentes ha presentado
cuadros repetitivos de infecciones de vías respiratorias altas en todos ellos ha recibido diferentes
tratamientos. A la exploración física presenta ambas membranas timpánicas opacas e íntegras a
la rinoscopía presenta mucosa pálida y violácea con moco hialino en forma abundante,
orofaringe sin alteraciones, cuello sin adenomegalias, la radiografía lateral de cuello muestra
datos francos de obstrucción nasofaringea.

En este paciente la sintomatología se explica por:

a) Infección local mal tratada.


b) Traumatismos frecuentes.
c) Cuerpo extraño en oído externo.
d) Infecciones respiratorias recurrentes

Los factores de riesgo tradicionalmente relacionados con la aparición de los episodios de OMA
durante la infancia han sido múltiples y van a depender tanto de las características del huésped
como del ambiente que le rodea.
Entre los mismos, incrementarían el riesgo de aparición y recidiva de OMA: edad, raza, sexo,
predisposición genética, historia familiar, número de hermanos, nivel socioeconómico,
infecciones respiratorias del tracto superior, asistencia a guardería, hábito de fumar en los
padres, uso de biberones nocturnos y chupete. Por otro lado, también se han descrito algunas
intervenciones que pueden prevenir la aparición de
OMA, como: lactancia materna, uso de xilitol y vacuna antineumocócica.
En un meta-análisis realizado en 1996(21), que incluía un total de 69 estudios válidos, fueron
analizados algunos de estos factores y resultaron significativos: presencia de un familiar con
OMA (RR 2.63 IC 1.86-3.72), asistencia a guardería (RR 2.45
IC 1.51-3.98), hábito de fumar en los padres ( RR 1.66 IC 1.33-2.06) y uso de chupete ( RR 1.14
IC 1.06-2.94). Por otro lado, la lactancia materna durante al menos 3 meses protegía de la
aparición de OMA (RR 0.87 IC 0.79-0.95).

Manejo de la otitis media aguda en la infancia. Guía de práctica clínica basada en la evidencia /
Pedro Martín
Muñoz y Juan Ruiz-Canela Cáceres . -- Sevilla: Agencia de Evaluación de Tecnologías
Sanitarias de Andalucía,
2001.
ISBN: 84-923802-8-4
1. OTITIS MEDIA / TERAPIA 2. NIÑOS 3. GUÍAS DE
PRÁCTICA CLÍNICA 4. MEDICINA BASADA EN LAS
PRUEBAS I. MARTÍN MUÑOZ, Pedro II. RUIZ-CANELA
CÁCERES, Juan III. ANDALUCÍA. Agencia de Evaluación de Tecnologías Sanitarias de
Andalucía. (ed.)Dirección

69.- Acude por primera vez al servicio un lactante con facies grotesca, con macroglosia,
edema palpebral, mínima hipotonía, con hernia umbilical, la madre refiere constipación
recurrente. El diagnóstico más probable del paciente es:

a) Trisomia 21
b) Hipertiroidismo
c) Enfermedad de Gaucher
d) Hipotiroidismo

El hipotiroidismo congénito

CONCEPTO Y CLASIFICACIÓN

Se define el hipotiroidismo como la situación resultante de una disminución de la actividad


biológica de las hormonas tiroideas a nivel tisular, bien por una producción deficiente o bien por
resistencia a su acción en los tejidos diana, alteración de su transporte o de su metabolismo.

El hipotiroidismo puede ser debido a causas prenatales (congénito) o postnatales (adquirido), sin
embargo desde un punto de vista operacional e independiente de la etiología, se puede
considerar que el HC es el fallo tiroideo detectado en el cribado neonatal, y el adquirido aquel
que no se detecta en el mismo.

En base al concepto de su definición, el hipotiroidismo se puede dividir en dos grandes grupos:

A. Hipotiroidismo por disminución de la producción hormonal. En función de la localización


del transtorno causal se clasifica en:

1. Primario o tiroideo, cuando la causa radica en la propia glándula tiroides.

2. Hipotálamo-hipofisario o central, cuando el transtorno está localizado en la


hipófisis (déficit de TSH) en cuyo caso se denomina hipotiroidismo secundario o
en el hipotálamo (déficit de TRH) conociéndose como hipotiroidismo terciario. A
su vez, el hipotiroidismo puede ser esporádico o hereditario, y bajo el punto de
vista evolutivo, permanente o transitorio.

B. Síndromes de sensibilidad reducida a las hormonas tiroideas. Bajo este amplio término
se engloba a aquellas entidades en las que la producción de hormonas tiroideas no está
descendida sino que está reducida su sensibilidad. A este grupo pertenece:

1. El síndrome de resistencia a las hormonas tiroideas: gen TRß, para el que se


sigue restringiendo el término "resistencia".

2. Defecto del transporte celular de las hormonas tiroideas: gen MCT8.

3. Defecto del metabolismo de las hormonas tiroideas: gen SECISBP2.


.

SINTOMAS Y SIGNOS DE HIPOTIROIDISMO CONGENITO

En el período neonatal

Gestación > 42 semanas


Peso de Nacimiento > 4 kg
Ictericia prolongada > 3 d
Hiperbilirrubinemia no conjugada en hipotiroidismo primario
Hiperbilirrubinemia conjugada y no conjugada en hipotiroidismo 2rio o 3rio
Edema
Hipotermia
Dificultad en la alimentación
Fontanela posterior > 5 mm
Hernia umbilical
Distensión abdominal
Distress respiratorio

Durante el primer mes

Cianosis periférica y livedo reticularis


Piel áspera y seca
Constipación
Letargia e hipoactividad
Llanto ronco
Macroglosia
Mixedema generalizado

Otros antecedentes que sugieren el diagnóstico son:

Vivir en zona con carencia de yodo


Enfermedad tiroídea autoinmune materna.
Antecedente familiar de dishormonogénesis.
Uso de yodo y drogas antitiroídeas durante el embarazo.
Consumo de alimentos bociógenos.

ETIOLOGIA DEL HIPOTIROIDISMO CONGENITO


I. Hipotiroidismo permanente
I.a.- Primario
Disgenesia tiroídea: Agenesia
Ectopia
Hipoplasia
Dishormonogénesis: Defecto en la captación de yodo
Defecto en la organificación
Defecto de la yodinasa
Defecto de la síntesis de tiroglobulina
Defecto del receptor de TSH
Iatrogenia Exposición materna a I 131
I.b.- Secundarios o Terciarios
Deficiencias múltiples de hormonas hipotalámicas
Idiopática
Familiar
Asociada a defectos de la línea medi
Deficiencia aislada de TRH
Deficiencia aislada de TSH
II. Hipotiroidismo transitorio
Carencia de yodo
Exposición materna o neonatal a yodo
Terapia materna con drogas
antitiroídeas
Enfermedad tiroídea autoinmune
materna.
Nefrosis congénita
Hipertirotropinemia idiopática
Hipotiroidismo primario idiopático

1. LaFranchi S. Congenital hypothyroidism: etiologies, diagnosis, and management. Thyroid


1999; 9: 735-40.
2. Van Vliet G. Neonatal Hypothyroidism: treatment and outcome. Thyroid 1999; 9: 79-84.
3. Congenital hypothyroidism: long-term outcome. Thyroid 1999; 9: 741-8.

Zimmerman D. Fetal and neonatal hyperthyroidism. Thyroid 1999; 9: 727-33.

70.- Se trata de paciente masculino de 4 semanas de vida, primogénito, que tiene vómitos
progresivos gástricos, posprandiales inmediatos, desde la segunda semana de vida; E.F.
distensión abdominal, peristalsis presente .La posibilidad diagnóstica es:

a) Mala técnica en la alimentación


b) Reflujo gastroesofágico
c) Estenosis hipertrófica de píloro
d) Bandas de Ladd

La estenosis hipertrofia del píloro (EHP) es una emergencia médico-quirúrgica y representa una
de las causas más frecuentes de cirugía abdominal en las primeras semanas de vida1'2.La
exacta causa de la hipertrofia del músculo pilórico es desconocida1'3'4. Se caracteriza por
obstrucción del vaciamiento gástrico, lo que clínicamente se expresa como vómitos progresivos,
frecuentemente de carácter explosivo que pueden llevar a deshidratación grave con riesgo vital
del paciente1' 3"5.
ESTENOSIS HIPERTR ÓFICA DE
HIPERTRÓFICA
PÍLORO
PÍLORO
V
 ómito
Vómito
– Nunca biliar
– Alimentario
– Progresivo  Proyectil.
Proyectil.
Proyectil.
– Pozos de café
café..
café
– Posprandial inmediato.

 Hambre despué
 después del vó
después vómito.
vómito.

ESTENOSIS HIPERTRÓFICA DE
HIPERTRÓFICA
P ÍLORO
PÍLORO
5. DIAGNÓ
DIAGN ÓSTICO (PUNTOS CLAVE)
DIAGNÓSTICO
 V
 ómito no biliar
Vómito
 Peristalsis visible.

 Oliva piló
 pilórica palpable.
pilórica
 Hipocloremia,
 Hipocloremia, hipokalemia.
hipokalemia.
 Alcalosis metabó
 metab ólica.
metabólica.

Bibliografía:

Urgencias en Pediatría, Interamericana.McGraw – Hill. Capítulo: Urgencias Médico Quirúrgicas,


Sección XXIII, pág. 718-721.
Operative Pediatric Surgery. Moritz M. Ziegler. International Edition, pág. 583- 588.
Cirugía Pediátrica, Ashcraft - Holder Interamericana.McGraw – Hill pág. 297– 313

71.- Se trata de lactante de 2 semanas, con un desarrollo y crecimiento normales, actualmente


recibe lactancia materna, por datos de ictericia se realizan paraclínicos con datos de
hiperbilirrubinemia de 10mg/dl, la medida más adecuada para el tratamiento del menor es:

a) Fototerapia
b) Exanguineotransfusión
c) Fenobarbital
d) Esperar un par de días y repetir la prueba.
La fototerapia es el empleo de luz visible para el tratamiento de hyperbilirubinemia en el recién
nacido (RN). Esta terapia relativamente común baja el nivel de bilirrubina en el suero por
transformación de la bilirrubina en isómeros solubles en agua que pueden ser eliminados sin la
conjugación en el hígado
La fototerapia convierte la bilirrubina que está presente en los capilares superficiales y espacio
intersticial a isómeros solubles en agua que son excretables sin pasar por el metabolismo del
hígado (Fig. 4). Maisels, un notable experto en bilirrubina, sugiere que la fototerapia se parece
mucho a una droga percutanea. Cuando la fototerapia ilumina la piel, una infusión de fotones de
energía, como moléculas de una medicina, es absorbida por la bilirrubina de la misma manera
que una molécula de medicina se une a un receptor.
Las moléculas de bilirrubina en la piel expuestas a la luz sufren las reacciones fotoquímicas
relativamente rápido, configurational isomerization, isomerización estructural, y la forma de
fotooxidación no tóxica, isómeros excretables. Estos isómeros de bilirrubina tienen formas
diferentes del isómero natal, son más polares, y pueden ser excretados del hígado en la bilis sin
sufrir la conjugación o requerir transporte especial para su excreción. La eliminación urinaria y
gastrointestinal son ambas importantes en reducir la carga de bilirrubina.

Figura 4. El mecanismo de fototerapia. Cuando las moléculas de bilirrubina absorben la luz, 2


reacciones fotoquímicas principales ocurren: el natural 4Z, 15Z-bilirubin se convierte a 4Z, 15E
bilirubin (también conocido como photobilirrubina) y a lumirrubina. A diferencia de 4Z, 15Z la
bilirrubina, photobilirrubina puede ser excretado vía hepática sin la conjugación, pero su
clearance es muy lento, y su conversión es reversible. En el intestino (lejos de la luz),
photobilirrubina es convertida atrás a bilirubina natal.
La lumirrubina no es reversible. Aunque mucho menos lumirrubina que photobilirrubina es
formado, lumirrubina es eliminado del suero mucho más rápidamente, y es probable que la
formación de lumirrubina es principalmente responsable de la disminución en el suero de la
bilirrubina. Las pequeñas cantidades de bilirrubina natal también son oxidadas a monopyrroles y
dipyrroles que pueden ser excretados en la orina. Esto es un proceso lento y sólo un contribuidor
menor a la eliminación de bilirrubina durante la fototerapia. Cortesía de diagrama de María
Puchalski.

El objetivo de la fototerapia es disminuir la bilirrubina sérica y prevenir su acumulación tóxica en


el cerebro, donde puede causar serias complicaciones neurológicas permanente conocido
como kernicterus. La fototerapia ha reducido enormemente la necesidad de exanguíneo
transfusión para tratar la hiperbilirrubinemia.

La fototerapia es usada de 2 modos principales: profiláctica y terapéuticamente.


 En RN prematuros o aquellos con un conocido proceso hemolítico, a menudo es
usado profilacticamente, para prevenir un rápido aumento de la bilirrubina sérica.
 En pretérminos pequeños o RN de término, es administrada en dosis terapéuticas
para reducir niveles de bilirrubina excesivos y evitar el desarrollo de kernicterus.
La fotoisomerización de bilirrubina comienza casi al instante cuando la piel es expuesta a la luz.
A diferencia de la bilirrubina no conjugada, los fotoproductos de estos procesos no son
neurotóxicos. Por lo tanto, ante una hiperbilirrubinemia severa del RN, es importante comenzar la
fototerapia sin retraso.

Referencias
Stokowski LA. Early recognition of jaundice and kernicterus. Adv Neonatal Care 2002;2:101-114.
Maisels MJ. A primer on phototherapy for the jaundiced newborn. Contemp Pediatr. 2005; 22(6):
passim. (OR, 38, 40, 44, 47, 48, 53, 54,57).
1. McDonagh AF. Phototherapy: from ancient Egypt to the new millenium. J Perinatol
2001;21:S7-S12.
2. Maisels MJ. Phototherapy-traditional and nontraditional. J Perinatol 2001; 21(Suppl
1):S93-S97.
72.- Dentro de los eventos adversos podemos esperar por la aplicación de vacuna: Difteria
Pertusis Tetanos es:

a) Adenitis supuras y encefalitis.


b) Parálisis flácida aguda.
c) Episodio hipotónico e hiporreactivo y llanto incontrolable y persistente por más de
3 horas.
d) Exantema maculopapular y fiebre.

Los sistémicos se presentan dentro de las 48 horas después de la vacunación, se han notificado
(fiebre en el 40% de los vacunados, en el 5 % llanto persistente e incontrolable por más de tres
horas, somnolencia, irritabilidad y malestar general. En menos del 3% cefalea, convulsiones,
calosfrío, mialgias y artralgias.

Committee on Infectious Diseases 2006. Red Book 2006, 27th Ed.


American Academy of Pediatrics; 2006

73.- Lactante masculino de 8 meses de edad, con 8 Kg. de peso, antecedentes de ser sano, de
manera súbita inicia con cólico intenso acompañado de sudoración y palidez, rechazo de
alimentos, vómitos y evacuaciones mucosanguinolentas, a la E.F. Se palpa masa en colon
transverso, la primera posibilidad diagnóstica es:

a) Apendicitis aguda.
b) Divertículo de Meckel.
c) Invaginación intestinal.
d) Malrotación intestinal.

Invaginación intestinal

Obstrucción intestinal aguda provocada por la introducción del tubo digestivo dentro de sí mismo.

Clínica: lactante sano y bien nutrido, que de forma súbita, cada 10-15 minutos, durante unos
minutos, presenta crisis de llanto, acompañadas de síntomas vagales (palidez, sudoración,
decaimiento), irritabilidad y rechazo del alimento. Inicialmente permanece asintomático entre los
episodios de llanto. De forma progresiva aparecen vómitos, anorexia, decaimiento, sangre roja
en heces, e incluso colapso vascular y shock.

Exploración física:

• Fosa iliaca derecha vacía. Signo de Dance (13%).

• Palpación de masa en hipondrio derecho y colon transverso “signo de morcilla” (24 a


90%)

• Tacto rectal, “hocico de tenca” (0.5 al 3%)


Puede presentar sangre en el tacto rectal, aumento de ruidos hidroaéreos y excepcionalmente
prolapso rectal de la invaginación.

Rx. Simple:

• Patrón anormal de aire


• Opacidad en CID 25-60%
• Datos de oclusión

• Distensión de asas

• Niveles hidroaéreos

• Ausencia de aire en recto

• Tratamiento:

• Ayuno.

• Terapia hídrica.

• Esquema Antimicrobiano. Ampicilina-Amikacina.

• Sonda a derivación

• Catéter Venoso.

• Sonda Urinaria.

• O’NEILL PEDIATRIC SURGERY 1998 URG. PED. HIM 5 ED 2002


OLDHAM SURGERY OF INFANTS AND CHILDREN. 1997

74.- Se trata de masculino de 14 años que acude a su consultorio con datos clínicos compatibles
con orquiepididimitis, el tratamiento de elección en ésta patología es:

a) Exploración quirúrgica de inmediato


b) Vigilancia y exploración quirúrgica en 24 hrs
c) Administración de antibióticos y de antiinflamatorios.
d) Administración de AINES.

El tratamiento de una orquiepididimitis consiste en:


— Medidas generales: reposo en cama durante al menos 5 días o hasta que ceden los síntomas
y utilización de un suspensorio.
— Antiinflamatorios y analgésicos a dosis habituales.
— Antibióticos:
Empíricamente si no conocemos germen causal, se asocian:

Pacientes con práctica sexual:


Ceftriaxona o azitromicina en mono dosis.
Continuar con doxiciclina por 14 días ( chlamydya T . Neiseria g.)
Levo u ofloxacino 10 a 14 días

Pacientes sin práctica sexual:


TMP/SMZ 10 A 14 DÍAS.
Se ha reportado resistencia por 50% por lo que se puede utilizar:
Amoxicilina 500 mg vo. X3x 10
Cefalexina 500 mg x3 x10
Ceftriaxona 1gr i.m. 1x1x10
Amikacina 15 mg /kg divido cada 12 hrs 10 días

NOM 2008

Referencias:

1. Young Y., Miller R Incidence of malignant tumours in U.S. children. J Pediatr 1975; 86: 254-
258.
2. Reiter A., Schrappe M. Chemotherapy in 998 unselected childhood acute lymphoblastic
leukemia patients. Results and conclusions of the multicenter trial ALL-BFM 86. Blood 1994;
84: 3122-3133.
3. Schorin M., Blattner S., Gelber R., et al.
Treatment of childhood acute lymphoblastic leukemia: Results of Dana Farber Cancer
Institute/Children s Hospital acute lymphoblastic
leukemia consortium protocol 85-01. J Clin
Oncol 1994; 12: 740-747.

75.- Un niño de 4 años de edad previamente sano aunque no vacunado presenta inicio súbito
de fiebre alta estridor inspiratorio, rechazo para beber agua y se aprecia posición tripoide. De las
siguientes causas de estridor inspiratorio, el que mejor se ajusta a este cuadro clínico es:

a) Epiglotitis
b) Anillo vascular
c) Aspiración de cuerpo extraño
d) Tumor laríngeo

La epiglotitis aguda (EA) es una inflamación de la epiglotis y las estructuras adyacentes de


instauración brusca y rápidamente progresiva, que se produce sobre todo en niños pequeños.
Su consecuencia más importante es la capacidad de provocar una obstrucción severa e incluso
total de la vía aérea superior, pudiendo causar la muerte.
La introducción de la vacuna contra el Haemophilus influenzae tipo B (Hib), el principal patógeno
implicado en la EA, ha reducido notablemente su incidencia, especialmente en los niños
menores de 5 años.

Clínica
El caso típico de EA es el de un varón de 2 a 4 años de edad que presenta en cualquier
momento del año una historia de 6 a 12 horas de fiebre elevada y disfagia. La odinofagia, más
frecuente en niños mayores y adultos, se observa en el 85% de éstos3. De forma relativamente
rápida, se instaura dificultad respiratoria, que hace que el paciente adopte una postura hacia
adelante, con el cuello en hiperextensión y la boca abierta con la lengua fuera, presentando una
postura clásicamente llamada "en trípode". Se muestra ansioso e inquieto. Es típico el babeo,
aunque no constante, pudiendo faltar en la mitad de los casos. Al contrario que en las laringitis
agudas,el estridor inspiratorio no es tan ruidoso, sino de tono bajo y húmedo, y es rara la tos.
BIBLIOGRAFIA
1. AGUDELO, Bertha. Tratamiento Del Croup Basado En La Evidencia. Revista
Colombiana De Neumología. Volumen 12 Nº 4, Noviembre De 2000
2. REYES, Aristizábal, Leal. Neumologia Pediatica: Infeccion, Alergia Y Enfermedad
Respiratoria Del Niño. 4° Edición, Bogotá: Editorial Medica Panamericana, 2001

76.- Lactante menor próximo a cumplir 2 meses, atendido en consulta para control de niño sano.
Se refiere asintomático. E.F.: dentro de límites normales. Antecedente: esquema de vacunación
completo para la edad.
De acuerdo a la cartilla nacional de vacunación, que vacunas debe recibir en esta consulta:

a) Pentavalente celular, hepatitis b, rotavirus, neumococo conjugada


b) Pentavalente acelular, hepatitis b, rotavirus, neumococo conjugada.
c) Pentavalente acelular, hepatitis b, rotavirus, neumococo polisacárida
d) Pentavalente acelular, hepatitis a, rotavirus, neumococo polisacárida.

77.- You are called to the nursery to see a baby who has noted to be jaundiced and has a
serum bilirrubin concentration of 13 mg/dl at 18 hours of age. The baby is a 3500 g boy who
was born at term to a 27-year- old primigravida 16 hours after membranes ruptured. There
were no prenatal complications. Breast-feeding has been well tolerated. Of the following, which
is more likely to be responsible for jaundice in this baby ?

a) Rh or ABO hemolytic disease


b) Physiologic jaundice
c) Sepsis
d) Congenital spherocytic anemia
Ictericia fisiológica. Ictericia monosintomática de inicio a partir del segundo día de vida, con
un pico máximo de B de 12-15 MG/Dl. en el 3°-5° día, no persistiendo más allá del 7día. No
requiere tratamiento pero sí observación y seguimiento por si se tratase de una ictericia
patológica. Se debe a una limitación del hígado para metabolizar el exceso de B producida en
los primeros días de vida.

Metabolismo de la bilirrubina e ictericia fisiológica.

Durante la etapa fetal, la mayor parte de la bilirrubina atraviesa la placenta y es metabolizada


por el hígado materno. En el momento del nacimiento este proceso se corta bruscamente y al
igual que lo que ocurre con otros procesos fisiológicos, debe ser asumida por los órganos y
sistemas del recién nacido los cuales deben adaptarse y madurar para asumir el proceso en
forma eficiente. La Figura 7.1 muestra las distintas etapas del metabolismo de la bilirrubina.
Ellas nos permitirán comprender las causas que llevan a la elevación de la bilirrubina sérica en
los primeros días de vida.

Metabolismo de la Bilirrubina

Producción de la bilirrubina: En el período neonatal, la producción de bilirrubina está


aumentada debido a la alta masa eritrocitaria del neonato (hematocritos de 61% ± 7,4) y a que
la vida media del glóbulo rojo es más corta, alrededor de 90 días comparada con 120 días en el
adulto.

Transporte de la bilirrubina: La bilirrubina no conjugada (BNC) circula en el plasma unida a la


albúmina. Normalmente en estas condiciones no atraviesa la barrera hematoencefálica. Puede
aparecer BNC libre (no unida a la albúmina) en condiciones en que la cantidad de bilirrubina
supera la capacidad de unión de la albúmina. Esto puede ocurrir porque hay cifras muy altas de
bilirrubina, hipoalbuminemia o presencia de substancias y factores que desplazan o debilitan la
unión de la bilirrubina con la albúmina. La presencia de BNC libre es siempre anormal y resulta
en paso de esta al SNC y eventual daño del cerebro

Captación, conjugación y excreción hepáticas: La bilirrubina es captada por receptores


específicos del polo sinusoidal del hepatocito y luego transportada por proteínas específicas al
retículo endoplasma. Aquí es conjugada con ácido glucurónico por acción de la enzima
glucuronil transferasa. Se obtiene así la llamada bilirrubina conjugada (BC) que se caracteriza
por ser soluble en agua y no difundir a través de las membranas celulares. La actividad de la
glucuronil transferasa es más baja en los primeros días de vida El principal estímulo para
aumentar su actividad son los niveles séricos de bilirrubina. También puede ser estimulada con
fenobarbital.

La excreción de la BC es un proceso de transporte activo a través de la membrana del


hepatocito hacia los canalículos biliares. Luego es transportada como un componente de la bilis
al intestino.

Circulación enterohepática de la bilirrubina: La BC que llega al duodeno es en parte


eliminada por las deposiciones, previa transformación en urobilinógeno y similares, por la
acción de las bacterias y en parte reabsorbida pasando nuevamente a la circulación, luego de
haber sido desconjugada del ácido glucurónico en el intestino por acción de la enzima
betaglucuronidasa. En el recién nacido hay varios factores que favorecen la reabsorción
intestinal de la bilirrubina, especialmente en los primeros días de vida:

 ausencia de bacterias.
 menor movilidad especialmente si hay ayuno.
 niveles altos de betaglucuronidasa, enzima que hidroliza la BC en BNC, la cual puede
ser reabsorbida.

En resumen los principales factores que explican la ictericia fisiológica del recién nacido son:

 Aumento de la producción de la bilirrubina


 Limitaciones en la captación y conjugación de la bilirrubina
 Aumento de la reabsorción intestinal de la BNC

Bibliografía

 AAP Subcommittee on Neonatal Hyperbilirubinemia. Neonatal jaundice and kernicterus.


Pediatrics 2001 Sep;108(3):763-5.
 American Academy of Pediatrics. Practice Parametres: Management of
Hiperbilirubinemia in the Healthy Term Newborn. Pediatrics 1994; 94; 558-565.
 Bezerra JA, Balistreri WF. Cholestatic syndromes of infancy and childhood. Semin
Gastrointest Dis 2001 Apr;12(2):54-65.
 Harris MC, Bernbaum JC, Polin JR, Zimmerman R, Polin RA. Developmental follow-up
of breastfed term and near-term infants with marked hyperbilirubinemia, Pediatrics 2001
May;107(5):1075-80
 Connolly AM y Volpe JJ: Clincal Features of Bilirubin Encephalopathy. Clin Perinatol
1990; 17: 371-379
 Gartner LM, Herschel M. Jaundice and breastfeeding.Pediatr Clin North Am. 2001
Apr;48(2):389-99.
 Geiger AM, Petitti DB, Yao JF Rehospitalisation for neonatal jaundice: risk factors and
outcomes. Paediatr Perinat Epidemiol 2001 Oct;15(4):352-8
 Kemper K, Forsyth B, McCarthy P: Jaundice, terminating breast-feeding, and the
vulnerable child. Pediatrics. 1989; 84; 773-778.
 Kramer Ll: Advancement of dermal icterus in the jaundiced newborn. AJDC. 1969; 118:
454.
 Hansen TW. Bilirubin brain toxicity. J Perinatol 2001 Dec;21 Suppl 1:S48-51

78.- Se trata de R/N masculino prematuro 33 sdg, hijo de madre de 29 años de edad gesta 2,
con control prenatal regular, diabética controlada y nace por parto. Apgar 8/9. Inicia con
dificultad respiratoria la cual se presenta progresiva. Peso 1.950KG. La probabilidad diagnóstica
es:

a) Enfermedad de membranas hialinas


b) Taquipnea transitoria del Recién Nacido
c) Transición
d) Neumonía

ENFERMEDAD DE MEMBRANA HIALINA


Definición:
El Síndrome de distress respiratorio idiopático (SDRI), conocido también como enfermedad
de membrana hialina, se origina en la deficiencia de surfactante pulmonar. Éste último,
mezcla de fosfolípidos, principalmente dipalmitoil fosfatidil colina, es el responsable de la
estabilización distal del alvéolo a volúmenes pulmonares bajos al final de la espiración,
gracias a que reduce la tensión superficial. Cuando existe déficit de surfactante, el recién
nacido puede no ser capaz de generar el aumento de la presión inspiratoria requerido para
insuflar las unidades alveolares, lo que resulta en el desarrollo de atelectasia progresiva (1).
EMH
 Conjunto de signos y síntomas clínicos, radiológicos y gasométricos atribuibles a la
deficiencia o ausencia de surfactante pulmonar .
 Incidencia inversamente proporcional con edad gestacional (60% a 80% en menores de
28 SDG, 15% A 30% de 32 a 36 SDG, 5% de 36 a 37 SDG)
MAYOR RIESGO:
 Hijo de madre diabética sin compromiso vascular (macrosómico)

 Asfixia (induce vasoconstricción pulmonar, persistencia de circulación fetal)


 Hermanos afectados previamente

Fisiopatología:
La hipoxemia se produce inicialmente por una alteración de la relación ventilación perfusión
debido al colapso difuso del pulmón, contribuyendo posteriormente al desarrollo de shunts
intra y extrapulmonares.

El déficit de surfactante también origina inflamación pulmonar y daño del epitelio respiratorio
conduciendo a edema pulmonar y aumento de la resistencia de la vía aérea. Esto último
contribuye al daño pulmonar con mayor deterioro de la función pulmonar. Paralelamente, el
pulmón dañado disminuye su capacidad de reabsorción de líquido, la que resulta
ineficiente, contribuyendo al edema pulmonar. Finalmente, éste último puede estar,
además, exacerbado por las respuestas sistémicas al síndrome de distress respiratorio que
contribuyen a la retención de líquidos.

El déficit de surfactante y el edema pulmonar conducen a anormalidades en la función


pulmonar que llevan a hipoxemia. Las anormalidades primarias en la mecánica pulmonar
son la disminución de la compliance y la disminución del volumen pulmonar que se refleja
en la disminución de la capacidad residual funcional.

Cuadro clínico:
Los recién nacidos con este síndrome casi siempre son prematuros. Las manifestaciones
clínicas son el resultado de la función pulmonar anormal y la hipoxemia. El síndrome de
dificultad respiratoria y la cianosis se presentan rápidamente después del nacimiento. Los
recién nacidos afectados presentan taquipnea y distintos grados de dificultad respiratoria.
La taquipnea, al acortar el tiempo espiratorio, reduce la pérdida del volumen pulmonar en el
pulmón con déficit de surfactante. Otros signos típicos son el quejido, un esfuerzo
compensatorio para prevenir el colapso alveolar al final de la espiración; aleteo nasal que
reduce la resistencia nasal y refleja la utilización de musculatura respiratoria accesoria; y
retracción esternal, subcostal e intercostal debido a la disminución de la compliance
pulmonar asociado a una pared torácica muy complaciente. A la auscultación, los sonidos
respiratorios estarán disminuidos. Los recién nacidos aparecerán pálidos con disminución
de los pulsos periféricos. La diuresis habitualmente es baja durante las primeras 24 a 48
horas y es común el edema.

Historia natural:
Dado que corresponde a un trastorno primario del desarrollo de la producción de
surfactante, el síndrome de dificultad respiratoria se presenta típicamente al nacer. Si no se
trata empeora progresivamente durante las primeras 48 horas de vida. En algunos casos,
los recién nacidos pueden no manifestar enfermedad inmediatamente después de nacer y
desarrollan dificultad respiratoria y cianosis dentro de las primeras horas de vida. Estos
recién nacidos pueden tener una cantidad límite de surfactante pulmonar que se consume
o se inactiva rápidamente. La historia natural de la enfermedad se modifica enormemente
por el tratamiento con surfactante exógeno. Antes del uso de surfactante, la enfermedad de
membrana hialina no complicada progresaba típicamente durante las primeras 48 a 72
horas, lo que iba seguido de una mejoría de la función respiratoria asociada a la producción
de surfactante endógeno para resolverse alrededor de la semana de vida. La mejoría era
precedida de una diuresis marcada. La administración de surfactante generalmente acorta
el curso clínico de la enfermedad.
Tratamiento:

 El objetivo es ventilar los pulmones atelectasiados y mantener las medidas generales


adecuadas hasta su resolución

MEDIDAS GENERALES
 Colocar en incubadora o cuna de calor radiante. Monitorear con oximetría de pulso
 Control estricto de líquidos y glucosa (60-80 ml/k/d, glucosa al 10%, incrementar a 120-
160 ml/kg/d para el día 5 de vida), muy prematuros pueden requerir hasta 200 ml/k/d
 Manipulación mínima
MANEJO DE LA VÍA AEREA
 Posición de olfateo
 Posición prona mejora el trabajo ventilatorio
 Aspiración gentil de secreciones en fase exudativa (inicia a las 48 horas de vida)
ADMINISTRACION DE OXIGENO
 Debe administrarse humidificado, caliente. Se puede administrar en casco cefálico,
mascarilla, puntas nasales, o cánula orotraqueal.
 Mantener PaO2 entre 50 y 80 Torr y/o saturaciones entre 85% y 95% PCO2 entre 40 y
55 Torr

MENEJO NO VENTILATORIO

 Catéteres umbilicales
 Hematocrito
 Acidosis
 Alimentación
 Infección
 Aminofilina
 Indometacina
MANEJO VENTILATORIO

 Casco cefálico FIO2- 88% y 96%


 CPAP si FIO2 mayor de 30% tomar gasometría y RX de tórax
 Si >1,500g evaluar 30 min. Tomar RX y gasometría
CONSIDERAR INTUBACION
 Hipercarbia (PaCo2 > 55 Torr)
 Hipoxemia (PaO2 < 50 Torr)
 Pobre esfuerzo ventilatorio o apnea
 Incremento de la dificultad respiratoria
 Plan de administrar surfactante
 Acidosis metabólica persistente (<7,25)

ESTRATEGIA VENTILATORIA MANEJO CON SIMV

 PEEP entre 4 a 6 cm H2O


 Tiempo Inspiratorio entre 0.25 y 0.35 segundos
 PIM adecuada para asegurar un volumen corriente entre 4 y 7 ml/kg
 Ajustar los ciclados para mantener la PaCo2 en límites fijados, no exceder 60 ciclados
por minuto
 Flujo necesario para llevar al PIM fijado

SUFRACTACTE.
Existen dos tipos de surfactantes:
1) El derivado de animales o “natural”
2) Los sintéticos

 SURFACTANTES NATURALES
Pueden obtenerse de pulmones porcinos o bovinos. Los bovinos son: BERACTANT
(SURVANTA), SURFACTANT TA (SURFACTEN), CALFACTAN T (INFASURF), SF-RI1
(ALVEOFACT). Los porcinos son: PORACTANT (CUROSURF)

 SURFACTANTES SINTÉTICOS
COLFOSCERIL (EXOSURF), COMPONENTE ARTIFICIAL DE EXPANSIÓN PULMONAR
(PULMACTANT), LUCINACTANT

79.- Se trata de paciente femenino de 2 años, se encuentra en sala de espera de consulta con,
fiebre, presenta crisis convulsiva caracterizada por sacudidas de brazos y piernas que duran 3
minutos cede espontáneamente. Antecedente: cuadro catarral de tres días de evolución. E.F.:
FC: 110 LPM. FR: 28 RPM, TA 85/70, TEMP: 39.0 ºC. PESO 14 KG., actualmente se observa
somnolienta pero se puede despertar, nariz con rinorrea cristalina, oídos normales, faringe
hiperémica, buen esfuerzo respiratorio y buena perfusión.
El diagnóstico más probable en este paciente es:

a) Tumor intracraneal.
b) Meningitis.
c) Ingesta de tóxico.
d) Crisis convulsiva febril.
Descripción detallada de las crisis
convulsivas
 Síntomas motores o
1. Estado de sensoriales
consciencia
 Desviación de la
2. Duración mirada
3. Aura
 Movs. Clónicos
4. Postura unilaterales
5. Cianosis
 Parestesias
6. Control de esfínteres
(urinario)  Dolor localizado en
una área
7. Estado postictal
(sueño, cefalea,
hemiparesias)

Crisis Febriles
 Predisposición genética
 Edad: 9 meses a 5 años
 media: 14 a 18 meses
 3 a 4%
 Gen: cromosoma 1ap y 8q13-21
 Descartar: sepsis , meningitis

Crisis Febriles  30a 50%


recurrentes
 Temperatura > 39° C <12m
 Generalizada tónica /  Historia familiar
clónica  Riesgo de
 Segundos a <15min
epilepsia si:
 Alteración neurológica
 Periodo postictal previa
 Crisis febril <12m
 Retraso en el desarrollo
 Historia familiar

80.- Femenino de 19 meses, es llevada por su madre al servicio de urgencias de Pediatría por
presentar exantema generalizado. Antecedentes: Fiebre de 38.5 , Irritabilidad, anorexia, y haber
recibido tratamiento a base de acetaminofen 48 hrs. previas a la consulta. Exploración física:
Temp. 36.8, TA 100/65mmHg, FC 97 x´, FR 26 x´, exantema en tronco, cuello y cara
La aparición exantemática que apoya el diagnóstico en este paciente es:

a) Cefalocaudal
b) Caudocefálica
c) Toracoabdominal
d) Centrípeta

Roséola: Es una enfermedad febril exantemática leve, que ocurre casi en exclusiva durante la
lactancia. El período prodrómico de la roséola suele ser asintomático, pero puede presentarse
con signos leves del tracto respiratorio superior, entre ellos rinorrea mínima, inflamación
faríngea ligera y enrojecimiento conjuntival leve, adenoparias cervicales.
La enfermedad clínica suele ser precedida por temperatura alta, la fiebre persiste durante 3 a 5
días, y después es típica la resolución brusca. El exantema aparece en 12-24 hrs siguientes a la
resolución de la fiebre, comienza en forma de lesiones rosadas, un poco elevadas, pequeñas, y
delimitadas en tronco, y se suele extender al cuello, cara, y regiones proximales de las
extremidades, no suele ser pruriginoso.
Nelson, Tratado de Pediatría, 17ª Edición, Ed. Elsevier, Pág. 1069-1071

También podría gustarte